USMLE Platinum Notes Step 2, 2e PDF
USMLE Platinum Notes Step 2, 2e PDF
USMLE Platinum Notes Step 2, 2e PDF
PLATINUM NOTES
The Complete Preparatory Guide
Second Edition
Ashfaq UI Hassan
MBBS MS
Consultant Anatomy
SKIMS Medical College
Bemina, Srinagar, Jammu and Kashmir, India
Overseas Offices
J.P. Medical Ltd. Jaypee-Highlights Medical Publishers Inc.
83, Victoria Street, London City of Knowledge, Bld. 237, Clayton
SW1H 0HW (UK) Panama City, Panama
Phone: +44-20 3170 8910 Phone: +1 507-301-0496
Fax: +44-(0)20 3008 6180 Fax: +1 507-301-0499
Email: info@jpmedpub.com Email: cservice@jphmedical.com
JP Medical Inc. Jaypee Brothers Medical Publishers (P) Ltd.
325 Chestnut Street, Suite 412 17/1-B, Babar Road, Block-B, Shaymali
Philadelphia, PA 19106, USA Mohammadpur, Dhaka-1207, Bangladesh
Phone: +1215-713-7181 Mobile: +08801912003485
Email: jrusko@jpmedus.com Email: jaypeedhaka@gmail.com
Jaypee Brothers Medical Publishers (P) Ltd.
Bhotahity, Kathmandu, Nepal
Phone: +977-9741283608
Email: Kathmandu@jaypeebrothers.com
Website: www.jaypeebrothers.com
Website: www.jaypeedigital.com
© 2016, Jaypee Brothers Medical Publishers
The views and opinions expressed in this book are solely those of the original contributor(s)/author(s) and do not necessarily
represent those of editor(s) of the book.
All rights reserved. No part of this publication may be reproduced, stored or transmitted in any form or by any means, electronic,
mechanical, photocopying, recording or otherwise, without the prior permission in writing of the publishers.
All brand names and product names used in this book are trade names, service marks, trademarks or registered trademarks of
their respective owners. The publisher is not associated with any product or vendor mentioned in this book.
Medical knowledge and practice change constantly. This book is designed to provide accurate, authoritative information about
the subject matter in question. However, readers are advised to check the most current information available on procedures
included and check information from the manufacturer of each product to be administered, to verify the recommended dose,
formula, method and duration of administration, adverse effects and contraindications. It is the responsibility of the practitioner
to take all appropriate safety precautions. Neither the publisher nor the author(s)/editor(s) assume any liability for any injury and/
or damage to persons or property arising from or related to use of material in this book.
This book is sold on the understanding that the publisher is not engaged in providing professional medical services. If such advice
or services are required, the services of a competent medical professional should be sought.
Every effort has been made where necessary to contact holders of copyright to obtain permission to reproduce copyright
material. If any have been inadvertently overlooked, the publisher will be pleased to make the necessary arrangements at the
first opportunity.
Inquiries for bulk sales may be solicited at: jaypee@jaypeebrothers.com
USMLE Step 2 CK Platinum Notes
First Edition: 2013
Second Edition: 2016
ISBN: 978-93-5250-172-4
Printed at
Contributors
Ashfaq UI Hassan
MBBS MS
Consultant–Anatomy
SKIMS Medical College
Bemina, Srinagar, Jammu and Kashmir, India
Ashfaq Ul Hassan
ashhassan@rediffmail.com
Preface to the First Edition
It gives me a great pleasure to introduce Platinum Notes for USMLE Step 1 and Step 2.
A need was felt by most of the students for a comprehensive book for USMLE for preparing extensively and
simultaneously getting a very good score.
My efforts are directed to benefit the students to the maximum.
For a student preparing, revising, forgetting and consulting different books is a part of the game. In this journey some
students tend to get nervous and anxious and I thought of putting all my best efforts into one book which would make the
preparation simpler, interesting, more lucid and palatable.
High yield points, important topics, clinical correlations, in-depth focus on subjects, easy retention and quick revision
are the points on which I have focused taking into account the latest trends of the examinations.
The matter in the book is highly concentrated, needs multiple revisions and deep concentration for retention.
The book has been framed to simulate the study pattern of USMLE examinations to its best.
The questions in the text and after every subject are almost similar to the questions put in the actual USMLE examination
which gives a student a good chance to understand the standard of USMLE as well as to prepare for the examinations in a
better way.
I feel that the students would be benefitted through my effort and a feedback from the students would be highly
appreciated at ashhassan@rediffmail.com
I wish you good luck for your academic pursuits.
Ashfaq Ul Hassan
ashhassan@rediffmail.com
Acknowledgments
No words would be enough to express my regards to Prof. Showkat A Zargar, Director, Sher-i-Kashmir Institute of Medical
Sciences (SKIMS), Srinagar, Jammu and Kashmir, India for his constant encouragement.
I am also thankful to my father Prof. Ghulam Hassan for his constant encouragement in all my academic pursuits and
guiding me throughout the process of the completion of this book.
I convey my sincere thanks to Jaypee Brothers Medical Publishers (P) Ltd, New Delhi, India for their efforts and
suggestions, especially Shri Jitendar P Vij (Group Chairman), Ms Chetna Vohra (Associate Director), and Ms Payal Bharti
(Project Manager) for helping me through my idea.
Abbreviations
1° Primary bid Twice a day
2° Secondary BMR Basal metabolic rate
# Fracture BP Blood pressure
ACE Angiotensin converting enzyme BPH Benign prostatic hypertrophy
ACE-I Angiotensin converting enzyme inhibitor BUN Blood urea nitrogen
ACTH Adrenocorticotropic hormone B/L Bilateral
ACh Acetylcholine BM Bone marrow, basement membrane
Adr Adrenaline b/n or b/w Between
AD Autosomal dominant C/S Culture and sensitivity
ADH Anti-diuretic hormone Ca Calcium
2+
2. Pediatrics 217
3. Psychiatry 327
4. Dermatology 369
5. Surgery 421
6. Orthopedics 609
7. Ophthalmology 625
Copper
Copper is an integral part of numerous enzyme systems including:
•• Amine oxidases
•• Ferroxidase (ceruloplasmin)
•• Cytochrome-c oxidase
•• Superoxide dismutase and
•• Dopamine hydroxylase
•• Dietary copper deficiency is relatively rare, although it has been described in premature infants fed milk diets and in infants with
malabsorption
4 USMLE Step 2 CK Platinum Notes
Selenium
•• Selenium, in the form of selenocysteine, is a component of the enzyme glutathione peroxidase, which serves to protect proteins,
cell membranes, lipids, and nucleic acids from oxidant molecules. Selenocysteine is also found in the deiodinase enzymes, which
mediate the deiodination of thyroxine to the more active triiodothyronine
•• Keshan disease is an endemic cardiomyopathy found in children and young women residing in regions of China where dietary
intake of selenium is low. Low blood levels of selenium in various populations have been correlated with an increase in coronary
artery disease and certain cancers, although the data are not consistent.
Chromium
•• Chromium potentiates the action of insulin in patients with impaired glucose tolerance, presumably by increasing insulin
receptor-mediated signaling
•• Chromium deficiency has been reported to cause glucose intolerance, peripheral neuropathy and confusion.
Magnesium
Magnesium deficiency is a common clinical problem:
•• Reduced renal reabsorption due to loop diuretics and alcohol use is a common cause of hypomagnesemia
•• Vomiting and nasogastric suctioning, fluid loss from diarrhea
•• Hypomagnesemia is prevalent in alcoholics
The clinical manifestations of hypomagnesemia are similar to those of severe hypocalcemia
The signs and symptoms of hypomagnesemia include:
•• Muscle weakness
•• Prolonged PR and QT intervals
•• Cardiac arrhythmias
•• Positive Chvostek’s and Trousseau’s signs
•• Carpopedal spasm can also occur with hypomagnesemia
•• Remember: Magnesium is important for effective PTH secretion as well as the renal and skeletal responsiveness to PTH; thus,
hypomagnesemia is often associated with hypocalcemia due to impaired PTH secretion and function.
Ultratrace Elements
Are those for which the need is <1 mg/d
•• Molybdenum is necessary for the activity of sulfite and xanthine oxidase, and molybdenum deficiency may result in skeletal
and brain lesions
•• Arsenic (impaired growth, infertility)
•• Boron (impaired energy metabolism, impaired brain function)
•• Nickel (impaired growth and reproduction), silicon (impaired growth) and
•• Vanadium (impaired skeletal formation) might also be essential.
Medicine 5
Cadmium
•• Environmental exposure to cadmium can result from the ingestion of basic foodstuffs, especially grains, cereals, and leafy
vegetables, which readily absorb cadmium occurring naturally or in soil contaminated by sewage sludge, fertilizers, and
polluted groundwater
•• Cigarette smoke contains cadmium
•• Occupational exposure takes place in the metal-plating, pigment, battery, and plastics industries
•• Most absorbed cadmium is concentrated in the liver and kidneys. In erythrocytes and soft tissues, cadmium is bound to
metallothionein. The toxicity of cadmium may involve its binding to key cellular sulfhydryl groups, its competition with other
metals (zinc and selenium) for inclusion in metalloenzymes, and its competition with calcium for binding sites on regulatory
proteins such as calmodulin
Acute high-dose cadmium inhalation can cause severe respiratory irritation with pleuritic chest pain, dyspnea, cyanosis, fever,
tachycardia, nausea, and life-threatening noncardiogenic pulmonary edema
Acute exposure through ingestion can cause severe nausea, vomiting, salivation, abdominal cramps, and diarrhea
Chronic effects of cadmium exposure are dose-dependent and include anosmia, yellowing of the teeth, emphysema, minor
changes in liver function, microcytic hypochromic anemia unresponsive to iron therapy, renal tubular dysfunction character-
ized by proteinuria and increased urinary excretion of b2-microglobulin and (with prolonged poisoning) osteomalacia leading
to bone lesions and pseudofractures. In follow-up studies of occupationally exposed workers, b2-microglobulinuria was found
to be irreversible
Chelation therapy is not useful, and dimercaprol is contraindicated as this agent may exacerbate nephrotoxicity. Avoidance of further
exposure and supportive therapy (including vitamin D if osteomalacia exists) are the mainstays of management.
Deficiency of Minerals
•• Iron deficiency •• Microcytic anemia
•• Iodine •• Goiter
•• Zinc •• Acrodermatitis enteropathica
•• Copper •• Menkes disease
•• Selenium •• Cardiomyopathy
•• Chromium •• Impaired glucose tolerance
Arsenic-related Diseases
Chronic arsenic (As) exposure has been linked to many cancers. Most common cancers associated are:
•• Basal cell carcinoma
•• Squamous cell carcinoma
•• Angiosarcoma of liver
•• Lung Ca
•• Kidney Ca
•• Colon Ca
•• Noncirrhotic portal fibrosis is a medical condition associated with As exposure
ACID-BASE/ELECTROLYTE DISORDERS
Medically Important Electrolyte Imbalances
Potassium Deficit and Hypokalemia Potassium Excess and Hyperkalemia
1. K depletion usually is due to excessive losses of K in the Hyperkalemia (serum K > 5.5 mEq/L)
urine or stool Must be distinguished from pseudohyperkalemia due to
2. Bartter’s syndrome is characterized by sodium wasting, hemolysis of the blood sample, or the release of K from
excessive production of renin and aldosterone, and erythrocytes, leukocytes, and platelets during the clotting
normotension
Contd...
6 USMLE Step 2 CK Platinum Notes
Contd...
Potassium Deficit and Hypokalemia Potassium Excess and Hyperkalemia
3. Cushing’s syndrome due to adrenal cancer or the ectopic process in patients with marked leukocytosis (> 50,000/µL) or
ACTH syndrome and hyperaldosteronism thrombocytosis (> 500,000/µL)
4. In association with diuretics (e.g. thiazides, bumetanide, Hyperkalemia may occur when:
furosemide, ethacrynic acid, but not spironolactone or 1. Acidosis (due to accumulation of inorganic acids)
triamterene) 2. Hyperglycemia (in the presence of insulin deficiency)
5. Hypomagnesemia in osmotic diuresis (e.g. diabetic 3. Moderately heavy exercise (particularly in the presence of
ketoacidosis); in renal tubular disease, such as type I or ß-blockade)
type II renal tubular acidosis, Fanconi’s syndrome
4. Digitalis intoxication, Beta blockers
6. Excessive licorice ingestion and Liddle’s syndrome
5. Acute tumor lysis
7. Amphotericin B and with antipseudomonal penicillins
6. Acute intravascular hemolysis or
(e.g. carbenicillin) or high-dose penicillin treatment
7. Hyperkalemic familial periodic paralysis produces a shift of
8. Gastrointestinal losses usually are due to diarrhea,
K out of cells into the ECF; or as a consequence of K excess
chronic laxative abuse or clay ingestion, vomiting
or gastric suction (renal K wasting with developing 8. Since the kidneys normally excrete K loads rapidly, sus-
metabolic alkalosis is of primary importance) and bowel tained hyperkalemia usually implies diminished renal
diversion K excretion
9. Villous adenoma of the colon is a rare cause of K loss 9. K excess is particularly common in acute oliguric states
from the GI tract (especially acute renal failure) associated with severe crush
injuries, burns, bleeding into soft tissue or the GI tract, or
10. Beta 2-Adrenergic agonists such as albuterol or
adrenal insufficiency
terbutaline may produce hypokalemia due to cellular K
uptake TREATMENT:
11. Cystic fibrosis Acute emergency: Calcium gluconate, sodium bicarbonate, insulin,
albuterol
12. Theophylline intoxication
Nonemergency: Furosemide, resins, hemodialysis, peritoneal dialysis
13. Cortisol
Hypokalemia
•• Severe hypokalemia (serum K < 3 mEq/L) may produce:
•• Muscular weakness and lead to paralysis and respiratory failure. Muscular malfunction may result in respiratory
hypoventilation, paralytic ileus
•• The characteristic ECG changes are
–– ST segment depression
–– Increased U wave amplitude and
–– T wave amplitude <U wave amplitude
–– Severe hypokalemia may produce premature ventricular and atrial contractions and ventricular and atrial
tachyarrhythmias.
Hyperkalemia
The ECG Changes are:
•• Shortening of the QT interval and
•• Tall, peaked T waves (serum K > 5.5 mEq/L)
•• Progressive hyperkalemia produces nodal and ventricular arrhythmias, widening of the QRS complex (serum K > 6.5 mEq/L)
•• PR interval prolongation and disappearance of the P wave and finally
•• Degeneration of the QRS complex to a sine wave pattern and ventricular asystole or fibrillation.
Medicine 7
Hypomagnesemia
•• Serum magnesium concentration < 1.6 mEq/L (< 1.9 mg/dL)
•• Magnesium depletion usually results from:
–– Inadequate intake/alcoholism
–– Impairment of renal or gut absorption
–– Prolonged parenteral feeding, usually in combination with loss of body fluids via gastric suction or diarrhea
–– Lactation (increased requirement for Mg)
–– Conditions of abnormal renal conservation of Mg, such as hypersecretion of aldosterone, ADH, or thyroid hormone,
hypercalcemia, diabetic acidosis, and cisplatin or diuretic therapy
–– It is often associated and presents with hypocalcemia and hypophosphatemia
•• Anorexia, nausea, vomiting, lethargy, weakness, personality change, Tetany (e.g. positive Trousseau’s or Chvostek’s sign, or
spontaneous carpopedal spasm), tremor, and muscle fasciculations may be present. The neurologic signs, particularly
tetany, correlate with the development of hypocalcemia and hypokalemia
•• Myopathic potentials are found on electromyography.
Hypermagnesemia
Serum magnesium > 2.1 mEq/L (> 2.5 mg/dL)
•• Symptomatic hypermagnesemia is common in Renal failure when patients are taking magnesium-containing antacids, laxatives,
enemas, or infusions
•• Acute rhabdomyolysis. Volume depletion
•• Lithium
•• Ketoacidosis without treatment
•• Renal failure
Hypermagnesemia leads to generalized impairment of neuromuscular transmission, probably as the result of inhibition of
acetylcholine release at the neuromuscular junction. ECG shows:
•• Prolongation of the PR interval,
•• Widening of the QRS complex, and increased T wave amplitude
Deep tendon reflexes disappear; hypotension, respiratory depression and narcosis develop with progression of the
hypermagnesemia, cardiac arrest occurs
Treatment of Severe Mg Intoxication:
•• Consists of circulatory and respiratory support, with IV administration of 10 to 20 mL of 10% calcium gluconate. The latter may
reverse many of the Mg-induced changes, including the respiratory depression
•• Administration of IV furosemide or ethacrynic acid increases Mg excretion, if continuous and adequate hydration is maintained
and renal function is adequate
•• Hemodialysis may be of value in severe hypermagnesemia
•• When hemodialysis is impractical, peritoneal dialysis may be effective.
8 USMLE Step 2 CK Platinum Notes
Hypocalcemia
•• Hypoparathyroidism: Characterized chemically by low serum calcium and high serum phosphorus levels usually follows
accidental removal of or damage to several parathyroid glands during thyroidectomy.
–– Idiopathic hypoparathyroidism, in which the parathyroids are absent or atrophied, is uncommon. It may occur
sporadically as an isolated or inherited condition, or in association with the DiGeorge syndrome
–– It also occurs as part of a Genetic syndrome of hypoparathyroidism, Addison’s disease and mucocutaneous candidiasis.
–– Pseudohypoparathyroidism (PHP) differs from the postoperative and idiopathic forms and is characterized not by deficiency
of PTH but by target organ (bone and kidney) unresponsiveness to its action.
•• Vitamin D deficiency, due to inadequate dietary intake, decreased exposure to sunlight, hepatobiliary disease, or intestinal
malabsorption and associated with rickets or osteomalacia. Functional vitamin D deficiency may occur during prolonged
anticonvulsant therapy with barbiturates and phenytoin, presumably the result of increased catabolism of 25-(OH) D3.
•• Vitamin D-dependent rickets (VDDR), in which the formation of 1,25-DHCC is defective (Type I), or there is marked resistance
of target organs to the effects of 1,25-DHCC and other vitamin D metabolites (Ttype II) and familial hypophosphatemic
(Vitamin D-resistant) rickets.
•• Renal tubular disease, including Fanconi’s syndrome due to nephrotoxins (e.g. heavy metals) and distal renal tubular acidosis.
•• Renal failure, where diminished formation of 1, 25-DHCC coupled with hyperphosphatemia produces hypocalcemia.
•• Magnesium depletion occurring with intestinal malabsorption or dietary deficiency, which causes hypocalcemia by relatively
deficient secretion of PTH, as well as end-organ resistance to its action.
•• Acute pancreatitis, which lowers serum Ca levels when Ca is chelated by lipolytic products.
•• Hypoproteinemia of any cause with reduction in protein-bound Ca (e.g. nephrotic syndrome, cirrhosis, protein-losing
enteropathy). Hypocalcemia due to diminished protein-binding is asymptomatic, since the ionized Ca fraction is unaltered and
the effects of Ca upon membrane excitability are produced by the ionized fraction.
•• During periods of increased Ca utilization coupled with inadequate intake (e.g. after surgical correction of hyperparathyroidism
with healing bone lesions), hypocalcemia can also develop. It may develop in oxalic acid poisoning and during sodium edetate
therapy.
•• Septic shock may be associated with hypocalcemia due to suppression of PTH release and 1, 25-DHCC formation.
Causes of Hypercalcemia
Total serum calcium>10.5 mg/dL.
•• Primary hyperparathyroidism usually characterized by hypercalcemia, hypophosphatemia, and excessive bone resorption.
•• Sarcoidosis is associated with hypercalcemia in up to. Hypercalcemia and/or hypercalciuria have also been described in other
granulomatous diseases (e.g. tuberculosis, leprosy, berylliosis, histoplasmosis, and coccidioidomycosis). Similarly, elevated
serum levels of 1, 25-DHCC have been reported in hypercalcemic patients with tuberculosis, silicosis, and Hodgkin’s and non-
Hodgkin’s lymphoma.
•• Milk-alkali syndrome, excessive amounts of Ca and absorbable alkali are ingested, usually during peptic ulcer therapy, resulting
in increased Ca absorption and hypercalcemia.
•• Idiopathic hypercalcemia of infancy is recognized by the combination of suppressed levels of PTH, hypercalciuria, and in
some severely affected patients, the somatic abnormalities of ‘Williams’ syndrome’ (e.g. supravalvular aortic stenosis, mental
retardation, and an elfin facies).
•• Myeloma should be suggested by the syndrome of anemia, azotemia, and hypercalcemia. This diagnosis is confirmed by bone
marrow examination or finding a monoclonal gammopathy or free light chains in the serum or urine on immunoelectrophoresis
•• Lithium causes hypercalcemia.
A 40-year-old male from California who presented with polyuria, pain abdomen, nausea, vomiting, and altered sensorium was found
to have Bronchogenic carcinoma. The electrolyte abnormality seen in him would be Hypercalcemia
3. Respiratory acidosis
4. Respiratory alkalosis
Ans. 1. Metabolic Acidosis
The laboratory results indicate that the arterial pH, arterial PCO2, and plasma HCO3– concentrations are all low. These changes
clearly demonstrate metabolic acidosis, which occurs commonly when a carbonic anhydrase inhibitor is administered.
Remember:
•• Respiratory Acidosis: Any cause of hypoventilation causes Respiratory Acidosis (COAD, CNS Depression, Neuromuscular disease)
•• Respiratory Alkalosis: Hyperventilation, Hypoxia, Hyperthyroidism, High Altitude (H4)
•• Metabolic Alkalosis: Conditions causing Hypokalemia usually
•• Metabolic Acidosis: Lactic acidosis, Ketoacidosis, Liver failure, Cardiac failure, Ethylene glycol, Renal tubular acidosis, Diarrhea.
Hypoalbumenemia
Lithium toxicity
lgG myeloma
Hypercalcemia
Hypermagnesemia
Hyperkalemia
Lactic Acidosis
An increase in plasma L-lactate may be secondary to:
Poor tissue perfusion (Type A):
•• Circulatory insufficiency
•• Shock, circulatory failure
•• Severe anemia, mitochondrial enzyme defects
•• Inhibitors (carbon monoxide, cyanide)
Aerobic disorders (Type B): Malignancies, diabetes mellitus, renal or hepatic failure, severe infections (cholera, malaria), seizures,
AIDS, or drugs/toxins:
•• Biguanides
•• Ethanol
•• Methanol
•• Isoniazid
•• AZT analogues
•• Fructose
HYPOTHERMIA
Degrees of Hypothermia
•• Mild Hypothermia 95–90 °F
•• Moderate Hypothermia 90–82.4 °F
•• Severe Hypothermia less than 82.4 °F
Accidental hypothermia occurs when there is an unintentional drop in the body’s core temperature below 35 °C (95 °F)
Causes
•• Primary accidental hypothermia is geographically and seasonally pervasive. Although most cases occur in the winter months
and in colder climates, it is surprisingly common in warmer regions as well.
•• Multiple variables make individuals at the extremes of age, the elderly and neonates, particularly vulnerable to hypothermia.
Dementia, psychiatric illness, and socioeconomic factors often compound these problems by impeding adequate measures to
prevent hypothermia.
•• Individuals whose occupations or hobbies entail extensive exposure to cold weather are clearly at increased risk for hypothermia.
Military history is replete with hypothermic tragedies. Hunters, sailors, skiers, and climbers are also at great risk of exposure,
whether it involves injury, changes in weather, or lack of preparedness.
•• Ethanol, Amphetamine, chlorpromazine
•• Wernicke’s encephalopathy, phenothiazines, barbiturates, benzodiazepines, cyclic antidepressants, and many other
medications reduce centrally mediated vasoconstriction.
•• Several types of endocrine dysfunction can lead to hypothermia. Hypothyroidism, Adrenal insufficiency and hypopituitarism
can also increase susceptibility to hypothermia. Hypoglycemia, most commonly caused by insulin or oral hypoglycemic drugs, is
associated with hypothermia, in part the result of neuroglycopenic effects on hypothalamic function.
•• Neurologic injury from trauma, cerebrovascular accident, subarachnoid hemorrhage, or hypothalamic lesions increases
susceptibility to hypothermia. Agenesis of the corpus callosum, or Shapiro syndrome is one cause of ‘episodic hypothermia’,
characterized by profuse perspiration followed by a rapid fall in temperature. Acute spinal cord injury disrupts the autonomic
pathways that lead to shivering and prevents cold-induced reflex vasoconstrictive responses.
•• Hypothermia is confirmed by measuring the core temperature, preferably at two sites. Rectal probes should be placed to a
depth of 15 cm and not adjacent to cold feces.
Osborn waves (also known as camel-hump sign, late delta wave or current of injury) are usually observed on the electrocardiogram
of people suffering from hypothermia, though they may also occur in people with high blood levels of calcium (hypercalcemia), brain
injury, vasospastic angina, or ventricular fibrillation. Osborn waves are positive deflections occurring at the junction between
the QRS complex and the ST segment, where the S point, also known as the J joint, has a myocardial infarction-like elevation
QT INTERVAL PROLONGED
•• Neuroleptic malignant syndrome (NMS) is a complex of extrapyramidal muscular rigidity, high-core temperature, altered level
of consciousness, and elevated creatine kinase levels occurring as an acute or subacute reaction to therapy with neuroleptic
medications
•• Malignant hyperthermia is a hypermetabolic, myopathic syndrome, chemically or stress-induced, and is manifested by an abrupt
rise in core temperature, vigorous muscle contractions, metabolic and respiratory acidosis, and ventricular arrhythmias, usually
when inducing anesthesia.
RESPIRATORY MEDICINE
Importance of Different Presentations
•• Wheeze (Rhonci): Monophonic Obstruction in Airway
•• Wheeze: Polyphonic Asthma
•• Crepitations (Crackles): Fine and high-pitched: Pulmonary edema
•• Persistent coarse crepitations: Bronchiectasis
•• Pleural rub: Pleuritis, pulmonary infarction
•• Silent chest: Severe bronchospasm
Sputum
•• Copious, pink, frothy: Pulmonary edema
•• Copious, pungent, purulent: Bronchiectasis, lung abscess
•• Black: Coal dust
Respiratory Failure
Type I respiratory failure: PaO2: ↓, PaCO2: n or ↓, PA–aO2↑Hypoxemia with decreased PaCO2
Caused by:
•• Parenchymal diseases
•• Pneumoniae
•• ARDS
•• Emphysema
•• R-L shunts
Type II respiratory failure: Pa O2: ↓, PaCO2: ↑, PA–a O2 n. Hypoxemia with increased PaCO2
Caused by:
•• COAD
•• Interstitial lung diseases
•• Musculoskeletal problems: Polymyositis, kyphoscoliosis
Asthma
•• Category of COAD/COPD
•• Hyperesponsiveness of airways with bronchoconstriction
•• Incidence is increasing day by day
•• Hypoxia is a common feature
•• Leukotrienes ↑
•• Hyperesponsiveness of airways
•• Mast cell stabilizer used: Ketotifen
•• Hypersensitive lung
•• Constriction of small airways
–– Creola bodies seen
–– Charcot-Leyden crystals seen
–– Curschmann’s spirals seen
•• FEV1 is the parameter to improve maximum on bronchodilator therapy
•• Mast cell stabilizers are used in treatment (chronic)
•• Omalizumab is used now for treatment
Medicine 15
Speech difficulty
Diaphoresis
Altered sensation Bad signs of asthma
Cyanosis
Silent chest
Severe asthma
Bronchiectasis
Bronchiectasis describes a permanent dilatation of the airways secondary to chronic infection or inflammation.
Bronchiectasis
•• Cylindrical bronchiectasis—the bronchi appear as uniformly dilated tubes that end abruptly at the point that smaller airways are
obstructed by secretions
•• Varicose bronchiectasis—the affected bronchi have an irregular or beaded pattern of dilatation resembling varicose veins
•• Saccular (cystic) bronchiectasis—the bronchi have a ballooned appearance at the periphery, ending in blind sacs without
recognizable bronchial structures distal to the sacs
–– Nodular bronchiectasis is seen in infection with mycobacterium avium
–– Feature of Kartagener’s syndrome
–– Tram-track lines on CXR
–– Commonest in left lower lobe
–– Clubbing is a feature
–– Does not predispose to lung cancer
–– HRCT is the diagnostic technique of choice
16 USMLE Step 2 CK Platinum Notes
Causes
•• Postinfective: Tuberculosis, measles, pertussis, pneumonia
•• Cystic fibrosis
•• Bronchial obstruction, e.g. lung cancer/foreign body
•• Immune deficiency: Selective IgA, hypogammaglobulinemia
•• Allergic bronchopulmonary aspergillosis (ABPA)
•• Ciliary dyskinetic syndromes: Kartagener’s syndrome, Young’s syndrome
•• Yellow nail syndrome
–– Dyspnea is a common and prominent complaint in patients with ILD, especially the idiopathic interstitial pneumonias,
hypersensitivity pneumonitis, COP, sarcoidosis, eosinophilic pneumonias, and PLCH
–– HRCT is superior to the plain chest X-ray for early detection and confirmation of suspected ILD
–– Drug Therapy: Glucocorticoids are the mainstay of therapy for suppression of the alveolitis present in ILD
–– Many cases of ILD are chronic and irreversible despite the therapy discussed above and
–– Lung transplantation may be considered.
Pleural Effusion
Transudate Exudate
•• Protein < 3 gm/100 ml •• Protein > 3 gm/100 ml
•• LDH < 200 IU •• LDH > 200 IU
•• LDH Ratio < 0.6 •• LDH Ratio > 0.6
•• Pleural fluid/serum protein < 0.5 •• Pleural fluid/serum protein > 0.5
–– CHF –– Pneumoniae
–– Cirrhosis –– Pulmonary infarction
–– Nephrotic syndrome –– Tuberculosis
–– Meig’s syndrome –– Rheumatoid arthritis
–– Pulmonary embolism –– Myxedema
–– Lupus erythematosus
–– Pulmonary embolism
–– Bronchogenic cancer
QUESTIONS ASKED
Pleural Fluid Characteristics
•• Bloodstained: Pulmonary infarction, metastatic carcinoma, tuberculosis
•• Low Glucose: Rheumatoid arthritis, empyema
•• PH < 7.2: Empyema
•• PH < 6.0: Esophageal rupture
•• ↑Amylase: Esophageal perforation, pancreatitis
•• Chylous effusion: Lymphomas, chest trauma
•• Cholesterol: Hypothyroidism, TB, rheumatoid arthritis
•• Left sided effusion: Pancreatitis, esophageal rupture
•• Tuberculous pleuritis: The pleural fluid is an exudate with predominantly small lymphocytes. The diagnosis is established by
demonstrating high levels of TB markers in the pleural fluid (adenosine deaminase > 45 IU/L, gamma interferon > 140 pg/mL,
or positive PCR for tuberculous DNA).
18 USMLE Step 2 CK Platinum Notes
•• Effusion secondary to malignancy treat by: (1) tube thoracostomy with the instillation of a sclerosing agent such as talc;
(2) outpatient insertion of a small indwelling catheter; or (3) thoracoscopy with pleural abrasion or the insufflation of talc
•• Parapneumonic effusion parapneumonic effusions are associated with bacterial pneumonia, lung abscess, or bronchiectasis and
are probably the most common exudative pleural effusion
–– Loculated pleural fluid
–– Pleural fluid pH below 7.20
–– Pleural fluid glucose less than 60 mg/dL
–– Positive Gramstain or culture of the pleural fluid
–– The presence of gross pus in the pleural space.
Pneumonias
‘Typical’ pneumonia syndrome is characterized by the sudden onset of fever, cough productive of purulent sputum, shortness
of breath, and (in some cases) pleuritic chest pain; signs of pulmonary consolidation
•• Dullness
•• Increased fremitus
•• Egophony
•• Bronchial breath sounds, and rales) may be found on physical examination in areas of radiographic abnormality
The typical pneumonia syndrome is usually caused by the most common bacterial pathogen in community-acquired pneumonia,
S pneumoniae, but can also be due to other bacterial pathogens, such as H influenzae and mixed anaerobic and aerobic components
of the oral flora.
‘Atypical’ pneumonia syndrome is characterized by a more gradual onset, a dry cough, shortness of breath, a prominence
of extrapulmonary symptoms (such as headache, myalgias, fatigue, sore throat, nausea, vomiting, and diarrhea), and
abnormalities on chest radiographs despite minimal signs of pulmonary involvement (other than rales) on physical examination
Atypical pneumonia is classically produced by:
•• M pneumoniae
•• L pneumophila
•• C pneumoniae
•• Oral anaerobes
•• P carinii
Less frequently encountered pathogens. C psittaci, Coxiella burnetii, Francisella tularensis, H capsulatum, and Coccidioides
immitis.
Remember
•• Mycoplasma pneumonia: May be complicated by erythema multiforme, hemolytic anemia, bullous myringitis, encephalitis,
and transverse myelitis.
•• Staph aureus: Postinfluenza and pneumatocele formation
•• Klebsiella pneumonia: It is characterized by red current jelly sputum
•• Legionella pneumonia: It is frequently associated with diarrhea, deterioration in mental status (delirium), renal and hepatic
abnormalities, and marked hyponatremia; (walking pneumonia), failure to respond to beta lactam antibiotics
•• Self-limiting form of legionella infection is called pontaic fever (Flu-like illness without pneumonia)
•• C pneumonia: Pneumonia, sore throat, hoarseness, and wheezing are relatively common.
•• The atypical pneumonia syndrome in patients whose HIV infection suggests Pneumocystis infection.
Mycoplasma Pneumonia
•• Cause of atypical pneumonia which often affects younger patients
•• It is associated with a number of complications such as Erythema multiforme and Cold autoimmune hemolytic anemia
Features
•• Flu-like symptoms classically precede a dry cough
•• Bilateral consolidation on X-ray
•• Serology useful in diagnosis
Complications:
•• Cold agglutins (IgM) may cause hemolytic anemia, thrombocytopenia
•• Erythema multiforme, erythema nodosum
•• Meningoencephalitis, Guillain-Barre syndrome
•• Bullous myringitis: painful vesicles on the tympanic membrane
•• Pericarditis/myocarditis
20 USMLE Step 2 CK Platinum Notes
Nosocomial Pneumonia
The usual criteria for nosocomial pneumonia, which include new or progressive pulmonary infiltrates, purulent tracheobronchial
secretions, fever, and leukocytosis, are frequently unreliable in those patients who often have preexisting pulmonary disease,
endotracheal tubes that irritate the tracheal mucosa and may elicit an inflammatory exudate in respiratory secretions, or multiple
other problems likely to produce fever and leukocytosis.
Aspiration of Solids
•• Usually food. Obstruction of major airways typically results in atelectasis and moderate nonspecific inflammation. Therapy consists
of removal of the foreign body.
Mendelson’s Syndrome
•• Results from regurgitation of stomach contents and aspiration of chemical material, usually gastric juices. Pulmonary
inflammation including the destruction of the alveolar lining, with transudation of fluid into the alveolar space, occurs
with remarkable rapidity
•• Typically this syndrome develops within hours, often following anesthesia when the gag reflex is depressed. The patient
becomes tachypneic, hypoxic, and febrile. The leukocyte count may rise, and the chest X-ray may evolve suddenly from normal
to a complete bilateral ‘whiteout’ within 8 to 24 hours. Sputum production is minimal. The pulmonary signs and symptoms can
resolve quickly with symptom-based therapy or can culminate in respiratory failure, with the subsequent development of bacterial
superinfection over a period of days.
Necrotizing Pneumonitis
This form of anaerobic pneumonitis is characterized by numerous small abscesses that spread to involve several pulmonary segments.
The process can be indolent or fulminating. This syndrome is less common than either aspiration pneumonia or lung abscess and
includes features of both types of infection.
FREQUENTLY ASKED
Differentiate
Tropical eosinophilia
It is usually caused by filarial infection; however, eosinophilic pneumonias also occur with other parasites such as Ascaris,
Ancyclostoma sp, Toxocara sp, and Strongyloides stercoralis, Wuchereria bancrofti or W malayi.
Eosinophilia-myalgia syndrome
Caused by dietary supplements of L-tryptophan, is occasionally associated with pulmonary infiltrates.
Loeffler’s syndrome
It is a benign, acute eosinophilic pneumonia of unknown cause characterized by migrating pulmonary infiltrates and minimal clinical
manifestations
Acute eosinophilic pneumonia has been described recently as an idiopathic acute febrile illness lasting less than 7 days with severe
hypoxemia, pulmonary infiltrates, and no history of asthma
Chronic eosinophilic pneumonia presents with significant systemic symptoms including fever, chills, night sweats, cough, anorexia, and
weight loss lasting for several weeks to months. The chest X-ray classically shows peripheral infiltrates resembling a photographic negative
of pulmonary edema. Some patients also have bronchial asthma of the intrinsic or nonallergic type. Dramatic clearing of symptoms and chest
X-rays is often noted within 48 h after initiation of glucocorticoid therapy.
•• Noncontagious
•• Upper lobe dominance
•• Effusion is rare
Nocardiosis
The clinical manifestations are nonspecific and include fever, cough, weight loss, and dyspnea. The range of pulmonary involvement
extends from transient or inapparent infection to confluent bronchopneumonia with complete consolidation. Radiographic examination
of the chest may reveal one or more of the following: fluffy infiltrates, multiple abscess formation with cavitation, bulging fissures, masses,
nodules, and empyema.
Caused by:
•• Filamentous, aerobic Gram-positive Bacteria:
•• Partially acid causes disseminated infections in immunocompromised
•• The risk of pulmonary or disseminated disease is greater than usual among people with deficient cell-mediated immunity especially
that associated with lymphoma, transplantation, or AIDS
•• In persons with AIDS, Nocardiosis usually presents at a CD4+ lymphocyte concentration of <250/ul
•• Prophylaxis with sulfamethoxazole and trimethoprim appears to reduce the risk of nocardiosis in persons with AIDS or transplanted
organs
•• Nocardiosis has also been associated with pulmonary alveolar proteinosis, tuberculosis and other mycobacterial diseases, and
chronic granulomatous disease.
Pneumonia is by far the most common respiratory tract nocardial disease. Nocardial pneumonia is typically subacute; symptoms
have usually been present for days or weeks at presentation. The onset may be more acute in immunosuppressed patients. Cough is
prominent and produces small amounts of thick, purulent sputum that is not malodorous. Fever, anorexia, weight loss, and malaise
are common; dyspnea, pleuritic pain, and hemoptysis are less common. Remissions and exacerbations over several weeks are frequent
The most common site of dissemination is the brain.
Medicine 23
Pulmonary Tuberculosis
•• Ghon Complex: Primary subpleural granuloma in the inferior upper lobe/superior lower lobe region (Ghon Focus) along with
draining Hilar nodes
•• Puhl’s lesion: Isolated lesion of chronic pulmonary TB in apex of lung
•• Assmann’s Focus: Infraclavicular lesion of chronic pulmonary TB
•• Rancke Complex: Combination of calcified peripheral lesion (Ghon Complex) and calcified Hilar nodes seen in primary TB
Obligate aerobe
Acid fast
Acid fastness is due to mycolic acid and cell wall
Lung is the mc organ involved
Koch’s phenomenon is seen in Tuberculosis
Postprimary Disease
•• Also called adult-type, reactivation, or secondary tuberculosis, postprimary disease results from endogenous reactivation
of latent infection and is usually localized to the apical and posterior segments of the upper lobes, where the high oxygen
concentration favors mycobacterial growth
•• In addition, the superior segments of the lower lobes are frequently involved
•• The extent of lung parenchymal involvement varies greatly, from small infiltrates to extensive cavitary disease
•• Hemoptysis, however, may also result from rupture of a dilated vessel in a cavity (Rasmussen’s aneurysm) or from aspergilloma
formation in an old cavity
–– Popcorn calcification can occur
–– Patients of atopic asthma are prone to develop TB
–– ‘Hemorrhagic’ pleural effusions can ocuur in TB
–– ‘Cavitatory’ lesions in lung can occur
Pleural Tuberculosis
•• Involvement of the pleura is common in primary tuberculosis and results from penetration by a few tubercle bacilli into the
pleural space
–– The fluid is straw-colored and, at times, hemorrhagic
–– It is an exudate with a protein concentration > 50% of that in serum, a normal-to-low glucose concentration
–– A pH that is generally < 7.2
–– And detectable white blood cells (usually 500 to 2500/ml). Neutrophils may predominate in the early stage, while
mononuclear cells are the typical finding later
–– Mesothelial cells are generally rare or absent. AFB are very rarely seen on direct smear, but cultures may be positive for
M tuberculosis in up to one-third of cases
•• Necrotic Lymph nodes with peripheral rim enhancement is due to TB
•• PCR and BACTEC are now used for diagnosis.
24 USMLE Step 2 CK Platinum Notes
Tuberculous Empyema
It is a less common complication of pulmonary tuberculosis
It is usually the result of the rupture of a cavity, with delivery of a large number of organisms into the pleural space, or of a bronchopleural
fistula from a pulmonary lesion
Tuberculous empyema may result in severe pleural fibrosis and restrictive lung disease.
•• A rare presentation which has a chronic course characterized by mild intermittent fever, anemia, and ultimately meningeal
involvement preceding death
•• An acute septicemic form, nonreactive miliary tuberculosis, occurs very rarely and is due to massive hematogenous dissemination
of tubercle bacilli
•• Pancytopenia is common in this form of disease, which is rapidly fatal
•• At postmortem examination, multiple necrotic but nongranulomatous (nonreactive) lesions are detected
In miliary TB:
•• Large cavities are formed
•• Radiologically diffuse consolidation
•• Pleural effusions seen
Pneumocystis Carinii
•• Pneumocystis carinii, recently suggested to be a fungus rather than a parasite, is usually dormant in the host lung, causes disease
when defenses are compromised
•• Fungus causing pneumonia in AIDS
•• Damage of type I pneumocytes
•• Hypertrophy of type II pneumocytes
•• Interstitial pneumonitis
•• Nearly all patients have immunologic deficiencies
•• The most common being defects in cell-mediated immunity as with hematologic malignancies, lymphoproliferative
diseases, cancer chemotherapy, and AIDS
•• Among patients with HIV infection, about 60% have P. carinii pneumonia as the initial AIDS-defining diagnosis, and >80% of AIDS
patients have this infection at some time during their course
•• Most patients have a history of fever, dyspnea, and a ‘dry, nonproductive cough’ that may evolve in a subacute fashion over
several weeks or acutely over several days
•• The chest X-ray characteristically shows diffuse, bilateral, perihilar infiltrates, but 10 to 20% of patients have normal X-rays.
26 USMLE Step 2 CK Platinum Notes
•• Arterial blood gases show ‘hypoxemia, with a marked increase in the alveolar-arterial O2 gradient’, and pulmonary function
shows altered diffusing capacity
•• Patients with HIV become vulnerable to P. carinii pneumonia when the CD4 helper cell count is <200/µL
•• The diagnosis requires histopathologic demonstration of the organism with methenamine silver, Giemsa, Wright-Giemsa,
modified Grocott, Gram-Weigert, or monoclonal antibody stain, with specimens obtained by transtracheal aspiration,
transthoracic needle aspiration, open lung biopsy, induced sputum, or bronchoscopy
•• The preferred diagnostic method is induced sputum when technical expertise is available
•• Alternatively, the preferred method is bronchoscopy with bronchioalveolar lavage and brush biopsy.
Sarcoidosis
•• Sarcoidosis is a systemic disease, and thus the clinical manifestations may be generalized or focused on one or more organs.
However, because the lung is almost always involved, most patients have symptoms referable to the respiratory system
•• The first manifestation of the disease is an accumulation of mononuclear inflammatory cells, mostly CD4+ TH1 lymphocytes
and mononuclear phagocytes, in affected organs. The giant cells within the granuloma can be of the Langhans’ or foreign-body
variety and often contain inclusions such as noncaseating granulomas with bilateral hilar lymphadenopathy
–– Schaumann bodies (conch-like structures)
–– Asteroid bodies (stellate-like structures) and
–– Residual bodies (refractile calcium-containing inclusions)
Medicine 27
Lymphangioleiomyomatosis (LAM)
•• Lymphangioleiomyomatosis (LAM) is a rare lung disease that results in disorderly smooth muscle proliferation throughout the
bronchioles, alveolar septa, perivascular spaces, and lymphatics, resulting in the obstruction of small airways (leading to pulmonary
cyst formation and pneumothorax) and lymphatics (leading to chylous pleural effusion)
•• LAM occurs in a sporadic form, which only affects females, who are usually of childbearing age. LAM also occurs in patients who
have tuberous sclerosis
•• Symptoms are dyspnea, cough, chest pain, and hemoptysis; spontaneous pneumothorax is common
•• Diagnosis is suspected on the basis of symptoms and chest X-ray and is confirmed by high-resolution CT. Prognosis is uncertain,
but the disorder is slowly progressive and over the years often leads to respiratory failure and death. Primary treatment is lung
transplantation
•• Lymphangioleiomyomatosis (LAM) is not an interstitial lung disease, but patients are occasionally misdiagnosed as having
interstitial lung disease (and also asthma or COPD).
Lung Cancer
Lung Cancer
•• Lung Ca with worst prognosis Small cell Ca
•• Lung Ca most responsive to radiotherapy Small cell Ca
•• Lung Ca most responsive to chemotherapy Small cell Ca
–– Most common type of lung Ca Adenocarcinoma
–– Most commonly metastasizing to opposite lung Adenocarcinoma
–– Most common type in females Adenocarcinoma
–– Most common type in nonsmokers Adenocarcinoma
–– Most common in young Adenocarcinoma
–– Most common in peripheral location Adenocarcinoma
•• Second most common lung Ca Squamous cell carcinoma
•• Most common cavitating lung Ca Squamous cell carcinoma
•• Best prognosis among lung Ca Squamous cell carcinoma
•• Most common to produce hypercalcemia Squamous cell carcinoma
C associated with squamous cell carcinoma
Medicine 29
•• Central in location
•• Cavitation
•• Clubbing
•• Calcium level↑
Lambert-Eaton Syndrome
Lambert-Eaton myasthenic syndrome is seen in association with small cell lung cancer, and to a lesser extent, breast and ovarian
cancer. It may also occur independently as an autoimmune disorder. Lambert-Eaton myasthenic syndrome is caused by an antibody
directed against presynaptic voltage-gated calcium channel in the peripheral nervous system.
Features:
•• Repeated muscle contractions lead to increased muscle strength *(in contrast to myasthenia gravis)
•• Limb girdle weakness (affects lower limbs first)
•• Hyporeflexia
•• Autonomic symptoms: Dry mouth, impotence, difficultly micturating
•• Ophthalmoplegia and ptosis not commonly a feature (unlike in myasthenia gravis)
EMG
–– Incremental response to repetitive electrical stimulation
Management
–– Guanidine is sometimes used
Pulmonary Embolism
•• Dyspnea most common symptom and earliest manifestation
•• Tachypnea most common sign
•• Tachycardia, S3Q3T3 (an S wave in lead I, a Q wave in lead III, and an inverted T wave in lead III Pattern in ECG) Abnormalities include
sinus tachycardia; new-onset atrial fibrillation or flutter; and an inverted T wave in lead III. Often, the QRS axis is greater than 90.
T wave inversion in leads V1 to V4 reflects right ventricular strain
–– West Mark’s Sign: Focal oligemia on CXR
–– Hamptons’s Sign: Wedge shaped density on CXR
–– Palla’s Sign: Enlarged Right Descending Pulmonary Artery
•• Plasma D-dimer enzyme-linked immunosorbent assay (ELISA) level is elevated in more than 90% of patients with PTE, reflecting
plasmin’s breakdown of fibrin
•• Pulmonary angiography: Selective pulmonary angiography is the most specific test available for establishing the definitive
diagnosis of PTE and can detect emboli as small as 1 to 2 mm
•• A definitive diagnosis of PTE depends upon visualization of an intraluminal filling defect in more than one projection
•• Secondary signs of PTE include:
–– Abrupt occlusion (cut-off) of vessels
–– Segmental oligemia or avascularity
–– A prolonged arterial phase with slow filling
–– Or tortuous, tapering peripheral vessels
30 USMLE Step 2 CK Platinum Notes
A Hypercoagulable state is characteristic of pregnancy, and deep venous thrombosis (DVT) is a common complication. Indeed,
pulmonary embolism is the most common cause of maternal death in the United States. Activated protein C resistance caused by the
factor V Leiden mutation increases the risk for DVT and pulmonary embolism during pregnancy
•• Well’s grading of pulmonary embolism is based on: clinical symptoms
•• Definitive method of diagnosing pulmonary embolism is pulmonary angiography
•• Most reliable method for diagnosis of PE is angiography
Pneumothorax
•• Pneumothorax is the presence of gas in the pleural space
•• Sudden dyspnea following cough and chest pain is seen
•• Spontaneous pneumothorax occurs without antecedent trauma to thorax common in males/smokers
•• Rupture of subpleural bleb is a common cause
•• Primary spontaneous pneumothorax occurs in the absence of underlying lung disease, while
•• Secondary spontaneous pneumothorax occurs in its presence. A traumatic pneumothorax results from penetrating or
nonpenetrating chest injuries
–– Tension pneumothorax is a pneumothorax in which the pressure in the pleural space is positive throughout the respiratory
cycle
–– Primary Spontaneous Pneumothorax: Primary spontaneous pneumothoraces are usually due to rupture of apical pleural
blebs, small cystic spaces that lie within or immediately under the visceral pleura. Primary spontaneous pneumothoraces
occur almost exclusively in smokers
–– Secondary Spontaneous Pneumothorax: Most secondary spontaneous pneumothoraces are due to chronic obstructive
pulmonary disease
–– Traumatic Pneumothorax: Traumatic pneumothoraces can result from both penetrating and nonpenetrating chest trauma.
Traumatic pneumothoraces should be treated with tube thoracostomy unless they are very small.
Medicine 31
–– Tension Pneumothorax: This condition usually occurs during mechanical ventilation or resuscitative efforts. The positive
pleural pressure is life-threatening both because ventilation is severely compromised and because the positive pressure is
transmitted to the mediastinum, which results in decreased venous return to the heart and reduced cardiac output
•• Decreased breath sounds
•• Hyper-resonant percussion note
•• Intrapleural pressure is equal to atmospheric pressure in Open Pneumothorax.
Tension Pneumothorax
This is a life-threatening emergency that needs to be managed immediately with either chest tube thoracostomy or needle
thoracocentesis to relieve the tension on the affected side of the thorax. Physical examination reveals tachycardia, tachypnea,
decreased or absent breath sounds over the involved hemithorax, increased resonance to percussion, subcutaneous
emphysema, and deviation of the trachea to the opposite side. Tension pneumothorax develops when air leaking into the chest
increases intrathoracic pressure, completely collapsing the lung on that side. It results in displacement of the mediastinum
and trachea to the opposite side of the chest and impedes venous return.
Remember
•• Shrinking lung syndrome is seen in SLE
•• Bovine cough is a feature of laryngeal palsy
•• White lung is a feature of asbestosis
•• Sequestration lung is best diagnosed by angiography
•• Shock lung/DAD (Diffuse Alveolar Damage) is ARDS
Features of ARDS
32 USMLE Step 2 CK Platinum Notes
•• Stiff lungs
•• Hypoxemia
•• Hypercapnia
•• Pulmonary edema
•• Normal PCWP
•• ↑Pulmonary artery pressure
•• No response to oxygen
•• Ground glass appearance
•• Acute onset of respiratory failure
•• Air bronchogram sign is positive
•• Associated with
•• Pancreatitis
•• Trauma
•• Septicemia
•• Embolism
•• Multiple blood transfusions
•• Bronchogenic cancer
•• COLD
•• Bronchiectasis
•• Lung abscess
•• Empyema
•• Pulmonary fibrosis
•• Mediastinal tumors
•• Pleural tumors
•• Cryptogenic organizing pneumonia
CXR demonstrating bihilar adenopathy CXR of a squamous cell carcinoma of the left upper lobe of lung
CARDIOVASCULAR MEDICINE
Different Types of Pulse
•• Small volume, slow rising pulse: (Anacrotic): Aortic Stenosis
•• Collapsing and slow rising (Bisferiens): AS with AR
•• Collapsing Pulse: Hyperdynamic circulation, AR, Thyrotoxicosis, PDA, AR
•• Pulsus alternans. LVF
•• Jerky pulse: HOCM
•• Pulsus paradoxus: Asthma, cardiac tamponade, pericarditis
Waves in JVP
•• ‘a’ wave – 1st positive wave and it is due to atrial systole. Absent in AF
•• ‘x’ wave – fall of pressure in atrium, coincides with atrial diastole
•• ‘c’ wave – it is due to rise in atrial pressure during isometric contraction during which the AV valves bulge into atrium
•• ‘x1’ wave – occurs during ejection period, when AV ring is pulled towards ventricles causing distension of atria
•• ‘v’ wave – occurs during isometric relaxation period or during atrial diastole
•• ‘Y’ wave – due to opening of AV valve and emptying of blood into ventricle.
Heart Sounds
Heart sounds Cause
First Closure of AV valves (Mitral Tricuspid)
High-pitched duration: 14 seconds
SOFT S1 is seen in:
–– MS (long-standing, calcified)
–– Obesity
–– MR
–– Pleural effusion
–– Loud S1 is heard in:
anemia, anxiety, fever, Thyrotoxicosis
ASD, PDA
MS (Polonged flow through AV valve)
Second Closure of semilunar valves (aortic, pulmonary)
Wide split of S2 is seen in:
ASD, MR, PS
Reverse splitting of S2 is seen in:
LBBB, HTN, AS
S3
•• S3 is low-pitched and because of ventricular filling
•• Diastolic occurring after S2
•• Best heard at apex
•• Best heard with bell of stethoscope
Causes of S3:
•• In children, hyperdynamic states
•• LVF, RVF
•• Regurgitant lesions
S4
•• S4 is low-pitched
•• Presystolic, produced during second rapid filling phase (before S1)
•• Best heard with bell of stethoscope
Seen in:
•• Hypertension
•• AS
36 USMLE Step 2 CK Platinum Notes
•• HOCM
•• Ischemic heart diseases
•• Acute MR
Parameters in shock
Conduction Abnormalities
Normal ECG
Prolonged
•• Lev’s disease: There is calcification and sclerosis of the fibrous cardiac skeleton, which frequently involves the aortic and mitral
valves, the central fibrous body, and the summit of the ventricular septum
•• Lenegre’s disease appears to be a primary sclerodegenerative disease within the conducting system itself with no involvement
of the myocardium or the fibrous skeleton of the heart
•• First-degree AV block, more properly termed prolonged AV conduction, is classically characterized by a PR interval > 0.20 s
•• Second-degree heart block (intermittent AV block) is present when some atrial impulses fail to conduct to the ventricles
•• Mobitz type I second-degree AV block (AV Wenckebach block) is characterized by progressive PR interval prolongation prior to
block of an atrial impulse. This type of block is almost always localized to the AV node and associated with a normal QRS duration,
although bundle branch block may be present
•• There is progressive lengthening of the PR interval and a drop beat occurs
•• Mobitz type II second-degree AV block, conduction fails suddenly and unexpectedly without a preceding change in PR intervals.
It is generally due to disease of the His-Purkinje system and is most often associated with a prolonged QRS duration
•• Third-degree AV block is present when no atrial impulse propagates to the ventricles. If the QRS complex of the escape rhythm
is of normal duration, occurs at a rate of 40 to 55 beats per minute, and increases with atropine or exercise, AV nodal block is
probable. Congenital complete AV block is usually localized to the AV node. If the block is within the His bundle, the escape
pacemaker is usually less responsive to these perturbations. If the escape rhythm of the QRS is wide and associated with rates 40
beats per minute, block is usually localized in, or distal to, the His bundle and mandates a pacemaker, since the escape rhythm in
this setting is unreliable
•• Some patients with infra-His bundle block are capable of retrograde conduction. In such patients, a ‘pacemaker syndrome’ may
develop if a simple ventricular pacemaker is used. Dual-chamber pacemakers eliminate this potential problem.
38 USMLE Step 2 CK Platinum Notes
‘Pacemaker’ Syndrome
The pacemaker syndrome consists of fatigue, dizziness, syncope, and distressing pulsations in the neck and chest and can be
associated with adverse hemodynamic effects. The pathophysiologic contributors to the pacemaker syndrome include:
•• Loss of atrial contribution to ventricular systole
•• Vasodepressor reflex initiated by cannon A waves, which are caused by atrial contractions against a closed tricuspid valve and
observed in the jugular venous pulse and
•• Systemic and pulmonary venous regurgitation due to atrial contraction against a closed AV valve
The symptoms associated with the pacemaker syndrome can be prevented by maintaining AV synchrony by dual-chamber pacing
or, in the case of a ventricular demand pacemaker, by programming an escape rate of 15 to 20 beats per minute below that of the
paced rate.
Relative Bradycardia
•• Typhoid fever
•• Brucellosis
•• Leptospirosis
•• Drug-induced fever
•• Factitious fever
ST Elevation is seen in
•• Acute MI
•• Prinzmetal’s angina
•• Acute pericarditis
•• Ventricular aneurysm
•• Early repolarization variant
Torsades De Pointes
Wolff-Parkinson-White Syndrome
•• The term Wolff-Parkinson-White syndrome is applied to patients with both pre-excitation on the ECG and paroxysmal tachycardias.
AV bypass tracts can be associated with certain congenital abnormalities, the most important of which is Ebstein’s anomaly
•• AV bypass tracts that conduct in an antegrade direction produce a typical ECG pattern of a:
–– Short PR interval (< 0.12 s)
–– A slurred upstroke of the QRS complex (delta wave) and
–– A wide QRS complex
•• During PSVT in WPW, the impulse is usually conducted antegradely over the normal AV system and retrogradely through the
bypass tract
•• Procainamide is helpful
•• Radiofrequency ablation is done.
40 USMLE Step 2 CK Platinum Notes
Sinus Bradycardia
Atrial Fibrillation
Ventricular Tachycardia
42 USMLE Step 2 CK Platinum Notes
Ventricular Fibrillation
VF: Here heart rate is greater than 300, irregular rhythm, absent P wave, fibrillatory base line
Myocardial Infarction
Anteroseptal MI
Old Inferior MI
Anteroseptal MI
In hypothermia, a small extra wave is seen immediately after the QRS complex (best seen in Lead I in this example)
This wave is called a J-wave, or Osborn wave. This wave disappears with the warming of body temperature
Murmurs
Ejection systolic Pan-systolic Late systolic Early diastolic Mid-diastolic
•• Aortic stenosis •• Mitral regurgitation •• HOCM •• Aortic •• Mitral stenosis
•• Pulmonary stenosis •• Tricuspid •• Mitral valve prolapse regurgitation •• Austin-Flint murmur
•• HOCM regurgitation •• Coarctation of aorta (severe aortic
•• ASD •• Mitral prolapse regurgitation)
•• Fallot’s •• VSD
Medicine 45
REMEMBER
Coarctation of the aorta
Coarctation of the aorta occurs in two patterns:
In the infantile type, the stenosis is proximal to the insertion of the ductus arteriosus (preductal); this pattern is associated with
Turner’s syndrome
In the adult form, the stenosis is distal to the ductus arteriosus (postductal) and is associated with notching of the ribs (secondary
to continued pressure from the aorta on them), hypertension in the upper extremities, and weak pulses in the lower extremities.
Headache, cold extremities, and lower extremity claudication with exercise are typical if the patient is symptomatic (many adults with
mild distal coarctation may remain asymptomatic for years). Upper extremity hypertension with weak pulses in the lower extremities,
and a midsystolic (or continuous) murmur over the chest or back may be the only obvious signs in some.
•• Sydenham’s chorea (St. Vitus’ dance): A characteristic series of rapid movements without purpose of the face and arms
•• This can occur very late in the disease
Minor Criteria
•• Fever
•• Arthralgia: Joints pain without swelling
•• Raised erythrocyte sedimentation rate or C-reactive protein
•• Leukocytosis
Mitral Stenosis
Mitral stenosis: It is said that the causes of mitral stenosis are rheumatic fever, calcium deposits and narrowing of motral valve in
Newborn babies. Rarer causes that may be seen include mucopolysaccharidoses, carcinoid and endocardial fibroelastosis
Features:
•• Mid-diastolic murmur (Best heard in expiration)
•• Loud S1, opening snap
•• Low volume pulse
•• Malar flush
•• Atrial fibrillation
•• Hemoptysis seen
•• Hoarseness of voice is seen in MS
•• Left atrial myxoma is an important differential diagnosis
•• Pulmonary hypertension occurs
Features of severe MS:
Length of murmur increases, opening snap becomes closer to S2
Echocardiography:
Most useful in MS is echocardiography
•• The normal cross-sectional area of the mitral valve is 4–6 sq cm
•• A ‘tight’ mitral stenosis implies a cross-sectional area of < 1 sq cm
•• < 0.6 cms of mitral vavle area is incompatible with life.
Aortic Stenosis
Aortic stenosis:
Features
•• Ejection systolic murmur
•• Slow rising pulse
•• Narrow pulse pressure
•• Sustained heaving apex is seen
•• Angina is a feature
•• Sudden death is a feature
•• Exercise testing is contraindicated in AS
Causes of Aortic Stenosis:
•• Degenerative calcification
•• Bicuspid aortic valve
•• William’s syndrome (supravalvular aortic stenosis)
•• Postrheumatic disease
•• Subvalvular: HOCM
Management:
•• If asymptomatic, then observe the patient as a general rule
•• If symptomatic, then valve replacement
•• If asymptomatic but valvular gradient > 50 mm Hg and with features such as left ventricular systolic dysfunction, then consider
surgery
•• Balloon valvuloplasty is limited to patients with critical aortic stenosis who are not fit for valve replacement.
Aortic Regurgitation
Aortic regurgitation:
Features
•• Early diastolic murmur
•• Collapsing pulse
•• Wide pulse pressure
•• Mid-diastolic Austin-Flint murmur in severe AR—due to partial closure of the anterior mitral valve cusps caused by the
regurgitation streams
Causes (due to valve disease)
•• Rheumatic fever
•• Infective endocarditis
•• Connective tissue diseases e.g. RA/SLE
•• Bicuspid aortic valve
Causes (due to aortic root disease)
•• Aortic dissection
•• Spondylarthropathies (e.g. ankylosing spondylitis)
•• Hypertension
•• Syphilis
•• Marfan’s, Ehler-Danlos syndrome
Signs seen in AR
•• Corrigan’s sign Dancing carotids
•• De Musset’s sign Head movements with cardiac pulse
•• Quincke’s sign ↑Capillary pulsations
•• Traube’s sign Pistol shot sounds over femorals
48 USMLE Step 2 CK Platinum Notes
AR
An 20-year-old male comes to the physician for a routine physical examination. His height is 198 cm. He has long fingers
and toes. Blood pressure is 150/65 mm Hg, and pulse is 66/min. On auscultation, there is a grade 2/6, long, high-frequency
diastolic murmur at the second right intercostal space.
CARDIOMYOPATHIES
HOCM
Hypertrophic cardiomyopathy is a common cause of sudden cardiac death in young patients. It usually causes problems during
exertion. Clues to the diagnosis include: dyspnea, palpitations, bifid apical impulse, coarse systolic murmur at the left sternal border,
and ventricular hypertrophy with asymmetric septal thickening on echocardiogram. Left ventricular outflow obstruction typically
plays an important role in the pathophysiology of this condition. Maneuvers that decrease preload, such as the Valsalva maneuver, will
accentuate the heart murmur because they result in less ventricular filling, contributing to greater outflow obstruction.
HOCM: Hypertrophic obstructive cardiomyopathy (HOCM) is an autosomal dominant disorder of muscle tissue caused by defects in
the genes encoding contractile proteins
Features:
•• Dyspnea, angina, syncope
•• Sudden death (most commonly due to ventricular arrhythmias), arrhythmias, heart failure
•• Jerky pulse
•• Large ‘A’ waves
•• Double apex beat
•• Ejection systolic murmur: Increases with Valsalva maneuver and decreases on squatting
Associations: Friedreich’s ataxia, WPWS
Medicine 49
ECHO:
•• Systolic anterior motion (SAM) of the anterior mitral valve leaflet
•• Asymmetric hypertrophy (ASH)
•• Mitral regurgitation
•• Hypertrophic obstructive cardiomyopathy (HOCM) caused by mutations in the myosin heavy chain b gene
•• Udually follows AD inheritance
•• Sudden death is a feature
•• Ejection systolic murmur of HOCM ↑STANDING, ↓Lying down, supine
•• Avoid digoxin
•• Cardiac catheterization shows Banana shaped left biventricular cavity in systole
•• Usual Premature closure of Aortic cusps
•• Predominantly Left ventricular outflow tract gradient.
Dilated Cardiomyopathy
Dilated cardiomyopathy
Basics:
•• Dilated heart leading to systolic (+/– diastolic) dysfunction
•• All 4 chambers affected but LV more so than RV
•• Features include arrhythmias, emboli, mitral regurgitation
•• Absence of congenital, valvular or ischemic heart disease
Causes:
•• Infections, e.g. coxsackie A and B, HIV, diphtheria, parasitic
•• Endocrine, e.g. hyperthyroidism
•• Infiltrative*, e.g. hemochromatosis, sarcoidosis
•• Neuromuscular, e.g. Duchenne muscular dystrophy, Friedreich’s ataxia
•• Nutritional, e.g. kwashiorkor, pellagra, thiamine/selenium deficiency
Restrictive Cardiomyopathy
Restrictive cardiomyopathy is the least prevalent form of cardiomyopathy
•• Amyloidosis
•• Hemochromatosis
•• Sarcoidosis and
•• Fabry’s disease
The main hemodynamic consequence of these pathologic states is a rigid, noncompliant chamber with a high-filling pressure
•• Systolic function may deteriorate if compensatory hypertrophy is inadequate to compensate for infiltrated or fibrosed chambers
•• Mural thrombosis and systemic emboli can complicate either the restrictive or obliterative variety
Echocardiography shows normal systolic function
•• The atria are often dilated
•• Myocardial hypertrophy is often seen in restrictive myopathy
•• High atrial pressure with a prominent ‘y’ descent
•• Kussmaul’s sign is seen
•• Normal-sized ventricular cavities with normal or decreased systolic shortening
•• Biopsy can demonstrate endocardial fibrosis and thickening, myocardial infiltration with iron or amyloid or chronic myocardial
fibrosis.
50 USMLE Step 2 CK Platinum Notes
Infective Endocarditis
Infective Endocarditis
•• Patients affected by endocarditis
•• Previously normal valves (50%, typically acute presentation)
•• Rheumatic valve disease (30%)
•• Prosthetic valves
•• Congenital heart defects
•• Intravenous drug users (IVDUs, e.g. typically causing tricuspid lesion)
Causes:
•• Streptococcus viridans (most common cause – 40–50%)
•• Staphylococcus epidermidis (especially prosthetic valves)
•• Janeway’s lesions, (nontender maculopapular lesions in palms/soles)
•• Splinter hemorrhages (NAILS)
•• Roth’s spots seen (RETINA)
LOW RISK:
•• ASD
•• MVP without MR
Atherosclerosis
Risk factors for atherosclerosis:
•• Smoking
•• Hypertension
•• Low HDL, raised apolipoprotein A
•• Familial hypercholesterolemia
•• Familial hypertriglyceridemia
•• Familial dysbetalipoproteinemia
•• Diabetes mellitus
•• Family history
•• Obesity
•• Physical inactivity
Associated with coronary atherosclerosis:
•• CMV
•• Chlamydiae
•• H. pylori
Amino acid associated with atherosclerosis: homocysteine
Medicine 51
Sequence of MI
•• Eisenmenger’s syndrome
Describes the reversal of a left to right shunt in a congenital heart defect due to pulmonary hypertension associated with:
–– VSD
–– ASD
–– PDA
Features
•• Original murmur may disappear
•• Cyanosis
•• Clubbing
•• Right ventricular failure
•• Hemoptysis, embolism
•• Carey Coombs Murmur: Transient soft mid-diastolic murmur of acute rheumatic fever due to mitral valvulitis
•• Graham Steel Murmur: Early diastolic murmur of Pulmonary Regurgitation
•• Means Murmur: Pulmonary systolic murmur in Thyrotoxicosis
•• Seagull’s Murmur: Aortic regurgitation murmur.
Medicine 53
Digoxin Toxicity
•• Digoxin toxicity
Features: Generally unwell, lethargy, N/V, confusion, yellow-green vision, arrhythmias (e.g. AV block, bradycardia, VF, VT)
Idioventricular rhythm is a feature:
Precipitating factors
•• Classically: hypokalemia*
•• Myocardial ischemia
•• Hypomagnesemia, hypercalcemia, hypernatremia, acidosis
•• Hypoalbuminemia
•• Hypothermia
•• Hypothyroidism
•• Drugs: amiodarone, quinidine, verapamil, spironolactone (compete for secretion in distal convoluted tubule, therefore,
reduce excretion)
Management
•• Withdrawal of digitalis, potassium supplements
•• Correct arrhythmias, phenytoin used, digitalis antibody (fab fragments)
•• Monitor K+
•• Tricyclic antidepressants
•• Chloroquine
•• Terfenadine: A nonsedating antihistamine and classic cause of prolonged QT in a patient, especially if also taking P450 enzyme
inhibitor, e.g. patient with cold takes terfenadine and erythromycin at the same time
Other causes
•• Electrolyte: Hypocalcemia
•• Hypokalemia
•• Hypomagnesemia
•• Hypothermia
•• Acute MI
•• Myocarditis
•• Subarachnoid hemorrhage
Management
•• Beta-blockers
•• Implantable cardioverter defibrillators in high-risk cases.
Acute Pericarditis
Pain, a pericardial friction rub, electrocardiographic changes, and pericardial effusion with cardiac tamponade and paradoxic
pulse are cardinal manifestations:
•• Chest pain is an important but not invariable symptom in various forms of acute pericarditis
•• The pericardial friction rub is the most important physical sign of acute pericarditis; it may have up to three components
per cardiac cycle and is high-pitched, scratching, and grating
•• The electrocardiogram: Widespread elevation of the ST segments, often with upward concavity.
Cardiac Tamponade
The accumulation of fluid in the pericardium in an amount sufficient to cause serious obstruction to the inflow of blood to the ventricles
results in cardiac tamponade
The three most common causes of tamponade are neoplastic disease, idiopathic pericarditis, and uremia. Tamponade may also
result from bleeding into the pericardial space either following cardiac operations and trauma (including cardiac perforation during
diagnostic procedures) or from tuberculosis and hemopericardium. The latter may occur when a patient with any form of acute
pericarditis is treated with anticoagulants
The three principal features of tamponade are:
•• Elevation of intracardiac pressures
•• Limitation of ventricular filling and
•• Reduction of cardiac output
Look for
•• Electrical alternans
•• Pulsus paradoxus
•• JVP↑
•• Prominent x descent
•• Absent y descent
•• Beck’s triad: hypotension, silent heart, ↑JVP
•• Absent Kussmaul’s sign
Medicine 55
Constrictive Pericarditis
•• Right heart failure occurs
•• Raised JVP seen
•• Ascites precox seen
•• Prominent x descent
•• Prominent y descent
•• Thickened pericardium
•• Kussmaul’s sign +
•• Square root sign+
Heart failure
Pericardial Effusion
Pericardial effusion nearly always has the physical characteristics of an exudate:
Bloody fluid is commonly due to:
•• Tuberculosis
•• Tumor
•• Effusion of rheumatic fever
•• Postcardiac injury
•• Postmyocardial infarction (especially following the administration of anticoagulant) and in uremic pericarditis
•• Transmural myocardial infarction
•• Aortic aneurysm dissection
•• Metastasis to pericardium.
A man with fever, dull-aching retrosternal chest pain reported to the emergency. On examination, he was found to show
pulsus paradoxus, Kussmaul’s sign and increased jugular venous pressure. His apex beat was impalpable and heart sounds
were muffled. Probable diagnosis is: Pericardial effusion.
HEMATOLOGY
Iron deficiency anemia of long duration is characterized by:
Microcytic Anemia
Causes:
•• Iron deficiency
•• Thalassemia
•• Sideroblastic anemia
•• Anemia of chronic disease
•• Lead poisoning
Blood Changes:
•• Hypochromia
•• Microcytosis
•• Poikilocytosis
•• Bone marrow with ↑ normoblasts
•• Microcytic hypochromic anemia not responding to iron is thalassemia usually.
•• Hypochromic microcytic anemia with ↓Sr, Fe, ↑TIBC: Iron deficiency anemia
•• Hypochromic microcytic anemia with ↓Sr, Fe, ↓TIBC: Anemia of chronic disease
•• ↓ Sr Ferritin is the early indicator of iron deficiency
Megaloblastic Anemia
Causes:
•• Liver diseases
•• Hypothyroidism
•• Orotic aciduria
•• Thiamine deficiency
•• B12 deficiency
•• Folic acid deficiency
•• Hemodialysis
•• Fish tapeworm infestation
•• Drugs: Phenytoin, primidone, nitrous oxide, phenobarbitone, folate antagonists.
Hematological findings
•• Decreased red blood cell (RBC) count and hemoglobin levels
•• Increased mean corpuscular volume (MCV, >95 fl) and mean corpuscular hemoglobin (MCH)
•• Normal mean corpuscular hemoglobin concentration (MCHC, 32–36 g/dL)
•• The reticulocyte count is decreased due to destruction of fragile and abnormal megaloblastic erythroid precursor
•• The platelet count may be reduced
•• Neutrophil granulocytes may show multisegmented nuclei (senile neutrophil). This is thought to be due to decreased production
and a compensatory prolonged lifespan for circulating neutrophils, which increase number of nuclear segments with age
•• Anisocytosis (increased variation in RBC size) and poikilocytosis (abnormally shaped RBCs)
•• Macrocytes (larger than normal RBCs) are present
•• Howell-Jolly bodies (chromosomal remnant) also present
•• Pancytopenia can also occur.
Folate Deficiency
It is commonly seen in alcoholics (poor intake) and pregnant women (increased need). All women of reproductive age should
take folate supplements to prevent neural tube defects in their offspring. Rare causes of folate deficiency include poor diet (e.g.
tea and toast), methotrexate, prolonged therapy with trimethoprim-sulfamethoxazole, anticonvulsant therapy (especially
phenytoin), and malabsorption. Look for macrocytes and hypersegmented neutrophils (either one should make you think of
the diagnosis) with no neurologic symptoms or signs and low folate levels in serum or red blood cells. Treat with oral folate.
Features of Hemolysis
•• Elevated lactate dehydrogenase (LDH)
•• Elevated bilirubin (unconjugated as well as conjugated if the liver is functioning)
•• Jaundice
•• Low or absent haptoglobin (intravascular hemolysis only)
•• Urobilinogen, bilirubin, and hemoglobin in urine (only conjugated bilirubin shows up in the urine, and hemoglobin shows
up in the urine only when haptoglobin has been saturated, as in brisk intravascular hemolysis).
Sickle-cell Crises
•• AR condition
•• Glutamine-valine change
TO BE EXACT: Replacement of glutamate by valine at position 6 beta chain of Hb A
Features:
•• Bone pain commonest. (acute presentation)
•• Pulmonary HTN
•• Cardiomegaly
•• Fish mouth vertebrae
•• Osteomyelitis, priaprism, renal pappilary necrosis
–– Hemosidenuria
–– ↑LDH
–– ↑unconjugated bilirubin
–– ↓haptoglobin
–– Leucocytosis
–– Target cells seen
•• Autosplenectomy
•• Crewcut hair skull along with thalassemia
•• Improved with Hb F
60 USMLE Step 2 CK Platinum Notes
Thrombotic crises
•• Also known as ‘painful crises’ or ‘vaso-occlusive crises’
•• Precipitated by infection, dehydration, deoxygenation
•• Infarcts occur in various organs including the bones (e.g. avascular necrosis of hip, hand-foot syndrome in children, lungs, spleen
and brain
Sequestration crises
•• Sickling within organs such as the spleen or lungs causes pooling of blood with worsening of the anemia
•• Acute chest syndrome: Dyspnea, chest pain, pulmonary infiltrates, low pO2—the most common cause of death after childhood
Aplastic crises
•• Caused by infection with parvovirus
•• Sudden fall in hemoglobin
Hemolytic crises
•• Rare
•• Fall in hemoglobin due an increased rate of hemolysis
Ans. 5. Parvovirus
This patient is presenting with aplastic crisis, organism is parvovirus B19, a virus implicated in erythema infectiosum (fifth
disease). The virus causes a transient red cell aplasia; bone marrow often shows lack of erythroid precursors.
–– Chloroquine
–– Dapsone
–– Nitrofurantoin
–– Nalidixic acid
Hereditary Spherocytosis
•• Primary abnormality in the cytoskeletal proteins that form RBC membrane
•• Cytoskeletal proteins: maintain RBC shape, strength and flexibility
•• Spherical RBC due to a molecular defect in one of the proteins in the cytoskeleton of the RBC membrane, leading to a loss of
membrane
•• Decreased ratio of surface area to volume
•• Usually has an autosomal dominant inheritance pattern
•• The major clinical features of hereditary spherocytosis are anemia, splenomegaly, and jaundice
•• Assessed by measurement of the osmotic fragility of the RBC (↑)
•• Aplastic crisis is triggered by parvovirus
•• About 50% of patients have a defect in ankyrin, the protein that forms a bridge between protein 3 and spectrin
•• A mutation of protein 3, resulting in a deficiency of that protein and mild anemia with dominant inheritance
•• Mutations of spectrin, leading to impaired synthesis or self-association
•• Paladin defect
•• Splenectomy is curative
Hereditary pyropoikilocytosis: This condition results from a deficiency of spectrin and an abnormality of spectrin self-assembly.
Hemolysis is usually severe, is recognized in childhood, and is partially responsive to splenectomy
Hereditary stomatocytosis: Stomatocytes are cup-shaped RBCs (concave on one face and convex on the other).
•• ↓LAP score
•• Hypercellular marrow
Diagnosis: Ham test: Acid-induced hemolysis (normal red cells wouldn’t)
Management: Blood product replacement, anticoagulation, stem cell transplantation
HAM test is a complement-based test
Analysis of GPI-linked proteins is used as a diagnostic test.
Sideroblastic Anemia
•• Red cells contain abnormal iron granules in the mitochondria around the nucleus: ring sideroblasts
•• Peripheral blood cells are hypochromic whilst increased marrow iron
•• High or normal transferrin saturation, ↑Serum ferritin, ↑Sr iron
•• May be congenital or acquired
•• Congenital cause: Delta-aminolevulinate synthase-2 deficiency
Acquired causes
•• Myelodysplasia
•• Alcohol
•• Lead
•• Anti-TB medications
Investigations
•• Hypochromic microcytic anemia (more so in congenital)
•• Bone marrow: Sideroblasts and increased iron store
•• Management: Supportive, pyridoxine may help.
Pyridoxine-responsive conditions
Homocystinuria
Oxaluria
Xanthinuria
Convulsions
Pernicious Anemia
Deficiency of intrinsic factor of Castle:
•• Antigastric parietal cell antibodies in 90%
•• Anti-intrinsic factor antibodies in 50%
•• Macrocytic anemia: Low WCC and platelets
•• LDH may be raised due to ineffective erythropoiesis
•• Also low-serum B12, hypersegmented polymorphs on film, megaloblasts in marrow
•• Schilling test
64 USMLE Step 2 CK Platinum Notes
Remember
•• Uremia •• Burr cells are seen
•• G6PD deficiency •• Bite cells
•• Helmet cells •• Hemolytic Uremic Syndrome
•• Cirrhosis •• Spur cells are seen
•• Iron depletion •• Seen in iron-deficiency anemia, polycythemia vera
•• Thalassemia minor •• Decreased osmolysis, microcytic hypochromic anemia, increased HbA2.
•• Sickle-cell anemia •• Tactoids and sickling seen
•• Acquired spherocytosis •• Schistocytes present, Coombs positive
•• Hereditary spherocytosis •• Coombs negative
•• Aplastic anemia •• Platelets maximum affected and last to recover, pancytopenia present
•• Pernicious anemia and folate deficiency •• Anisocytosis, poikilocytosis, fragmented RBCs, neutrophil lobes increased,
anemia platelets normal. Absolute reticulocyte count low.
•• Sideroblastic anemia •• Ring sideroblasts present (These are iron granules in mitochondria
around the nucleus), microcytic, hypochromic RBCs, macrocytic hypo- or
normochromic RBCs
Autoimmune gastritis causes a lack of the intrinsic factor needed to absorb vitamin B12. Autoantibodies that are often present include
those directed against the microsomal fraction of parietal cells and those capable of neutralizing intrinsic factor.
•• Venesection—first-line treatment
•• Hydroxyurea-slight increased risk of secondary leukemia
•• Phosphorus-32 therapy
•• Prognosis
•• Progress to myelofibrosis
•• Progress to acute leukemia
Myelophthisic Anemia
It is due to a space-occupying lesion in the bone marrow. The common causes are malignant invasion that destroys bone
marrow (most common) and myelodysplasia or myelofibrosis. On the peripheral smear, look for marked anisocytosis (different
size), poikilocytosis (different shape), nucleated red blood cells, giant and/or bizarre-looking platelets, and teardrop-shaped
red blood cells. A bone marrow biopsy may reveal no cells (‘dry tap’ if the marrow is fibrotic) or malignant-looking cells.
Thrombocytopenia
Causes of ‘severe’ thrombocytopenia:
•• ITP
•• DIC
•• TTP
•• Hematological malignancy
Causes of ‘moderate’ thrombocytopenia:
•• Heparin-induced thrombocytopenia (HIT)
•• Drug-induced, (e.g. quinine, diuretics, sulphonamides, aspirin, thiazides)
•• Alcohol
•• Liver disease
66 USMLE Step 2 CK Platinum Notes
•• Hypersplenism
•• Viral infection (EBV, HIV, hepatitis)
•• Pregnancy
•• SLE/antiphospholipid syndrome
•• B12 deficiency
•• Hess’ (tourniquet) test is used for ITP
Hemophilia
•• Decreased VIII level
•• Increased PTT
•• Bleeding into soft tissue
•• Anti-factor VIII antibodies are seen in hemophilia who have received infusion of plasma concentrates
Coagulation Defects
•• Hemophilia A Deficiency of factor VIII C
•• Hemophilia B (Christmas disease) Deficiency of factor IX (X-linked)
•• Hemophilia C Deficiency of factor XI (autosomal inherited defect)
•• Parahemophilia Deficiency of factor V
•• von Willebrand’s disease Deficiency of v WF along with VIII C
Ristocetin-induced platelet aggregation—Ristocetin-induced platelet aggregation (RIPA) measures the affinity with which vWF
binds to the platelet receptor GP Ib by limiting the concentration of ristocetin in the assay. It is used primarily to look for the type 2B
variant of vWF which has mutations in the binding site for GP Ib such that the type 2B vWF binds to GP Ib more readily than normal.
Hemophilia
•• It is an X-linked clotting disorder that occurs in two forms:
–– Hemophilia A due to deficient factor VIII and
–– Hemophilia B due to deficient factor IX
•• Some individuals with hemophilia have levels of these factors that are 5% of normalor even higher, and have relatively
mild disease, only requiring replacement therapy during surgical procedures or other situations in which significant
bleeding might occur
•• In contrast, individuals with factor levels less than 1% of normal have severe bleeding problems throughout life that
usually become apparent by 18 months of age. In these individuals, excessive bleeding into joints and tissues may cause
crippling musculoskeletal disorders.
•• The excessive clotting that occurs in this disorder is almost always restricted to the veins, where the clotting may cause a deep
vein thrombosis (DVT). If the venous clots break off, these clots can travel through the heart to the lung, where they block a
pulmonary blood vessel and cause a pulmonary embolism
•• Women with the disorder have an increased risk of miscarriage and stillbirth. It is extremely rare for this disorder to cause the
formation of clots in arteries that can lead to stroke or heart attack, though a ‘mini-stroke’, known as a transient ischemic attack, is
more common.
Antiphospholipid Syndrome
Antiphospholipid syndrome is an acquired disorder characterized by a predisposition to both venous and arterial thromboses,
recurrent fetal loss and thrombocytopenia. It may occur as primary disorder or secondary to other conditions, most commonly
systemic lupus erythematosus (SLE)
A key point for the exam is to appreciate that antiphospholipid syndrome causes a paradoxical rise in the APTT. This is due to an ex-
vivo reaction of the lupus anticoagulant autoantibodies with phospholipids involved in the coagulation cascade
Features: Can be asymptomatic
•• Venous/arterial thrombosis NOT BLEEDING
•• Recurrent fetal loss (2nd trimester abortions)
•• Livedo reticularis
•• Thrombocytopenia
•• Prolonged APTT
•• Other features: preeclampsia, pulmonary hypertension
•• Not Bleeding Disorder
Leukemoid Reaction
The leukemoid reaction describes the presence of immature cells, such as myeloblasts, promyelocytes and nucleated red cells in the
peripheral blood. This may be due to infiltration of the bone marrow causing the immature cells to be ‘pushed out’ or sudden demand
for new cells
Causes:
•• Severe infection
•• Severe hemolysis
•• Massive hemorrhage
•• Metastatic cancer with bone marrow infiltration
Medicine 69
A relatively common clinical problem is differentiating chronic myeloid leukemia from a leukemoid reaction. The following differences
may help:
Leukemoid reaction:
•• High leucocyte alkaline phosphatase score
•• Toxic granulation (DÖhle bodies) in the white cells
•• ‘Left shift’ of neutrophils, i.e. three or less segments of the nucleus
Chronic myeloid leukemia
•• Low leucocyte alkaline phosphatase score.
Others:
•• ARDS
•• Hypothermia
•• In nonmismatch blood transfusion reaction, direct Coombs test should be done.
70 USMLE Step 2 CK Platinum Notes
Thrombophilia
Inherited
•• Activated protein C resistance (Factor V Leiden)
•• Antithrombin III deficiency
•• Protein C deficiency
•• Protein S deficiency
Acquired
•• Antiphospholipid syndrome
•• OCP
Congestive splenomegaly (Banti’s syndrome): Chronically increased splenic venous pressure may result from hepatic cirrhosis,
portal or splenic vein thrombosis, or certain malformations of the portal venous vasculature. Associated bleeding from esophageal
varices may be worsened by the superimposed thrombocytopenia induced by splenomegaly.
Hodgkin’s Disease
•• Bi modal age distribution (late 20 years and after 50 years)
•• The commonest presentation of Hodgkin’s Lymphoma is painless enlargement of lymph nodes
•• Prognosis is directly proportional to number of RS cells and inversely proportional to number of lymphocytes
•• Spread is to be contigious to adjacent lymph nodes
•• Extranodal spread is uncommon
•• EBV is causative in some cases
•• Malignant cell is Reed-Sternberg Cell. (Owl-eyed, bilobed nucleus with prominent nucleoli)
•• RS cells are positive for CD 15 and CD 30. (Except lymphocyte predominant)
•• CNS involvement is uncommon
•• Cerebellar degeneration is seen in HD (anti-Tr antibodies)
Hodgkin’s lymphoma is a malignant proliferation of lymphocytes characterized by the presence of the Reed-Sternberg cells. It has a
bimodal age distributions being most common in the third and seventh decades
Burkitt’s lymphoma
This type of lymphoma is a high-grade B-cell lymphoma that occurs in endemic form in and sporadically in the United States and
Europe. The sporadic form is often in an abdominal site and occurs in young adults. The African form of Burkitt’s lymphoma has been
strongly associated with antibodies directed against Epstein-Barr virus; the association is weaker in sporadic cases. A characteristic
translocation, t(8;14) (q24.l3;q32.33) has been described.
Features:
•• Pancytopenia
•• Splenomegaly
•• Skin vasculitis in 1/3rd patients
•• ‘Dry tap’ despite bone marrow hypercellularity
•• Tartrate-resistant acid phosphotase (TRAP) stain positive
•• Cladribine is DOC
•• Pentostatin and interferon alpha are also used
•• Popcorn type of Reed Sternberg cell is seen in: Lymphocyte-predominant type of Hodgkin’s lymphoma
•• ABVD and MOPP Regimes are used in Hodgkin’s lymphoma
•• B-cell lymphomas are hairy cell leukemia, mantle cell lymphoma, Burkitt’s Lymphoma
•• HTLV 1 is associated with Adult T cell lymphoma
•• IgA lymphoma involves stomach
Castleman’s disease, which can present with localized or disseminated lymphadenopathy; some patients have systemic symptoms. The
disseminated form is often accompanied by anemia and polyclonal hypergammaglobulinemia, and the condition seems to be related to
an overproduction of interleukin 6, possibly produced by human herpesvirus 8.
Sinus histiocytosis with massive lymphadenopathy (Rosai-Dorfman’s disease) usually presents with bulky lymphadenopathy in
children or young adults. The disease is usually nonprogressive and self-limited, but patients can manifest autoimmune hemolytic
anemia.
Lymphomatoid papulosis is a cutaneous lymphoproliferative disorder that is often confused with anaplastic large-cell lymphoma
involving the skin. The cells of lymphomatoid papulosis are similar to those seen in lymphoma and stain for CD30, and T cell receptor
gene rearrangements are sometimes seen. However, the condition is characterized by waxing and waning skin lesions that usually
heal, leaving small scars.
Takayasu’s Arteritis
•• Also called ‘Pulseless Disease’ or ‘Aortic Arch Syndrome’
•• It is a vasculitis common in Asia, especially in young and middle-aged females, affecting medium to large size arteries including
aorta and its branches.
•• Characterized by granulomatous inflammation leading to arterial thrombosis, stenosis or aneurysm.
•• Clinical features are:
–– Aortic involvement with palpable purpura and panarteritis
–– Loss of pulse in upper extremities, (radial pulse not felt)
–– Systemic features: fever, weight loss, arthalgias, fatigue
–– Anemia and increased ESR
–– Visual loss or field defects, retinal hemorrhages
–– Neurological abnormalities like headache, pulsating chest pain, hypertension
Treatment is by steroids (Drug of choice)
Kawasaki Disease
•• Kawasaki disease (also known as ‘lymph node syndrome’, ‘mucocutaneous node disease’, ‘infantile polyarteritis’)
•• Kawasaki disease is an inflammation (vasculitis) of the middle-sized arteries
•• Kawasaki disease affects many organs, including the skin, mucous membranes, lymph nodes, and blood vessel walls, but the
‘most serious’ ‘effect is on the heart where it can cause ‘severe aneurysmal dilations’
Classically, five days of fever plus four of five ‘diagnostic criteria’ must be met in order to establish the diagnosis
The criteria are:
•• Erythema of the lips or oral cavity or cracking of the lips
•• Rash
•• Swelling or erythema of the hands or feet
•• Red eyes (conjunctival injection)
•• Swollen lymph node in the neck of at least 15 millimeters
76 USMLE Step 2 CK Platinum Notes
Polymyositis/Dermatomyositis
•• A systemic connective tissue disease characterized by inflammatory and degenerative changes in the muscles (polymyositis) and
frequently also in the skin (dermatomyositis), leading to symmetric weakness and some degree of muscle atrophy, principally
of the limb girdles
•• Dermato + Myositis: Involves skin and muscles usually
•• Females are more commonly affected. Features are ‘rash and muscle weakness’
•• ‘Proximal’ muscle weakness with loss of fine motor movements is seen
•• Dermatomyositis can be associated with malignancy in 10–20% cases
•• Rash involves:
–– Eyelids with edema (Heliotrope rash)
–– Erythema of knuckles with scaly eruption (Gottron’s rash)
–– Neck and anterior chest (V sign)
–– Back and shoulder (Shawl sign)
•• Mucocutaneous lesions, small, painless superficial ulcers are commonly seen on the oral mucosa, tongue, and glans penis (balanitis
circinata). Patients may also develop hyperkeratotic skin lesions of the palms and soles, and around the nails (keratoderma
blennorrhagica)
•• Cardiovascular involvement with aortitis, aortic insufficiency, and conduction defects occurs rarely
Temporal Arteritis
Temporal Arteritis (Giant Cell Arteritis; Cranial Arteritis)
•• A chronic inflammatory disease of large blood vessels, particularly those with a prominent elastica, occurring primarily in the
elderly
•• More common in females
•• Giant cell arteritis most often involves arteries of the carotid system, particularly the cranial arteries
•• Segments of the aorta, its branches, the coronary arteries, and the peripheral arteries may also be affected
•• The disease has a predilection for arteries containing elastic tissue; it is rarely seen in veins
•• The histologic reaction is a granulomatous inflammation of the intima and inner part of the media
•• Presentations are diverse, depending on the distribution of the arteritis, but typically include:
–– Severe headache (MC)
–– (Especially temporal and occipital), scalp tenderness, and visual disturbances (amaurosis fugax, diplopia, scotoma,
ptosis, and vision blurring)
–– Claudication of the masseter, temporalis, and tongue muscles are characteristic
–– Blindness due to ischemic optic neuropathy
–– On physical examination, there may be swelling and tenderness with nodularity over the temporal arteries and
–– Bruits over the large vessels
•• ESR is usually markedly elevated (often >100 mm/h, Westergren) during the active phase, but is normal in about 1% of patients
•• Normochromic and normocytic anemia are often present and, at times, profound. Alkaline phosphatase may be elevated
•• Other nonspecific findings include ‘polyclonal hyperglobulinemia and leukocytosis.’
•• Diagnosis can be made clinically but should be verified by temporal artery biopsy because of the need for prolonged corticosteroid
treatment
•• Even a temporal artery that is normal to palpation and without tenderness or swelling may be abnormal on biopsy
Erythromelalgia
A rare syndrome of paroxysmal vasodilation with burning pain, increased skin temperature, and redness of the feet and, less
often, the hands
Causes of secondary erythromelalgia:
•• Myeloproliferative disorders
•• Hypertension
•• Venous insufficiency
•• Diabetes mellitus, SLE, RA
•• Lichen sclerosus et atrophicus, gout
•• Spinal cord disease, or multiple sclerosis
Acrocyanosis
Persistent, painless, symmetric cyanosis of the hands and, less commonly, the feet, caused by vasospasm of the small vessels of
the skin
The etiology is unknown but may be related to increased tone of the arterioles associated with dilation of capillaries and venules
The disorder usually occurs in women and is not associated with occlusive arterial disease
The digits and hands or feet are persistently cold and bluish, and sweat profusely; they may swell
Cyanosis is usually intensified by cold and lessened by warming.
Renal Manifestations
•• Most patients with SLE have immunoglobulins deposited in glomeruli, but only one-half have clinical nephritis, defined by
proteinuria
•• Early in the disease, most are asymptomatic, although some develop the edema of nephrotic syndrome. Urinalysis shows
hematuria, cylindruria, and proteinuria
•• Papillary necrosis seen
•• Most patients with mesangial or mild focal proliferative nephritis maintain good renal function. Patients with diffuse proliferative
nephritis develop renal failure if untreated
•• Wire loop lesions seen
•• Renal involvement is the most common cause of death
Nervous System Manifestations
•• Mild cognitive dysfunction is the most frequent manifestation
•• Headaches are common and may be migraine-like or nonspecific
•• Seizures of any type may occur
•• Less frequent manifestations include psychosis, acute confusional states, demyelinating disorders, cerebrovascular disease,
movement disorders, aseptic meningitis, myelopathy, mononeuropathy or polyneuropathy of cranial or peripheral nerves,
autonomic dysfunction, acute demyelinating polyneuropathy (Guillain-Barre), mood disorders, optic neuritis, subarachnoid
hemorrhage, pseudotumor cerebri, and hypothalamic dysfunction with inappropriate secretion of vasopressin. Depression and
anxiety are frequent.
Vascular System
•• Thrombosis in vessels of any size can be a major problem
•• Although vasculitis may underlie thrombosis, there is increasing evidence that antibodies against phospholipids lupus
anticoagulant (LA), anticardiolipin (ACL) are associated with clotting without inflammation.
Hematologic Manifestation
•• Anemia of chronic disease occurs in most patients when lupus is active
•• Autoimmune hemolytic anemia seen
•• Coombs’ positive hemolysis occurs
•• Cold antibodies
•• Leukopenia (usually lymphopenia)
•• Mild thrombocytopenia is common
•• Pancytopenia can occur
•• Clinical manifestations of lupus anticoagulant and anticardiolipin include thrombocytopenia, recurrent venous or arterial
clotting, recurrent fetal loss, and valvular heart disease usually respond to glucocorticoids; clotting syndromes do not
Cardiopulmonary System
•• Pericarditis is the most frequent manifestation of cardiac lupus
•• Effusions can occur and occasionally lead to tamponade; constrictive pericarditis is rare
•• Myocarditis can cause arrhythmias, sudden death, and/or heart failure
•• Valvular insufficiency (usually aortic or mitral) can occur, with or without Libman-Sacks endocarditis
•• Myocardial infarcts usually result from degenerative disease, although they can result from vasculitis
•• Cavitation in lung
•• Pulmonary fibrosis
•• Pleurisy and pleural effusions are common manifestations of SLE
•• Shrinking lung syndrome
•• Lupus pneumonitis causes fever, dyspnea, and cough; X-rays show fleeting infiltrates and/or areas of plate-like atelectasis; this
syndrome responds to glucocorticoids. However, the most common cause of pulmonary infiltrates in patients with SLE is infection
•• Polyserositis
80 USMLE Step 2 CK Platinum Notes
Lab Tests
•• ANAs are the best screening test
•• Antibodies to double-stranded DNA (dsDNA) and to CM are relatively specific for SLE. High serum levels of ANAs and anti-
dsDNA and low levels of complement usually reflect disease activity, especially in patients with nephritis.
•• Very low levels of CH50 with normal levels of C3 seen
•• Hematologic abnormalities include anemia (usually normochromic normocytic but occasionally hemolytic), leukopenia,
lymphopenia, and thrombocytopenia
•• The Westergren erythrocyte sedimentation rate correlates with disease activity in some patients
•• Urinalysis should be performed and serum creatinine levels should be measured periodically in patients with SLE
•• With active nephritis, the urinalysis usually shows proteinuria, hematuria, and cellular or granular casts.
Drug-induced Lupus
•• Several drugs can cause a syndrome resembling SLE, including:
–– Procainamide
–– Hydralazine
–– Isoniazid
–– Chlorpromazine
–– D-penicillamine, practolol
–– Methyldopa, quinidine
–– Interferon A and
–– Possibly hydantoin, ethosuximide, and oral contraceptives
•• The syndrome is rare with all but procainamide, the most frequent offender, and hydralazine. There is genetic predisposition to
drug-induced lupus, partly determined by drug acetylation rates
•• Most common are systemic complaints and arthralgias; polyarthritis and pleuropericarditis
•• Renal and CNS involvement are rare
•• All patients have ANA and most have antibodies to histones
•• ‘Best marker’ is antibodies to histones.
PSS is about 4 times more common in women than men and is comparatively rare in children
•• The disease varies in severity and progression, ranging from generalized cutaneous thickening (PSS with diffuse scleroderma) with
rapidly progressive and often fatal visceral involvement, to a form distinguished by restricted skin involvement (often just the fingers
and face) and prolonged passage of time, often several decades, before full manifestation of characteristic internal manifestations
(CREST syndrome: Calcinosis, Raynaud’s phenomenon, Esophageal dysfunction, Sclerodactyly, Telangiectasia).
Medicine 81
•• Cardiorespiratory system: Lung fibrosis, with exertional dyspnea, its most prominent symptom, is associated early with an
impairment in gas exchange. Pleurisy and pericarditis with effusion may occur. Recent studies indicate generally indolent
progression of lung involvement, with substantial individual variability. Pulmonary hypertension may develop as a result of
long-standing interstitial and peribronchial fibrosis or intimal hyperplasia of small pulmonary arteries; the latter is associated with
the CREST syndrome. Cardiac arrhythmias, conduction disturbances, and other ECG abnormalities are common
•• The kidneys: Severe renal disease may occur as a consequence of intimal hyperplasia of interlobular and arcuate arteries, and
usually is heralded by the abrupt onset of accelerated or malignant hypertension
•• An antibody that reacts with centromeric protein (anticentromere antibody) is found in the serum of a high proportion of
patients with the CREST syndrome
•• Localized forms of scleroderma occur as circumscribed patches (morphea) or linear sclerosis of the integument and immediately
subjacent tissues without systemic involvement.
Sjogren’s Syndrome
•• Occurs as primary form/isolated disorder
•• Secondary form with other CT disorders
•• Sjogren’s syndrome is a chronic, slowly progressive autoimmune disease characterized by lymphocytic infiltration of the
exocrine glands resulting in xerostomia and dry eyes
•• A small but significant number of the patients may develop malignant lymphoma
•• The disease affects predominantly middle-aged women (female-to-male ratio 9:1)
•• Sera of patients with Sjogren’s syndrome often contain a number of autoantibodies directed against non-organ-specific antigens,
such as immunoglobulins (rheumatoid factors) and extractable nuclear and cytoplasmic antigens (Ro/SS-A, La/SS-B)
Lymphoma is a well-known manifestation of Sjogren’s syndrome that usually presents later in the illness. Persistent parotid gland
enlargement, lymphadenopathy, cutaneous vasculitis, peripheral neuropathy, lymphopenia, and cryoglobulinemia are manifestations
suggesting the development of lymphoma. Most lymphomas are extranodal, marginal zone B cell, and low grade. Salivary glands are
the most common site of involvement.
•• Dry eye or Keratitis Sicca or Aqueous deficiency dry eye occurs in Sjogrens syndrome
•• Dry mouth
•• Dry nose
82 USMLE Step 2 CK Platinum Notes
•• Dry eyes
•• Dry skin
•• Dry throat
•• Dry sex (vaginal involvement)
•• Involvement of other exocrine glands occurs less frequently and includes a decrease in mucous gland secretions of the upper and
lower respiratory tree, resulting in dry nose, throat, and trachea (xerotrachea), and diminished secretion of the exocrine glands of the
gastrointestinal tract, leading to esophageal mucosal atrophy, atrophic gastritis, and subclinical pancreatitis. Dyspareunia due to
dryness of the external genitalia and dry skin also may occur.
•• Renal failure is the most common cause of death in the rare patient who dies of Henoch-Schönlein purpura
•• Routine laboratory studies generally show a mild leukocytosis, a normal platelet count, and occasionally eosinophilia. Serum
complement components are normal, and IgA levels are elevated in about one-half of patients
•• Treatment: The prognosis of Henoch-Schönlein purpura is excellent. Most patients recover completely, and some do not
require therapy.
Differentiate
•• Gynecomastia Ectopic production of hCG is the most common cause of paraneoplastic gynecomastia; hepatoma or a germ
cell tumor with choriocarcinoma elements contains aromatase enzyme activity that converts circulating
androgens to estrogen. Other tumors rarely associated with ectopic gynecomastia include carcinoid tumors
of the bronchus, intestine, and small cell lung cancer
About 5% of men with testicular choriocarcinoma present with an enlarging breast mass.
•• Non-islet Can occur with large, slow-growing sarcomas, mesotheliomas, and hepatomas.
cell tumor
hypoglycemia
Hematologic Syndromes
•• Erythrocytosis Renal cell cancer, 10% of patients with hepatoma, and 15% of patients with cerebellar hemangioblastomas
have erythrocytosis
•• Granulocytosis Tumors and tumor cell lines from patients with lung, ovarian, and bladder cancers have been documented
to produce granulocyte colony-stimulating factor (G-CSF), granulocyte-macrophage colony-stimulating
factor (GM-CSF)
•• Thrombocytosis Lung and gastrointestinal cancers, 20% of patients with breast, endometrial, and ovarian cancers, and 10%
of patients with lymphoma
•• Eosinophilia Eosinophilia is present in 10% of patients with lymphoma, 3% of patients with lung cancer, and occasional
patients with cervical, gastrointestinal, renal, and breast cancer
•• Thrombophlebitis Lung, pancreatic, gastrointestinal, breast, ovarian, and genitourinary cancers, lymphomas, and brain tumors
Medical Oncology
Cytotoxic Mechanism of action Adverse effects
•• Vincristine Inhibits formation of microtubules Peripheral neuropathy (reversible)
•• Cisplatin Causes cross-linking in DNA Ototoxicity, peripheral neuropathy
•• Bleomycin Degrades preformed DNA Lung fibrosis
•• Doxorubicin Stabilizes DNA—topoisomerase II complex inhibits DNA Cardiomyopathy
and RNA synthesis
•• Methotrexate Inhibits purine synthesis Myelosuppression, mucositis
•• Cyclophosphamide Alkylating agent—causes cross-linking in DNA Hemorrhagic cystitis,
myelosuppression
Pulmonary Infiltrates
Patients with cancer may present with dyspnea associated with diffuse interstitial infiltrates on chest radiographs. Such infiltrates
may be due to progression of the underlying malignancy, treatment-related toxicities, infection, and/or unrelated diseases. The cause
may be multifactorial.
Typhlitis
Neutropenic enterocolitis (Typhlitis) is a necrosis of the cecum and adjacent colon that may complicate the treatment of acute
leukemia. The patient develops right lower quadrant abdominal pain, often with rebound tenderness and a tense, distended
abdomen, in a setting of fever and neutropenia
Watery diarrhea (often containing sloughed mucosa) and bacteremia are common, and bleeding may occur).
Hemorrhagic Cystitis
Hemorrhagic cystitis can develop in patients receiving cyclophosphamide and ifosfamide. Both drugs are metabolized to ‘Acrolein’
which is a strong chemical irritant that is excreted in the urine. Prolonged contact or high concentrations may lead to bladder irritation and
hemorrhage. Symptoms include gross hematuria, frequency, dysuria, burning, urgency, incontinence, and nocturia. Maintaining
a high rate of urine flow minimizes exposure. In addition, ‘2-Mercaptoethane sulfonate’ (Mesna) detoxifies the metabolites and can be
coadministered with the instigating drugs.
MUSCLE DISORDERS
Toxic Myopathies
Drugs and chemicals may produce focal or generalized damage to skeletal muscle
•• The most common cause of focal damage is the injection of narcotic analgesics. Three agents in particular: pentazocine,
meperidine, and heroin
•• D-Penicillamine induces a condition simulating the clinical and pathologic picture of polymyositis. A similar condition has been
reported with cimetidine
•• Procainamide may cause myositis as part of a systemic lupus erythematosus-like reaction
•• Chloroquine administration may cause a vacuolar myopathy
•• Zidovudine, used in the treatment of AIDS, produces proximal weakness and pain
•• Fibric acid derivatives (clofibrate, gemfibrozil), 3-hydroxy-methyl-glutaryl-coenzyme A reductase inhibitors (lovastatin,
simivastatin, pravastatin), and niacin have all been implicated in myopathies, occasionally
•• Emetine hydrochloride (used for treatment of amebiasis).
86 USMLE Step 2 CK Platinum Notes
Polymyositis
PM is a subacute inflammatory myopathy affecting adults, and rarely children, who do not have any of the following:
•• No rash
•• No involvement of the extraocular and facial muscles
•• No family history of a neuromuscular disease
•• No cutaneous manifestations
•• No history of exposure to myotoxic drugs or toxins
•• No endocrinopathy
•• No neurogenic disease
•• No muscular dystrophy
No biochemical muscle disorder (deficiency of a muscle enzyme), or PM may occur either in isolation, in association with a systemic
autoimmune or connective tissue disease, or with known viral or bacterial infection
Proximal limb muscles are involved
Pharyngeal muscles are involved
Pain is a feature
Focal necrosis on biopsy is seen
CPK is increased
Congenital myopathies have been named by their characteristic findings on muscle biopsy:
Central core disease
Centronuclear myopathy
Nemaline myopathy
These children present with delayed walking and mild proximal muscle weakness that is not progressive
The biochemical defect is unknown
Pattern of inheritance
•• Diagnosis is suggested by a modest elevation of the serum potassium level during attacks in nearly half of patients; at times,
however, the serum potassium level is normal or even low. The so-called hyperkalemic and normokalemic forms of this disorder
are not separate entities
•• Paramyotonia Congenita Paramyotonia congenita (PC) causes attacks of paralysis either spontaneously or with cold provocation.
PC with periodic paralysis is similar to hyperKPP, except that paradoxical myotonia (i.e. myotonia worsening with activity) and
objective cold sensitivity are more prominent in PC
•• Potassium-aggravated Myotonia Some patients with muscle sodium channel defects have severe muscle stiffness but no
paralytic episodes. The stiffness is accentuated by elevations in serum potassium levels.
GIT
•• Migratory glossitis Geographic tongue
•• Atrophic glossitis Iron deficiency
•• Leutic glossitis Syphilis
Esophagitis
Most common causes are:
–– Reflux esophagitis (mc)
–– Infectious esophagitis
–– Eosinophilic esophagitis
–– Drug-induced esophagitis
–– Radiation esophagitis
–– Corrosive esophagitis
Plummer-Vinson Syndrome
•• Also called the Paterson-Brown-Kelly syndrome or sideropenic dysphagia
•• Describes young women with koilonychias, iron-deficiency anemia and dysphagia referred high in the neck
–– Commonest type of esophageal carcinoma: Squamous cell carcinoma
–– Commonest site of squamous cell carcinoma: Lower end of esophagus
–– Commonest feature of achalasia cardia: Dysphagia
–– Commonest feature of cancer esophagus: Progressive dysphagia
–– Difficulty in swallowing: Dysphagia
–– Pain during swallowing: Odynophagia
The metaplasia of esophageal squamous epithelium to columnar epithelium (Barrett’s esophagus) is a complication of severe
reflux esophagitis, and it is a risk factor for esophageal adenocarcinoma
Barrett’s epithelium progresses through a dysplastic stage before developing into adenocarcinoma.
•• Multiple sacculations and diverticulae with corckscrew esophagus: diffuse esophageal spasm
•• Rat tail esophagus with dilated proximal and narrow distal end: achalasia
•• Narrow and irregular esophageal lume: esophageal cancer
•• Stricture/ulcer in esophagus: esophageal reflux—GERD
Gastritis
The term gastritis should be reserved for histologically documented inflammation of the gastric mucosa
•• Acute Gastritis: The most common causes of acute gastritis are infectious. Acute infection with H. pylori induces gastritis
•• Chronic Gastritis is identified histologically by an inflammatory cell infiltrate consisting primarily of lymphocytes and plasma
cells, with very scant neutrophil involvement. Chronic gastritis is also classified according to the predominant site of involvement
•• Type A refers to the body-predominant form (autoimmune) and
•• Type B is the central-predominant form (H. pylori-related)
•• The AB gastritis has been used to refer to a mixed antral/body picture
Medicine 91
•• Lymphocytic gastritis is characterized histologically by intense infiltration of the surface epithelium with lymphocytes. The
infiltrative process is primarily in the body of the stomach and consists of mature T cells and plasmacytes
•• A subgroup of patients has thickened folds noted on endoscopy. These folds are often capped by small nodules that contain a
central depression or erosion; this form of the disease is called Varioliform gastritis
•• Marked eosinophilic infiltration involving any layer of the stomach (mucosa, muscularis propria, and serosa) is characteristic of
Eosinophilic gastritis
•• Several systemic disorders may be associated with Granulomatous gastritis. Gastric involvement has been observed in Crohn’s
disease. Involvement may range from granulomatous infiltrates noted only on gastric biopsies to frank ulceration and stricture
formation. Gastric Crohn’s disease usually occurs in the presence of small intestinal disease. Several rare infectious processes can
lead to granulomatous gastritis, including histoplasmosis, candidiasis, syphilis, and tuberculosis.
Menetrier’s Disease
•• Menetrier’s disease characterized by large, tortuous gastric mucosal folds
•• The mucosal folds in Menetrier’s disease are often most prominent in the body and fundus
•• Histologically, massive foveolar hyperplasia (hyperplasia of surface and glandular mucous cells) is noted, which replaces most of
the chief and parietal cells
•• Protein-losing enteropathy is common
•• Gastric acid secretion is not increased
•• Premalignant: Increased risk of adenocarcinoma
•• Hyperplasia and hypertrophy produce the prominent folds observed
•• The pits of the gastric glands elongate and may become extremely tortuous
•• Although the lamina propria may contain a mild chronic inflammatory infiltrate, Menetrier’s disease is not considered a form of
gastritis.
Schindler’s Disease
•• Acid hypersecretion
•• Giant rugae
•• Normal protein balance
H. pylori
H. pylori accounts for most of the peptic ulcer cases
More prevalent in developing countries
•• It is a gram-negative bacillus
•• Microaerophilic
•• Urease-producing
•• pH-gated urea channel in H. pylori bacterium is called Urel
Causes
•• Gastritis
•• Gastric Ca
•• Gastric lymphoma
•• Peptic ulcer disease
•• Iron-deficiency anemia
•• Hepatocellular carcinoma
•• Cholangiocarcinoma
•• Inflammatory Bowel diseases
•• Raynaud’s Phenomenon
GI BLEEDING: CAUSES
ABCDEFGHI
•• Angiodysplasia
•• Bowel cancer
•• Colitis
•• Diverticulitis/Duodenal ulcer
•• Epistaxis/Esophageal (cancer, esophagitis, varices)
•• Fistula (anal, aortoenteric)
•• Gastric (cancer, ulcer, gastritis)
•• Hemorrhoids
•• Infectious diarrhea/IBD/Ischemic bowel
Medicine 93
•• Hematemesis indicates an upper GI source of bleeding (above the ligament of Treitz). Melena indicates that blood has been
present in the GI tract for at least 14 h. Thus, the more proximal the bleeding site, the more likely melena will occur
•• Hematochezia usually represents a lower GI source of bleeding, although an upper GI lesion may bleed so rapidly that blood
does not remain in the bowel long enough for melena to develop
•• GIB of obscure origin. Obscure GIB is defined as recurrent acute or chronic bleeding for which no source has been identified
by routine endoscopic and contrast studies. Push enteroscopy, with a specially designed enteroscope or a pediatric colonoscope to
inspect the entire duodenum and part of the jejunum, is generally the next step. Push enteroscopy may identify probable bleeding
sites in 20 to 40% of patients with obscure GIB. If enteroscopy is negative or unavailable, a specialized radiographic examination of the
small bowel (e.g. enteroclysis) should be performed
•• Occult GIB is manifested by either a positive test for fecal occult blood or iron-deficiency anemia
•• Peptic ulcers are the most common cause of UGIB, accounting for about 50% of cases. Mallory-Weiss tears account for 5 to 15%
of cases. Hemorrhagic or erosive gastropathy [e.g. due to nonsteroidal anti-inflammatory drugs (NSAIDs) or alcohol] and erosive
esophagitis often cause mild UGIB, but major bleeding is rare.
MALABSORPTION
Tropical Sprue
Tropical sprue: Malabsorptive disease in tropical regions with unknown etiology
Similar features to celiac sprue and responds to antibiotics, Vitamin B12, Folate.
Whipple’s Disease
Whipple’s disease: Malabsorptive disease caused by Tropheryma whippeli
•• PAS positive, rod-shaped bacilli fill lamina propria, Tropheryma whipelli
•• Biopsy is specific test
•• Malabsorption, fever, lymphadenopathy, arthralgia
•• CNS involvement with dementia, seizures, coma, myoclonus
•• Treatment is with antibiotics
•• Trimethoprim sulfamethaxozole
Celiac Disease
Celiac disease: It is caused by sensitivity to the protein gluten. Repeated exposure leads to villous atrophy which, in turn, causes
malabsorption. Conditions associated with celiac disease include dermatitis herpetiformis (a vesicular, pruritic skin eruption) and
autoimmune disorders
•• Type 1 diabetes mellitus
•• Autoimmune hepatitis
•• Myasthenia gravis
•• Sjogren’s syndrome
•• Lymphomas, Addison’s, Grave’s disease
•• Diagnosis is made by a combination of immunology and jejunal biopsy. Villous atrophy and immunology should reverse
on a gluten-free diet. Immunology:
–– Anti-endomyseal antibody
–– Alpha-gliadin antibody
–– TTG (tissue transglutinamise) antibodies
–– Anti-casein antibodies are also found in some patients
•• Jejunal biopsy shows:
–– Villous atrophy
–– Crypt hyperplasia
–– Increase in intraepithelial lymphocytes
–– Lamina propria infiltration with lymphocytes
–– Thickness of mucosa unaltered.
In Nutshell
•• Celiac sprue is gluten-sensitive enteropathy or nontropical sprue
•• Hypersensitivity to gluten/gliadin occurs with loss of villi and malabsorption
•• RICE is safe to use
•• Genetic predisposition: HLA B8, DR 3 and DQ
•• Pathology: Loss of villi, increased intraepithelial lymphocytes
•• Increased plasma cells in lamina propria
•• Presentation: Malabsorption, abdominal distension, bloating, diarrhea, steatorrhea, weight loss
•• Association with dermatitis herpetiformis.
Bloody diarrhea
•• Salmonella
•• Shigella
•• Yersinia
•• EIEC
•• Campylobacter jejuni
Medicine 95
Verner-Morrison Syndrome
WDHA Syndrome
Pancreatic cholera is due to increased secretion of VIP, GIP
Delta cell tumor causes pancreatic cholera which is not an infective condition but simulates cholera due to increased secretion of
water and salt
Diarrhea, weight loss and flushing occur
Electrolyte abnormalities include:
•• Hypokalemia
•• Hypochlorrhydia
•• Hyperglycemia
•• ‘Hypercalcemia’ and not Hypocalcemia
VIRAL HEPATITIS
Incubation period
•• Hepatitis A – incubation period 2–6 weeks
•• Hepatitis B4 – 8 weeks
•• Hepatitis C2 – 22 weeks
•• Hepatitis D4 – 8 weeks
•• Hepatitis E2 – 9 weeks
HDV
•• HDV is ‘defective’ because it does not have genes for its proteins
•• HDV can replicate in cells only infected with HBV
•• HDV uses surface antigen of HBV (HBsAg) as its envelop protein
•• HBV is a helper virus for HDV
Fitz-Hugh-Curtis Syndrome
•• Fitz-Hugh-Curtis syndrome is a condition in which bacteria, usually from a pelvic infection, spread through the abdomen and
cause inflammation of the tissue surrounding the liver. It occurs in 15–30% of women with pelvic inflammatory disease (PID),
but may also occur in women without PID, and in men
•• In Fitz-Hugh-Curtis syndrome, the inflammation of the liver tissue leads to the (adhesions) between the outside of the liver and the
inside of the abdomen. In some individuals, these adhesions cause no symptoms. Others have severe pain in the upper right part
of the abdomen, over the gallbladder. The pain may move to the right shoulder. Sometimes the pain increases with coughing,
sneezing, or movement. Since the source of Fitz-Hugh-Curtis syndrome is most often a pelvic infection, symptoms such as nausea,
vomiting, chills, fever, and headaches may be present
•• Diagnosis: The presence of a pelvic infection would also provide a clue to the diagnosis, but without PID, the diagnosis may be
difficult, since many conditions can cause abdominal pain. In women, cervical cultures for two particular pelvic infections
need to be done. Chlamydia and gonorrhea are the most common causes of Fitz-Hugh-Curtis syndrome. If infection is present,
the white blood cell (WBC) count in the blood will be high, as will the erythrocyte sedimentation rate (ESR)
•• Laparoscopy confirms diagnosis: The physician can see the adhesions on the outside of the liver, which have a typical stringy
look (called ‘violin-string’ adhesions).
98 USMLE Step 2 CK Platinum Notes
Wilson’s Disease
•• It is an AR disorder
•• Chromosome 13 is involved
•• The diagnosis is confirmed by the demonstration of either:
–– A serum ceruloplasmin level < 20 mg/dl and Kayser-Fleischer rings or
–– A serum ceruloplasmin level < 20 mg/dl and a concentration of copper in a liver biopsy sample > 250 mg/g dry weight or
–– Most symptomatic patients excrete >100 mg copper per day in urine and have histologic abnormalities on liver biopsy
Pathognomonic features are:
•• Hepatic dysfunction: Jaundice, Hepatomegaly, Cirrhosis, Portal hypertension, Hepatitis occur
•• Eye Changes: Brownish or grayish Kayser-Fleisher ring in Descement’s membrane in cornea
•• These golden deposits of copper in Descemet’s membrane of the cornea do not interfere with vision but indicate that copper has
been released from the liver and has probably caused brain damage
•• If a patient with frank neurologic or psychiatric disease does not have Kayser-Fleischer rings when examined by a trained observer
using a slit lamp, the diagnosis of Wilson’s disease can be excluded
•• Rarely, Kayser-Fleischer rings may be accompanied by sunflower cataracts
•• The neurologic manifestations include resting and intention tremors, spasticity, rigidity, chorea, drooling, dysphagia, and
dysarthria. Babinski responses may be present, and abdominal reflexes are often absent. Sensory changes never occur, except
for headache
•• Psychiatric disturbances are present in most patients with neurologic symptoms. Schizophrenia, manic-depressive psychoses,
and classic neuroses may occur, but the commonest disturbances are bizarre behavioral patterns that defy classification
Treatment consists of removing and detoxifying the deposits of copper as rapidly as possible and must be instituted once the diagnosis
is secure, whether the patient is ill or asymptomatic
•• Zinc is the DOC
•• Penicillamine is administered orally in an initial dose of 1 g daily in single or divided doses at least 30 min before and 2 h after
eating
•• The dose of Trientine is 1 g/d on an empty stomach is more potent than pencillamine and used in patients not tolerating
pencillamine
Zinc acetate or gluconate are effective as maintenance therapy, at doses of 150 mg/d of elemental zinc, for patients who are
asymptomatic or have improved maximally on penicillamine or trientine.
Medicine 99
Hemochromatosis
•• It is a common disorder of iron storage
•• In which there is an appropriate increase in intestinal iron absorption
•• Results in deposition of excessive amounts of iron in parenchymal cells with eventual tissue damage and
•• Impaired function of organs, especially the liver, pancreas, heart, joints, and pituitary
–– Hereditary or genetic hemochromatosis: This disorder is most often caused by inheritance of a mutant HFE gene, which is
tightly linked to the HLA-A locus on chromosome 6p
–– Autosomal dominant
–– Secondary iron overload: Tissue injury usually occurs secondary to an iron-loading anemia such as thalassemia or
sideroblastic anemia, in which increased erythropoiesis is ineffective. In the acquired iron-loading disorders, massive iron
deposits in parenchymal tissues can lead to the same clinical and pathologic features as in hemochromatosis
In hemochromatosis, mucosal absorption is inappropriate to body needs and amounts to 4 mg/d or more. The progressive
accumulation of iron causes an:
–– Early elevation in plasma iron
–– An increased saturation of transferrin, and
–– Progressive elevation of plasma ferritin level.
Clinical Manifestations
Initial symptoms include weakness, lassitude, weight loss, change in skin color, abdominal pain, loss of libido, and symptoms of diabetes
mellitus
The liver is usually the first organ to be affected, and hepatomegaly is present in more than 95% of symptomatic patients.
Hepatocellular carcinoma develops in about 30% of patients with cirrhosis, and it is the most common cause of death in
treated patients; hence the importance of early diagnosis and therapy. Its incidence increases with age, is more common in men, and
occurs almost exclusively in cirrhotic patients. Splenomegaly occurs in approximately half of symptomatic cases
Excessive skin pigmentation is present in over 90% of symptomatic patients at the time of diagnosis. The characteristic metallic or
slate gray hue is sometimes referred to as bronzing and results from increased melanin and iron in the dermis. Pigmentation usually is
diffuse and generalized, but it may be more pronounced on the face, neck, extensor aspects of the lower forearms, dorsa of the hands,
lower legs, genital regions, and in scars
Bronze color
Diabetes mellitus is more likely to develop in those with a family history of diabetes, suggesting that direct damage to the pancreatic
islets by iron deposition occurs in combination with a genetic predisposition
Bronze Diabetes
Arthropathy develops in 25 to 50% of patients. It usually occurs after age 50, but may occur as a first manifestation, or long after
therapy. The joints of the hands, especially the second and third metacarpophalangeal joints, which are usually the first joints
involved may ensue
Acute brief attacks of synovitis may be associated with deposition of calcium pyrophosphate (chondrocalcinosis or pseudogout),
mainly in the knees.
100 USMLE Step 2 CK Platinum Notes
Cardiac involvement is the presenting manifestation in about 15% of patients. The most common manifestation is congestive heart
failure, which occurs in about 10% of young adults with the disease, especially those with juvenile hemochromatosis
Hypogonadism occurs in both sexes and may antedate other clinical features. Manifestations include loss of libido, impotence,
amenorrhea, testicular atrophy, gynecomastia, and sparse body hair. These changes are primarily the result of decreased
production of gonadotropins due to impairment of hypothalamic-pituitary function by iron deposition; however, primary testicular
dysfunction may be seen in some cases. Adrenal insufficiency, hypothyroidism, and hypoparathyroidism may also occur.
Diagnosis
The association of:
1. Hepatomegaly
2. Skin pigmentation
3. Diabetes mellitus
4. Heart disease
5. Arthritis and
6. Hypogonadism should suggest the diagnosis
Transferrin saturation and iron-binding capacity are used as initial screening tests
Treatment: Venesection, Phlebotomy (TOC) and desferroxamine.
Carcinoid Syndrome
•• Kulchitsky Type I (typical carcinoid)
•• Kulchitsky Type II (atypical carcinoid)
•• Kulchitsky Type III (small cell lung cancer)
•• ‘Carcinoid syndrome’ occurs in less than 10% of patients with carcinoid tumors
The carcinoid syndrome is encountered when venous drainage from the tumor gains access to the systemic circulation so that
vasoactive secretory substances escape hepatic degradation
This situation obtains in three circumstances:
1. When hepatic metastases are present
2. When venous blood from extensive retroperitoneal metastases drains into paravertebral veins, and
3. When the primary carcinoid tumor is outside the gastrointestinal tract, e.g. a bronchial, ovarian, or testicular tumor
Hepatorenal Syndrome
Definition and Pathogenesis Hepatorenal syndrome is a serious complication in the patient with cirrhosis and ascites and is
characterized by worsening azotemia with avid sodium retention and oliguria in the absence of identifiable specific causes of
renal dysfunction
•• The kidneys are structurally intact
•• Urinalysis and pyelography are usually normal
•• Renal biopsy is also normal, and
•• Kidneys from such patients have been used successfully for renal transplantation
Clinical features and diagnosis:
•• Worsening azotemia
•• Hyponatremia
•• Progressive oliguria
•• Hypotensions are the hallmarks of the hepatorenal syndrome
This syndrome, which is distinct from prerenal azotemia, may be precipitated by severe gastrointestinal bleeding, sepsis, or overly
vigorous attempts at diuresis or paracentesis; it may also occur without an obvious cause.
Remember
•• Commonest cause of lower GI bleed is: Hemorrhoid
•• BUT commonest cause of massive bleed per rectum in elderly is diverticulosis
•• Hypergastrenemia with hypochlorrydia is a feature of pernicious anemia
•• Clostridium difficile has been associated with Crohn’s disease.
RENAL SYSTEM
Significance of Casts
RBC Casts: Glomerulonephritis
Dysmorphic RBC: Glomerular pathology
WBC Casts: Pyelonephritis
Hyaline Casts: Normal, ARF
Coarse granular casts: ATN
Waxy casts: CRF
Tom Harsfall protein: normal, tubular in origin
Causes of CRF:
•• Membranous GN
•• Membranoproliferative GN
•• FSGN
Diabetes Mellitus
Epidemiology IDDM NIDDM
Age at onset Younger, lean patients Older, obese patients
Heredity Usually < 25 years Usually > 40 years
HLA-DR 3 and/or DR 4 (95%) No HLA links
30–50% concordance in identical twins 90% concordance in identical twins
Autoimmunity Presence of islet cell antibody, Insulin autoantibody. Association with None
other organ-specific autoimmune diseases
Insulin deficiency
May develop ketoacidosis
Insulin
Clinical Partial insulin deficiency, Insulin
treatment resistance
May develop nonketotic
hyperosmolar state
Weight loss, drugs, insulin sometimes
Remember
•• Glucose levels rise in the early morning (Dawn phenomenon)
•• Rebound hyperglycemia may appear from 1/2 to 24 hours after moderate to severe hypoglycemia (Somogyi phenomenon)
•• Syndrome X is a term used to describe a constellation of metabolic derangements that includes insulin resistance, hypertension,
dyslipidemia, central or visceral obesity, endothelial dysfunction, and accelerated cardiovascular disease
•• Chronic complications can be divided into vascular and nonvascular complications. The vascular complications of DM are further
subdivided into:
–– Microvascular (retinopathy, neuropathy, nephropathy) and
–– Macrovascular complications (coronary artery disease, peripheral vascular disease, cerebrovascular disease)
–– Nonvascular complications include problems such as gastroparesis, sexual dysfunction, and skin changes.
104 USMLE Step 2 CK Platinum Notes
DRUGS IN DIABETES
•• Α-Adrenergic blockers slightly improve insulin resistance and positively impact the lipid profile, whereas beta blockers and
thiazide diuretics can increase insulin resistance
•• Beta blockers cause masking of hypoglycemic symptoms
•• Central adrenergic antagonists and vasodilators are lipid- and glucose-neutral
NONALCOHOLIC STEATOHEPATITIS
Nonalcoholic steatohepatitis (NASH) is a term used to describe liver changes similar to those seen in alcoholic hepatitis in the absence
of a history of alcohol abuse. Associated factors:
•• Obesity
•• Hyperlipidemia
•• Type 2 diabetes mellitus
•• Jejunoileal bypass
•• Sudden weight loss/starvation
Features: Usually asymptomatic, hepatomegaly, AST > ALT
MODY
Maturity-onset diabetes of the young (MODY) is characterized by the development of type 2 diabetes mellitus in patients <25 years
old. It is typically inherited as an autosomal dominant condition. Over six different genetic mutations have so far been identified as
leading to MODY. Ketosis is not a feature at presentation
MODY 2: 20% of cases, due to a defect in the glucokinase gene
MODY 3: 60% of cases, due to a defect in the HNF-1 alpha gene
Diabetic Nephropathy
Diabetic nephropathy is a clinical syndrome characterized by:
•• Persistent albuminuria (> 300 mg/d or > 200 mcg/min) that is confirmed on at least 2 occasions 3–6 months apart (early test)
•• A relentless decline in the glomerular filtration rate (GFR)
•• Elevated arterial blood pressure
•• HbA1C is used to monitor diabetic control
•• As a result of injury to the glomerular capillary wall, urinalysis typically reveals red blood cell casts, dysmorphic red blood cells,
leukocytes, and subnephrotic proteinuria of < 3.5 g per 24 h (nephritic urinary sediment)
•• Hematuria is often macroscopic.
Kidney in HIV
Aggressive form of ‘focal segmental glomerulosclerosis’ (FSGS) characterizes HIV infection along with microcystic tubular dilatation
and interstitial fibrosis.
Medicine 107
Wegner’s Granulomatosis
•• Necrotizing vasculitis of small arteries and veins together with granuloma formation that can be either intravascular or
extravascular
•• Lung involvement: Bilateral nodular cavitary infiltrates demonstrate necrotizing granulomatous vasculitis
•• The renal biopsy lesion is that of a pauci-immune necrotizing and crescentic GN.
Goodpasture’s Syndrome
•• Autoimmune disease in which autoantibodies are directed against type IV collagen
•• The clinical complex of anti-GBM nephritis and lung hemorrhage is referred to as Goodpasture’s syndrome. Patients with
Goodpasture’s syndrome are typically young males (5 to 40 years; male-female ratio of 6:1)
•• The target antigen is a component of the noncollagenous (NCI) domain of the α3 chain of type IV collagen, the α3 chain being
preferentially expressed in glomerular and pulmonary alveolar basement membrane
Anti-GBM disease commonly presents with:
•• Hematuria
•• Nephritic urinary sediment, subnephrotic proteinuria and
•• Rapidly progressive renal failure over weeks, with or without pulmonary hemorrhage
•• About 20% of patients have low titers of ANCA, usually a perinuclear (p ANCA)
•• Renal biopsy is the gold standard for diagnosis of anti-GBM nephritis
•• The typical morphologic pattern on light microscopy is diffuse proliferative glomerulonephritis, with focal necrotizing lesions and
crescents in >50% of glomeruli (crescentic glomerulonephritis)
•• Immunofluorescence microscopy reveals linear ribbon-like deposition of IgG along the GBM
•• C3 is present in the same distribution in 70% of patients. Prominent IgG deposition along the tubule basement membrane and
tubulointerstitial inflammation is found occasionally
•• Electron microscopy reveals nonspecific inflammatory changes without immune deposits. Typical features on lung biopsy include
alveolar hemorrhage.
Rhabdomyolysis
Causes:
•• Crush injury
•• Burns
•• Heroin, barbiturates
•• Cocaine, amphetamines
•• Hypokalemia
•• Hypophosphatemia
•• Hypothyroidism
•• Diabetic ketoacidosis
•• Clostridium, streptococcus
Liddle’s Syndrome
Liddle’s syndrome is a rare familial disease with a clinical presentation of ‘hyperaldosteronism’, consisting of:
•• Hypertension
•• Hypokalemia and
•• Metabolic alkalosis
However, aldosterone levels are undetectable in these patients, and a nonaldosterone mineralocorticoid has not been isolated
Increased distal tubule sodium reabsorption, due to activating mutations in the amiloride-sensitive sodium channel, has been
described in multiple families
Pharmacologic agents that block distal tubule sodium uptake, such as amiloride and triamterene, are effective in treating the
hypertension and electrolyte abnormalities.
Differentiating features
Bartter’s Syndrome
Antenatal Bartter’s syndrome is characterized by polyhydramnios and premature delivery
The infants also have a characteristic facies consisting of a triangular face with prominent eyes and ears. Prostaglandin E production
is very high
Characteristic of Bartter’s syndrome:
•• Hypokalemia
Medicine 109
Pheochromocytomas
Rule of ‘10’ in pheochromocytomas:
•• 10% malignant*
•• 10% bilateral*
•• 10% extra-adrenal*
•• 10% extra-abdominal*
•• 10% occur in children
•• 10% familial
•• 10% not associated with hypertension*
•• Most common sign in pheochromocytoma = Hypertension*
•• Commonest cause of paroxysmal hypertension
Syndromes associated with pheochromocytoma:
•• MEN type I (Sipple’s syndrome)
•• MEN type II (Mucosal neuroma syndrome)
•• von Hippel–Lindau’s syndrome
•• von Recklinghausen’s neurofibromatosis
110 USMLE Step 2 CK Platinum Notes
Tuberous Sclerosis
Patients with this multisystem disease most commonly present with skin lesions and benign tumors of the central nervous system
Renal involvement is common
Angiomyolipomas are the most frequent abnormality and are usually bilateral
Renal cysts may be present as well and can give an appearance similar to that of ADPKD
Histologically, the cysts are unique. The cyst-lining cells are large with an eosinophilic-staining cytoplasm and may form
hyperplastic nodules that can fill the cyst space.
Diabetes Insipidus
Causes of cranial DI:
•• Idiopathic
•• Post-head injury
•• Pituitary surgery
•• Craniopharyngiomas
•• Histiocytosis X
DIDMOAD is the association of cranial Diabetes Insipidus, Diabetes Mellitus, Optic Atrophy and Deafness (also known as Wolfram’s
syndrome)
Causes of nephrogenic DI:
•• Genetic (primary)
•• Electrolytes: hypercalcemia, hypokalemia
•• Drugs: demeclocycline, lithium
•• Tubulointerstitial disease: obstruction, sickle-cell, pyelonephritis
Investigation
•• High-plasma osmolarity, low-urine osmolarity, water deprivation test
3. Liddle’s Syndrome
4. Addison’s disease
5. Diabetes insipidus
6. Diabetes mellitus
7. Fanconi syndrome
Ans. 5. Diabetes lnsipidus
It is characterized by the excretion of abnormally large volumes of dilute urine (polyuria) with a commensurate increase in fluid
intake (polydipsia). The most common type is due to inadequate secretion of antidiuretic hormone (also called vasopressin) and is
usually referred to as ‘neurogenic’ diabetes insipidus. This condition rarely causes severe problems as long as the person has plenty of
water to drink. Placing the patient on overnight water restriction caused severe dehydration and a greatly elevated plasma sodium
concentration. The possibility of diabetes mellitus which can also be associated with polyuria and polydipsia is easily excluded by the
lack of glucosuria.
Dialysis
Commonly accepted criteria for putting patients on dialysis include:
•• The presence of uremic syndrome
•• The presence of hyperkalemia unresponsive to conservative measures
•• Extracellular volume expansion
•• Acidosis refractory to medical therapy
•• A bleeding diathesis; and a creatinine clearance of < 10 cc/min per 1.73 m2
•• In chronic renal failure (ESRD), the options include:
–– Hemodialysis (in center or at home)
–– Peritoneal dialysis, either as continuous ambulatory peritoneal dialysis (CAPD) or
–– Continuous cyclic peritoneal dialysis (CCPD)
•• Hemodialysis is the most common therapeutic modality
•• Peritoneal dialysis is favored in younger patients.
Cushing’s Syndrome
ACTH-dependent causes
•• Cushing’s disease (80%): pituitary tumor secreting ACTH producing adrenal hyperplasia
•• Ectopic ACTH production (5–10%): e.g. small cell lung cancer
ACTH-Independent causes
•• Iatrogenic: steroids
•• Adrenal adenoma (5–10%)
114 USMLE Step 2 CK Platinum Notes
Tests
•• The first test to perform is the dexamethasone suppression, which consists of administering 1 mg of dexamethasone at 11 PM,
and then measuring serum cortisol levels in a blood sample drawn at 8 AM the next day. Abnormally high cortisol levels after this
test confirm hypercortisolism
•• The next step is to find the source of excessive cortisol or ACTH
Baseline plasma ACTH measurement should be performed after a diagnosis:
•• If hypercortisolism has been confirmed by dexamethasone suppression test. This test is used to determine whether excessive
cortisol production is secondary to increased ACTH levels
•• CT scans of the chest and abdomen are performed to look for ectopic
•• Sources of ACTH. These radiologic studies are especially useful in finding neoplasms that may manifest with inappropriate ACTH
secretion, the most frequent of which is small cell carcinoma of the lungs.
Cushings disease
Difference
Medicine 115
Thyroid Endocrinology
Wolff-Chaikoff Effect
•• Excess iodide transiently inhibits thyroid iodide organification, a phenomenon known as the Wolff-Chaikoff effect
Sick euthyroid syndrome
•• Any acute, severe illness can cause abnormalities of circulating TSH or thyroid hormone levels in the absence of underlying thyroid
disease, making these measurements potentially misleading
•• The major cause of these hormonal changes is the release of cytokines. Unless a thyroid disorder is strongly suspected, the routine
testing of thyroid function should be avoided in acutely ill patients
The most common hormone pattern in sick euthyroid syndrome (SES) is a decrease in total and free T3 levels
(Low T3 syndrome) with normal levels of T4 and TSH
HYPERTHYROIDISM:
•• Wide pulse pressure
•• AF
•• Tachycardia
•• Cardiomyopathy
•• Diarrhea
•• Heat intolerance
•• Improper tissue handling in surgeries of thyroid can lead to thyroid storm
Hormonal changes in Primary Thyrotoxicosis are:
•• TSH↓
•• T3 and T4↑
•• RAIU↑
Hormonal changes in Secondary Hyperthyroidism are:
•• TSH↑
•• T3 and T4↑
•• RAIU↑
Hypothyroidism
•• Hung-up reflexes
•• Carpal tunnel syndrome
•• Menorrhagia
•• Constipation
•• Subnormal body temperature
•• Tendency for↑ sleep
•• Cold intolerance
•• ↑TSH
•• Lithium, PAS, Amidarone implicated in causation
Commonest cause of congenital hypothyroidism is thyroid agenesis
↓ T3↓T4↑TSH are a feature of primary hypothyroidism
Hashimoto’s disease is autoimmune throiditis
Thyroxine is used for treatment
Severe cases lead to myxedemic coma characterized by:
•• Slow pulse
•• Hypotension
Treated with:
•• IV fluids
•• Hydrocortisone
•• T3 injections
Primary Hyperparathyroidism
Primary hyperparathyroidism is stereotypically seen in elderly females with an unquenchable thirst and an inappropriately normal or
raised parathyroid hormone level. It is most commonly due to a solitary adenoma
Causes of Primary Hyperparathyroidism:
•• 80%: solitary adenoma
•• 15%: hyperplasia
•• 4%: multiple adenoma
•• 1%: carcinoma
Features—‘bones, stones, abdominal groans and psychic moans’
•• Polydipsia, polyuria
•• Peptic ulceration/constipation/pancreatitis
•• Bone pain/fracture
Medicine 117
Hypoparathyroidism
Primary hypoparathyroidism
•• Decrease PTH secretion, e.g. secondary to thyroid surgery
•• Low calcium, high phosphate
•• Treat with alfacalcidol
Pseudohypoparathyroidism
•• Target cells being insensitive to PTH
•• Due to abnormality in a G protein
•• Associated with low IQ, short stature, shortened 4th and 5th metacarpals
•• Low calcium, high phosphate, high PTH
•• Diagnosis is made by measuring urinary cAMP and phosphate levels following an infusion of PTH. In hypoparathyroidism this will
cause an increase in both cAMP and phosphate levels. In pseudohypoparathyroidism type I, neither cAMP nor phosphate levels are
increased whilst in pseudohypoparathyroidism type II, only cAMP rises
Pseudo-pseudo-hypoparathyroidism
•• Similar phenotype to pseudohypoparathyroidism but normal biochemistry.
Conn’s Syndrome
•• Hyperaldosteronism
•• ↑Aldosterone↑renin
•• Hypertension
•• ↑sodium
•• ↓potassium
•• Polyuria, polydipsia
•• Weakness
•• Proximal myopathy
Addison’s Disease
•• Addison’s disease is primary adrenocortical insufficiency. Associated electrolyte abnormalities:
–– Hyperkalemia, hyponatremia
118 USMLE Step 2 CK Platinum Notes
–– Hypoglycemia
–– Metabolic acidosis
•• 90% gland must be destroyed to manifest the disease
•• Autoimmune causes are responsible for most cases
•• Clinical signs include: fatigability, weakness, anorexia, weight loss, cutaneous and mucosal pigmentation, hypotension and
hypoglycemia
•• Pigmentation is usually increased except in adrenal insufficiency secondary to pituitary failure. Increased pigmentation is
characterized by diffuse tanning of both exposed and nonexposed portions of the body, especially on pressure points (bony
prominences), skin folds, scars, and extensor surfaces
•• Black freckles over the forehead, face, neck, and shoulders; areas of vitiligo; and bluish-black discolorations of the areolae
and of the mucous membranes of the lips, mouth, rectum, and vagina are common
•• In a patient with suspected Addison’s disease, the definite investigation is a short ACTH test. Plasma cortisol is measured before
and 30 minutes after giving Synacthen 250 mg IM. Adrenal autoantibodies such as anti-21-hydroxylase may also be demonstrated
•• Busulfan causes Addisonian-like features.
Adrenal Insufficiency
A 44-year-old female from Ohio has unexplained fatigue, weight loss, and diffuse muscle pains over the last 4 months. She
has been using oral prednisone (10 mg/day) that was started early this year. Three months ago her breathing seemed to be
getting a lot better, so she decided to stop the prednisone. Her temperature is 37.0 °C (98.6 °F), blood pressure is 90/40 mm
Hg, pulse is 100/min, and respirations are 14/min. On physical examination, she has a tanned appearance with an otherwise
normal skin examination. Her heart is tachycardic and regular without murmurs. Her lungs have a few scattered wheezes.
Laboratory studies show: hypoglycemia, hyponatremia, and hyperkalemia.
Waterhouse-Friderichsen Syndrome
Fulminant, life-threatening meningitis caused by meningococcus is with a vasculitic purpura and disseminated intravascular
coagulation. Complicated by adrenal involvement with coagulopathy, hypotension, adrenal cortical necrosis, and sepsis is Waterhouse-
Friderichsen syndrome.
SIADH
CAUSED BY:
•• Carbamezapine
•• Vincristine
•• Chlorpropamide
•• Oxytocin
•• Head injury
•• Encephalitis
•• Oat cell carcinoma of lung
•• Hypothyroidism
•• Hyponatremia is a feature
Insulinomas
•• Insulinomas are endocrine tumors of the pancreas derived from β cells that autonomously secrete insulin, which results in
hypoglycemia
•• The most common clinical symptoms are due to the effect of the hypoglycemia on the central nervous system (neuroglycemic
symptoms) and include confusion, headache, disorientation, visual difficulties, irrational behavior, or even coma
•• Also, most patients have symptoms due to excess catecholamine release secondary to the hypoglycemia, including sweating,
tremor, and palpitations. Characteristically these attacks are associated with fasting.
•• Insulinomas are generally small (> 90% are < 2 cm in diameter)
•• Usually solitary (90%), and only 5 to 15% are malignant
•• They almost invariably occur only in the pancreas, distributed equally in the pancreatic head, body and tail
•• Insulinomas should be suspected in all patients with hypoglycemia, especially with a history suggesting attacks provoked by
fasting or with a family history of MEN-1
•• In insulinomas, in addition to elevated plasma insulin levels, elevated plasma proinsulin levels are found and
C-peptide levels can be elevated.
Glucagonomas
•• Glucagonomas are endocrine tumors of the pancreas that secrete excessive amounts of glucagon that causes a distinct
syndrome characterized by dermatitis, glucose intolerance or diabetes, and weight loss
•• Glucagonomas mainly occur in persons between 45 and 70 years old. They are heralded clinically by a characteristic dermatitis
(migratory necrolytic erythema; accompanied by glucose intolerance weight loss , anemia, diarrhea and thromboembolism
•• The characteristic rash usually starts as an annular erythema at intertriginous and periorificial sites, especially in the groin or
buttock. It subsequently becomes raised and bullae form; when the bullae rupture, eroded areas form. The lesions can wax and
wane.
120 USMLE Step 2 CK Platinum Notes
Somatostatinoma Syndrome
•• Somatostatinomas are endocrine tumors that secrete excessive amounts of somatostatin, which causes a syndrome
characterized by diabetes mellitus, gallbladder disease, diarrhea, and steatorrhea
•• Somatostatinomas occur primarily in the pancreas and small intestine, and the frequency of the symptoms differs in each
•• Somatostatinomas occur in the pancreas in 56 to 74% of cases, with the primary location being in the pancreatic head.
VIPomas
•• These are endocrine tumors that secrete excessive amounts of VIP, which causes a distinct syndrome characterized by large-
volume diarrhea, hypokalemia, and dehydration
•• This syndrome is also called Verner-Morrison syndrome, pancreatic cholera, or WDHA syndrome (watery diarrhea,
hypokalemia, and achlorhydria).
NEUROLOGY
Alzheimer’s Disease
•• Mc cause of dementia in elderly
•• Cortical Dementia not Subcortical Dementia
•• Causes brain atrophy in advanced cases
•• Atrophy usually involves frontal, temporal and parietal lobes
•• Neurofibrillary tangles, Neuritic plaques, Granulovacuolar degeneration, Hirano bodies are pathological features
•• Nucleus Basalis of Meynert is effected
–– Aphasia
–– Amnesia
–– Agnosia
–– Acalculia
–– Alexia are features
Medicine 121
Huntington’s Disease
•• Huntington’s disease (HD) is a autosomal dominant, degenerative brain disorder
•• The two clinical hallmarks of the disease are chorea and behavioral disturbance, the movement disorder is usually slowly
progressive and eventually may become disabling.
122 USMLE Step 2 CK Platinum Notes
•• There are frequent, irregular, sudden jerks and movements of any of the limbs or trunk. Grimacing, grunting, and poor articulation
of speech may be prominent
•• The gait is disjointed and poorly coordinated and has a so-called dancing (choreic) quality.
MS (Multiple Sclerosis)
•• Multiple scarred are as visible on macroscopic examination of the brain
–– These lesions, termed plaques, are sharply demarcated gray or pink areas easily distinguished from surrounding white matter
–– Plaques vary in size from 1 or 2 mm to several centimeters
–– CSF abnormalities consist of:
- Abnormally increased levels of intrathecally synthesized IgG, oligoclinal banding, and mononuclear cell pleocytosis
- The ratio of IgG to albumin in the CSF is divided by the ratio in the serum ‘the CSF IgG index’. Oligoclonal banding of CSF
IgG is detected by agarose gel electrophoresis techniques
•• Optic neuritis is the mc manifestation
•• Also seen are:
–– Nystagmus on adducting eye
–– One and half syndrome
–– Pendular nystagmus
•• Two or more oligoclonal bands are found in 75 to 90% of patients with MS. Three treatment options for patients with RRMS
are approved for use:
–– IFN-1b
–– IFN-1a (Avonex) and
–– Glatiramer acetate
Neuromyelitis optica (Devic’s syndrome) is characterized by separate attacks of acute optic neuritis and myelitis. Optic neuritis
may be unilateral or bilateral and precede or follow an attack of myelitis by days, months or years
Acute MS
(Marburg’s variant) is a rare acute fulminant process that generally ends in death from brainstem involvement within one
year. There are no remissions. Diagnosis can be established only at postmortem examination; widespread demyelination, axonal loss,
edema, and macrophage infiltration are characteristic.
Learn to Differentiate
•• Normal pressure hydrocephalus is a potentially reversible cause of dementia that causes gait disturbances (unsteady or
shuffling gait), urinary incontinence, and dementia. Enlargement of the ventricles with increased cerebrospinal fluid (CSF)
pressure is found, and therapeutic lumbar punctures may significantly improve symptoms.
•• Creutzfeldt-Jakob disease is a rare diffuse degenerative disease that usually affects people in their 50s, and the usual course
is about one year. The terminal stage is characterized by severe dementia, generalized hypertonicity, and profound speech
disturbances. It is one of the several diseases caused by prions.
•• Huntington’s disease is a hereditary disease associated with progressive degeneration of the basal ganglia and the cerebral
cortex. It is transmitted in an autosomal dominant pattern. The onset of Huntington’s disease occurs between 35 and 50 years
of age, or later in rare cases. This disease is characterized by progressive dementia, muscular hypertonicity, and bizarre
choreiform movements.
•• Parkinson’s disease is characterized primarily by motor dysfunction, but dementia may be a part of the disorder. The
characteristic motor symptoms (bradykinesia, flat facies, resting tremor, shuffling gait, etc. are caused by degeneration of
the nigrostriatal dopaminergic tract.
Medicine 123
•• Pick’s disease causes a slowly progressive dementia. It is associated with focal cortical lesions, primarily of the frontal lobe.
Pathological examination of the brain reveals intraneuronal inclusions called Pick bodies.
•• Tumor of the Falx or the parasagittal meninges typically produces sensory or motor dysfunction of the leg.
•• Tumor of hippocampus might produce memory problems or seizures.
•• Tumor of the posterior fossa produces unilateral deafness, tinnitus, vertigo, and sometimes sensory loss in the distribution
of cranial nerves V or VII.
•• Tumor of the sphenoidal ridge can cause cranial nerve palsies involving III, IV, V, or VI.
Bulbar palsy
•• B/L LMNL of 9,10,11,12 Cranial Nerves
Features:
•• Dysarthria
•• Dysphagia
•• Dysphonia
Pseudobulbar palsy:
•• B/L UMNL of 9, 10, 11, 12 Cranial Nerves
Syringomyelia:
•• Dissociated sensory loss
•• Wasting of small muscles of hand
•• Charcot’s joints
Syringobulbia:
•• Facial pain/Sensory loss
•• Facial Palsy
•• Vertigo, Nystagmus, Horner’s Syndrome
Lennox-Gastaut Syndrome
•• Lennox-Gastaut syndrome occurs in children and is defined by the following triad:
–– Multiple seizure types (usually including generalized tonic-clonic, atonic, and atypical absence seizures);
–– An EEG showing slow (<3 Hz) spike-and-wave discharges and a variety of other abnormalities;
–– Impaired cognitive function in most but not all cases
•• Lennox-Gastaut syndrome is associated with CNS disease or dysfunction from a variety of causes, including developmental
abnormalities, perinatal hypoxia/ischemia, trauma, infection, and other acquired lesions.
•• High-resolution magnetic resonance imaging (MRI) can detect the characteristic hippocampal sclerosis that appears to be an
essential element in the pathophysiology of MTLE for many patients.
•• Recognition of this syndrome is especially important because it tends to be refractory to treatment with anticonvulsants but
responds extremely well to surgical intervention.
SSPE
•• SSPE is a chronic, progressive, demyelinating, late CNS complication of Measles.
•• It appears after a latent periods of 6–8 years.
•• It was first described by Dawson in 1933.
•• The incidence of SSPE has decreased over years since the introduction of live attenuated measles vaccine.
Early in the course of disease, the electroencephalogram (EEG) may be normal or show only moderate nonspecific slowing.
•• In the myoclonic stage, most patients with SSPE have ‘burst-suppression’ episodes. However, this pattern is not unique to SSPE.
•• Later in the illness, EEG becomes increasingly disorganized.
•• Diagnosis is confirmed by elevated levels of measles antibodies in the serum and CSF. CSF also shows elevated oligoclonal bands
(IgG). CSF pleocytosis is absent or minimal.
•• MRI or CT scans of patients with SSPE show variable cortical atrophy and ventricular enlargement
•• Myoclonic seizures are a feature.
•• No definitive therapy is available.
•• Inosoplex alone or win combination with interferon has been shown to be beneficial.
Prion Diseases
•• Prions are a unique class of infectious proteins associated with a group of neurodegenerative diseases, the transmissible spongiform
encephalopathies
•• In humans, these diseases include:
–– Kuru
–– Creutzfeldt-Jakob disease
–– Gerstmann-Straussler-Scheinker syndrome and
–– Fatal familial insomnia
•• In animals:
–– Scrapie and
–– Bovine spongiform encephalopathy (madcow disease)
•• PrPSc is a pathogenic, transmissible spongiform encephalopathy-specific form of the host-encoded prion protein (PrP);
•• PrPSc differs from PrP in that it contains a high amount of b-pleated sheet structure and is insoluble and resistant to proteolytic
enzymes
•• PrPSc deposits either consist of or can be readily converted to amyloid fibrils.
Creutzfeldt-Jakob Disease
•• Caused by Prion Protein (Infectious proteins)
•• Most common infectious prion disease in humans
•• Neurodegenerative disease
•• On light microscopy, the pathologic hallmarks of CJD are spongiform degeneration and astrogliosis
•• The lack of an inflammatory response in CJD and other prion diseases is an important pathologic feature of these degenerative
disorders
•• Spongiform degeneration is characterized by many 1- to 5-mm vacuoles in the neuropil between nerve cell bodies
•• In (new variant) nvCJD, a characteristic feature is the presence of ‘florid plaques’. These are composed of a central core of PrP
amyloid surrounded by vacuoles in a pattern suggesting petals on a flower
Features:
–– Rapidly progressive dementia
–– Myoclonus
–– Rapid progression are common features
–– Can be transmitted by corneal implants and is inheritable
•• Brain biopsy is diagnostic
Guillain-Barré Syndrome
Guillain-Barré syndrome is characterized by the three As: acute, areflexic, ascending. It is an autoimmune-mediated, demyelinating
ascending paralysis that is often idiopathic but is also associated with HIV and Campylobacter infections. A lumbar puncture can often
help clinch the diagnosis, although nerve conduction studies, if available, are often more consistent, as CSF findings can be ambiguous
early in the disease. The classic finding is CSF with few cells and a high total protein
Remember the disease by features:
•• Motor paralysis
•• Ascending type
•• Areflexia seen
126 USMLE Step 2 CK Platinum Notes
•• No sensory loss
•• Flaccidity seen
•• Legs affected more than arms
•• Deep tendon reflexes absent usually
•• Albumin cytological dissociation seen
Plasmapheresis and immunoglobulins used for treatment
A 28-year-old man of weakness. He reports a 4-day history of weakness and tingling in his legs, which has since progressively involved
his upper body. Physical examination is remarkable for profound weakness in his lower extremities and torso, with normal cranial
nerves and moderately reduced strength in his upper extremities. Deep tendon reflexes are absent in his lower extremities and
reduced in his upper extremities. All findings are symmetrical. Cerebrospinal fluid (CSF) analysis shows increased total protein.
Guillain-Barre-syndrome is characterized by the three As: Acute, areflexic, ascending. It is an autoimmune-mediated, demyelinating
ascending paralysis that is often idiopathic but is also associated with HIV and campylobacter infection. A lumbar puncture can often
help clinch the diagnosis, although nerve conduction studies, if available, are often more consistent, as CSF findings can be ambiguous
early in the disease. The classic finding is CSF with few cells and a high total protein.
Miller-Fisher Syndrome
•• Variant of Guillain-Barre syndrome
•• Associated with areflexia, ataxia, ophthalmoplegia
•• Anti-GQ1b antibodies are present in 90% of the cases
Friedreich’s Ataxia
•• Most common form of inherited ataxia
•• AR Inheritance
•• The classic form of Friedreich’s ataxia has been mapped to (Ch 13)
•• And the mutant gene, frataxin contains expanded GAA triplet repeats in the first intron
•• Earliest feature is ataxia
•• Dorsal root ganglia are affected earliest
•• Presents before 25 years of age with progressive staggering gait, frequent falling, and titubation. The lower extremities are more
severely involved than the upper ones
•• The neurologic examination reveals nystagmus, loss of fast saccadic eye movements, truncal titubation, dysarthria, dysmetria,
and ataxia of extremity and truncal movements
•• Extensor plantar responses (with normal tone in trunk and extremities), absence of deep tendon reflexes, and weakness (greater
distally than proximally) are usually found. Loss of vibratory and proprioceptive sensation occurs
•• Absent knee jerk and extensor plantar response are features
•• Pyramidal tract is involved
•• Cardiac involvement occurs in 90% of patients. Cardiomegaly, symmetric hypertrophy, murmurs, and conduction defects are
reported. (CARDIOMYOPATHY)
•• Musculoskeletal deformities are common and include pes cavus, pes equinovarus, and scoliosis.
A 13-year-old boy having difficulty in running and a staggering gait, poor articulation in speech. The physical examination
revealed an unsteady, broad-based stance and sudden lurching when walking. Loss of vibratory and position sense was
observed bilaterally in all extremities. There was a tremor of the upper extremity as the patient reached for objects. Speech
was slurred, slow and with an uneven pattern. Hyporeflexia was present but there was only a slight indication of muscle
atrophy, though there were bilateral Babinski’s signs.
Medicine 127
Ataxia-telangiectasia
•• Patients usually present in the first decade of life with progressive telangiectatic lesions associated with deficits in cerebellar
function and nystagmus
•• Truncal ataxia, extremity ataxia, dysarthria, extensor plantar responses, myoclonic jerks, areflexia, and distal sensory deficits may
develop
•• There is a high incidence of:
–– Recurrent pulmonary infections
–– Neoplasms of the lymphatic and reticuloendothelial system and
–– Increased incidence of cancer
•• Especially an increased incidence of lymphomas, Hodgkin’s disease, and acute leukemias of the T cell type
•• Thymic hypoplasia with cellular and humoral (IgA and IgG2) immunodeficiencies, premature aging, and endocrine disorders
such as insulin-dependent diabetes mellitus are described
•• The immunologic defects and increased susceptibility to cancer have been causally linked to cellular disorders in AT. Exposure
of cultured cells to ionizing radiation slows the rate of DNA replication and increases the frequency of chromosomal
aberrations
•• The most striking neuropathologic changes include:
–– Loss of Purkinje, granule, and basket cells in the cerebellar cortex as well as of neurons in the deep cerebellar nuclei
–– Loss of anterior horn neurons in the spinal cord and of dorsal root ganglion cells associated with posterior column spinal cord
demyelination
•• A poorly developed or absent thymus gland is the most consistent defect of the lymphoid system
•• The gene for AT (the ATM gene) has been positionally mapped to chromosome 11
•• Defective DNA repair in AT fibroblasts exposed to ultraviolet light has been demonstrated.
Choreoathetoid movements, slurred speech, ophthalmoplegia, and progressive mental retardation characterize the disease
at it advances. Telangiectasias are a helpful diagnostic clue. These children also have vulnerable recurrent sinopulmonary
infections. Immunologic evaluation may demonstrate a lack of IgA and IgE, cutaneous anergy, and a progressive cellular
immune defect. Other features of the syndrome include endocrine disorders and a predisposition for certain cancers
(leukemias, brain cancer, and gastric cancer).
Newer Concepts
Mitochondrial Ataxias
•• Spinocerebellar syndromes have been identified with mutations in mitochondrial DNA (mtDNA)
Xeroderma Pigmentosum
•• Xeroderma pigmentosum is a rare autosomal recessive neurocutaneous disorder
•• Caused by the inability to repair damage to DNA, such as that produced by ultraviolet radiation
•• In addition to skin lesions, patients may show progressive mental deterioration, microcephaly, ataxia, spasticity,
choreoathetosis, and hypogonadism
•• Nerve deafness, peripheral neuropathy (predominantly axonal), electroencephalographic abnormalities and seizures are reported
Cockayne Syndrome
•• This is a rare autosomal recessive disorder first described by Cockayne in 1936
•• Clinical features are mental retardation, optic atrophy, dwarfism, and neural deafness, hypersensitivity of skin to sunlight,
cataracts, and retinal pigmentary degeneration
•• Cerebellar, pyramidal, and extrapyramidal deficits and peripheral neuropathy may occur, with a ‘bird-headed’ facial appearance
and normal-pressure hydrocephalus
•• Skin fibroblasts exposed to ultraviolet light demonstrate defective DNA repair.
128 USMLE Step 2 CK Platinum Notes
Bacterial Meningitis
•• S. pneumoniae is the most common cause of meningitis in adults > 20 years
•• N. meningitidis accounts for nearly 60% of bacterial meningitis cases in children and young adults
•• Staphylococcus aureus and coagulase-negative staphylococci are predominant organisms causing meningitis that follows
invasive neurosurgical procedures, particularly shunting procedures for hydrocephalus
•• Group B-streptococcus or S. agalactiae is responsible for meningitis predominantly in neonates
•• L. monocytogenes has emerged as an important cause of bacterial meningitis in the elderly and in individuals with impaired cell-
mediated immunity.
–– Purulent exudates seen
–– Cloudy CSF
–– Ventricular spread seen
–– Ventricular enlargement seen
Medicine 129
The classic clinical triad of meningitis is fever, headache, and nuchal rigidity (stiff neck). Each of these signs and symptoms occurs
in > 90% of the cases.
Alteration in mental status occurs in >75% of patients and can vary from lethargy to coma. Nausea, vomiting, and photophobia are
also common complaints.
Nuchal rigidity is the pathognomonic sign of meningeal irritation and is present when the neck resists passive flexion.
Kernig’s sign is elicited with the patient in the supine position. The thigh is flexed on the abdomen, with the knee flexed; attempts to
passively extend the leg elicit pain when meningeal irritation is present.
Brudzinski’s sign is elicited with the patient in the supine position and is positive when passive flexion of the neck results in
spontaneous flexion of the hips and knees.
Unlike meningitis occurring in normal children, ventriculoperitoneal shunt infections are most commonly caused by
coagulase-negative Staphylococcus, such as Staphylococcus epidermidis. S. epidermidis causes 40 to 60% of all CSF infections
in persons with ventriculoperitoneal shunts. Coagulase-negative Staphylococcus presents a significant threat to people who
have indwelling devices or catheters. Most S. epidermidis isolates are resistant to multiple antibiotics, including nafcillin and
oxacillin. Vancomycin is the drug of choice to S. epidermidis infection. Removal of the indwelling medical device and parenteral
antibiotic treatment are often necessary.
Viral Meningitis
Enteroviruses account for 75 to 90% of aseptic meningitis cases in most series. Viruses belonging to the Enterovirus genus are members
of the family Picornaviridae and include the coxsackieviruses, echoviruses, polioviruses, and human enteroviruses 68 to 71
The most important laboratory test in the diagnosis of meningitis is examination of the CSF. The typical profile in cases of viral
meningitis is:
•• Lymphocytic pleocytosis (25 to 500 cells per microliter)
•• A normal or slightly elevated protein level [0.2 to 0.8 g/l (20 to 80 mg/dl)]
•• A normal glucose level
•• A normal or mildly elevated opening pressure (100 to 350 mm H2O)
•• Organisms are not seen on Gram’s or acid-fast stained smears or India ink wet mounts of CSF
CSF polymerase chain reaction (PCR) tests, culture, and serology, a specific viral cause can be found in 75 to 90% of cases of viral
meningitis
CSF cultures are positive in 30 to 70% of patients, the frequency of isolation depending on the specific viral agent.
130 USMLE Step 2 CK Platinum Notes
HIV meningitis
•• Should be suspected in any patient with known or identified risk factors for HIV infection
•• Aseptic meningitis is a common manifestation of primary exposure to HIV and occurs in 5 to 10% of cases
•• In some patients, seroconversion may be delayed for several months; however, detection of the presence of HIV genome by PCR
or p24 protein establishes the diagnosis
•• HIV can be cultured from CSF in some patients
•• Cranial nerve palsies, most commonly involving cranial nerves V, VII, or VIII, are more common in HIV meningitis than in other viral
infections.
Neuroimaging
This is a normal sagittal MRI scan demonstrating the midline with the frontal lobe and parietal lobe and occipital lobe and cerebel-
lum and genu of corpus callosum and splenium of corpus callosum and mammillary body and thalmus and midbrain and pons
and medulla and cervical spinal cord and tongue and nasal cavity
TCT scan without contrast demonstrating a large epidural hematoma with right to left shift and ventricular narrowing
Medicine 131
MRI scan of the head in coronal view demonstrates a cysticercus cyst of the brain
This computed tomographic (CT) scan of the head in transverse view demonstrates an area of
hemorrhage arising in the basal ganglia on the left
CT scan without contrast demonstrating a large subdural hematoma with left to right shift and ventricular narrowing
132 USMLE Step 2 CK Platinum Notes
Encephalitis
In distinction to meningitis, where the infectious process and associated inflammatory response are limited largely to the
meninges, in encephalitis, the brain parenchyma is also involved. Many patients with encephalitis also have evidence of
associated meningitis (meningoencephalitis) and, in some cases, involvement of the spinal cord or nerve roots (encephalomyelitis,
encephalomyeloradiculitis)
•• HSV is the mc cause of sporadic viral encephalitis
•• Hemorrhagic lesions seen
•• Temporal lobe involvement is common
•• PCR of HSV virus is diagnostic
The initial manifestations are similar to those seen in SSPE and include decline in school performance, behavioral alterations, and
seizures, followed by severe progressive dementia, prominent ataxia, pyramidal signs (spasticity, hyperreflexia, extensor plantar
responses), and visual deterioration
In the terminal stages of the illness, patients are globally demented, mute, and quadriparetic, often with associated ophthalmoplegia
Diagnostic Studies: CSF shows a mild lymphocytic pleocytosis, slightly elevated protein level, and rubella virus-specific
oligoclonal bands. CT scan may show enlarged ventricles, cortical and cerebellar atrophy, and hypodensity in the white matter.
Herpes simplex virus (HSV): It is the most common etiologic agent of sporadic viral encephalitis. The clinical presentation is variable
but usually abrupt, and the rapid onset of confusion and seizures are frequent manifestations. The most characteristic pathologic
lesions, which are demonstrated by CT studies, consist of hemorrhagic necrosis of the temporal lobes. Herpes encephalitis is almost
always due to HSV type 1 and affects previously healthy individuals. Up to 50% of newborns delivered vaginally from mothers with
active HSV infection develop a severe generalized form of herpetic encephalitis.
Arbovirus encephalitis: It is the most important cause of epidemic viral encephalitis. Many different species may be involved, each
with a specific regional geographic distribution. The clinical course is milder and prognosis is better than herpetic encephalitis.
Echovirus encephalitis: It is one of the most common etiologic agents of the so-called lymphocytic meningoencephalitis, Symptoms
are mild, often limited to headache and malaise. CSF is usually normal or shows mild lymphocytosis.
•• Septic cavernous sinus thrombosis: The oculomotor nerve, the trochlear nerve, the abducens nerve, the ophthalmic and
maxillary branches of the trigeminal nerve, and the internal carotid artery all pass through the cavernous sinus
•• The symptoms of septic cavernous sinus thrombosis are fever, headache, frontal and retroorbital pain, and diplopia. The classic
signs are ptosis, proptosis, chemosis, and extraocular dysmotility due to deficits of cranial nerves III, IV, and VI
•• Hypo- or hyperesthesia of the ophthalmic and maxillary divisions of the fifth cranial nerve and
•• A decreased corneal reflex may be detected
•• There may be evidence of dilated, tortuous retinal veins and papilledema.
–– Gradiengo’s Syndrome: It is irritation of ophthalmic division of Trigeminal (V) nerve and Abducens (VI) nerve in which
Petrositis of petrous temporal bone occurs.
–– Tolsa-hunt Syndrome: Inflammatory condition involving Cavernous Sinus presenting with pain, loss of ocular movements
and loss of sensations in territory of ophthalmic division of Trigeminal (V) nerve.
•• Raeder’s Syndrome:
–– It is Paratrigeminal Syndrome
–– Pain in Ophthalmic and Maxillary Divisions of Trigeminal Nerve
–– Ptosis and Miosis+IV and VI Cranial Nerve involvement
–– It occurs in lesions of Middle Cranial Fossa
•• Costen’s Syndrome: Aching pain around ear aggravated by chewing due to malalignment of Temporomandibular joint with
‘Altered BITE’
•• Symptomatic Neurosyphilis: Although mixed features are common, the major clinical categories of symptomatic neurosyphilis
include meningeal, meningovascular, and parenchymatous syphilis. The last category includes general paresis and tabes
dorsalis
Features:
•• Dorsal column involvement
•• Ptosis
•• Miosis
•• Ataxia
•• Sensory loss
•• Argyll-Robertson pupil
•• Bladder dysfunction
Pseudotumor Cerebri
•• Increased intracranial pressure
•• Papilledema
•• 6th cranial nerve palsy
•• Visual field defects
Causes:
•• Tetracycline
•• OCP
•• Hypervitaminosis A
5 Ds of Charcot’s joints:
•• Disorganization
•• Density increased
•• Debris within joint capsule
•• Destruction of bone
•• Deformity
Clinical Manifestations
•• Neuropathic joint disease usually begins in a single joint and then progresses to involve other joints, depending on the underlying
neurologic disorder
•• The involved joint progressively becomes enlarged from bony overgrowth and synovial effusion
•• Loose bodies may be palpated in the joint cavity
•• Joint instability, subluxation, and crepitus occur as the disease progresses. Neuropathic joints may develop rapidly, and a totally
disorganized joint with multiple bony fragments may evolve in a patient within weeks or months
•• The amount of pain experienced by the patient is less than would be anticipated based on the degree of joint involvement. Patients
may experience sudden joint pain from intra-articular fractures of osteophytes or condyles.
Medicine 135
Intracerebral Leukocytostasis
Ball’s disease is a potentially fatal complication of acute leukemia (particularly myelogenous leukemia) that can occur when the
peripheral blast cell count is greater than 100,000/ml
•• High blast cell counts
•• Blood viscosity is increased
•• Blood flow is slowed
•• Primitive leukemic cells are capable of invading through endothelium and causing hemorrhage into the brain
Patients may experience stupor, dizziness, visual disturbances, ataxia, coma, or sudden death.
Occlusion of the vertebral artery may cause Medial Medullary Syndrome which is characterized by:
Paralysis or atrophy of tongue on the side of lesion (XII nerve involvement)
Paralysis of arm and leg on opposite side
Impaired tactile and proprioceptive sense on opposite side. (involvement of pyramidal tract and medial lemniscus.
Parkinsonism
Parkinson’s disease has a classic tetrad of (1) slowness or poverty of movement, (2) muscular (‘lead pipe’ and ‘cog-wheel’)
rigidity, (3) ‘pill-rolling’ tremor at rest (which disappears with movement and sleep), and (4) postural instability (manifested
by the classic shuffling gait and festination)
Parkinsonism is a syndrome consisting of a combination of:
•• Tremor
•• Rigidity
•• Bradykinesia and
•• Characteristic disturbance of gait and posture
Associated:
•• On-off phenomenon
•• Decreased blinking
Medicine 137
•• Symptoms of Parkinson’s disease are caused by loss of nerve cells in the pigmented substantia nigra, pars compacta and the
locus coeruleus in the midbrain
•• Parkinsonism can be induced in primates by exposure to 1-methyl-4-phenyl-1,2,3,6-tetrahydropyridine (MPTP)
•• The 4- to 6-Hz tremor is typically most conspicuous at rest and worsens with emotional stress
•• Festinating gait is a feature
•• It often begins with rhythmic flexion-extension of the fingers, hand, or foot, or with rhythmic pronation-supination of the forearm
–– There is fixity of facial expression, with widened palpebral fissures and infrequent blinking
–– There may be:
•• Blepharoclonus (fluttering of the closed eyelids)
•• Blepharospasm (involuntary closure of the eyelids) and
•• Drooling of saliva from the mouth
•• The voice is hypophonic and poorly modulated. Power is preserved, but fine or rapidly alternating movements are impaired
•• They walk with small, shuffling steps, have no arm swing, are unsteady (especially on turning), and may have difficulty in
stopping. Some patients walk with a festinating gait, i.e. at an increasing speed to prevent themselves from falling because of
their abnormal center of gravity
•• The tendon reflexes are unaltered, and the plantar responses are flexor
•• Repetitive tapping (at about 2 Hz) over the glabella produces a sustained blink response (Myerson’s sign)
Parkinsonism:
The cause is thought to be a loss of dopaminergic neurons, especially in the substantia nigra, which project to the basal
ganglia. The result is decreased dopamine in the basal ganglia. Drug therapy, which aims to increase dopamine, includes:
•• Levodopa with carbidopa
•• Bromocriptine
•• Pergolide
•• Monoamine oxidase-B inhibitors (selegiline), and
•• Amantidine
Remember
•• A resting tremor may be due to: hyperthyroidism, anxiety, drug withdrawal or intoxication, or benign (essential) hereditary
tremor
•• Benign hereditary tremor is usually autosomal dominant; look for a positive family history, and use beta blockers to
reduce the tremor
•• Also watch for Wilson’s disease (hepatolenticular degeneration), which can cause chorea-like movements; asterixis (slow,
involuntary flapping of outstretched hands) may be seen in patients with liver and kidney failure.
138 USMLE Step 2 CK Platinum Notes
•• Aneurysms can undergo small ruptures and leaks of blood into the subarachnoid space, so-called sentinel bleeds
•• Noncontrast CT is used as best initial investigation. Nimodepine is used to prevent vasospasm
•• Surgical clipping of aneurysm is the modality of treatment used.
Headache of sudden onset (‘thunderclap’ headache), rapid deterioration of mental status and blood in the CSF are virtually
diagnostic of ruptured berry aneurysms. Note the characteristic hyperdensity on CT of the suprasellar cistern, indicating blood
in the subarachnoid space. Rupture of a berry aneurysm is the most common cause of subarachnoid bleeding. Berry aneurysms
develop as a result of congenital weakness at branching points of the arteries in the circle of Willis. These outpouchings tend
to expand progressively, but in most cases they remain asymptomatic. Hypertension facilitates development and rupture of
berry aneurysm. One-third of patients recover, one-third die, and one-third develop re-bleeding. Rapid onset of coma is an
ominous sign.
•• Basilar migrane seen in adolescent females beginning with total blindness: Bickersaffs migrane
•• Cluster headache: Reader’s syndrome, sphenopalatine neuralgia.
Glossopharyngeal Neuralgia
•• A rare syndrome characterized by recurrent attacks of severe pain in the posterior pharynx, tonsils, back of the tongue, and
middle ear
•• As in trigeminal neuralgia, intermittent attacks of:
–– Brief
–– Severe
–– Excruciating pain occurs paroxysmally, either spontaneously or
–– Precipitated by movement (e.g. chewing, swallowing, talking, sneezing)
–– The pain, lasting seconds to a few minutes, usually begins in the tonsillar region or at the base of the tongue and may
radiate to the ipsilateral ear. The pain is strictly unilateral. Occasionally, increased vagus nerve activity causes cardiac
sinus arrest with syncope. Attacks may be separated by long intervals
•• Treatment:
•• As in trigeminal neuralgia, carbamazepine is the drug of choice
•• Phenytoin, baclofen, amitriptyline, or trazodone may be added if necessary.
•• The involved side is flat and expressionless, but patients may complain instead about the seemingly twisted intact side
•• In severe cases, the palpebral fissure is wide, and the eye cannot be closed
•• Corneal protection is necessary
•• The patient may complain of a numb or heavy feeling in the face, but no sensory loss is demonstrable
•• A proximal lesion may affect salivation, taste, and lacrimation, and may cause hyperacusis.
PERIPHERAL NEUROPATHY
Peripheral neuropathy may be divided into conditions which predominately cause a motor or sensory loss.
Remember
•• Balaclava type of sensory loss: Syringomyelia
•• Dissociated sensory loss: Syringomyelia
•• Mask-like facies: Parkinsonism
•• Tic douloureux: Trigeminal neuralgia
•• Punch drunk: Alcoholism
•• Picket fence fever: Lateral sinus thrombosis
•• Astasia-Abasia: Hysterical conversion
•• Opsoclonus/myoclonus: Neuroblastoma
Fevers
Fever Causative Organism
•• Sandfly fever •• Arbovirus
•• Lassa fever •• Arena virus
•• Glandular fever •• Ebstein-barr virus
•• Boutonneuse fever •• Rickettsiae conorii
•• Oroya fever •• Bartonella
•• Pretibial fever •• Leptospirosis
•• Canicola fever •• Leptospirosis
•• Swamp fever •• Leptospirosis
•• Seven-day fever •• Leptospirosis
•• Haverhill fever •• Streptobacillus moniliformis
•• Scarlet fever •• Streptococci
•• Pontiac fever •• Legionella
142 USMLE Step 2 CK Platinum Notes
Parasitic Diseases
Parasitic Diseases
‘Most Likely’ clinical scenarios associated with parasitic diseases
•• ANA: SLE
•• Anti-smith, anti-dsDNA: Specific for SLE
•• Anti-histone: Drug induced SLE
•• Anti-centromere: CREST Syndrome (c-c)
•• Anti-Scl 70: Scleroderma (scl-scl)
•• Anti-SSA, Anti-SSB: Sjogren’s syndrome (s-s)
•• Anti-Jo 1: Polymyositis
•• Anti-mitochondrial: Primary biliary cirrhosis
•• Anti-glidian, Anti-transglutaminase: Celiac disease
•• Anti-GIL: Hemolytic transfusion reactions
•• Anti-sacchromyces cerevisiae: Crohn’s disease
•• Anti-epithelial cell: Pemphigus vulgaris
•• Anti-IgG: Rheumatoid arthritis
•• Antibodies to filaggrin
•• Antibodies to citrulline
•• Antibodies to calpastatin
•• Antibodies to components of the spliceosome (RA-33), and an unknown antigen, Sa
Important Eponyms
•• Hebra nose Rhinoscleroma
•• Rubber man Ehler-Danlos syndrome
•• Sailors skin Solar elastosis
•• Petrified man Myositis ossificans
Rare Syndromes
LEOPARD syndrome:
•• Lentiges
•• ECG abnormalities
•• Ocular hypertelorism
•• Pulmonary stenosis
•• Abnormal genitalia
•• Retarded growth
•• Deafness
LAMB syndrome:
•• Lentiges
•• Atrial myxoma
•• Blue nevus
NAME syndrome:
•• Nevus
•• Atrial myxoma
•• Myxoid neurofibroma
•• Ephelids
Carney syndrome: LAMB and NAME syndrome
POEMS syndrome:
•• Polyneuropathy
•• Organomegaly
•• Endocrinopathy
•• M protein
•• Skin change
Concept Clarifications
•• Hutchinson’s freckle Lentigo maligna (melanoma variant)
•• Hutchinson’s pupil Blown pupil in uncal herniation
•• Hutchinson’s sign Herpes zoster ophthalmicus, vesicles at tip of nose
•• Hutchinson’s teeth Small, widely spaced teeth in congenital syphilis
•• Hutchinson’s triad Interstitial keratitis, notched incisors, VIII nerve deafness
•• Hutchinson’s # # radial styloid process
•• Hutchinson’s book Book on medicine (not asked in exams)
CD Markers
•• Hairy cell leukemia CD 103+
•• Mantle cell lymphoma CD 5+, CD 103–
•• CLL CD 23+
•• Apoptosis CD 95+
•• Brainstem lesion:
Cranial nerve involvement+sensory loss of ½ of face+contralateral ½ of body
•• Thalamic lesion:
Sensory loss of ½ of face+same ½ of body
•• Internal capsule lesion:
Loss of face, arm, leg without higher cortical dysfunction
Remember
•• Nonenhancing lesion in AIDS PML
•• Solitary weakly ring-enhancing lesion CNS lymphoma
•• Multiple ring-enhancing lesions Toxoplasmosis
•• This patient has cerebral toxoplasmosis, which represents one of the most common opportunistic infections in AIDS
•• Toxoplasma gondii is a protozoon that infects humans who ingest the oocysts from cat feces or incompletely cooked lamb or pork
•• Only immunodepressed patients and fetuses are vulnerable to this infection. In the fetus, toxoplasmosis causes extensive damage
to brain parenchyma and retina
•• Toxoplasmosis associated with AIDS manifests with necrotizing lesions surrounded by chronic inflammation
•• A ring-enhancing lesion is a mass that contains a rim of contrast enhancement (bright signal on MRI) surrounding a dark core
corresponding to central necrosis
•• In AIDS, the most frequent causes of a ring-enhancing lesion are primarily brain lymphoma and toxoplasmosis.
Cardiac Tamponade
•• X-descent
•• Pulsus paradoxus
Medicine 147
Constrictive Pericarditis
•• Kussmaul sign
•• Steep y-descent
•• Square root sign
•• M contour sign
•• Pericardial knock
•• Myelodysplastic syndrome: 5q
•• Cri-du-chat syndrome: 5p
•• A 3-year-old boy from rural America is brought to the ENT emergency room in extreme Epiglottitis
respiratory distress, with a temperature of 104 degrees Fahrenheit. He is drooling and
has great difficulty swallowing and on physical examination, an inspiratory stridor is noted.
An immediate lateral X-ray done shows swelling of the epiglottis. He has had no previous
vaccinations. Most likely disease is:
•• A highly encapsulated organism is found to cause bronchopneumonia with patchy Klebsiella pneumoniae
infiltrates involving one or more lobes with red sputum in a debilitated alcoholic. Most likely
disease is:
•• An 8-year-old girl is bitten in the leg by a dog. She presents the next day with fever and Pasteurella
bone pain localized to her right calf. X-ray reveals a lytic lesion of the left femur. Results of
the bone culture are pending. Infecting organism is most likely to be:
•• An organism is causing urinary tract infections. It has ability to break down urea and is Proteus species
thought to contribute to the development of struvite kidney stones due to the elevation of
urine pH by production of ammonia. The said organism is also having swarming motility. The
organism is identified as:
•• A 44-year-old man from rural Indian village is brought to a rural hospital with severe Plague
bronchopneumonia. He suffered sudden onset of chills, fever, and headache several days
ago. Two days later, he complained of chest pain and difficulty breathing, and coughed up
blood-tinged sputum. Chest X-ray reveals patchy infiltrates and segmental consolidation.
The most likely cause of this man’s pneumonia and the disease itself is due to yersinia.
Disease itself is:
•• A disease is caused in a gardener by virtue of organism entering through skin breaks in Sporotrichosis
the fingers or hands, causing a chancre, papule, or subcutaneous nodule with erythema
and fluctuance. Ulcerating lesions appear along lymphatic channels, but the lymph nodes
are not commonly infected. Potassium iodide is used for the treatment of the subcutaneous
manifestations. The disease is common in gardeners. Most likely disease is:
•• A 66-year-old alcoholic man from New with brain and pulmonary abscess and is treated with Clostridium difficile
antibiotics for last two weeks. He develops nausea, vomiting, abdominal pain, and voluminous
green diarrhea. The condition is diarrhea due to:
•• An organism identified on dark ground microscopy caused testicular involvement with Syphilis
gumma formation, endarteritis, and/or a prominent plasma cell infiltrate. Most likely
disease is:
•• A disease which may be spread by handling rabbits or rabbit skins, or by bites from ticks Tularemia
that feed on the blood of wild rabbits, a gram-negative coccobacillus. The disease begins
as a rupturing pustule followed by an ulcer, with involvement of regional lymph nodes.
More serious cases can be complicated by bacteremia, splenomegaly, rash, pneumonia, or
endotoxemic shock. Most likely disease is:
•• A gram-positive spore-forming anaerobic rod causing (a spastic paralysis caused by Tetanus caused by clostridia
toxin which blocks the release of the inhibitory neurotransmitters glycine and gamma- tetani
aminobutyric acid [GABA]) is most likely:
•• A gram-negative rod that is a zoonotic agent causing an undulating febrile disease with Brucellosis
malaise, lymphadenopathy and hepatosplenomegaly. The normal route of exposure is via
ingestion of the organism. Most likely disease is:
•• A 65-years-old male from Chicago presents with low back pain, anemia and fatigability. ESR •• Multiple myeloma
is 90 and serum creatinine: 3.2 mg%. Likely diagnosis is:
•• A child brought by his mother from Mexico presents with ascending flaccid paralysis. There •• Guillain-Barré syndrome
is subsequent respiratory muscle involvement. CSF examination shows albuminocytological
dissociation. Likely diagnosis is:
•• A 56-year-old engineer from Michigan presents with pain, numbness and impaired sensation •• Wallenberg’s syndrome
over half of the face along with ataxia, nystagmus, dysphagia and hoarseness of voice. His
pain and thermal sensations over opposite half are impaired. Horner’s syndrome is present.
Likely cause is:
Medicine 149
•• A 25-year-old male scholar from New Zealand had pigmented macules over the palm, sole and •• Peutz-Jeghers syndrome
oral mucosa. He also had anemia and pain in abdomen. The most probable diagnosis is:
•• A 64-year-old lady admitted in neurology unit of Philadelphia complains of severe unilateral •• Temporal arteritis
headache on the right side and blindness for 2 days. On examination, there is a ‘thick cord’
like structure on the lateral side of the head. The ESR is 80 mm/hr in the first hour. The most
likely diaganosis is:
•• A 45-years-old female in Texas presented with hypertension, palpitation. CT scan finds 7 cm •• Pheochromocytoma
suprarenal mass. Most likely diagnosis is:
•• A young female presents with history of dyspnea on exertion. On examination, she has wide, •• Ostium primum atrial sep-
fixed split S2 with ejection systolic murmur (III/VI) in left second intercostal space. Her EKG tal defect
shows left axis deviation. The most probable diagnosis is:
•• A 29-year-old woman from Ohio was found to have hemoglobin of 7.8 g/dl with a reticulocyte •• Iron-deficiency anemia
count of 0.8%. The peripherial blood smear showed microcytic hypochromic anemia,
hemoglobin A2 and hemoglobin F levels were 2.4% and 1.3% respectively. The serum irom
and the total iron-binding capacity were 15 micro g/dl, and 420 micro g/dl, respectively.
The most likely cause of anemia is:
•• A 33-year-old lady working as a manager in Ohio presents with polydipsia and polyuria. Her •• Psychogenic polydipsia
symptoms started soon after a road traffic accident 6 months ago. The blood pressure is 120/80
mm Hg with no postural drop. The daily urinary output is 6–8 liters. Investigation showed, Na
130 mEq/l, K.3.5 mEq/l, urea 15 mg/dl, sugar 65 mg/dl. The plasma osmolality is 268 mosmol/l
and urine osmolality 45 mosmol/l. The most likely diagnosis is:
•• A 4½-year-old girl brought by her dear mother from Washington always had to wear warm •• Coarctation of aorta
socks even is summer season. On physical examination, it was noticed that she had high
blood pressure and her femoral pulse was weak as compared to radial and carotid pulse.
A chest radiograph showed remarkable notching of ribs along with their lower borders. The
most likely diagnosis is:
•• A 56-year-old immunocompromised man who was given steroids by his physician in •• Herpes zoster
Ohio has painful vesicular rash over his right upper eyelid and forehead for the last
48 hours. He underwent chemotherapy for nonhodgkin’s lymphoma one year ago. His tepmperature
is 98 °F, blood pressure 138/76 mm Hg and pulse is 80/minute. Examination shows no other
abnormalities. Which of the following is the most likely diagnosis:
•• A 40-years-male from Calafornia presents with polyuria, pain abdomen, nausea, vomiting, •• Hypercalcemia
altered sensorium was found to have Bronchogenic carcinoma. The electrolyte abnormality
seen in him would be:
•• A 65-year-old female is seen in surgical casualty of New Orleans. The patient has a progressive •• Nelson’s syndrome
skin pigmentation, amenorrhea, and visual field disturbances from a functioning
pituitary tumor. A CT scan shows a pituitary tumor. History is significant for bilateral
adrenalectomy for Cushing’s disease. High plasma concentrations of ACTH and MSH were
demonstrated by bioassay. Most likely diagnosis is:
•• A 75-year-old working in some factory in China presents with symptoms of nonpleuritic chest •• Mesothelioma
pain, dyspnea secondary to pleural effusion, weight loss, fever, and cough also occur with
high frequency. Physical examination is notable for dullness to percussion and diminished
breath sounds. They stain positive for keratin. The chest X-ray shows pleural plaques,
pleural effusion. Computed tomography demonstrates extrapleural soft tissue invasion and
diaphragmatic involvement. The most likely diagnosis is:
•• A 55-year-old man has abdominal fullness, nonspecific symptoms of fatigue, weakness, •• Hairy cell leukemia
and weight loss, easy bruising. It is found that he has a type of leukemia. This leukemia is
characterized by presence of malignant cells that have irregular, filamentous cytoplasmic
projections on light microscopy that give the cells a hairy appearance. The most likely
diagnosis is:
•• In a 55-year-old of Chicago, a painless firm fibrotic thickening of the fascia of the •• Peyronie’s disease
corpora cavernosa is observed involving the dorsolateral aspects of the penile shaft or the
intracavernous septum between the corpora cavernosa. Most likely diagnosis is:
150 USMLE Step 2 CK Platinum Notes
•• A 54-year-old alcholic after a trauma on the highway in California has hypoxia, confusion. •• Fat embolism
Petechiae are found over the chest, conjunctivae, and uvula. Neurological examination shows
confusion, agitation, stupor. Respiratory abnormalities include tachypnea, profound hypoxia.
Blood examination shows thrombocytopenia, hypofibrinogenemia, and prolongation of the
partial thromboplastin time. Electrocardiogram shows prominent S waves in lead 1, prominent
Q waves in lead 3, ST segment depression, and right axis strain. Arterial PO2 is 50 mm Hg on
room air. Most likely diagnosis is:
•• A pathologist from Leicester (UK) reports that a disease is confined to the mucosal and •• Ulcerative colitis
submucosal layers of the colonic wall, with superficial ulcers, increased cellular infiltration of
the lamina propria, and crypt abscesses beginning in the rectum and advancing proximally
to involve the entire colon. On gross inspection, the colonic mucosa demonstrates healed
granular superficial ulcers superimposed on a friable and thickened mucosa with
increased vascularity. The most likely cause is:
•• A syndrome is associated with the rare occurrence of malignant infiltration of the colonic •• Ogilvie syndrome
sympathetic nerve supply in the region of the celiac plexus in which sometimes massive
cecal and colonic dilation is seen in the absence of mechanical obstruction. Such an entity
is described as:
•• A 6-year-old boy seen by a neurologist in Washington DC has been complaining •• Craniopharyngioma
of headache, ignoring to see the objects on the sides for four months.
On examination, he is not mentally retarded, his grades at school are good, and visual acuity
is diminished in both the eyes. Visual charting showed significant field defect. CT scan of the
head showed suprasellar mass with calcification. Most probable diagnosis is:
•• A patient from National Athletic Association of New York has nuchal lymphedema •• Turner’s syndrome
at birth, short stature, webbed neck, widely spaced nipples, and primary
ovarian failure. A buccal smear done would classically reveal absent Barr bodies.
Most likely diagnosis is:
•• A tall Patient from Third year Class of a medical school with arachnodactyly, hyperextensible •• Homocystinuria
joints, mitral valve prolapse, dislocation of the lens of the eye, and a high risk for thoracic
aortic dissection is most likely having:
•• CD 19 positive, CD 22 positive, CD 103 positive monoclonal B-cells with bright kappa •• Hairy cell leukemia
positivity were found to comprise 60% of the peripheral blood lymphoid cells on flow
cytometric analysis in a 55-year-old man with massive splenomegaly and a total leucocyte
count of 3.3 x 109/l. Most likely diagnosis is:
•• A 36-year-old female with symptoms of hyperparathyroidism, tumor in pancreas, adrenal •• MEN I
cortical hyperplasia, pituitary adenomas, islet cell tumor with cutaneous angiofibromas. Most
likely diagnosis is:
•• A 30-year-old male patient presents with complaints of weakness in right upper and •• Polyarteritis nodosa
both lower limbs for the last 4 months. He developed digital infarcts involving 2nd
and 3rd fingers on right side and 5th finger on left side. On examination, BP was
160/140 mm Hg, all peripheral pulses were palpable and there was asymmetrical neuropathy.
Investigations showed Hb 12 gm%, TLC 12,000 per cu mm platelets 4,30,000 and ESR 49 mm.
Urine examination showed Proteinuria and RBC-10–15/hpf with no casts. The most likely
diagnosis:
•• A 34-year-old female, nonhypertensive working in White House presents with severe headache •• Subarachnoid hemorrhage
and neck stiffness of abrupt onset. She tells she never had ‘such a severe headache’ before. On
further asking, she tells she is nauseated and has photophobia. Diagnosis is:
•• A 16-year-old girl from Texas with primary amenorrhea attends OPD. She has normal sexual •• Androgen insensitivity syn-
development and normal breast but with absent pubic and axillary hair. Examination drome
shows B/L inguinal hernias
•• USG shows absent uterus and blind vagina. Diagnosis will be:
Medicine 151
•• A 20-year-old male comes to the physician for a routine physical examination. •• Aortic regurgitation
His height is 198 cm. He has long fingers and toes. Blood pressure is 150/65 mm Hg,
and pulse is 66/min. On auscultation, there is a grade 2/6, long, high-frequency diastolic
murmur at the second right intercostal space. Echo would reveal most likely a diagnosis of
valvular lesion of:
•• A 66-year-old woman reports to a physician because of aches and stiffness in her neck, •• Polymyalgia
shoulders, and hips for two-and-a-half months. Her symptoms are more in the morning
hours. She has had chronic fatigue and low-grade fever during this period. Range of motion
of the neck, shoulders, and hips is normal. The muscles are minimally tender to palpation.
Muscle strength, sensation, and deep tendon reflexes are normal. Erythrocyte sedimentation
rate is 88 mm/h. Serum rheumatoid factor and antinuclear antibody assays are negative.
The most likely diagnosis is:
•• A 20-year-young man presents with exertional dyspnea, headache, and giddiness. On •• Coarctation of the aorta
examination, there is hypertension and LVH. X-ray picture shows notching of the anterior
ends of the ribs. The most likely diagnosis is:
•• A 30-year-old HIV-positive patient from New York robbery gang presents with fever, dyspnea •• Pneunocystis carinii pneu-
and nonproductive cough. Patient is cyanosed. His chest X-ray reveals bilateral, symmetrical monia
interstitial infiltrates. The most likely diagnosis is:
•• A 40-year-old male working as a bartender in a local pub, with history of daily alcohol •• Delerium tremens
consumption for the last 7 years, is brought to the hospital with acute onsent of not recognizing
family members, violent behavior and tremulousness for few hours. There is history of his
having missed the alcohol drink since 2 days. Examination reveals increased blood pressure,
tremors, increased psychomotor activity, fearful affect, hallucinatory behavior, disorientation,
impaired judgment and insight
•• He is most likely to be suffering from:
•• A 50-year-old man, an alcoholic and a smoker presents with a 3-hour history of increasing •• Acute aortic dissection
shortness of breath. He started having this pain while eating, which was constant and
radiated to the back and interscapular region. He was a known hypertensive. On
examination, he was cold and clammy with a heart rate of 130/min, and a BP of 80/40
mm Hg. JVP was normal. All peripheral pulses were present and equal. Breath sounds
were decreased at the left lung base and chest X-ray showed left pleural effusion. The
most likely diagnosis is:
•• A patient with ‘leukemia on chemotherapy’ in a Chemotherapy unit of Washington •• Neutropenic colitis
develops acute right lower abdominal pain associated with anemia, thrombocytopenia
and leukopenia. The most likely diagnosis is:
•• A patient presents with chronic small bowel diarrhea, duodenal biopsy shows villous •• Gluten enteropathy
atrophy. Antiendomysial antibodies are positive. Most likely diagnosis is:
•• A 24-year-old male presents with abdominal pain, rashes, palpable purpura and arthritis. •• Henoch-Schonlein Purpura
The most probable diagnosis is: (HSP)
•• A 7-year-old girl presents with repeated episodes of bleeding into joints. APTT is •• von Willebrand disease
prolonged and PT is normal. The most likely diagnosis is:
•• In a lady with bilateral superior temporal quadrantopia and galactorrhea, the most •• Pituitary macroadenoma
probable cause is:
•• A 40-year-old engineer presented with hepatosplenomegaly, hilar lymphadenopathy •• Sarcoidosis
and hypercalcemia. Most probable diagnosis is:
•• A man with fever, dull-aching retrosternal chest pain reported to the emergency. On •• Pericardial effusion
examination, he was found to show pulsus paradoxus, Kussmaul’s sign and increased jugular
venous pressure. His apex beat was impalpable and heart sounds were muffled. Probable
diagnosis is:
152 USMLE Step 2 CK Platinum Notes
•• A 33-year-old lady presents with polydipsia and polyuria. Her symptoms started soon after •• Psychogenic polydipsia
a road traffic accident 6 months ago. The blood pressure is 120/80 mm Hg with no postural
drop. The daily urinary output is 6–8 liters. Investigation showed Na 130 mEq/l, K.3.5
mEq/l, urea 15mg/dl, sugar-65 mg/dl. The plasma osmolality is 268 mosmol/l and urine
osmolality 45 mosmol/l. The most likely diagnosis is:
•• A 33-year-old HIV-positive male from New York complains of pain on swallowing. Physical •• Candidiasis
examination is remarkable for white plaque-like material on his tongue and buccal
mucosa, which is scraped and sent to the laboratory. The man is diagnosed with acquired
immunodeficiency syndrome (AIDS)
•• The most likely diagnosis is:
•• A 4-year-old boy presents with a 1-day history of loose stools, fever, abdominal •• Salmonella
cramping, headache, and myalgia. He has no blood in the stool. The incubation period is
36 hours after ingestion of contaminated food or water. Most likely the causative agent
of his diarrhea is:
•• A 4-year-old girl with a history of hydrocephalus is brought to the neurologist by her •• Staph epidermis infection
parents with a severe headache and fever. The girl underwent a surgery for a ventricular-
peritoneal shunt 2 months ago, and the neurologist suspects that an infection has occurred.
Infection is most likely due to:
Medicine 153
2. Fibrosis of the bone marrow, extramedullary hematopoiesis, presence in the peripheral blood of immature erythroid and
granulocyte precursors and massive splenomegaly
3. No fibrosis of the bone marrow, no extramedullary hematopoiesis, presence in the peripheral blood of immature erythroid
and granulocyte precursors and massive splenomegaly
4. No fibrosis of the bone marrow, extramedullary hematopoiesis, presence in the peripheral blood of immature erythroid and
granulocyte precursors and no splenomegaly
Ans. 2 Fibrosis of the bone marrow, extramedullary hematopoiesis, presence in the peripheral blood of immature erythroid
and granulocyte precursors and massive splenomegaly
3. AntiLKM antibody
4. Antiphospholipid antibody
5. Antimitochondrial antibody
Ans. 5. Antimitochondrial antibody
3. 1, 4, and 6 months
4. 0, 3, and 6 months
Ans. 1. 1, 2, and 6 months
1. Iron-deficiency anemia
2. Wegner’s granulomatosis
3. Sjögren’s syndrome
4. Osteoarthritis
5. Thrombophlebitis
Ans. 3. Sjögren’s syndrome
This woman has a classic presentation of primary biliary cirrhosis. Primary biliary cirrhosis is often seen in individuals with
other autoimmune diseases, such as Sjögren’s syndrome, pernicious anemia and Hashimoto thyroiditis.
3. Blood gas
4. Sleep study
5. Exercise tolerance test
Ans. 4. Sleep study
The diagnosis of obstructive sleep apnea can be made with a sleep study. In sleep apnea, there is gross obesity and airways
obstruction, occasionally leading to type II respiratory failure.
3. Homonymous hemianopia
4. Dressing apraxia
5. Finger agnosia
6. Chorea
7. Perseverance
8. Total blindness
Ans. 7. Perseverance
Deficits following frontal lobe damage are attention disorder (distractibility and poor attention), aphasia, reduced activity,
particularly a diminution of spontaneous activity, lack of drive, inability to plan ahead, and lack of concern.
7. Pseudotumor cerebri
8. Normal pressure hydrocephalus
9. Cerebellar disease
10. Midbrain infarct
Ans. 9. Cerebellar disease
2. SLE
3. Polymyositis
4. Mixed connective tissue disease
Ans. 2. SLE
Rheumatoid factor is not specific for Rheumatoid arthritis. Combination of oral ulcer, thrombocytopenia, prolonged APTT
suggests SLE. Anti-Sm antibodies are not sensitive for SLE.
1. Sarcoidosis
2. Lofgren’s syndrome
3. Heerd Ford Waldenstroms syndrome
4. Tuberculosis
Ans. 2. Lofgren’s syndrome
4. The K-ras oncogene is located on chromosome 15, the DCC (deleted in colon cancer) located on chromosome 20 and the
p53 tumor suppressor gene located on chromosome 17p
Ans. 1. The K-ras oncogene is located on chromosome 12, the DCC (deleted in colon cancer) located on chromosome 18 and the
p53 tumor suppressor gene located on chromosome 17p
4. Ventricular extrasystole
5. First-degree heart block
Ans. 2. Supraventricular tachycardia
Out of the following options, the most likely rhythm which may cause symptoms are supraventricular tachycardia. The other
rhythms may cause palpitations but would be unusual to cause light-headedness/presyncope.
1. Sarcoidosis
2. Tuberculosis
3. Lymphoma
4. Silicosis
Ans. 2. Tuberculosis
3. Pseudomonas
4. Salmonella
5. Campylobacter
6. EHEC
7. EIEC
8. ETEC
Ans. 8. ETEC
4. Give ergometrine
5. Give latanoprost
6. Low-dose prednisone treatment
7. High-dose prednisone treatment
8. Temporal artery biopsy
Ans. 7. High-dose prednisone treatment
The patient needs urgent treatment with high-dose prednisone for giant cell arteritis (i.e. temporal arteritis)
5. Coccidioidomycosis
6. Aspergillus fumigatus
Ans. 6. Aspergillus fumigatus
3. Asthma
4. Fibrotic lung
5. Pulmonary eosinophilia
6. Pulmonary embolus
Ans. 4. Fibrotic lung
•• In rheumatoid arthritis:
•• Exudative pleural effusions
•• Fibrotic lung disease
•• Caplan’s syndrome (pneumoconiosis, pulmonary nodules)
•• Obstructive lung disease in the form of bronchiolitis obliterans (obstruction of bronchiolar lumen with inflammatory exudate) may
occur
3. A very high total protein level but without any increase in the number of cells in the CSF
4. A very high total protein level along with an increase in the number of cells in the CSF
Ans. 3. A very high total protein level but without any increase in the number of cells in the CSF
3. Hirsutism
4. Malignant melanoma
5. Arterial thrombosis
Ans. 5. Arterial thrombosis
Behçet’s syndrome is inflammatory disorder (associated with certain HLA B and DR types) causing:
•• Mouth ulceration
•• Arthritis
•• Eyes (anterior uveitis, retinal vein occlusion)
•• Vasculitis (thrombophlebitis) and thrombosis
•• CNS vasculitis involvement may lead to TIA, meningoencephalitis, Parkinson’s and dementia
1. HLA-DR4 positivity
2. HLA-B27 positivity
3. HLA-DR2 positivity
4. HLA-DR5 positivity
Ans. 2. HLA-B27 positivity
1. Anthrax
2. Trichinosis
3. Brucellosis
4. Leptosporiosis
5. Toxoplasmosis
6. Histoplasmosis
Ans. 3. Brucellosis
Brucellosis abortus is the most common species to cause endocarditis. The aortic valve is most commonly involved, followed
by the mitral valve, and then both valves. Most cases of brucellosis are associated with occupational exposure in persons such
as veterinarians.
3. Lymphoma
4. Tonsillitis
5. Retropharyngeal abscess
6. Ludwig’s angina
7. Quinsy
Ans. 2. Glandular fever
3. EBV
4. HPV
5. HIV
6. Herpes
Ans. 6. Herpes
1. SLE
2. Dermatomyositis
3. Polymyositis
4. Sjögren’s syndrome
5. MCTD
6. Amyloidosis
Ans. 4. Sjögren’s syndrome
4. Ulcerative colitis
5. Pseudomembranous colitis
Ans. 3. Crohn’s disease
•• Sensorineural injury
•• Respiratory failure
•• Necrotizing enterocolitis
•• Cholestatic liver disease
Late
Neurological:
•• Mental retardation
•• Seizures
•• Microcephaly
•• Poor school performance
•• Hearing impairment, visual impairment, myopia
Respiratory:
•• Bronchopulmonary dysplasia
•• Cor pulmonale, recurrent pneumonia
GIT:
•• Short bowel syndrome
•• Malabsorption
•• Infectious diarrhea
Liver: Cirrhosis, liver failure, carcinoma
Nutrient deficiency: Osteopenia, anemia, growth failure
Others:
•• SIDS
•• Inguinal hernia
•• GERD
•• Hypertension
•• Craniosynostosis
•• Nephrocalcinosis
Differentiate Between
Cephalohematoma Caput succedaneum
•• Not present at time of birth •• Present over presenting part
•• Subperiosteal swelling •• Soft, boggy swelling
•• Does not cross suture lines •• Can cross suture line
•• Disappears late (2 weeks–3 months) •• Disappears early
•• Down’s syndrome
•• Congenital Rubella syndrome
•• Edward’s syndrome
•• Patau’s syndrome
•• Beckwith-Wiedemann syndrome
•• Cornelia de Lange syndrome
•• Velocardiofacial syndrome
•• Cockayne syndrome
•• Charge syndrome
Teratogens
•• Carbamazepine •• Cleft lip, cleft palate
•• Valproic acid •• Neural tube defects
•• Warfarin •• Chondrodysplasia punctata
•• Carbimazole •• Fetal cutis aplasia
•• Lithium •• Ebstein’s anomaly
•• Thalidomide •• Phocomelia
•• Chloramphenicol •• Gray baby syndrome
•• DES •• Clear-cell cancer
•• Mental retardation
•• Cardiac malformation
•• Hyperkinetic movements
Premature Neonate
Features are:
•• Sutures widely separated
•• Skin appears shiny
•• Abundant lanugo
•• Subcutaneous fat is reduced
•• Sluggish neonatal reflexes
•• Small face
224 USMLE Step 2 CK Platinum Notes
Breast Milk
Has got many protective factors against bacteria, viruses, parasites
•• Lactoferrin is particularly important against Plasmodium falciparum, E. coli, Giardia
•• Bile salt-stimulated lipase is important against E. histolytica
•• Lactadherin against Rotavirus
•• PABA is also anti-infective for malaria
•• Can transmit tuberculosis
Breastfed infant can be deficient in:
•• Vitamin B1
•• Vitamin B12
•• Vitamin D
•• Vitamin K
•• Folic acid
Contraindications to Breastfeeding
•• Galactosemia is an absolute contraindication to breastfeeding
•• Chronic medical illness such as decompensated heart failure
•• Open tuberculosis of mother is also a contraindication to breastfeeding in developing countries
•• Severe anemia
•• Chronic nephritis
•• Puerperal psychosis
•• Patient on antiepileptic, antithyroid drugs
Pediatrics 225
MALNUTRITION
Marasmus
•• It is a state in which virtually all available body fat stores have been exhausted due to starvation. The diagnosis is based on:
–– Severe fat and muscle wastage resulting from prolonged calorie deficiency. Diminished skin-fold thickness reflects the loss of
fat reserves
–– Reduced arm muscle circumference with temporal and interosseous muscle wasting reflects the catabolism of protein
throughout the body, including vital organs such as the heart, liver, and kidneys
–– The laboratory findings in marasmus are relatively unremarkable
–– The creatinine-height index (the 24-hour urinary creatinine excretion compared with normal values based on height) is
low, reflecting the loss of muscle mass, immunocompetence, wound healing, and the ability to handle short-term stress are
reasonably well-preserved in most patients with marasmus.
Kwashiorkor
•• In contrast to marasmus, kwashiorkor in developed countries occurs mainly in connection with acute, life-threatening illnesses
such as trauma and sepsis, and chronic illnesses that involve acute-phase inflammatory responses. In its early stages, the physical
findings of kwashiorkor are few and subtle
•• Fat reserves and muscle mass are initially unaffected, giving the deceptive appearance of adequate nutrition
•• Signs that support the diagnosis of kwashiorkor include easy hair pluck ability, edema, skin breakdown, and poor wound healing
•• The major sine qua non is severe reduction of levels of serum proteins such as albumin (< 2.8 g/dl) and transferrin (< 150 mg/dl) or
iron-binding capacity (< 200 g/dl).
•• Cellular immune function is depressed, reflected by lymphopenia (< 1500 lymphocytes/l in adults and older children) and lack of
response to skin test antigens (anergy)
•• Unlike treatment in marasmus, aggressive nutritional support is indicated to restore better metabolic balance rapidly.
Marasmic Kwashiorkor
•• Marasmic kwashiorkor, the combined form of PEM, develops when the cachectic or marasmic patient experiences acute stress
such as surgery, trauma, or sepsis, superimposing kwashiorkor onto chronic starvation
•• An extremely serious, life-threatening situation can occur because of the high risk of infection and other complications. It is
important to determine the major component of PEM so that the appropriate nutritional plan can be developed
•• If kwashiorkor predominates, the need for vigorous nutritional therapy is urgent; if marasmus predominates, feeding should be
more cautious.
226 USMLE Step 2 CK Platinum Notes
Remember
•• Gomez syndrome: It is a nutritional recovery syndrome due to sudden rise of estrogen leading to hepatomegaly, jaundice and
spider naevi.
•• Keshans disease: It is cardiomyopathy due to selenium deficiency
•• Kashin-Beck disease: It is osteoarthritis due to selenium deficiency
PEM
•• Hypothermia
•• Hypoglycemia
•• Hypomagnesemia
•• ↑Total body water
•• ↑Cortisol and↑GH
Rickets
The clinical manifestations of rickets are the result of skeletal deformities:
•• Susceptibility to fractures, weakness and hypotonia, and disturbances in growth
•• Features:
•• Parietal flattening
•• Frontal bossing develops in the skull
•• Craniotabes: The calvariae are softened
•• Sutures may be widened
•• Rachitic rosary: Prominence of the costochondral junctions is called the rachitic rosary
•• Harrison’s groove: Indentation of the lower ribs at the site of attachment of the diaphragm
•• Knock Nee + Coxa Vara
•• Bow legs
•• Pot belly
•• Forward projection of sternum (pectus carinatum)
•• Caput quadratum
•• Lumbar lordosis
•• Short stature, genu valgum, coxa vara, kyphoscoliosis
•• Triradiate pelvis
Causes
•• Nutritional rickets: Vitamin D deficiency, malabsorption
•• Accelerated loss of Vitamin D: Phenytoin, rifampicin, barbiturates
•• Impaired hydroxylation in liver and kidney
•• Liver disease, hypoparathyroidism, renal failure, renal tubular acidosis
•• Vitamin D-resistant rickets Fanconi’s syndrome, Wilson’s disease
Biochemical
Serum calcium: Normal or low
Serum phosphate: Low
Alkaline phosphatase: High
PTH : High
Scurvy
•• Vitamin C deficiency causes scurvy
•• Symptoms of scurvy primarily reflect impaired formation of mature connective tissue and include bleeding into skin (petechiae,
ecchymoses, perifollicular hemorrhages); inflamed and bleeding gums; and manifestations of bleeding into joints, the
peritoneal cavity, pericardium and the adrenal glands
•• Wimberger’s sign seen
•• Rosary seen
•• Pseudoparalysis seen
228 USMLE Step 2 CK Platinum Notes
Vitamin C, or ascorbic acid, is a necessary cofactor for platelet function. A deficiency, most commonly due to a dietary cause, results in
petechiae, microhemorrhages, gingival bleeding, and perifollicular hemorrhages. Classically, this disease was called ‘scurvy,’ and British
sailors who did not have access to fresh fruits and vegetables suffered from this condition. Today, scurvy is rare and is seen mostly in
elderly patients with poor dietary habits (such as those who may live alone). Treatment requires the daily administration of vitamin C
for 3 to 4 weeks and is quite successful.
Maculopapular Rashes
Measles
Measles rash is blotchy, red or pink in color, raised in places, and starts behind the ears and on the face, spreading downwards.
The lesions tend to become confluent on the upper part of the body and remain more discrete lower down. The rash fades, usually
after 2–3 days. The skin becomes brown and although desquamation occurs, this is not usually seen on the hands and feet as it is in
the case of scarlet fever.
Rubella
Rubella results in a pink rash which progresses caudally. The lesions are normally discrete and the rash develops more quickly and
disappears earlier than in measles. Desquamation is not a characteristic.
Scarlet fever
The eruption is dark red and punctiform
The rash tends to be most prominent on the neck and in the major skinfolds
A distinctive feature is circumoral pallor as a result of the rash sparing the area around the mouth. As with measles, desquamation is
seen but the hands and feet are involved
True scarlet fever is associated with inflammation of the tongue (white and red strawberry tongue)
Scarlatina refers to the rash which may occur alone in milder streptococcal infection, and is often shortlived.
Pediatrics 229
Kawasaki disease
Although several features are required for the diagnosis of this condition, which is of unknown etiology, the rash may be confused with
that of scarlet fever. Discrete red maculopapules are seen on the feet, around the knees and in the axillary and inguinal skin
creases. Desquamation of the hands and feet is a common feature.
Erythema infectiosum or fifth disease
Infection caused by parvovirus B19 is associated with a rash which develops in two stages
The cheeks appear red and flushed, giving rise to a ‘slapped cheek’ appearance. A maculopapular rash develops 1–2 weeks later,
predominantly over the arms and legs which, as it fades, appears lace-like
Causes nonimmune hydrops is caused by parvo virus.
Roseola infantum
The main cause is human herpesvirus 6 (HHV-6)
Roseola infantum is characterized by a widespread morbilliform (measles-like) rash, seen in its most florid form on the trunk. The
lesions tend to be discrete. As the rash appears, the fever, which is normally present over the previous 4 days, resolves and the child
looks well (in contrast to measles, in which the child is febrile and unwell when the rash appears).
Viral infections
Many viral infections, particularly those associated with the enteroviruses, may cause maculopapular rashes.
Kawasaki Disease
A 2-year-old boy was admitted to a pediatric hospital with a 7-day history of high fever, lymphadenopathy, conjunctivitis and an
erythematous exfoliative rash affecting his trunk and extremities.
Types of Rash
•• Anaphylactoid purpura •• Vasculitis
•• Dermatomyositis •• Violaceous
•• Salmonella •• Evanescent
•• Erythema infectiosum •• Slapped face appearance
•• Infectious mononucleosis •• Drug (ampicillin) induced rash
230 USMLE Step 2 CK Platinum Notes
Henoch-Schönlein Purpura
A 7-year-old boy from Kansas is brought to the emergency department. His parents assert that over the past 3 days, he has
become progressively ill with generalized fatigue and mid-abdominal pain that have become steadily worse. On examination,
he has a maculopapular rash on his thighs and feet with some spread of the rash to his buttocks. The rash does not blanch and
some lesions near the ankles look petechial. His temperature is 103.4 °F. He has dark stool, which is positive for occult blood.
The boy has not voided since early morning and is unable to provide a urine sample. The physician determines that the boy is
dehydrated.
•• Also referred to as anaphylactoid purpura, is a distinct systemic vasculitis syndrome that is characterized by:
–– Palpable purpura (most commonly distributed over the buttocks and lower extremities)
–– Arthralgias
–– Abdominal pain
–– Glomerulonephritis
•• It is a small vessel vasculitis
•• Henoch-Schönlein purpura is usually seen in children; most patients range in age from 4 to 7 years
•• A seasonal variation with a peak incidence in spring has been noted
•• Ig A is the antibody class most often seen in the immune complexes and has been demonstrated in the renal biopsies of these
patients
•• Gastrointestinal involvement, which is seen in almost 70% of pediatric patients, is characterized by colicky abdominal pain
usually associated with nausea, vomiting, diarrhea, or constipation and is frequently accompanied by the passage of blood and
mucus per rectum; bowel intussusception may occur rarely
•• It usually resolves spontaneously without therapy. Rarely, a progressive glomerulonephritis will develop
•• Renal failure is the most common cause of death in the rare patient who dies of Henoch-Schönlein purpura
•• Routine laboratory studies generally show a mild leukocytosis, a normal platelet count, and occasionally eosinophilia. Serum
complement components are normal, and IgA levels are elevated in about one-half of patients
•• Treatment: The prognosis of Henoch-Schönlein purpura is excellent. Most patients recover completely, and some do not
require therapy.
Kawasaki Disease
•• Kawasaki disease (also known as ‘lymph node syndrome’, ‘mucocutaneous node disease’, ‘infantile polyarteritis’)
•• Kawasaki disease is an inflammation (vasculitis) of the middle-sized arteries
•• Kawasaki disease affects many organs, including the skin, mucous membranes, lymph nodes, and blood vessel walls, but the ‘most
serious’ effect is on the heart where it can cause ‘severe aneurysmal dilations’.
Classically, five days of fever plus four of five ‘diagnostic criteria’ must be met in order to establish the diagnosis
•• The criteria are:
Treatment:
Intravenous immunoglobulin + aspirin. Long-term therapy: aspirin. Only condition in children in which aspirin is given. (Causes
Reye’s syndrome.)
Pediatrics 231
Rashes
•• Mnemonic ‘’V’ery ‘S’ick ‘P’atient ‘M’ust ‘T’ake Double Tea.
•• Day 1 of fever: ‘v’aricella zoster
•• Day 2 of fever: ‘s’carlet fever
•• Day 3 of fever: small’p’ox
•• Day 4 of fever: ‘m’easles
•• Day 5 of fever: ‘t’yphus
•• Day 6th- dengue fever
•• Day 7th- typhoid
Kawasaki Disease
•• It is a systemic vasculitis of unknown origin that remains a leading cause of acquired heart disease in infants and children
•• It is a multisystemic disease, also known as mucocutaneous lymph node syndrome. An (unidentified) infectious origin and a T-cell
immune activation play a prominent role in disease pathogenesis
•• Tumor necrosis factor alpha (TNF-alpha) receptor levels correlate with the degree of vascular damage and likelihood of coronary
artery aneurysm formation
•• The cardiovascular complications account for most of the morbidity and mortality
•• Fever, bilateral nonexudative conjunctivitis, mucous membrane changes (injected pharynx, cracked lips, or strawberry
tongue), extremity changes (edema, desquamation, erythema, or rash), and cervical adenopathy are common at
presentation.
232 USMLE Step 2 CK Platinum Notes
•• The acute manifestations include myocarditis, valvular insufficiency, arrhythmias, pericardial effusion, and congestive heart
failure with coronary abnormalities; these develop in 15% to 25% of patients
•• Leukocytosis and an elevated C-reactive protein are associated with the development of coronary artery aneurysms
•• Treatment includes aspirin at 80-100 mg/kg/day in divided doses and IV immune globulin in high doses
•• Corticosteroids should be avoided because of the potential link with increasing the likelihood of coronary aneurysm development.
CHROMOSOMAL SYNDROMES
Down’s Syndrome
•• Common trisomy
•• Head circumference is small
•• Brachycephalic skull
•• The neck is short and thick
•• Hypotonicity is present
•• The palpebral fissures slope upwards (i.e. the outer canthus is higher than the inner canthus) and there may be marked epicanthic
folds (HYPOTELORISM)
•• Brushfield’s spots (whitish spots scattered round the periphery of the iris)
•• ↑ Nuchal fold thickness
•• There is an increased incidence of lens opacities. The ears are small with an overfolding helix. The nasal bridge is flat.
•• The tongue appears large and may protrude because the mouth is relatively small. Eruption of the teeth is frequently delayed
with abnormalities in dental positioning. The hair may be fine and sparse
•• Hypothyroidism and VSD can be present
•• Transient myeloproliferative disorder of newborn is seen
•• The hands are short and broad. The fifth finger is short and incurved (clinodactyly). Radiologically this feature is accompanied by
shortening of the shaft of the middle phalanx
•• Iliac index < 60
•• A single transverse palmar crease (or simian crease)
•• A deep plantar crease (sandal gap) between the first and second toe may also be a helpful diagnostic sign
•• Atrioventricular canal and ventricular septal defects are the commonest types of cardiac lesion seen. Intestinal atresia, in
particular, duodenal atresia. Leukemia (CML) is also common
•• Transient myeloproliferative disorder is seen
•• Initially a child with Down may be hypotonic, but the early developmental milestones are eventually reached. Hypothyroidism is
common
•• The ultimate IQ ranges from 20 to 75 with a mean around 50. The earlier assessment of development tends to be more favorable
than the formal measurement of IQ in later childhood
•• It is associated with Alzheimer’s dementia
•• Intellectual function shows a decline with age and this has been attributed to a presenile dementia
•• Triple test is done for Down’s syndrome
•• ↑nuchal fold thickness is a feature of Down’s syndrome
•• Maternal nondisjunction is the mc cause
A 48-year-old woman from Holland delivers a 3126 g newborn male. Her pregnancy was normal except that she noted decreased fetal
movement compared to her previous pregnancies. She declined an amniocentesis offered by her obstetrician. Physical examination
of the newborn reveals an infant with facial features suggestive of Down’s Syndrome. The infant then has bilious vomiting. An X-ray
film showing the kidneys, ureters, and bladder (KUB) is performed, which shows a distended and gas-filled stomach and proximal
duodenum and the absence of gas in the distal bowel.
Pediatrics 233
4p–(WOLF-HIRSCHHORN) Syndrome
•• The features include
–– Microcephaly
–– Hypertelorism
–– Low set simple ears
–– Coloboma cleft palate, renal abnormalities, heart defects and intrauterine growth retardation
•• The facial features are described as resembling a Greek helmet since the flat nasal bridge appears to run in continuity from the glabella
in much the same way that the protective nose piece is incorporated into a Greek helmet.
18q–syndrome
•• The cardinal features are intrauterine growth retardation, profound mental retardation and a ‘carp-shaped mouth’
•• Cleft palate, heart defects and ocular defects are also common. Genital hypoplasia may occur
•• Serum immunoglobulin A (IgA) is decreased in about half the affected individuals.
Deletions of 13q
•• Deletions of 13q14 are associated with retinoblastoma
•• They also characteristically have hypoplastic thumbs and anal atresia
A 5-year-old boy suffers from a condition characterized by recurrent fungal and viral infections, thymic hypoplasia, tetany, and
abnormal facies. Serum levels of immunoglobulins are mildly depressed, and lymph node biopsy shows lymphocyte depletion of
T-dependent areas.
•• The constellation of thymic hypoplasia, hypocalcemia with (Tetany) abnormal facies and congenital cardiac anomalies
defines the condition known as DiGeorge syndrome
•• This results from a developmental failure of third and fourth pharyngeal pouches, which gives rise to congenital absence or
anomalies of the parathyroid, thymus, lower face, and cardiac structures
•• Immune deficiency results from failure of T-lymphocytes to mature in the thymus. Thus, fungal and viral organisms, which are
normally controlled by T-mediated mechanisms, become frequent causes of opportunistic infections
•• The underlying gene defect is related to 22q11 deletion, which results in two partially overlapping conditions.
Williams Syndrome
It was initially referred to as infantile hypercalcemia as the biochemical change was the usual presentation, ‘elfin facies’
•• Short stature
•• Supravalvular aortic stenosis
•• Hyperacusis
•• Developmental delay
Prader-Willi syndrome usually presents with neonatal hypotonia with feeding difficulties and in males, undescended testes
Angelman syndrome presents with severe developmental delay, inappropriate laughter and complex seizures
•• Prader-Willi syndrome is due to paternal deletions
•• Angelman syndrome is due to maternal deletions
This phenomenon would not be predicted by Mendelian inheritance where the paternal origin of a mutation does not usually
matter and is an example of genetic imprinting. It appears that both paternal and maternal chromosome 15s are required in early
embryonic development and that paternal and maternal chromosome 15q12 have different functions. Thus, genetic imprinting
may be defined as the determination of gene expression depending on the parental origin.
•• Finger deformities
•• Coarctation of the aorta may also be present
•• Renal tract abnormalities, which include horseshoe kidneys and duplex ureters
•• At puberty, it may present for the first time with primary amenorrhea and failure of secondary sexual development. Streak
gonads are found with ultrasound and at laparotomy. The patients are almost invariably sterile, but menstruation and secondary
sexual development may be induced by estrogen replacement
•• Short fourth metacarpal
•• Have a normal life span
•• Patients with Turner’s syndrome have a 45XO karyotype
47XXX
•• The majority of girls with triple X (47XXX) will not have been brought to medical attention and have had their chromosomes tested.
However, prospective studies have identified some potential differences
•• Their height tends to be greater than average and in statistical terms, there is a slight lowering of mean IQ but no specific
abnormal behavioral patterns
•• Fertility is normal
•• At puberty, however, the secondary sexual characteristics develop poorly and body fat tends to take on a feminine distribution
with gynecomastia in 50% of cases
•• Beard growth is minimal and in adult life, patients will rarely need to shave more than twice a week
•• The testes remain small and infertility may be the presenting feature in adult males. Sexual function is normal, although the libido
is reduced
•• The testosterone levels are low and the gonadotrophin levels elevated and testosterone replacement may be helpful
•• Testicular histology shows an increase in Leydig cells and interstitial fibrosis. Height is usually increased with relatively long
limbs
•• Subnormal intelligence
It is best diagnosed with a 47, XXY karyotype, although variants with 46, XY exist. This condition is characterized by small
testes, disproportionately long arms and legs (secondary to delayed epiphyseal plate closure in puberty), infertility, and
gynecomastia. Mental retardation and learning disability are common. Many men are not diagnosed until adulthood when
they seek medical attention for infertility. Our patient clearly has these traits. Patients with Klinefelter’s syndrome often have
an unexplained increase in follicle-stimulating hormone and luteinizing hormone. This increase occurs even in patients
with normal testosterone levels. Low follicle-stimulating hormone, luteinizing hormone and testosterone are typical for a
secondary form of hypogonadism. Secondary hypogonadism is not associated with gynecomastia because patients don’t
have an increase in conversion of testosterone to estradiol, which is stimulated by follicle-stimulating hormone and luteinizing
hormone.
Torch Infections
The acronym TORCH has been used to summarize the infections toxoplasma, rubella, CMV and herpes, to which should
be added human immunodeficiency virus and varicella zoster.
•• Rubella
Rubella infection in the first 16 weeks of pregnancy can result in a severely damaged baby with:
•• Cataracts
•• Sensorineural deafness
•• Congenital heart disease (PDA), ASD, VSD, PDA
•• Microcephaly
•• Hepatosplenomegaly
•• Thrombocytopenia
The condition is preventable by achieving high immunization coverage.
Pediatrics 237
•• CMV
Can present in a dramatic form with jaundice and petechiae, the ‘blueberry muffin’ baby. Most babies who are symptomatic at birth
will be handicapped. Only small percentage of babies born with congenital CMV infection are symptomatic. However, most babies are
being clinically unaffected by the infection.
•• Varicella zoster
Can result in severe damage with cicatricial skin scarring, cataracts and seizures.
•• Herpes simplex virus
Can result in severe illness in newborns. There is a high incidence of meningitis and encephalitis.
•• Human immunodeficiency virus
This retrovirus can cause meningitis and encephalopathy. Neurological manifestations include encephalopathy and microcephaly.
•• Toxoplasma gondii
–– This protozoan can cause neurological sequelae when acquired in uterine life and the diagnosis is important because the
condition is treatable
–– The organism is mainly acquired from cat litter or consumption of undercooked meat
–– The classic is of hydrocephalus, chorioretinitis and intracranial calcification is seen
–– MC Manifestation: Chorioretinitis
Syphilis
Early Congenital Syphilis Snuffles or rhinitis is the early feature
•• Vesico bullous lesions (not found in any other variety of syphilis)
•• Snail track ulcers on oral mucosa
Late Congenital Syphilis
•• Hutchinson’s triad (Hutchinson’s teeth + 8th cranial nerve deafness+ Interstitial keratitis)
•• Moon’s molars and maldevelopment of the maxilla resulting in ‘bulldog’ facies are typical
•• Saddle nose
•• Saber tibia
•• Mulberry molars
•• Rhagades The infant may fail to thrive and have a characteristic ‘old man’ look, with fissured lesions around the mouth (rhagades)
and a mucopurulent or blood-stained nasal discharge causing snuffles
•• Frontal bossing of Parrot
•• Peroistitis (Higoumenakis sign)
•• IgM FTA ABS best diagnoses congenital syphilis
Jaundice in Pediatrics
Causes of ‘Unconjugated’ hyperbilirubenimia are:
•• Criggler najar Syndrome I, Criggler najar Syndrome II
•• Hemolytic anemia
•• Physiological jaundice
•• Hypothyroidism
•• Breast milk jaundice
•• Excessive RBC destruction (Rh incompatibility, G6PD deficiency, spherocytosis, vitamin K, sulfamethoxazole)
Remember
•• Jaundice appearing on the first day in any newborn and a bilirubin concentration >10 mg/dl in premature infants or >15 mg/dl in
full-term infants warrant investigation
•• Physiological jaundice usually appears on 3rd day and disappears by 7th day
•• When the blood level of bilirubin is about 4 to 5 mg/dl, jaundice becomes apparent
•• In kernicterus, unconjugated bilirubin is increased
•• In kernicterus, staining of brain is more in basal ganglia
•• With increasing bilirubin levels, visible jaundice advances in a head-to-foot direction
•• MC cause of jaundice in a newborn within 24 hours is erythroblastosis fetalis
•• Breast milk jaundice is due to pregnanediole
Physiological Jaundice
•• Jaundice not present on day 1
•• Total bilirubin level rises < 5 mg/dl/day
•• Total bilirubin does not exceed 13 mg/dl in term infants and 15 mg/dl in preterm infants
•• Jaundice should not last more than one week in term infants or 2 weeks in preterm infants
Gilbert’s Syndrome
•• Common, benign, inherited condition
•• Unconjugated hyperbilirubinemia
•• Common in males
•• ↓UDP glucoronyl transferase activity
•• Normal LFT
•• Normal liver histology
•• Stress, fatigue, alcohol use, ↓ caloric intake, illness ↑ serum bilirubin
Alagille Syndrome
•• Alagille syndrome is an autosomal dominant (AD) disorder
•• There is an error in development of bile ducts, heart, CNS and face
•• Neonatal intrahepatic cholestasis and hepatosplenomegaly with xanthomas with teratology of fallot and coarctation of
aorta are a feature
•• Supplementation of vitamins A, D, E, K and ursodeoxycholic acid is initially used for treatment
•• Liver transplant is selectively used
•• A liver biopsy may indicate too few bile ducts (bile duct paucity)
Other signs of Alagille syndrome include:
•• Congenital heart problems, particularly tetralogy of fallot
•• Unusual butterfly shape of the bones of the spinal column
•• Many people with Alagille syndrome have similar facial features, including a broad, prominent forehead, deep-set eyes, and a small
pointed chin. The kidneys and central nervous system may also be affected.
A full-term infant is brought to the office on her 6th day of life because her mother noted that she looked ‘yellow’. The mother states
that the infant is strictly breast-fed and has been eating every 2-3 hours. On examination, she is noted to be jaundiced over her trunk
and face. There is no scleral icterus. She is otherwise healthy. Both the mother and baby are Rh positive. Which of the following is the
most likely cause of this infant’s jaundice?
Breastfeeding jaundice
•• Symptoms of projectile vomiting occurring 10–20 min after a feed develops between the second and fourth weeks of life,
although they can occasionally occur either sooner or at up to 4 months of age
•• With progressive vomiting, the infants lose weight and may eventually become dehydrated and alkalotic (hypokalemic
metabolic alkalosis)
•• Compensation is by: paradoxical aciduria with hyponatremia and hypochloremia
•• On clinical examination, gastric peristaltic activity may be seen, and palpation of the right upper quadrant of the abdomen
during a test feed will reveal the pyloric tumor in most cases
•• The initial management of the patient after the diagnosis has been confirmed is intravenous rehydration and correction of any
acid-base disturbance
•• The patient should then undergo a Ramstedt’s pyloromyotomy where the pyloric muscle is split down to the mucosa.
Pyloric Stenosis
A four-week-old boy is brought to the pediatric emergency department by his father who states that he has been vomiting after
being fed for the past several days. She describes the vomitus as nonbilious, and he has had normal stools with no blood in them. On
examination, the infant appears to be mildly dehydrated, his abdomen is soft, and there is a palpable, olive-sized, firm movable mass
in the right upper quadrant.
Cystic Fibrosis
•• Cystic fibrosis is an autosomal recessive condition
•• The underlying defect results from one of a number of possible mutations, in a chloride transporter protein which lines ductular
epithelium
•• As a result, abnormal sweat chloride test
•• Lung is normal at birth
•• This results in inspissation of mucus in the ducts of the pancreas and in the bronchial and biliary trees with the resulting
development of pancreatic insufficiency, chronic respiratory disease and biliary cirrhosis
•• Causes intractable diarrhea in children
•• Pancreatic insufficiency: The inspissated concretions result in distension of the ducts and acini of the pancreas which with time
progress to the formation of small cysts with destruction and fibrosis of the exocrine tissue. As a consequence of this, the output
of pancreatic enzymes and bicarbonate falls, leading to pancreatic insufficiency
•• Patients with cystic fibrosis have pancreatic insufficiency with resulting malabsorption and steatorrhea
Meconium Ileus
•• Fibrosis may present in the first day or two of life with vomiting and abdominal distension due to meconium ileus
(mucoviscidosis)
•• This obstruction to the small intestine with thick tenacious meconium may be complicated by volvulus, atresia or peritonitis
•• Plain abdominal X-ray will show dilated loops of intestine with meconium, outlined by trapped air, present in the obstructed
segment of bowel and
•• Gastrografin enema will reveal a microcolon distal to the obstruction.
Pediatrics 241
Shwachman’s Syndrome
•• This autosomal recessive condition is characterized by the presence of neutropenia with impairment of neutrophil function
•• The diagnosis should be considered in any child with pancreatic insufficiency, a normal sweat test and neutropenia.
•• Aspermia and infertility are seen in 98% of adult men secondary to maldevelopment of the vas deferens. In women, fertility is
decreased secondary to viscid cervical secretions, but many women with CF have carried pregnancies to term. However, maternal
complications and fetal wastage show an increased incidence.
–– In infants without meconium ileus, onset is frequently heralded by a delay in regaining birthweight and inadequate weight
gain at 4 to 6 weeks of age.
–– Pancreatic insufficiency is clinically apparent in 85 to 90% of patients. It is usually present early in life but may be progressive.
–– Manifestations include the passage of frequent, bulky, foul-smelling, oily stools; abdominal protuberance; and poor
growth pattern with decreased subcutaneous tissue and muscle mass, despite a normal or voracious appetite.
–– Rectal prolapse occurs in 20% of untreated infants and toddlers.
–– In arid climates, infants may present with chronic metabolic alkalosis. Findings of salt crystal formation on the skin and a salty
taste on the skin are highly suggestive of CF.
–– Fifty percent of all patients present with pulmonary manifestations usually consisting of chronic cough and wheezing
associated with recurrent or chronic pulmonary infections.
Laboratory Findings
•• Since meconium in most neonates with CF contains large amounts of serum proteins (especially albumin), meconium examination
has been used as the basis of a newborn screening test. Pancreatic insufficiency is present in about 85% of patients with CF
•• The serum concentration of immunoreactive trypsin is elevated in newborns with CF
Intussusception
•• Intussusception is one of the acute abdominal emergencies occurring in children. In children, three months to three years of age
without prior abdominal surgery, intussusception is the most common cause of an acquired intestinal obstruction. It is believed
that enlarged intramural lymphoid tissue (Peyer’s patches) in the small bowel may act as a lead point for the development of an
idiopathic intussusception with invagination of the distal small bowel into the cecum and colon. Other structural lesions such as
a Meckel’s diverticulum, duplication cyst, polyp or lymphoma may be lead points for an intussusception. However, these
associated lead points are more frequently identified in children younger than three months of age, older than five years of age, or
in children with multiple recurrent episodes of intussusception
•• A child with intussusception typically has intermittent episodes of abdominal pain, emesis and bloody or ‘currant jelly’
stools. However, the clinical presentation may be varied, occasionally with diarrhea, a palpable abdominal mass or lethargy with
delay in treatment. Plain films may be diagnostic if a soft-tissue mass in the cecum or transverse colon is identified or strongly
suggestive of intussusception with a distal small bowel obstruction in a young child with no prior abdominal surgery. However,
plain films are frequently unrevealing. Therefore, if intussusception is suspected, clinically further imaging evaluation is warranted
•• Intussusception has a characteristic appearance on ultrasound (US). On transverse imaging through an intussusception, a
‘target’ or ‘donut’ sign is seen with a peripheral rim of hypoechoic tissue and a hyperechoic center. On longitudinal imaging
through the intussusception, the intussusceptum can be seen extending into the intussuscipiens.
Secondary causes:
•• Meckel’s diverticulum
•• Polyps/carcinomas
•• Duplication cysts
•• Lipoma
•• Purpura
•• Lymphoma
•• Intermittent colicky pain with screaming, vomiting with red currant jelly stools
•• Sausage-shaped periumbilical mass
•• Commonest cause of intestinal obstruction in children: intussusception
•• Empty right iliac fossa (signe de dance)
•• Step ladder peristalsis
•• Initially hydrostatic reduction is done
•• Laparotomy with reduction is done in complicated cases and those failing initially
Features of intussusception
Pediatrics 243
Hirschsprung’s Disease
•• There is absence of ganglion cells in the myenteric plexuses (both Auerbach’s and Meissner’s) of the most distal bowel and
extending proximally for a variable distance
•• Aganglionosis involving only the rectum or rectosigmoid is often termed ‘short segment’ Hirschsprung’s disease and affects
males five times more commonly than females
•• ‘Long segment’ Hirschsprung’s disease, extending above the sigmoid
•• Associated congenital anomalies are:
–– Down’s syndrome
–– Waardenburg’s syndrome
–– Cartilage-hair hypoplasia syndrome
•• Usually the symptoms of Hirschsprung’s disease are manifest within the first few days of life
•• Failure to pass meconium within the first 24 h, abdominal distention, bile-stained vomiting and reluctance to feed are the main
symptoms
•• Diarrhea may be the presenting feature of Hirschsprung’s enterocolitis, a devastating complication of the condition which
has a high mortality
•• A plain abdominal X-ray shows distended small and large bowel, sometimes with multiple fluid levels on an erect film. Distal
segment is constricted, proximal segment is dilated
•• A barium enema is best carried out without a previous rectal examination as then the narrow aganglionic bowel with dilation
proximally is demonstrated
•• Definitive diagnosis is by rectal biopsy.
Duodenal Atresia
•• Associated with Down’s syndrome
•• Atresia at the level of ampulla of vater
•• Double bubble sign on radiograph
•• Mc cause of acute intestinal obstruction in neonates
•• Duodenojujenostomy is the procedure of choice
Reye’s Syndrome
Reye’s syndrome is the best known example of secondary mitochondrial hepatopathy
Mitochondrial changes include:
•• Microvesicular steatosis
•• Swelling of mitochondrial matrix
•• Dissolution of cristae/appearance of granules
•• Ameboid shapes
244 USMLE Step 2 CK Platinum Notes
Wilson’s Disease
It is an AR disorder
•• Chromosome 13 is involved
•• The diagnosis is confirmed by the demonstration of either:
–– A serum ceruloplasmin level < 20 mg/dl and Kayser-Fleischer rings or
–– A serum ceruloplasmin level < 20 mg/dl and a concentration of copper in a liver biopsy sample >250 mg/g dry weight
–– Most symptomatic patients excrete > 100 mg copper per day in urine and have histologic abnormalities on liver biopsy
Pathognonic features are:
–– Hepatic dysfunction: Jaundice, hepatomegaly, cirrhosis, portal hypertension
–– Eye changes: Brownish or grayish Kayser-Fleischer ring in Descemet’s membrane in cornea. These golden deposits of
copper in Descemet’s membrane of the cornea do not interfere with vision but indicate that copper has been released from
the liver and has probably caused brain damage
–– If a patient with frank neurologic or psychiatric disease does not have Kayser-Fleischer rings when examined by a trained
observer using a slit lamp, the diagnosis of Wilson’s disease can be excluded
–– Rarely, Kayser-Fleischer rings may be accompanied by sunflower cataracts
–– The neurologic manifestations include resting and intention tremors, spasticity, rigidity, chorea, drooling, dysphagia, and
dysarthria. Babinski responses may be present, and abdominal reflexes are often absent. Sensory changes never occur,
except for headache
–– Psychiatric disturbances are present in most patients with neurologic symptoms. Schizophrenia, manic-depressive psychoses,
and classic neuroses may occur, but the commonest disturbances are bizarre behavioral patterns that defy classification
Treatment consists of removing and detoxifying the deposits of copper as rapidly as possible and must be instituted once the
diagnosis is secure, whether the patient is ill or asymptomatic
•• Zinc is the DOC
•• Penicillamine
•• Trientine
•• Zinc acetate or gluconate are effective as maintenance therapy.
Pediatrics 245
Biliary Atresia
•• It is a disorder of unknown cause where a destructive sclerosing inflammatory process causes obstruction of the extrahepatic
bile ducts and extends into the major intrahepatic bile ducts
•• By 6–10 weeks of age, the portal blood pressure is increased. Cirrhosis rapidly develops
Clinical features:
•• The first sign is prolongation of jaundice. The urine is always yellow. The stools contain no yellow or green pigment
•• Conjugated hyperbilirubinemia
•• Acholic stools
•• Biliary atresia must be considered in any infant remaining jaundiced beyond 14 days of age
•• Percutaneous liver biopsy showing the features described above in all portal tracts is suggestive. Diagnosis of it is confirmed at
laparotomy
•• Complications: Cholangitis occurs in a significant minority of cases in the first 2 years after surgery. Portal hypertension is present
in almost all cases
•• The prognosis is transformed by the Kasai operation and with liver.
A four-month-old presents with cholestasis. Biopsy shows inflammation and fibrosing stricture of the hepatic or common bile ducts;
inflammation of major intrahepatic bile ducts, with progressive destruction of the intrahepatic biliary tree, marked bile ductular
proliferation, portal tract edema and fibrosis, and parenchymal cholestasis); and periportal fibrosis and cirrhosis.
Choledochal Cyst
•• In this disorder, there is enlargement or dilatation of part or all the extrahepatic biliary system
•• Bile flow may be obstructed with intrahepatic changes similar to those of biliary atresia, progressing to cirrhosis
•• More common in females
–– Recurrent abdominal pain
–– Episodic jaundice
–– Right upper quadrant mass
–– Or silent up to childhood are features
•• Type 1 (dilatation of common bile duct) is commonest
•• Cholangitis
•• Cyst rupture
•• Pancreatitis
•• Gallstones
•• Carcinoma of the cyst wall
•• In infancy, it presents with features of neonatal cholestasis
–– In older children, there may be recurrent upper abdominal pain, recurrent jaundice and/or a palpable cystic mass but the
classical triad is rare
–– The treatment is surgical removal with biliary drainage via a Roux-en-Y loop
Caroli’s Disease
•• Congenital, autosomal recessive disorder
•• Communicating cavernous ectasia of intrahepatic biliary ducts. Kindly note this point. It differentiates the disease from
choledochal cyst
•• Cystic structures (multiple) converging towards porta hepatis
•• Central dot sign on CT scan
•• Cholangiography most diagnostic
•• Congenital hepatic fibrosis and cystic (polycystic) kidney disease are associations.
Clinical Scenarios. Highlighted text underlined. Don’t look at the answers initially. Try to make diagnosis reading the left
column only.
•• Within 6 hours of birth a baby with excessive salivation. NG tube insertion shows tube •• TE fistula
coiling back and air in GIT
•• A newborn is found to be tachypneic, cyanotic and grunting. Abdomen is scaphoid •• Congenital diaphragmatic hernia
with bowel sounds heard in chest
•• A newborn with green-colored vomitus in first day of life not passing meconium. X-ray •• Intestinal atresia
shows multiple air fluid levels and distended bowel
•• A prematurely born baby has feeding intolerance and early abdominal distension •• Necrotizing enterocolitis
•• A 2-year-old boy has fractured humerus, subdural hematoma and retinal hemorrhages •• Suspect child abuse
•• A 3-month-old has chronic constipation, abdominal distension. X-ray shows dilated •• Hirschsprung’s disease
bowel loops
•• A 3-week-old has non bilious, projectile vomiting with visible peristalsis and mass in •• Hypertrophic pyloric stenosis
right upper abdomen
•• A 7-year-old with bloody bowel movement •• Meckel’s diverticulitis
A 12-year-old boy has a productive cough characterized by large volumes of foul-smelling sputum. Three years ago, the patient was
diagnosed with pancreatic insufficiency, as evidenced by repetitive gastrointestinal symptoms of steatorrhea. After culture of the
sputum, colorless, oxidase-positive colonies with a fruity aroma develop on the agar. Most likely cause is:
Cystic Fibrosis
Pneumatosis Intestinalis
•• Pneumatosis intestinalis, defined as gas in the bowel wall, is often first identified on abdominal radiographs or computed
tomography (CT) scans
•• It is a radiographic finding and not a diagnosis, as the etiology varies from benign conditions to fulminant gastrointestinal disease
•• In neonates, pneumatosis is usually secondary to NEC and indicates a later stage of the disease
•• NEC is seen almost exclusively in premature infants and is associated with bowel ischemia caused by bacterial invasion and
hyperosmolar feeds. Mesenteric vascular occlusion leads to ischemic damage of the mucosa, which allows luminal gas and bacteria
to invade the bowel wall
•• Pneumatosis intestinalis is considered an ‘ominous finding’ in ischemia, especially in association with portomesenteric venous
gas.
Pediatrics 247
RESPIRATORY SYSTEM
Choanal Atresia
•• This rare anomaly is due to failure of breakdown of the nasobuccal membrane which normally occurs at 6 weeks’ fetal
development
•• Of the cases, 50% are associated with the CHARGE syndrome – choanal atresia with ear, eye, heart and genital defects
•• Gasping respiration in a neonate is suggestive of choanal atresia
•• Bilateral choanal atresia is a neonatal emergency
•• The diagnosis is by suspicion, by inability to pass a catheter along the nose and confirmation by endoscopic examination
•• The treatment consists of establishment of an airway, either oral or orotracheal.
A newborn infant is in respiratory distress and requires several attempts at resuscitation in the delivery room because of difficulty
breathing and frequent cyanosis. The neonatologist notes that during crying, her breathing improves and breath and heart sounds
are normal. Direct laryngoscopy is unremarkable as well. Deep inspirations by the neonate are ineffective. Which of the following is
the most effective intervention?
The patient most likely has choanal atresia, which is the presence of a congenital membrane between the nose and the pharynx.
Since most newborns are obligate nose breathers, spells of crying force mouth breathing, improving the ventilation. If unilateral, the
atresia may not cause symptoms. Intubation via the oropharynx will provide immediate relief and surgery should then be performed
to correct the atresia.
Foreign-Body Aspiration
A 2-year-old girl is rushed to the emergency department by her parents because of an acute attack of wheezing. At one point, they
noticed that something was wrong because she was gasping for breath and became very frightened and quiet. They looked in her
mouth, thinking she had swallowed something, as there were lots of snacks and food lying around, but they could not see anything.
The patient has an unremarkable medical history with no prior respiratory problems. She is not allergic to any foods or drugs. On
physical examination, the patient is in severe respiratory distress, drooling and continuously attempting to swallow. Nasal flaring and
intercostal retractions are present. She appears cyanotic. Breath sounds are decreased bilaterally.
Foreign-body aspiration is most commonly seen in children 1 to 3 years of age because of the size of the trachea, the lessening of
parental supervision secondary to increased independence of the child, and the tendency of small children to put objects in their
mouths. Foreign bodies include hot dogs, peanuts, beans, coins, buttons, nuts, deflated balloons, etc. When aspirated, these items
may go to the airway or the esophagus.
Diaphragmatic Hernia
•• The commonest herniation is the Bochdalek type, a posterolateral defect, possibly a failure of closure of the pleuroperitoneal
canal
•• A hernia through the Foramen of Morgagni (Retrosternal) is less common in neonates. This defect is retrosternal, to the right
or left of the midline
•• The third site for herniation is the Esophageal hiatus—the so-called hiatus hernia
•• Majority present with respiratory distress—cyanosis, dyspnea and tachypnea—either immediately after birth or within a few
hours
•• Most common on left side
•• Examination reveals a scaphoid abdomen, bowel sounds on auscultation of the affected side of the chest and a shift of
the apex beat to the right in the case of a left-sided hernia. Once air has been swallowed after birth, a chest X-ray confirms
the diagnosis, showing gas-filled loops of bowel on the affected side of the chest with displacement of the mediastinum to the
opposite side
•• Treatment: A nasogastric tube is passed to reduce the gaseous distention of the bowel with air.
Bronchiolitis
•• Caused by RSV mainly. Commonly in children less than 2 years
•• Self-limiting
250 USMLE Step 2 CK Platinum Notes
•• Ribavirin DOC
•• Leads to asthma
•• Humidified oxygen
Obliterative Bronchiolitis
•• Obliterative bronchiolitis (OB) has been described in association with infections caused by adenovirus types 3, 7 and 21; measles,
pertussis, influenza A, mycoplasma; after lung transplantation; and secondary to chronic aspiration
•• The clinical course is one of cough, wheeze and tachypnea in an infant or young child, which may be static if the cause is an
infection, or progress at a variable rate to respiratory failure and failure to thrive if there is ongoing aspiration
•• Chest radiography initially shows generalized hyperinflation. HRCT demonstrates patchy air trapping, particularly if the scan
is expiratory
•• Focal hyperlucency involving a lobe or a whole lung represents the radiological appearance of MacLeod’s syndrome, in which the
affected lung appears small and hyperlucent
•• Long-term oxygen dependency is common
•• Improvement after 2–3 years may be seen. The condition is usually stable once the initial effects of the devastating infection have
burnt out, but some may go on to require lung transplantation. OB developing after lung transplant is eventually fatal and the
results of second transplantation are very poor.
Follicular Bronchiolitis
•• Follicular bronchiolitis (FB) is part of the spectrum of lymphoid disorders of the lung, which includes lymphoid interstitial
pneumonia (LIP)
•• There is a polyclonal expansion of lymphoid tissue, probably part of the bronchus-associated lymphoid tissue (BALT), which
causes small airway obstruction
•• Presentation is with chronic respiratory distress, and HRCT demonstrates air trapping, sometimes with associated infiltrates.
Pulmonary lymphangiectasia
•• This typically presents with relentlessly progressive respiratory distress in a term baby
•• CXR may show dilatation of lymphatics and pleural effusions.
Pulmonary alveolar microlithiasis
•• This condition, which may be inherited, has a nonspecific presentation but a classical sandstorm appearance on CXR
•• Calcium carbonate stones are formed within the alveoli; the differential diagnosis is other form of pulmonary calcification
•• Pulmonary fibrosis may develop many years later when the patient becomes symptomatic.
•• Asphyxiating thoracic dystrophy (Jeune’s syndrome): This is a rare disorder of the costal cartilages in which the ribs are
shortened and the rib cage narrowed so that lung development is retarded and the lungs are small.
Kartagener’s Syndrome
•• Situs inversus in Kartagener’s Syndrome
It is assumed that during early embryonic life, ciliary beats in the growing embryo determine the type of laterality. When ciliary
movements are absent, laterality may develop fortuitously, thus affecting a situs inversus in about half the affected cases
•• Infertility in Kartagener’s Syndrome
Infertility is common due to defective ciliary action in the fallopian tube in affected females or diminished sperm motility in affected
males
•• The Chest in Kartagener’s Syndrome
Mucociliary clearance is an important primary innate defense mechanism that protects the lungs from deleterious effects of inhaled
pollutants, allergens and pathogens (bronchiectasis)
The main consequence of impaired ciliary function is reduced or absent mucus clearance from the lungs, and susceptibility to chronic
recurrent respiratory infections, including sinusitis, bronchitis, pneumonia and otitis media.
1. Ciliated columnar epithelia move mucus and other substances via cilia, and are found in the upper respiratory tract, the Fallopian
tubes, the uterus and the central part of the spinal cord.
2. Mucociliary clearance is an important primary innate defense mechanism that protects the lungs from deleterious effects of
inhaled pollutants, allergens, and pathogens.
3. Ciliated columnar epithelium lines the lumen of the uterine tube, where currents generated by the cilia propel the egg cell, toward
the uterus.
4. Cilia are also involved in maintaing left-right axis during embryogenesis.
Pediatric Cardiology
•• A 10-year-old girl with pulmonary flow systolic murmur and fixed split second hear sound ASD
•• A 3-year-old girl with failure to thrive has pansystolic murmur on left sternal border with prominent VSD
pulmonary vascular markings
•• A 5-day-old girl with machinery-like heart murmur and bounding peripheral pulses PDA
•• A 6-year-old with clubbing, cyanosis improved by squatting, systolic ejection murmur in 3rd intercostal Tetralogy of Fallot
space with right ventricular hypertrophy.
ASD:
•• Commonest: Ostium secondum defects
•• Systolic ejection murmur in left, mid and upper sternal borders
•• A wide fixed split of S2
•• Left parasternal heave is seen
•• CXR: Enlarged RA, RV not LA
•• Seen in Holt-Oram syndrome
VSD:
•• Holosystolic murmur at left lower sternal border
•• Thrill associated
•• LVF, biventricular hypertrophy with left axis deviation
•• Close spontaneously in 30-50% cases
PDA:
•• Failure of closure of ductus arteriosus
•• Blood flows from aorta to pulmonary artery both during systole and diastole
•• Wide pulse pressure
•• Seen in rubella
•• Hypoxia and prematurity predispose
•• ‘Differential cyanosis’ is seen
•• ‘Machinery murmur’ which is continuous. Heard at 2nd left intercostal space
•• CXR: Prominent pulmonary artery
•• PG inhibitors stimulate closure of PDA
Coarctation of Aorta
•• Mostly below origin of left subclavian artery
•• Acyanotic
•• Seen in Turner’s syndrome and associated with bicuspid aortic valve
•• Upper extremity, hypertension, headache, dizziness, intermittent claudication
•• Lower extremity hypotension
•• Ribnotching due to increased flow through collaterals
•• Ribnotching with double buldging is seen
•• Cause of death is usually complication of hypertension, CHF, endocarditis.
Trilogy of Fallot:
•• Pulmonary valve stenosis
•• Atrial septal defect
•• Right ventricular hypertrophy
254 USMLE Step 2 CK Platinum Notes
Tetralogy of Fallot:
•• Ventricular septal defect, communication between the two ventricles
•• Pulmonary stenosis, narrowing at the pulmonary valve or at the level of right ventricular infundibulum, which lies just below the
pulmonary valve
•• Overriding of the aorta, the aorta being positioned over the ventricular septal defect instead of in the left ventricle
•• Right ventricular hypertrophy
Pentalogy of Fallot:
•• The four characteristics of Fallot’s tetralogy syndrome, plus a patent foramen ovale or atrial septal defect.
•• Carey Coombs Murmur: Transient Soft middiastolic murmur of acute rheumatic fever due to mitral valvulitis, low-pitched
•• Austin Flint Murmur: Murmur of mitral stenosis at apex
•• Graham Steell Murmur: Early diastolic murmur of Pulmonary Regurgitation
•• Means Murmur: Pulmonary Systolic murmur in Thyrotoxicosis
•• Seagull’s Murmur: Aortic regurgitation murmur.
Kindly Memorize
•• Turner’s syndrome •• Coarctation of aorta
•• Noonan syndrome •• Pulmonary stenosis, ASD
•• Congenital rubella •• PDA
•• Marfan’s syndrome •• MVP
•• William’s syndrome •• Supravalvular aortic stenosis
•• Digeorges’s syndrome •• Aortic arch abnormalities
Marfan’s syndrome
•• It is an autosomal dominant disorder of the connective tissues
•• The genetic defect results in abnormal synthesis and secretion of fibrillin, which is an essential component in connective tissues
•• Patients with Marfan’s syndrome often have involvement in the skeletal, ocular and cardiovascular systems
•• Most of the morbidity and mortality with this disease are related to the cardiovascular manifestations; patients develop aortic root
dilatation. This may be associated with aortic insufficiency and aortic arch dissection. They may also have mitral valve prolapse
with significant hemodynamic consequences. Aortic root, aortic valve and mitral valve replacement may be needed during the
lifetime of the patient.
•• Patients with Marfan’s syndrome may frequently complain of chest pain; these patients must be viewed as having aortic dissection
until proven otherwise. In this population, a spontaneous pneumothorax is another potential cause of chest pain
•• Patients must avoid contact sports that cause acceleration-deceleration injury and isometric maneuvers that tend to increase
central blood pressure. Aerobic activity is encouraged, but those with established aortic root dilatation should have adequate
heart rate control with beta-blockers.
256 USMLE Step 2 CK Platinum Notes
Blood
•• Hematopoiesis is the process by which formed elements of the blood are produced
•• In the BM (Bone Marrow) first morphologically recognizable precursor is the PRO normoblast and ‘not normoblast’
•• Erythropoietin is produced by the peritubular cells within the kidney
•• There is daily replacement of 0.8–1% of all circulating red cells
•• ‘Hematopoiesis’ is the process by which formed elements of the blood are produced. Stem cells are capable of producing all
classes of cells
•• In the BM (Bone Marrow), first morphologically recognizable precursor is the’PRO normoblast.’ This cell can undergo 4–5 cell
divisions that result in production of 16–32 mature red cells
•• Hemoglobin first to appear in fetus: Hb Gowers
•• Switch over of fetal Hb to adult Hb begins at 14 week’s gestation
•• Fetal Hb is completely replaced by adult Hb by 6 months
•• Increased Fetal Hb is a feature of CML.
Iron-deficiency Anemia
Commonest type of anemia in children
Iron deficiency in children can be due to causes slightly different from adults:
•• Cow’s milk (poor source of iron)
•• Hookworm infestation
•• Polyposis colon
•• Meckel’s diverticulum
•• Malabsorption syndrome
Pediatrics 257
Thalassemias
(Mediterranean Anemia; Hereditary Leptocytosis; Thalassemia Major and Minor)
•• Fe deposits in heart muscle may cause dysfunction and ultimately heart failure
•• Hair on end appearance seen
•• Hepatic siderosis occurs typically, leading to functional impairment and cirrhosis
•• Electrophoresis is the diagnostic test.
258 USMLE Step 2 CK Platinum Notes
Thalassemia Major
•• ↑HbF
•• Normal Hb A2
•• Microcytosis and hypochromia
•• Anisocytosis and poikilocytosis
•• Target and tear drop RBC
•• Howel-Jolly bodies and Heinz bodies
•• Erythroblasts seen
α Thalassemia
Deletion of one α globin gene •• Silent hematologically
Deletion of two α globin gene •• Called α Thalassemia triat
•• Presents as mild microcytic anemia
•• ↑HbF
•• ↑Hb A2
Deletion of three α globin gene •• Hb H disease
•• Cases have chronic hemolysis with jaundice and hepatosplenomegaly
•• Gallstones and leg ulcers seen
Deletion of four α globin gene •• Hydrops fetalis occurs
•• Profound anemia
•• Most severe
LABORATORY TESTING: There are five tests commonly used to screen for vWD
•• Plasma vWF antigen (vWF:Ag)
•• Plasma vWF activity (ristocetin cofactor activity, vWF: RCo)
•• Factor VIII activity (FVIII) and activated partial thromboplastin time (APTT)
•• Platelet function analyzer (PFA) testing or bleeding time (BT)
If one or more of these tests is abnormal, or if there is still a high index of suspicion for vWD in the face of normal results, further testing
is carried out to evaluate the following:
•• vWF multimer distribution on gel electrophoresis
•• Ristocetin-induced platelet aggregation (RIPA)
Collagen-binding assay: Collagen binding has been introduced as a second type of functional assay for vWF (vWF:CB). The binding
of plasma vWF to collagen-coated plates is dependent upon the quantity of vWF in plasma and on the presence of higher molecular
weight multimers that bind more readily. This functional assay takes longer to perform, but is gaining wide acceptance in clinical
laboratories.
ITP
In many cases, the cause is not actually idiopathic but autoimmune, with antibodies against platelets being detected in approximately
60 percent of patients:
Most often these antibodies are against platelet membrane glycoproteins IIb-IIIa or Ib-IX, and are of the IgG type
•• The diagnosis of ITP is a process of exclusion
•• In approximately one percent of cases, autoimmune hemolytic anemia and immune thrombocytopenic purpura coexist.
This condition is called ‘Evans syndrome’
•• Bleeding time is prolonged in ITP patients
•• On examination of the bone marrow, an increase in the production of megakaryocytes may be observed and may help in
establishing a diagnosis of ITP
•• Platelet transfusions are not helpful in immunogenic variety.
260 USMLE Step 2 CK Platinum Notes
Renal System
Childhood Hypertension
•• Childhood hypertension is defined as arterial BP above 95th percentile with reference to age and sex
•• Essential hypertension is relatively uncommon in children
•• Causes of persistent hypertension in children: in decreasing order:
•• Chronic glomerulonephritis
•• Chronic pyelonephritis
•• Essential hypertension
•• Renal artery stenosis
•• Renal arteritis
•• Obstructive uropathy
Liddle’s Syndrome
Liddle’s syndrome is a rare familial disease with a clinical presentation of ‘hyperaldosteronism’, consisting of:
•• Hypertension (Compare with Bartter’s syndrome)
•• Hypokalemia
•• Metabolic alkalosis
Bartter’s Syndrome
Most cases of classic Bartter’s syndrome present during childhood
•• Symptoms such as weakness and cramps are secondary to hypokalemia
•• Polyuria and nocturia are common due to the hypokalemia-induced nephrogenic diabetes insipidus
•• Growth retardation may be seen.
Pediatrics 261
Nephrocalcinosis
•• Hypercalciuria
•• Hyperoxaluria
•• Distal renal tubular acidosis
•• Bartter’s syndrome
•• Dent’s disease
•• Primary hyperparathyroidism
Urachus
The urachus in the embryo connects the bladder to the allantois. It is normally obliterated to form the median umbilical ligament.
It may, however, persist as a patent urachus in the neonate, requiring repair. Occasionally the two extremities of the urachus close,
leaving a cyst in the middle which becomes filled with secretions and may present as a mass or, more commonly, when it becomes
infected, as an abscess.
VUR
•• Straining and dribbling of urine in a male child with recurrent UTI is suspicious of UTI
•• Mc cause of renal scarring in CHILDREN
•• Caused by congenital incompetence of vesicoureteral junction
Poststreptococcal Glomerulonephritis
•• Most cases are sporadic, though the disease can occur as an epidemic.
•• Glomerulonephritis develops, on an average, 10 days after pharyngitis or 2 weeks after a skin infection (impetigo) with a
nephritogenic strain of group Ab-hemolytic streptococcus.
•• The known nephritic strains include M types
•• Epidemic poststreptococcal glomerulonephritis is most commonly encountered in children of 2 to 6 years of age with pharyngitis
during the winter months.
•• The classic clinical presentation of poststreptococcal glomerulonephritis is full-blown nephritic syndrome with oliguric acute
renal failure; however, most patients have milder disease
•• Subclinical cases outnumber overt cases by four- to tenfold during epidemics.
Nephrotic Syndrome
The nephrotic syndrome is characterized by proteinuria of > 3.5 g per 1.73 m2 per 24 h (in practice, > 3.0 to 3.5 g per 24 h),
hypoalbuminemia, edema, hyperlipidemia, lipiduria, and hypercoagulability
•• Hyperlipidemia is a consequence of increased hepatic lipoprotein synthesis
•• Hypercoagulability is caused by increased urinary loss of antithrombin III, altered levels and/or activity of proteins C and
S, hyperfibrinogenemia due to increased hepatic synthesis, impaired fibrinolysis, and increased platelet aggregability.
Patients can develop spontaneous peripheral arterial or venous thrombosis, renal vein thrombosis, and pulmonary
embolism
•• Clinical features that suggest acute renal vein thrombosis include sudden onset of flank or abdominal pain, gross hematuria, a
left-sided varicocele (the left testicular vein drains into the renal vein), increased proteinuria, and an acute decline in GFR. Seen
most commonly in membranous glomerulonephritis
•• An increased susceptibility to infection may reflect low levels of IgG that result from urinary loss and increased catabolism
•• Low serum calcium is a feature
Six entities account for greater than 90% of cases of nephrotic syndrome in adults: Minimal change disease (MCD) is the commonest
cause. Focal and segmental glomerulosclerosis (FSGS), membranous glomerulopathy, MPGN, diabetic nephropathy, and amyloidosis
Renal biopsy is used for establishing a definitive diagnosis, guiding therapy, and estimating prognosis
Renal biopsy is not required in the majority of children with nephrotic syndrome as most cases are due to MCD and respond to
empiric treatment with glucocorticoids.
•• The onset is preceded by acute diarrheal or dysenteric illness by shigella or E. coli (Shigella dysentriae 1, Verocytotoxin producing
E. coli
•• Serum complement may be low
•• There is no significant direct involvement of CNS. Direct microvascular injury affects kidneys significantly.
Cat-Scratch Disease
•• Typical cat-scratch disease presents with painful regional lymphadenopathy persisting for weeks to months after a cat scratch
•• Only occasionally a generalized lymphadenopathy may occur
•• Tender regional lymphadenopathy occurs in 1–2 weeks after inoculation
•• Epitrochlear, axillary, pectoral and cervical lymph nodes may be affected
•• Lymph nodes become suppurative occasionally
•• Lymphadenopathy persists for weeks
•• Polyclonal hyperglobuliemia
•• Lymphocytic interstitial pneumonitis is seen
•• Ideal mode of delivery is cesarean section
•• Breastfeeding is contraindicated
264 USMLE Step 2 CK Platinum Notes
Hyperdiploidy
DNA index >16
PreB-cell type
Neuroblastoma
•• Most frequently presenting as a large abdominal mass and abdominal calcification
•• The commonest primary sites are the adrenal glands
•• It arises from primordial neural crest cells which form part of the sympathetic and rarely the parasympathetic nervous system
•• May present as lytic lesion in skull with sutural diasthesis
•• Spontaneous regression of malignant to benign ganglioneuroma occurs
•• It occurs with increased frequency in Beckwith-Wiedemann’s syndrome, neurofibromatosis, nesidioblastosis and in fetal
phenytoin syndrome
•• MC mass in postmediastinum of children
•• The commonest presentation is with a large firm abdominal mass often crossing the midline but with the features of marrow
infiltration including anemia, bruising, fever, lethargy and irritability. A syndrome of opsoclonus/myoclonus in which the patient
has acute cerebellar and truncal
•• Metastasize to bones most often
•• HVA, VMA, Catecholamines in urine are seen
•• Associated with hypertension
Wilms’ Tumor
•• Wilm’s tumor is the most common primary tumor of childhood
•• Wilms’ tumor is also called nephroblastoma
•• An embryonal adenomyosarcoma of the kidneys with heterogenous carcinomatous elements; the tumor occurs fetally and may
not manifest itself clinically for years
•• The diagnosis usually is made in children < 5 years of age, but the tumor occasionally can be detected in older children and rarely
in adults
•• The most frequent presenting finding is a palpable abdominal mass; other findings include abdominal pain, hematuria, fever,
anorexia, nausea, and vomiting
•• Hematuria (15 to 20% of cases) indicates invasion of the collecting system
•• Hypertension may occur secondary to ischemia from renal pedicle or parenchymal compression.
Rhabdomyosarcoma (RMS)
•• These are the commonest soft tissue sarcomas and arise from tissue which imitates striated muscle
•• They make up over 75% of the soft tissue sarcomas
•• Bladder and vaginal rhabdomyosarcomas predominantly occur in infancy and in the young
•• They are principally embryonal or botryoid in type
•• Rhabdomyosarcomas can arise at any site in the body as mass lesions. They are usually nontender and the presentation depends
on the site of the tumor
•• Head and neck—about 10% of tumors arise in the orbit producing proptosis and ophthalmoplegia. These usually present early,
before the tumor has metastasized. There is little lymphatic spread from the orbit and the prognosis is good. Parameningeal tumors
(about 20%) arise from the nasopharynx, paranasal sinuses, middle ear, mastoid and pterygopalatine fossa, producing nasal airway
and ear symptoms often with signs of secondary purulent or even bloody discharge
•• Commonest tumor of face in children
•• Genitourinary tract
•• These usually arise in the bladder (commonest) and prostate (12%) presenting as a polypoid mass inside the bladder leading
to hematuria, urinary obstruction or even extrusion of the tumor into the urethra in females. They tend to be localized. Prostatic
tumors lead to bladder outlet obstruction. Constipation may arise from obstruction of the rectum. Bladder tumors tend to occur in
younger patients. Vaginal and uterine rhabdomyosarcomas
•• Most are botryoid (grape-like) and present with a mucousy and sometimes bloody discharge.
•• Cystinosis
–– Small crystals may develop in the corneal epithelium, which may be painful.
•• Corneal clouding
–– Mucopolysaccharidoses, all of which show corneal clouding, except MPSII and MPSIII
–– Mucolipidoses
–– Fucosidosis
–– Mannosidosis
•• Wilson’s disease
–– KF ring. Copper deposition in the peripheral cornea may be detected on slit-lamp examination
•• Ophthalmology changes in leukemia
–– Retinal hemorrhages may be found in severe anemia and leukemia
•• Marfan’s syndrome
–– Screening for lens subluxation, ectopia lentis may be requested
•• Neurofibromatosis
–– The ophthalmologist may provide diagnostic information in cases of suspected neurofibromatosis types 1 and 2 (NF1, NF2). Lisch
nodules of the iris will be present in over 90% of children aged 5 years or more who have NF1. Children with confirmed NF1 also
require regular screening examinations for optic nerve gliomas and optic chiasma gliomas
•• In NF2, posterior subcapsular cataract is present in more than 50% of cases and is a useful diagnostic sign. Combined hamartoma
of the retina and pigment epithelium is a less common ocular association of NF2.
•• Tuberous sclerosis
–– Fundus examination for retinal hamartomas should be performed in children with seizures and delayed cognitive development.
Hamartomas will be found in approximately 50% of patients with tuberous sclerosis.
•• von Hippel-Lindau disease
–– Retinal angiomas are the earliest and the most common clinical features of von Hippel-Lindau disease. Careful annual fundus
examination is required in order to detect early lesions, which are most often found in the mid-periphery.
Endocrinology
Rokitansky–Küster–Hauser Syndrome
•• Girls with this condition are phenotypically normal, with a 46XX karyotype and normal ovaries
•• The degree of failure of development varies from total absence of Mullerian duct structures to varying degrees of uterine horn
development, very occasionally including endometrium within a small blind uterine horn when the girl will present with severe cyclical
abdominal pain
•• Vaginal reconstruction either by the use of dilators or surgically using a loop of bowel is required.
McArdle’s Disease
•• This disorder is characterized by increasing intolerance for strenuous exercise
•• Strenuous muscle activity is accompanied by severe cramps and may be followed by myoglobinuria, which can precipitate
anuria and renal failure. In middle life, the fatigue increases and muscle wasting and weakness predominate.
Galactosemia
•• Three enzyme deficiencies implicated
–– GPUT deficiency (Galactose 1 Po4 uridyl transferase) most common
•• AR condition:
–– Newborn presents with failure to thrive, vomiting, diarrhea and jaundice
–– Hepatomegaly
–– Cataract (oil-drop cataract)
–– Mental retardation
–– E. coli sepsis
•• Cataract because of accumulation of galactitol
•• Reducing sugar seen in urine
•• Rapid diagnosis and rapid removal of galactose from diet are essential.
Fabry Disease
•• This is an X-linked disorder
•• Results from a variety of mutations in the alphagalactosidase gene
•• Clinically, the disease manifests with:
–– Angiokeratomas (telangiectatic skin lesions)
–– Hypohidrosis
–– Corneal and lenticular opacities
–– Acroparesthesia
–– Small-vessel disease of the kidney, heart and brain
270 USMLE Step 2 CK Platinum Notes
•• Angiokeratomas are punctate, dark red to blue-black, flat or slightly raised, usually symmetric, and do not blanch with pressure.
They range from barely visible to several millimeters in diameter and have a tendency to increase in size and number with age
•• The acroparesthesia can be debilitating in childhood and adolescence, with a tendency to decrease after the third decade. Episodic
agonizing, burning pain of the hands, feet and proximal extremities can last from minutes to days and can be precipitated by exercise,
fatigue, or fever. Abdominal pain can resemble that from appendicitis or renal colic
•• Renal: Casts and microscopic hematuria can occur early, whereas proteinuria, isosthenuria, and progressive renal dysfunction
occur in the second to fourth decades. Progressive renal failure occurs and requires transplantation
•• CVS: Hypertension, left ventricular hypertrophy, anginal chest pain with or without myocardial ischemia or infarction, and
congestive heart failure can occur in the third to fourth decades
•• Hypohidrosis or anhidrosis, angiokeratomas, and the typical corneal and lenticular lesions provide a presumptive
diagnosis in males.
Maple syrup urine Branched chain α ketoacid •• Valine, leucine, isoleucine defect
disease (MSUD) dehydrogenase •• Maple syrup urine
•• Lethargic baby, loses weight, letosis, coma
Homocystinuria Cystathionine synthetase •• Mental retardation, MI, osteoporosis
•• Dislocation of lens
•• It is inherited as AR trait
•• Patients can be responsive to Vitamin B6 (pyridoxine)
Albinism Tyrosinase •• White hair
•• Photosensitivity
•• Strabismus
•• Nystagmus
•• Photophobia
Alkaptonuria
•• Alkaptonuria is inherited as an autosomal recessive trait
•• Patients have minimally increased concentrations of homogentisic acid in blood because it is rapidly cleared by the kidney.
However, homogentisic acid accumulates in cells and body fluids.
Pediatrics 271
•• Its oxidized polymers bind to collagen, leading to the progressive deposition of a gray to bluish-black pigment
•• Clinical Manifestations: Alkaptonuria may go unrecognized until middle life when degenerative joint disease develops. Prior to
this time, the tendency of the patient’s urine to darken on standing may go unnoticed, as may slight pigmentation of the sclerae
and ears. Foci of gray-brown scleral pigment and generalized darkening of the concha, anthelix, and finally, helix of the ear are
typical. Ear cartilages may be irregular and thickened
•• Ochronotic arthritis is heralded by pain, stiffness, and some limitation of motion of the hips, knees, and shoulders. Acute arthritis
may resemble rheumatoid arthritis, but small joints are usually spared. Limitation of motion and ankylosis of the lumbosacral
spine are common late manifestations. Pigmentation of heart valves, larynx, tympanic membranes, and skin occurs, and
occasional patients develop pigmented renal or prostatic calculi. Degenerative cardiovascular disease may be increased in
older patients
•• Diagnosis: The diagnosis is usually made from the triad of degenerative arthritis, ochronotic pigmentation, and urine that
turns black upon alkalinization
•• Homogentisic acid in urine may be identified presumptively by other tests: after addition of ferric chloride, a purple-black color
is observed; treatment with Benedict’s reagent yields a brown color; addition of a saturated silver nitrate solution produces an
intermediate black color
•• These screening tests can be confirmed by chromatographic, enzymatic, or spectrophotometric determinations of homogentisic
acid
•• X-rays of the lumbar spine show degeneration and dense calcification of the intervertebral disks and narrowing of the
intervertebral spaces (bamboo-like appearance)
•• There is no specific treatment for ochronotic arthritis. Joint manifestations might be mitigated if homogentisic acid accumulation
and deposition could be curbed by dietary restriction of phenylalanine and tyrosine. Ascorbic acid impedes oxidation and
polymerization of homogentisic acid in vitro, but the efficacy of this form of treatment has not been established.
Homocystinuria
•• Homocystinuria (elevated urinary levels of homocysteine) is a hallmark of several disorders in the metabolism of sulfur-containing
amino acids.
•• The classic form of the disease which is caused by defective activity of the enzyme cystathionine beta-synthase (CBS)
•• This autosomal recessive enzyme defect is the most common cause of homocystinuria
•• The classic presentation of cystathionine beta-synthase deficiency includes Marfanoid habitus, developmental delay, lens
dislocation and predisposition for blood clotting.
272 USMLE Step 2 CK Platinum Notes
•• Presentation is usually in the first decade with the exception of embolism which may occur later
•• The most characteristic feature of this disorder is subluxation of the ocular lens. Mental retardation is common, although
not always present
•• Most patients have osteoporosis and skeletal abnormalities similar to those seen in Marfan’s syndrome. In homocystinuria,
however, the joints tend to be limited in mobility rather than hypermobile
•• There is also lenticular subluxation in both conditions; however, in Marfan’s syndrome, the lens is usually displaced upwards,
whereas in homocystinuria it is displaced downwards and medially
•• Diagnosis
•• In CBS deficiency, elevated homocysteine can be detected in both urine and blood. Levels of methionine are usually also
elevated and levels of cystine are reduced. Because homocysteine is unstable, testing should be done on fresh urine
•• Screening tests for homocystinuria using cyanide nitroprusside reagent are available, but are not completely sensitive.
Measurement of total homocysteine in blood is useful
•• Because homocystinuria can be caused by several genetic defects, it is important to specifically confirm the diagnosis of CBS
deficiency by measuring the enzyme in liver, cultured skin fibroblasts or lymphoblasts
•• Treatment
•• All patients should initially be treated with large doses (100–500 mg/day) of pyridoxine to determine their degree of vitamin
responsiveness
•• Those who do not respond may be treated with a diet low in methionine and supplemented with L-cysteine.
All patients should probably receive high doses of folic acid. In addition, the compound betaine may be used to aid in the
conversion of homocysteine to methionine.
Cystinuria
•• Cystinuria is a complex genetic disorder involving at least three alleles governing dibasic amino acid COAL (cystine ornithine,
arginine and lysine) transport
•• Renal tubular cells demonstrate an inability to take up cystine from their brush border, but can do so from the basolateral surface.
Because the excretion of cystine can exceed the amount filtered, there is evidence of net cystine secretion by the tubule
•• The clinical manifestations are confined to individuals in whom the cystine concentration in urine exceeds its solubility product
and leads to calculus formation. Almost all untreated homozygotes will experience calculi at some time in their lives, a quarter of
them before 20 years of age. Obstruction and infection cause lasting damage to the urinary tract. Patients present with renal colic
or episodes of hematuria
•• The stones are radiopaque because of their high sulfur content. Ultrasound is a good way of identifying calculi in both the renal
collecting system and the bladder but often misses calculi in ureters
•• Microscopy of the urine reveals flat hexagonal birefringent crystals under polarized light, and the Nitroprusside test for
urinary disulfides is positive. Confirmation is by quantification of urinary proteins
•• The mainstay of treatment is to keep the urine volume sufficiently high so that cystine is kept below its solubility maximum
•• A second line of treatment, additional to and not a substitute for the first, is to prescribe bicarbonate or citrate to ensure that the
early morning urine pH is alkaline
•• Increase in cystine solubility can be achieved by forming a thiol is cysteine disulfide with agents such as D-penicillamine.
Mucopolysaccharidoses (MPS)
The various forms of MPS result from deficiencies of lysosomal enzymes needed for glycosaminoglycan (GAG) catabolism. GAGs are
long-chain, complex carbohydrates that are linked to proteins in connective tissue.
•• Affected individuals appear normal at birth but exhibit accelerated growth and mild coarsening of facial features in the first year.
Subsequently, there is slowing of growth, leading to short stature
•• In the first 2 years, clinical diagnosis is suggested by hepatosplenomegaly, corneal clouding, coarse features, large tongue, joint
stiffness, and characteristic dysostosis multiplex on skeletal X-rays
•• Instability of the cervical vertebral bodies can lead to paralysis, particularly with subluxation on hyperextension
MPS II (Hunter Syndrome)
•• This is an X-linked recessive disorder
•• Results from mutations in the gene encoding iduronate sulfatase
•• Clinically, MPS IH and II are similar, though corneal clouding is absent in MPS II
•• Clinical manifestations range from severe CNS and visceral involvement with death in late childhood to milder forms with normal
CNS function and survival into adulthood
MPS IIIA, IIIB, IIIC, and IIID (the Sanfilippo Syndrome)
•• Skeletal defects and hepatosplenomegaly are less pronounced in this group of MPS variants, though progressive behavioral
problems, mental retardation, and seizures are present
•• Affected patients can survive into the third or fourth decade with progressive CNS disease
MPS IV (Morquio Syndrome)
•• These MPS variants are autosomal recessive disorders characterized by severe skeletal diseases that resemble the
spondyloepiphyseal dysplasias
•• There is extreme shortening of the trunk due to multiple vertebral collapses
•• The long bones are relatively spared. Joint laxity can lead to osteoarthritis-like destruction of the joints
•• Upper cervical spinal cord compression due to atlantoaxial instability predisposes to subluxation and paralysis. Many patients have
mitral valve insufficiency that can be functionally significant
MPS VI (Maroteaux-Lamy Disease)
•• Mutations in the arylsulfatase B gene cause this autosomal recessive disorder. Although clinically variable, the general phenotype
resembles Hurler disease. Intelligence is normal, and the life span can extend beyond three decades. Cardiac valvular disease and
progressive pulmonary hypertension are frequent causes of death.
Lesch-Nyhan Syndrome
A complete deficiency of HPRT, the Lesch-Nyhan syndrome, is characterized by:
•• Hyperuricemia
•• Self-mutilative behavior
•• Choreoathetosis
•• Spasticity
•• Mental retardation
Kelley-Seegmiller syndrome
•• A partial deficiency of HPRT, the Kelley-Seegmiller syndrome, is associated with hyperuricemia but no central nervous system
manifestations
•• In both disorders, the hyperuricemia results from urate overproduction and can cause uric acid crystalluria, nephrolithiasis,
obstructive uropathy, and gouty arthritis
•• Purine metabolic defect
•• HPRT catalyzes the reaction that combines PRPP and the purine bases hypoxanthine and guanine to form the respective
nucleoside monophosphate IMP or GMP and pyrophosphate.
274 USMLE Step 2 CK Platinum Notes
Menkes’ Disease
•• Menkes’ disease is also known as kinky hair disease
•• X-linked neurodegenerative disease
•• Impaired copper transport. Occurs Named after noting the similarity of kinky hair to the brittle wool of Australian sheep raised
in areas with copper-deficient soil, he demonstrated abnormal levels of copper and ceruloplasmin in these patients
•• In Menkes’ disease, transport of dietary copper from intestinal cells is impaired, leading to the low serum copper levels. Abnormal
copper transport in other cells leads to paradoxical copper accumulation in duodenal cells, kidney, pancreas, skeletal muscle,
and placenta
•• Hair changes: Abnormal kinky hair, eyebrows, and eyelashes. Often lightly or abnormally pigmented; can be white, silver, or gray
•• Associated are abnormal facies, progressive cerebral degeneration, connective-tissue abnormalities. Loose skin at the nape
of the neck and over the trunk
•• Joint hypermobility, polypoid masses, which can be multiple, in the gastrointestinal tract, umbilical and inguinal hernias,
which can be bilateral, bladder diverticula, dilated ureters, emphysema, arterial rupture, brachial, lumbar and iliac artery
aneurysms, internal jugular vein aneurysms
•• Menkes’ kinky hair syndrome is associated with the defecive functioning of several copper-dependent enzymes due to impaired
copper absorption, transport, or metabolism. Lysyl oxidase is a copper-requiring enzyme
•• Decreased lysyl oxidase (LO) activity accounts for the connective-tissue fragility and vascular abnormalities in Menkes’
disease.
Congenital Hypothyroidism
•• Open posterior fontanelle
•• Large posterior fontanelle, sluggishness are a feature of newborn hypothyroid
•• Absent social smile, absent eyebrows are also features of a hypothyroid
•• Growth retardation, delayed puberty, umbilical hernia are a feature
Other important features:
–– Constipation
–– Pallor
–– Hypothermia
–– Cold extremeties in children are a common feature
–– Large tongue
–– Rough, dry skin
–– Hypotonia
–– Causes prolonged physiological jaundice
–– Causes delayed skeletal maturity
•• Epiphyseal dysgenesis is a feature
•• Seminiferous tubule dysgenesis is also seen
•• Pendred syndrome is hypothyroidism + deafness
•• Pendred syndrome has goiter + sensorineural type deafness
‘Holoprosencephaly’ is a complex malformation of the brain such that only a single large monoventricular cerebral hemisphere
exists. It is characterized by failed septation of the midline forebrain structures and is frequently accompanied by midfacial
abnormalities. Associated syndromic (including Smith-Lemli-Opitz, Hall-Pallister, pseudotrisomy 13) and structural anomalies are
common. Trisomy 13 is the most commonly identified cause.
‘Porencephaly’ is a severe cleft in the brain that allows the ventricles to communicate with the subarachnoid space.
It is a vascular malformation
Seen in cerebral infarction, Dandy-Walker syndrome.
Dandy-Walker malformation: Dandy-Walker malformation (DWM) describes agenesis or hypoplasia of the cerebellar vermis with
communication between the fourth ventricle and the cisterna magna, due to obstruction of foramen of Magendie and Lushka.
Lissencephaly is synonymous with ‘smooth brain.’
•• A rare condition in which there is defective neuronal migration from 12th week of gestation onwards resulting in ‘Agyria’
•• Such children are severely handicapped
•• Viral infections, mutation of ‘Reelin gene’ on ‘chromosome 7’ occurs
•• It occurs in association with:
–– Miller-Dieker syndrome
–– Walker-Warburg syndrome and
–– Muscle eye brain diseases
Remember the commonest cause of obstructive hydrocephalus in children is aqueductal stenosis.
Arnold-Chiari malformations are among the most frequent congenital anomalies of the CNS. The small posterior fossa is a crucial
diagnostic feature of Arnold-Chiari type 2. This change is probably responsible for downward displacement of the cerebellar vermis
and medulla through the foramen magnum. This leads to obstruction of the CSF flow and hydrocephalus. Important associated
abnormalities include lumbar myelomeningocele and syringomyelia.
Hydrocephalus
•• Internal or noncommunicating hydrocephalus: Excess of CSF within the ventricular system up to the level of the outlet
foramina of the fourth ventricle. The common sites of obstruction are at the outlet foramina of the fourth ventricle, the aqueduct
of Sylvius or at the foramen of Monro
•• External or communicating hydrocephalus: An increase in the ventricular volume and the subarachnoid spaces of the
cranium and spine. The sites of obstruction are at the arachnoid villi or in the basal cisterns
•• Panventricular hydrocephalus: Dilation of the lateral, third and fourth ventricles (in aqueduct stenosis, the fourth ventricle is
small or of normal size—‘triventricular hydrocephalus’). An isolated fourth ventricle (‘double compartment hydrocephalus’
or ‘trapped fourth ventricle’) occurs when there is outlet obstruction from that ventricle and stricture of the aqueduct.
•• Unilateral hydrocephalus: Abnormal dilation of the body, frontal and/or posterior horn of the lateral ventricle on one side.
This may be due to compression of the ventricular system on the opposite side, obstruction to one foramen of Monro, slit ventricle
syndrome or hemiparenchymal atrophy
•• Slit ventricles: A reduction in the size of the ventricular system seen on CT scan, usually in response to excessive CSF
drainage. The slit ventricle syndrome is distinguished from radiological slit ventricles by the presence of symptoms and clinical
signs attributable to this overdrainage.
276 USMLE Step 2 CK Platinum Notes
Bacterial Meningitis
The classic triad of symptoms of bacterial meningitis includes:
•• Fever
•• Stiff neck
•• Alterations of mental status
Poor feeding is the most common presentation of neonatal meningitis
Physical signs are:
•• Kernig’s sign
•• Brudzinski’s sign
•• Nuchal rigidity
A febrile, irritable, inconsolable infant with rash and fever with an altered state of alertness even in the absence of meningeal
signs should be suspected to have meningitis.
Seizures in Childhood
•• Commonest cause of convulsions in childhood is febrile seizures
•• Subtle seizures are the commonest type of seizures in newborn
Febrile Seizures
Hypoxic ischemic encephalopathy is mc cause of seizure in newborne
Commonest type of seizure in newborn: Subtle
Pediatrics 277
•• Seen between 9 months and 5 years. Do not usually last more than 10 minutes
•• Good prognosis
•• Generalized in nature
•• EEG normal after 2 weeks
•• Fifty percent recurrence
Asterixis:
•• Consists of coarse arrhythmic lapses of sustained posture
•• It is usually related to metabolic disorders and is most easily seen when the patient’s arms are outstretched
•• Asterixis occurs bilaterally
Lennox-Gastaut syndrome:
•• It is a variant of petit mal epilepsy
•• Consists of intellectual impairment, distinctive slow spike and wave pattern, and atonic postural lapses followed by minor tonic-
clonic spasms.
Pseudoseizures:
•• Are parts of conversion disorder and are not directly related to the use of antipsychotics
•• They are very much like real seizures except that there are no aura and no EEG abnormalities
•• The movements are asynchronous and nonstereotyped and they occur when the person is awake.
Infantile Spasms
Constitute a unique and very serious epilepsy syndrome confined to infants. The usual characteristic features of this syndrome are:
•• Tonic or myoclonic seizures
•• Hypsarrhythmic EEGs
•• Mental retardation
Infantile spasms may vary considerably in their clinical manifestations. Some seizures are characterized by brief head nods, whereas
other seizures consist of violent flexion of the trunk, arms, and legs. Most patients have more than one seizure type. Infantile spasms
can be classified into three major groups:
1. Flexor: Flexion of the neck, trunk, arms, and legs. Spasms of the muscles of the upper limbs result in either adduction of the arms
or a self-hugging motion, or adduction of the arms to either side of the head with the arms flexed at the elbow
2. Extensor: A predominance of extensor muscle contractions, which produce abrupt extension of the neck and trunk, along with
extensor abduction or adduction of the arms, legs, or both
3. Mixed flexor-extensor: Flexion of the neck, trunk, and arms and extension of the legs, or flexion of the legs and extension of the
arms with varying degrees of flexion of the neck and trunk.
Electroencephalography
Infantile spasms are usually associated with markedly abnormal EEGs. The most common interictal abnormality is hypsarrhythmia.
There are several variations:
•• Hypsarrhythmia with interhemispheric synchrony
•• Hypsarrhythmia with a consistent focus of abnormal discharge
•• Hypsarrhythmia with episodes of attenuation
•• Hypsarrhythmia consisting primarily of high-voltage slow activity with few sharp waves or spikes.
Physical Abuse
•• A physically abused child is defined as ‘any child who receives physical injury (or injuries) as a result of acts (or omissions)
usually on the part of his parents or guardians’
•• The definition includes actual or likely physical injury to a child, or failure to prevent physical injury (or suffering) to a child
•• Deliberate poisoning, suffocation and Munchausen syndrome by proxy are usually included in this category
•• Physical abuse (nonaccidental injury) is usually perpetrated by the child’s carers. It has been recognized as an assault, i.e. abuse,
from when ‘battered babies’ were first described
•• Common pattern of injuries in infancy includes skull, rib and long bone fractures, bruising anywhere (but note that the head,
neck and chest are more common), retinal hemorrhages and subdural hematomas
Battered child syndrome (BCS) is found at every level of society, although the incidence may be higher in low-income households,
where adult caregivers suffer greater stress and social difficulties, without having had the benefit of higher education. The child abuser
most often injures a child in the heat of anger that he was also often abused as a child himself. The incessant crying of an infant or child
may trigger abuse. Symptoms may include a delayed visit to the emergency room with an injured child; an implausible explanation
of the cause of a child’s injury; bruises that match the shape of a hand, fist or belt; cigarette burns; scald marks; bite marks; black eyes;
unconsciousness; bruises around the neck; and a bulging fontanel in infants.
A four-month-old infant is brought to a physician because of increased lethargy and irritability. The father of the infant says that the
child rolled off the bed and fell on the cemented floor. According to the father, the child has been previously healthy and is up-to-
date on his vaccinations. He has been meeting his development milestones. While in office, the patient develops a seizure. Initial
examination shows tense fontanellae and bruises on limbs.
Sexual Abuse
‘The sexual exploitation of children is referred to as the involvement of dependent, developmentally immature children and adolescents
in sexual activities that they do not fully comprehend, are unable to give informed consent to and that violates the social taboos of
family roles’.
Pediatrics 279
Contact involves touching, fondling or oral contact of breast or genitals. There may be insertion of fingers or objects into vulva or anus.
Masturbation may be by adult of him/herself in the presence of the child, including ejaculation onto the child, by adult of child or by
child of adult. Intercourse is vaginal, anal or oral, whether actual or attempted in any degree. This is usually with an adult as the active
party but, in some cases a child may be encouraged to penetrate the adult. Rape is attempted or achieved penile penetration of the
vagina. Other genital contacts include intercrural intercourse where the penis is laid between the legs or genital contact with any part
of the child’s body, e.g. a penis rubbed on child’s thigh
Prostitution involves any of the above abuses which includes the exchange of money, gifts or favors and applies to both boys (‘rent
boys’) and girls
Noncontact abuse involves exhibitionism (flashing), pornography (photographing sexual acts or anatomy), showing pornographic
images (photographs, films, videos) and erotic talk (telling children titillating or sexually explicit stories). Other sexual exploitations
include sadistic activities, e.g. burning a child’s buttocks or genital area.
Refsum Disease
•• The cardinal manifestations of Refsum disease include retinitis pigmentosa, cerebellar ataxia and polyneuropathy
•• The underlying metabolic etiology for Refsum disease is a deficiency of phytanoyl-coenzyme A (CoA) hydroxylase, an enzyme
that catalyzes an essential step in phytanic acid degradation
•• Deficiency of this enzyme results in the accumulation of phytanic acid
•• The sensory-motor polyneuropathy associated with Refsum disease typically begins in the distal lower extremities and, if
untreated, progresses to involve the upper extremities and trunk. Usually become evident later than the retinal degeneration and
polyneuropathy
•• Additional clinical features can include sensorineural hearing loss, anosmia, cardiac conduction abnormalities and
cardiomyopathy and skeletal manifestations (epiphyseal dysplasia, shortening or elongation of the third or fourth
metatarsals, hammer toes)
•• Cardiomyopathy is a frequent cause of sudden death in untreated patients
•• Therapy: Because phytanic acid is dietary in origin, therapy focuses on diets low in phytanic acid.
Nail-patella Syndrome
•• Nail-patella syndrome is an autosomal dominant disorder consisting of hypoplasia or absence of patellae, dystrophic nails,
dysplasia of the elbows and iliac horns, and renal disease
•• The nephropathy is usually benign and includes microhematuria and mild proteinuria, which usually appear in adolescence or
young adulthood. Some patients develop nephrotic syndrome and mild hypertension and are at risk of progression to end-stage
renal failure
•• Remember chronic glomerulonephritis is mc cause of childhood hypertension.
Kasabach–Merritt Syndrome
•• In the neonate, large or multiple hemangiomas are occasionally associated with a generalized bleeding disorder caused by
the trapping of platelets within them, which produces a profound thrombocytopenia
•• A course of prednisone, 2–4 mg/kg per 24 h can effect dramatic improvement
•• If this fails, embolization of the hemangioma can be considered.
Sacrococcygeal Teratoma
•• The sacrococcygeal teratoma is the commonest teratoma presenting in the neonatal period
•• They tend to be large and protrude from the space between the anus and the coccyx
•• The lesion is usually covered in skin but the most protuberant part may be necrotic due to vascular compromise
•• The tumor may also extend up into the pelvis and a large retrorectal component is palpable in all the cases. In a presacral teratoma,
there is no protrusion behind the anus and the presentation may be later in the first year of life
280 USMLE Step 2 CK Platinum Notes
•• The tumor may be both solid and cystic in nature. A very large tumor may give rise to dystocia and, if diagnosed antenatally, is
best delivered by cesarean section
•• Treatment is excision within the first few days of life
•• Adulthood recurrence of benign or malignant elements may occur.
Musculoskeletal System
Systemic Juvenile Rheumatoid Arthritis
Fractures in Children
•• The periosteum is extremely strong in children
•• Children’s bones are much more resilient and less brittle than those of adults
•• Bending moments applied to the bone of a child may cause a Greenstick fracture, in which there is distraction of the cortex on
the convex side and compression of bone on the concave side (Cortex intact.)
Ehlers-Danlos Syndrome
Ehlers-Danlos syndrome (EDS) consists of a group of disorders of connective tissue characterized by:
•• Joint hypermobility plus fragility and laxity of the skin
•• These conditions are characterized by abnormalities in collagen genes resulting in the production of abnormal collagen and
consequent tissue fragility
•• EDS has soft, hyperextensible skin, ‘cigarette paper’ scars, easy bruising and marked hypermobility. EDS type II is similar to
type I but less severe
•• Also seen is serious involvement with a high incidence of rupture of the arteries, the colon or the pregnant uterus. Management
of EDS consists of patient education and support, with genetic counseling for the more severe forms.
Osteogenesis Imperfecta
OI is also called brittle bone disease or ‘Lobstein syndrome’
•• Basic defect is in collagen type 1
•• It is transmitted either as AD or AR Inheritance
•• Type 1 OI is the most common type
•• A male presents with triad of lytic bony lesions on skull, diabetes Insipidus and exomphalos Hand-Schullerchristian disease
•• An infant presents with failure to thrive, pancytopenia. Bones are brittle and fracture easily. Infantile osteopetrosis
Defect is found in osteoclasts
•• A two-year-old presents with multiple fractures in different stages of healing brought by Battered baby syndrome
overconcerned parents
•• A six-year-old with b/l symmetrical fractures and blue sclera. Bones are osteopenic and Osteogenesis imperfecta
brittle
Sexual precocity in girls with polyostotic fibrous dysplasia and cutaneous pigmentation constitutes ‘McCune-Albright syndrome’.
Fibrous dysplasia
McCune-Albright syndrome represents a phenotypic spectrum of disorders caused by activating mutations in the GNAS1 gene
•• Mitochondrial myopathies
•• Down’s syndrome
•• Ehler-Danlos syndrome
•• Clostridium botulinum
Perthe’s Disease
•• It is a chronic disease with ischemia of upper end of femur causing ‘avascular necrosis.’
•• Osteochondritis of femoral head
•• Boys are affected more
•• Not obese usually
•• Pain is the initial and most common complaint
•• Limping is the associated feature
Ewing’s Sarcoma
•• Affects children <15 years usually
•• Diaphysial tumor
•• Onion skin lesions on X-ray
•• Translocation (t11:22)
•• Mimics osteomyelitis
•• PAS positive material (glycogen) found
•• Most radiosensitive bone tumor
Buzz Phrases
•• Cherry-red spot on macula ‘without’ hepatosplenomegaly: Tay-Sachs disease
•• Cherry-red spot on macula ‘with’ hepatosplenomegaly: Niemann-Pick’s disease
•• Bronze skin: Hemochromatosis
•• Heliotrope rash: Dermatomyositis
•• Salty tasting infant: Cystic fibrosis
•• Honey ingestion: Infant botulism
•• Currant jelly stool: Intussusception
•• Currant jelly sputum: Klebsiella Pneumoniae
•• A 3-month-old male infant developed otitis media, for which he was given a course of Cotrimoxazole. A few days later:
–– He developed extensive peeling of the skin;
–– There were no mucosal lesions and the baby was not toxic
The most likely diagnosis is:
Staphylococcal scalded skin syndrome
•• A two-year-old girl child is brought to the outpatient department (OPD) with features of:
–– Hand-wringing stereotype movements
–– Impaired language and communication development
–– Breath-holding spells
- Poor social skills and deceleration of head growth after 6 months of age
The most likely diagnosis is:
Rett’s syndrome
•• An 11-month-old boy, weighing 3 kg
–– Has polyuria, polydipsia and delayed motor milestones
–– Blood investigations show creatinine of 0.5 mg/dl, potassium 3 mEq/l, sodium 125 mEq/l, chloride 88 mEq/l, calcium
8.8 mg/dl, pH 7.46 and bircarbonate 26 mEq/l
–– Ultrasonography shows medullary nephrocalcinosis
The most likely diagnosis is:
Barter’s syndrome
•• A 13-year-old boy is referred for evaluation of nocturnal enuresis and short stature.
286 USMLE Step 2 CK Platinum Notes
Timothy Syndrome
•• It is a multisystem disorder characterized by cardiac, hand/foot, facial, and neuro developmental features
•• The two forms are type 1 (classic) and type 2, a rare form caused by mutations in an isoform of the same gene
•• Cardiac findings include a rate-corrected QT interval of between 480 ms and 700 ms and congenital heart defects (patent
ductus arteriosus, patent foramen ovale, ventricular septal defect, tetralogy of Fallot, hypertrophic cardiomyopathy)
•• Hand/foot findings are unilateral or bilateral cutaneous syndactyly variably involving fingers two (index), three (middle),
four (ring), and five (little) and bilateral cutaneous syndactyly of toes two and three
•• Facial findings include flat nasal bridge, low-set ears, thin upper lip, and round face. Neuropsychiatric involvement includes
global developmental delays and autism spectrum disorders. Ventricular tachyarrhythmia is the leading cause of death,
followed by infection and complications of intractable hypoglycemia
•• Diagnosis/testing: Timothy syndrome is diagnosed by clinical features and by the presence of one of the three known mutations
in CACNA1C, the gene encoding the CaV1.2 calcium channel. Molecular genetic testing is available on a clinical basis.
Landau-Kleffner Syndrome
•• It is a childhood disorder. A major feature of LKS is the gradual or sudden loss of the ability to understand and use spoken
language
•• All children with LKS have abnormal electrical brain waves that can be documented by an electroencephalogram (EEG), a
recording of the electric activity of the brain
•• Approximately 80 percent of the children with LKS have one or more epileptic seizures that usually occur at night. Behavioral
disorders such as hyperactivity, aggressiveness and depression can also accompany this disorder
•• LKS occurs most frequently in normally developing children who are between 3 and 7 years of age. For no apparent reason, these
children begin having trouble understanding what is said to them. Doctors often refer to this problem as auditory agnosia or
‘word deafness’. The auditory agnosia may occur slowly or very quickly. Parents often think that the child is developing a hearing
problem or has become suddenly deaf. Hearing tests, however, show normal hearing. Children may also appear to be autistic
or developmentally delayed
•• The inability to understand language eventually affects the child’s spoken language which may progress to a complete loss of the
ability to speak (mutism)
•• Children who have learned to read and write before the onset of auditory agnosia can often continue communicating through
written language.
•• Death can occur as early as a few hours after onset of signs and symptoms
•• Other symptoms may include: Loose, greenish stools, a refusal to suck, ashen color (implied by the name gray baby
syndrome) and lactic acidosis.
Wiskott-Aldrich Syndrome
•• It is an X-linked recessive immunodeficiency disease characterized by the triad of thrombocytopenia (hemorrhage may be
the presenting complaint), eczema, and recurrent infections (often respiratory)
•• The children have defects in both T- and B-cell function, and are vulnerable to pyogenic bacteria, viruses, fungi, and Pneumocystis
carinii
•• These patients have severe disease, and formerly often dies by the age of 15
•• Survivors past age 10 have a 10% incidence of cancer, particularly lymphoma and acute lymphoblastic leukemia
•• Modern treatment consists of splenectomy, continuous antibiotic therapy, IV immunoglobulin, and bone marrow
transplantation.
Kallmann’s Syndrome
•• It is an isolated gonadotropin deficiency or familial hypogonadotropic hypogonadism and can present with primary amenorrhea
•• Primary amenorrhea is defined as the absence of menses in a female by the age of 16
•• Associated findings in Kallmann’s syndrome may include anosmia or hyposmia, color blindness, and cleft lip or cleft palate.
These findings are attributable to the fact that during embryogenesis, the GnRH neurons originally develop in the epithelium of
the olfactory placode and normally migrate into the hypothalamus
•• Physical examination may reveal absent to minimal breast development. Treatment of the patient with Kallmann’s syndrome is
with exogenous estrogen and progestin replacement therapy. If pregnancy is desired, ovulation induction can be brought about
with the pulsatile administration of exogenous GnRH.
CHARGE Syndrome
Includes
•• Colobomas
•• Heart defects
•• Atresia, (choanal)
•• Retardation
•• Genitourinary abnormalities
•• Ear anomalies
The etiology of the CHARGE syndrome is unknown, but it may involve altered morphogenesis during the second trimester of pregnancy.
It is not genetically transmitted and is not associated with a teratogenic effect of any substance.
Sturge-Weber Syndrome
•• Facial nevus flames (usually in the distribution of 1st branch of trigeminal nerve)
•• Contralateral focal seizures
•• Calcification (rail-road track) of the cortex and subcortical structures
•• Glaucoma on the same side as skin lesions
•• Mental retardation due to cerebral atrophy
Segawa Syndrome
Segawa syndrome, also called dopa-responsive dystonia, is an inherited disease that can cause physical rigidity and
developmental delay. There are two forms—mild and severe.
In the mild form, symptoms typically begin in childhood. Children develop jerky movements that quickly progress to physical
rigidity. These children show spastic movements, and make very little voluntary movement. If untreated, children with Segawa
syndrome may have expressionless faces, drooping eyelids, tongue tremors, and drooling problems. They will show both mental
and physical developmental delays. Some children with Segawa syndrome show a ‘diurnal’ pattern, meaning their symptoms are
more or less severe on alternate days.
The severe form of the disease will appear in infancy, usually before six months of age. Affected infants have delayed motor skills,
weakness in the chest and abdomen, rigidity in the arms and legs, and problems with movement. These children will eventually
have learning disabilities, problems with speech, and behavioral/psychological problems. In addition, some people with the
disease have problems with their autonomic nervous system, which regulates unconscious functions, such as body temperature
regulation, digestion, blood sugar level, and blood pressure. Treatment of the severe form of the disease is often less successful
Segawa syndrome is caused by a deficiency in an enzyme called tyrosine hydroxylase. Without it, the amino acid tyrosine cannot
properly be converted to dopamine, a key neurotransmitter in the brain. Dopamine is important for many functions, including muscle
control and cognition.
290 USMLE Step 2 CK Platinum Notes
5. Pendred syndrome
6. Goldenhar’s syndrome
7. Alport’s syndrome
8. Pierrie Robinson syndrome
9. Treacher Collins syndrome
Ans. 7. Alport’s syndrome
1. Annular pancreas
2. Pancreas divisum
3. Agenesis of pancreas
4. Ectopic pancreas
Ans. 2. Pancreas divisum
COGNITIVE DISORDERS
Functions disturbed in ‘Organic Brain Syndrome’
•• Consciousness
•• Orientation
•• Abstract thinking
•• Recent memory
•• Intelligence
Delirium
•• Acute onset
•• Memory registration and retention impaired
•• Clouding of consciousness
•• Disorientation in time, place, person
•• Attention and concentration disturbed
•• Hallucinations common
•• Disturbed sleep wake pattern
•• Marked diurnal variation (IMP)
Dementia
•• Chronic onset usually
•• There is no disturbance of consciousness
•• There is cognitive impairment
•• Impaired judgment
•• Personality alteration
•• Impaired memory
•• Attention and concentration undisturbed
•• Hallucinations uncommon
•• Diurnal variation absent
•• Retrograde Amnesia is characterized by an inability to remember events prior to the onset of disease, with preservation of
the ability to form new memories
•• Anterograde Amnesia is characterized by an inability to register and form lasting memories of new or present events.
•• Proactive Inhibition is when something newly learned interferes with previously learned information.
New material inhibits old material
•• Retroactive Inhibition is when previously learned material interferes with the ability to acquire new information. Old
material inhibits new material
•• Transient Global Amnesia is characterized by confusion and impairment of recent memory. It often occurs in elderly people
and resolves without sequelaie
Alzheimer’s Disease
•• Cortical Dementia not Subcortical Dementia
•• Causes brain atrophy in advanced cases
•• Progressive dementia
•• Atrophy usually involves frontal, temporal and parietal lobes
•• Nucleus Basalis of Meynert is effected
–– Aphasia
–– Amnesia
–– Agnosia
–– Acalculia
–– Alexia are features
There is reduction in acetylcholine concentration
Associations with
Down’s syndrome, APP (Amyloid precursor Protein gene on Chromosome 21)
APO E gene
•• Presinilin 1 on Chromosome 14
•• Presinilin 2 on chromosome 1
•• Treatment is by
•• Tacrine
•• Donepezil
•• Memantine
–– Neurofibrillary tangles are intracytoplasmic filamentous inclusions found in Alzheimer’s disease and, to alesser extent, in
normal aging brains
–– Granulovacuolar degeneration
–– Hirano Bodies: They are Specific protein deposits associated with AGE (Advance Glycosylation End products) localized within
soma of neurons
–– Amyloid Plaques: They represent fragmented accumulations of proteins which are normally broken down but accumulate
in Alzheimer’s disease
–– Alzheimer’s Cells (Special cells)
–– Neurofibrillary tangles are insoluble twisted fibrils composed of Tau proteins
Pseudodementia
•• It is seen in depression
•• Severely depressed individuals may appear demented, a phenomenon called pseudodementia
•• Unlike cortical dementias, memory and language are usually intact when carefully tested in depressed persons
Psychiatry 333
•• The patients may feel confused and are unable to accomplish routine tasks
•• Vegetative symptoms are common, such as insomnia, lack of energy, poor appetite and concern with bowel function
•• The psychosocial milieu may suggest prominent reasons for depression
•• The patients respond to antidepressant treatment
Autism
•• Autism is characterized by onset of symptoms before 3 years of age
•• Difficulty in concentration and communication is typical
•• Lesion in frontal lobe, temporal lobe, cerebellum
•• Delayed speech and Language development (absence of babbling, presence of echolalia)
•• Problems in forming social relationships in early childhood. (Aloofness, absent social smile, lack of eye to eye contact)
•• Males are more affected in autism
•• Features:
–– Stereotyped movements
–– Poor speech
–– Lack of social interaction
Some autistic children develop well developed isolated skills atypically such as remote memory, calculating abilities and musical
abilities. This atypical presentation is called as ‘Idiot savant syndrome’
Children develop
–– Mental retardation
–– Epilepsy
–– EEG abnormalities
Differential Diagnosis
Asperger’s syndrome is a different entity
Characterized by autism without any delay in language or cognitive development
Better Prognosis
Retts Syndrome is mostly common in girls with deceleration of head growth between five months and 30 months
Stereotyped movements with hand clapping and head wringing are characteristic
334 USMLE Step 2 CK Platinum Notes
Attention deficit/hyperactivity disorder (ADHD) is characterized by impulsivity, hyperactivity, and inattention lasting at
least 6 months. To make the diagnosis, the disorder must have started before age 7 and six signs each of inattention and
impulsivity/hyperactivity need to be present.
In children, bipolar disorder often presents as extremely irritable and explosive mood, with poor psychosocial functioning,
decreased sleep, talkativeness, racing thoughts, and high energy.
Conduct disorder is diagnosed when the child has had symptoms of aggression toward people and animals, destruction of
property, deceitfulness or theft, and serious violation of rules. One symptom needs to last at least 6 months, even though the
symptoms may have been present intermittently for a year.
Post-traumatic stress disorder develops following trauma, and the child usually has symptoms of hypervigilance, increased
arousal avoidance and autonomic reactivity.
Rett syndrome belongs to the pervasive developmental disorders mostly seen in girls. The child has normal development
through the first 5 months of life. During the second year, however, she develops multiple deficits, including decreased head
growth, decreased hand skills, social impairment and impaired gait and trunk movements.
Psychiatry 335
•• Conduct disorder is a childhood/adolescent disorder defined as a pattern of behavior in which the basic rights of others are
violated with three or more of the following present in the past 12 months: destruction of property, cruelty to animals and people,
deceitfulness or theft, and serious violations of rules. It causes clinically significant impairment in social functioning and it is
reserved for patients younger than 18
•• Attention deficit/hyperactivity disorder requires the presence of six symptoms of inattention for at least 6 months to a degree
that is maladaptive and six symptoms of hyperactivity/impulsivity that cause social impairment. Symptoms are present in two or
more settings (e.g. home and school), and some of the symptoms are present before age 7. The symptoms are not due to a general
medical condition or other mental disorder
•• Intermittent explosive disorder is diagnosed in adults only after several episodes of failure to resist aggressive impulses that lead
to assaults or destruction of property. The degree of episodes is not proportionate to precipitating stressor. The disorder is not due
to any other mental disorder or general medical condition
•• Oppositional defiant disorder is a pattern of negativistic and defiant behavior lasting at least 6 months with four or more of the
following: loss of temper, arguments with adults, defying rules, deliberately annoying other people, blaming others for own faults,
presence of vindictive behavior, presence of anger and resentment
Delusion
Delusion is a false unshakable belief
Delusion is a disorder of thought
•• Capgras syndrome is delusion of doubles
•• Othello syndrome is delusion involving infidelity of spouse
•• Delusion of Dysmorphobia is body parts appearing ugly
•• Nihilistic delusion is seen in depression
•• ‘The female in a delusional state that the attractive person is in love with her. This is De Clerambaults Syndrome or erotomanic
delusion that a person of high status is in love with her.’
•• Delusion of Reference : Misinterpretation that events in the outside world are having direct personal relationship
336 USMLE Step 2 CK Platinum Notes
Hallucination
Disorder Most Common Delusion
Depression •• Nihilistic delusion
Mania •• Delusions of grandeur
Schizophrenia •• Paranoid delusions
•• Delusions of reference
Neurotic disorders
•• Insight present
•• Reality testing is intact
Psycotic disorder
•• Insight absent
•• Reality testing is not intact
•• Presence of delusions and hallucinations
SCHIZOPHRENIA
First Rank Symptoms in Schizophrenia
Hallucinations
Delusional perception
Thought Alienation phenomenon
•• Thought withdrawal
•• Thought insertion
•• Thought broadcasting
Passivity phenomenon
•• Made feelings
•• Made impulses
•• Made acts
•• Somatic passivity
Schizophrenia is a heterogeneous syndrome characterized by perturbations of language, perception, thinking, social activity,
affect, and volition
Patients may present with
•• ‘Positive symptoms’ (such as conceptual disorganization, delusions, or hallucinations) or
•• ‘Negative symptoms’ (loss of function, anhedonia, decreased emotional expression, impaired concentration and diminished
social engagement) and must have ‘at least two of these for a 1-month period’ and continuous signs for ‘at least 6 months’ to
meet formal diagnostic criteria
•• ‘Negative’ symptoms predominate in one-third of the schizophrenic population and are associated with a poor long-term
outcome and a poor response to drug treatment
•• Catatonic-type describes patients whose clinical presentation is dominated by profound changes in motor activity, negativism
and echolalia or echopraxia
•• Paranoid-type describes patients who have a prominent preoccupation with a specific delusional system and who otherwise do
not qualify as having disorganized-type disease
•• Disorganized-type in which disorganized speech and behavior are accompanied by a superficial or silly affect
•• Residual-type disease, negative symptomatology exists in the absence of delusions, hallucinations, or motor disturbance
•• The diagnosis of ‘Schizophreniform disorder’ is reserved for patients who meet the symptom requirements but not the duration
requirements for schizophrenia
•• The diagnosis of ‘Schizoaffective disorder’ is used for those whose symptoms of schizophrenia are independent of associated
periods of mood disturbance
•• About 10% of schizophrenic patients commit suicide
•• ‘The Dopamine Hypothesis of Schizophrenia’ is based on the fact that agents that diminish dopaminergic activity have beneficial
effects in reducing the acute symptoms and signs of psychosis, specifically agitation, anxiety and hallucinations
•• TREATMENT: Antipsychotic agents remain the cornerstone of acute and maintenance treatment of schizophrenia and are
effective in the treatment of hallucinations, delusions, and thought disorders, regardless of etiology
•• Older agents, such as chlorpromazine and thioridazine, are more sedating and anticholinergic and more likely to cause
orthostatic hypotension, while higher potency antipsychotics, such as haloperidol, perphenazine and thiothixene, carry a
higher risk of inducing extrapyramidal side effects
•• Acute onset
•• Late onset
•• Short duration
•• Female
•• Fatty physique
•• Family history of Mood disorder
•• Presence of stressor
•• Positive symptom predominance
•• Catatonic type
•• Good Social Support
•• Normal CT scan of head
Catatonic Schizophrenia
•• Mutism: Absence of speech
•• Rigidity seen
•• Waxy flexibility: Parts of body in uncomfortable position for a long period of time
•• Defect of conation
•• Stupor
•• Negativism
•• Echolalia: Repetition of phrases/words
•• Echopraxia
•• Mannerism and grimacing
•• Automatic obedience
•• Ambitendency
•• Verbigeration: Incomprehensible speech
•• Cataplexy not catalepsy
Psychiatry 339
Personality Disorders
•• Are pervasive, fixed, inappropriate patterns of relating to others that cause social and occupational impairment
•• Have limited insight
•• Do not have frank psychosis
The DSM-IV classifies Personality disorders into
•• Cluster A
•• Cluster B
•• Cluster C
Adjustment Disorders
Adjustment disorder Normal grief Acute stress disorder Post-traumatic stress disorder. (PTSD)
Emotional symptoms that Expected strong Pshycological symptoms Psycological symptoms lasting > 4 weeks
start within 3 months and emotional response lasting 2 days – 4 weeks/ after stressor
end within 6 months of usually sadness after within a month after
exposure to a stressor. a loss stressor
Kubler-Ross’s death and dying sequence is a stepwise process with 5 identified stages
The order in which these stages appear is the following
•• Denial
Psychiatry 341
•• Anger
•• Bargaining
•• Sadness
•• Acceptance
Somatoform Disorders
‘Somatoform disorders are characterized by physical symptoms without a sufficient organic cause. A person who has a somatoform
disorder is not malingering and not delusional but truly believes that he or she has a physical problem.’
The five major DSM-IV classifications of somatoform disorders are:
•• Somatization disorder
•• Conversion disorder
•• Hypochondriasis
•• Body dysmorphic features
•• Pain disorder
•• Undifferentiated somatoform disorders
Somatization Disorder
•• Patients with multiple somatic complaints that cannot be explained by a known medical condition. The somatoform disorders
include a variety of conditions that differ in terms of the specific symptoms that are present and in whether or not the symptoms
are intentionally produced
•• In somatization disorder the patient presents with multiple physical complaints referable to different organ systems
•• Onset is usually before age 30, and the disorder is persistent
•• Formal diagnostic criteria require the recording of
–– At least four pain
–– Two gastrointestinal
–– One sexual
–– One pseudoneurologic symptom
•• Patients with somatization disorder often present with dramatic complaints, but the complaints are inconsistent
•• Symptoms of comorbid anxiety and mood disorder are common and may be the result of drug interactions due to regimens
initiated independently by different physicians
•• Patients with somatization disorder may be impulsive and demanding and frequently qualify for a formal comorbid
psychiatric diagnosis
Conversion Disorder
•• Here symptoms focus on deficits that involve voluntary motor or sensory function and on psychological factors that initiate
or exacerbate the medical presentation
•• Like somatization disorder, the deficit is not intentionally produced or simulated, as is the case in factitious disorder (malingering)
•• Deficit is proceeded by conflicts/stressors
•• Deficit cannot be fully explained by medical condition or substance use
•• Deficit can cause significant distress/impairment
•• La Belle Indifference is noted
•• Primary and secondary gain are associated
Hypochondriasis
•• Here the essential feature is a belief of serious medical illness that persists despite reassurance and appropriate medical
evaluation
•• Abnormal preoccupation with normal body function
342 USMLE Step 2 CK Platinum Notes
Munchausen’s Syndrome
•• The patient consciously and voluntarily produces physical symptoms of illness
•• It is reserved for individuals with particularly dramatic, chronic, or severe factitious illness
•• A variety of signs, symptoms and diseases have been either simulated or caused by factitious behavior the most common including
chronic diarrhea, fever of unknown origin, intestinal bleeding or hematuria, seizures and hypoglycemia
•• Sole purpose is to obtain medical attention
•• Pseudologia fantastica is a feature
•• Grid abdomen with multiple scars is a feature
•• Patients are manipulative, convincing liars and have superficial knowledge of medical terminology
•• In somatoform disorders, the patient does not intentionally create symptoms (unconscious process)
•• In factitious disorders, patients intentionally create an illness or symptoms (e.g. they inject insulin to create hypoglycemia) and
subject themselves to procedures in order to assume the role of a patient (no financial or other secondary gain)
•• In malingering, patients intentionally create their illness for secondary gain (e.g. money, release from work or jail).
EATING DISORDERS
Anorexia Nervosa
Anorexia nervosa typically begins in mid to late adolescence
The disorder occasionally develops in early puberty, before menarche, but seldom begins after age 40
Despite being underweight, patients with anorexia nervosa rarely complain of hunger or fatigue and often exercise extensively
Further weight loss is viewed by the patient as a fulfilling accomplishment, while weight gain is seen as a personal failure
Physical Features
•• Amenorrhea is a feature
•• Patients with anorexia nervosa typically have few physical complaints but may note cold intolerance and constipation
•• Vital signs may reveal bradycardia, hypotension and hypothermia
•• Soft, downy hair growth (lanugo) sometimes occurs, especially on the back and alopecia may be seen
•• Salivary gland enlargement which is associated with starvation as well as with binge eating and vomiting may make the face
appear surprisingly full in contrast to the marked general wasting.
344 USMLE Step 2 CK Platinum Notes
•• Acrocyanosis of the digits is common, and peripheral edema can be seen in the absence of hypoalbuminemia, particularly
when the patient begins to regain weight
•• Some patients who consume large amounts of vegetables containing vitamin A develop a yellow tint to the skin (hypercarotenemia),
which is especially notable on the palms
Laboratory Abnormalities
•• Mild normochromic, normocytic anemia is frequent, as is mild to moderate leukopenia, with a disproportionate reduction of
polymorphonuclear leukocytes
•• Dehydration may result in slightly increased levels of blood urea nitrogen and creatinine
•• Serum liver enzyme levels may increase, especially during the early phases of refeeding. The level of serum proteins is usually
normal
•• Hypokalemic alkalosis suggests self-induced vomiting or the use of diuretics
•• Hyponatremia is common and may result from excess fluid intake and disturbances in the secretion of antidiuretic hormone
Endocrine Abnormalities
Amenorrhea is hypothalamic in origin and reflects diminished production of gonadotropin releasing hormone (GnRH)
Cardiac output is reduced, and congestive heart failure occasionally occurs during rapid refeeding
The electrocardiogram usually shows:
•• Sinus bradycardia
•• Reduced QRS voltage
•• Nonspecific ST-T-wave abnormalities
•• Some patients develop a prolonged QTc interval, which may predispose to serious arrhythmias
Anorexia Bulimia
•• Binge eating is uncommon •• Common
•• Weight loss common •• Mostly normal
•• Amenorrhea in 100% •• In 50%
•• Skin changes common •• Rare
•• Antisocial behavior rare •• Common
•• Restrict food intake •• Self induced vomiting
Bulimia Nervosa
It is a condition characterized by recurrent episodes of binge eating followed by a compensatory behavior to prevent weight
gain (vomiting, exercise, laxative abuse). Other features include stealing (food), alcohol and drug abuse, self-mutilation and
depression. The individuals are usually at or slightly over the normal weight for their height, sexual activity is normal or
increased and they continue to menstruate. Clinical findings that are caused by recurrent vomiting include dental caries,
periodontal disease and pharyngeal lacerations and nail changes. Metabolic alkalosis and hypokalemia are also present.
Complications include aspiration and rupture of the esophagus or stomach
Psychiatry 345
Signs of PPP
•• Hallucinations
•• Delusions
•• Illogical thoughts
•• Insomnia
•• Refusal to eat
•• Anxiety
•• Agitation
Increased Risk In
•• Positive family history of pshychosis
•• Bipolar disorder
•• Schizophrenia
Sleep Enuresis
•• It is another parasomnia that occurs during slow-wave sleep (stage IV) in the young
•• Before age 5 or 6, nocturnal enuresis should probably be considered a normal feature of development
•• The condition usually improves spontaneously at puberty
•• Persistence of enuresis into adolescence or adulthood may reflect a variety of underlying conditions
•• In older patients with enuresis a distinction must be made between primary and secondary enuresis, the latter being defined as
bedwetting in patients who have been fully continent for 6 to 12 months
•• Treatment of primary enuresis is reserved for patients of appropriate age (older than 5 or 6 years) and consists of bladder
training exercises and behavioral therapy. Urologic abnormalities are more common in primary enuresis and must be assessed
by urologic examination
•• Important causes of secondary enuresis include emotional disturbances
–– Urinary tract infections or malformations
–– Caudaequina lesions
–– Epilepsy
–– Sleep apnea
–– Certain medications
Symptomatic pharmacotherapy is usually accomplished with
•• Intranasal desmopressin
•• Oral oxybutynin chloride or
•• Imipramine
PHOBIA
Phobic Disorders
•• Marked and persistent fear of objects or situations, exposure to which results in an immediate anxiety reaction
•• The patient avoids the phobic stimulus, and this avoidance usually impairs occupational or social functioning
•• Panic attacks may be triggered by the phobic stimulus or may emerge spontaneously during the course of the illness
•• Unlike patients with other anxiety disorders, individuals with phobias experience anxiety only in specific situations
Drugs used:
•• Anxiolytic drugs
•• TCA
•• SSRI
•• MAO inhibitors
Psychiatry 347
•• Exposure therapy a type of behavior therapy is the most commonly used treatment of specific phobia. The therapist usually
desensitizes the patient by a gradual exposure to the phobic stimulus. Relaxation and breathing control are important parts of the
treatment
•• Hypnosis is used to enhance the therapist’s suggestions that the phobic object is not dangerous. At times, self-hypnosis can be
taught so that the patient uses it as a method of relaxation when confronted with the phobic stimulus
•• Insight-oriented psychotherapy was initially used to treat phobias, but analyzing unconscious conflicts did not resolve phobic
symptoms. It does help the patient understand the origins of the phobia and how to deal with anxiety-provoking stimuli
•• Medication is used in the treatment of a specific phobia only if it is associated with panic attacks and generalized anxiety. The
pharmacologic treatment is then directed toward the panic attacks
•• Supportive therapy may be used in helping the patient actively confront the phobic stimulus during treatment. It is usually used
in addition to an ongoing treatment
Panic Disorder
It is defined by the ‘presence of recurrent and unpredictable panic attacks, which are distinct episodes of intense fear and
discomfort associated with a variety of physical symptoms’, including:
–– Palpitations
–– Sweating
–– Trembling
–– Shortness of breath, chest pain, dizziness
–– A fear of impending doom or death
–– Paresthesias, gastrointestinal distress, and feelings of unreality are also common
Panic attacks have a sudden onset, developing within 10 min and usually resolving over the course of an hour and they occur in an
unexpected fashion
Insight present:
The frequency and severity of panic attacks varies, ranging from once a week to clusters of attacks separated by months of well-being
TREATMENT
•• Achievable goals of treatment are to decrease the frequency of panic attacks and to reduce their intensity. The cornerstone of drug
therapy is antidepressant medications
•• The tricyclic antidepressant (TCA) agents imipramine and clomipramine can benefit 75 to 90% of panic disorder patients
•• Selective serotonin reuptake inhibitors (SSRIs) are equally effective and do not have the adverse effects of TCAs. SSRIs should be started
at one-third to one-half of their usual antidepressant dose (fluoxetine, sertraline, paroxetine)
•• Monoamine oxidase inhibitors (MAOIs) are at least as effective as TCAs and may specifically benefit patients who have comorbid
features of atypical depression (i.e. hypersomnia and weight gain)
•• Because of anticipatory anxiety and the need for immediate relief of panic symptoms, benzodiazepines are useful early in the
course of treatment
•• Early psychotherapeutic intervention and psychoeducation aimed at symptom control enhances the effectiveness of drug
treatment
348 USMLE Step 2 CK Platinum Notes
Defense mechanisms
•• Are unconscious mental techniques used by ego to keep conflicts out of awareness, reduce anxiety and maintain persons
self-esteem and sense of safety and equilibrium
•• Although defense mechanisms protect the person, if used excessively, neurotic symptoms can occur
The defense mechanisms used are:
Isolation of effect
•• Does not experience or express emotions associated with stressful event, e.g. a man whose father dies in a road traffic accident
describes the circumstances of his death dispassionately
Rationalization
•• Gives reasonable explanation for unacceptable feelings, e.g. a candidate not passing MD/MS entrance examinations says that
passing was not good for me anyway
Undoing
•• Attempting to reverse past actions with present actions, e.g. a man diagnosed with lung cancer as a result of smoking buys
books on nutrition, stop smoking
Reaction formation
•• Denying unacceptable feelings and adopting opposite attitudes and behavior, e.g. a man who is angry with his friend
compliments his shirt
Major Depression
•• Major depression is defined as depressed mood on daily basis for a minimum duration of two weeks
•• Markedly diminished interest or pleasure in all/almost all activities most of the day, nearly everyday
•• Significant weight loss/weight gain
•• Insomnia/hypersomnia nearly everyday
•• Psychomotor retardation/agitation nearly everyday
•• Fatigue/loss of energy nearly everyday
•• Feeling of worthlessness nearly everyday
•• Diminished ability to think, concentrate or indecisiveness nearly everyday
•• Recurrent thoughts of death, suicidal ideation
•• Anhedonia (Greek a without hedone, pleasure): It is an inability to experience pleasure from normally pleasurable life events such
as eating, exercise, and social or sexual interaction
•• Anhedonia is recognized as one of the key symptoms of the mood disorder depression according to both the Diagnostic and
Statistical Manual of Mental Disorders Fourth Edition (DSM IV) and the International Statistical Classification of Diseases and
Related Health Problems (ICD)
•• Anhedonia is also seen in schizophrenia and schizoaffective disorder
Depression
•• Depression is the MC Pshyciatric disorder in India
•• Depression involves middle aged females mostly
•• Depression is associated with:
–– Post MI
–– Postpartum
–– AIDS
–– Myxedema
Neurotransmitter involved: serotonin and norepinephrine
MC cause for suicide is depression
MC type of post purpureal pshycosis is depression
In children anhedonia (loss of enthusiasm and interest in play, socializing, school activities, loss of pleasure and boredom are
manifestations of depression.)
•• Nihilistic ideas are seen in depression
•• Dysthymic disorder consists of a pattern of chronic (at least 2 years), ongoing, mild depressive symptoms that are less severe and
less disabling than those found in major depression; the two conditions are sometimes difficult to separate, however, and can
occur together (‘double depression’)
Psychiatry 351
•• A seasonal pattern of depression, called seasonal affective disorder, may manifest with onset and remission of episodes at
predictable times of the year
•• This disorder is more common in women, whose symptoms are anergy, fatigue, weight gain, hypersomnia, and episodic
carbohydrate craving. The prevalence increases with distance from the equator, and mood improvement may occur by altering
light exposure.
Associations of suicide:
•• Depression
•• Schizophrenia
•• Substance abuse
Point to Remember
Any patient with serious suicidal thoughts, suicidal intent, and a plan, must be hospitalized, against her will, if necessary. The
patient expressed a desire to die, she bought a poison and developed a believable plan. She needs to be hospitalized for her own
safety. Patients with bipolar disorder have a lifetime suicide rate of 10–15%.
Bipolar Disorder
It is characterized by unpredictable swings in mood from mania (or hypomania) to depression
Some patients suffer only from recurrent attacks of mania, which in its pure form is associated with:
•• Increased psychomotor activity
•• Excessive social extroversion
•• Decreased need for sleep
•• Impulsivity and impairment in judgment
•• Expansive, grandiose
•• Sometimes irritable mood
In severe mania, patients may experience delusions and paranoid thinking indistinguishable from schizophrenia. Half of patients with
bipolar disorder present with a mixture of psychomotor agitation and activation with dysphoria, anxiety, and irritability. It may be
difficult to distinguish mixed mania from agitated depression
In bipolar II disorder the full criteria for mania are lacking and the requisite recurrent depressions are separated by periods of mild
activation and increased energy (hypomania)
Chromosome 18 is involved
Cyclothymic Disorder
•• There are numerous hypomanic periods, usually of relatively short duration, alternating with clusters of depressive
symptoms that fail, either in severity or duration, to meet the criteria of major depression
•• The mood fluctuations are chronic and should be present for at least 2 years before the diagnosis is made.
Mania
Abnormally and persistently elevated, expansive or irritable mood lasting for at least one week
•• Inflated self-esteem, grandiosity
•• Decreased sleep
•• Flight of ideas
•• Distractibility
•• Increase in goal directed activity
•• Excessive involvement in pleasurable activities
•• More talkative
Korsakoff’s syndrome presents as profound and persistent anterograde amnesia (inability to learn new material) and a milder
retrograde amnesia. and confabulation (IMPAIRMENT) About 1% of alcoholics develop cerebellar degeneration, a syndrome of
progressive unsteady stance and gait often accompanied by mild nystagmus. Atrophy of the cerebellar vermis is seen on brain
computed tomography and magnetic resonance imaging scans, but the cerebrospinal fluid is usually normal
Alcoholics can show severe cognitive problems and impairment in recent and remote memory for weeks to months after an
alcoholic binge. Increased size of the brain ventricles and cerebral sulci are seen in 50% or more of chronic alcoholics, but these
changes are often reversible, returning toward normal after a year or more of abstinence
Permanent CNS impairment (alcohol-induced persisting dementia) can develop and accounts for up to 20% of chronically
demented patients
IN ALCOHOLIC PARANOIA, hallucinations are seen
Finally, almost every psychiatric syndrome can be seen temporarily during heavy drinking or subsequent withdrawal. These include
intense sadness lasting for days to weeks in the midst of heavy drinking in 40% of alcoholics, which is classified as an alcohol-induced
mood disorder in the Fourth Diagnostic and Statistical Manual of the American Psychiatric Association (DSM-IV); severe anxiety in 10
to 30% of alcoholics, often beginning during alcohol withdrawal and which can persist for many months after cessation of drinking
(alcohol-induced anxiety disorder); and auditory hallucinations and/or paranoid delusions in the absence of any obvious signs of
withdrawal, a state now called alcohol-induced psychotic disorder and reported at sometime in 1 to 10% of alcoholics
CAGE questionare is used in alcoholism.
Marchiafava-Bignami Disease: A rare demyelination of the corpus callosum that occurs in chronic alcoholics, predominantly males.
Patients become agitated and confused and show progressive dementia with frontal release signs.
Alcoholic Withdrawal
•• Hangover
•• Visual or tactile hallucinations
•• Alcoholic hallucinosis: Hallucinations usually auditory
•• Alcoholic seizures (Rum Fits):
–– Multiple seizures occurring during absistence usually 12–48 hours after heavy drink
•• Delerium tremens:
–– Occurs within 5 days of abstinence
–– Clouding of consciousness
–– Disorientation, anxiety, perceptual defect
–– Hallucinations
–– Autonomic disturbance
–– Agitation, insomnia
–– Mc symptom of withdrawal
–– Chlordiazepoxide is used for treatment.
Differentiate Between
Alcohol withdrawal Opioid withdrawal Cocaine withdrawal
•• Hangover •• Yawning •• Hypersomnia/insomnia
•• Hallucinations/illusions •• Insomnia, dusphoric mood •• Vivid unpleasant dreams
354 USMLE Step 2 CK Platinum Notes
Substance Feature
•• Alcohol •• Morbid Jealousy
•• Cocaine •• Magnus symptom
•• Cannabis (absent withdrawal symptoms) •• Amotivational syndrome, Run Amok
•• LSD (absent withdrawal symptoms) •• Flashbacks/Bad Trips (Loss of self-control)
•• Amphetamine •• Paranoid features
Drugs used
•• Methadone •• Used in opioid dependent for short-term and long-term detoxification.
•• Naloxone •• Used in opioid overdose treatment (DOC)
•• Naltrexone •• Used in opioid dependent for rapid detoxification and maintenance therapy to prevent
•• Toipromate relapse
•• Used to decrease craving and prevent relapse in heavy alcohol drinking
•• Alcohol dependence
•• Nalmifene (iv) •• Used in opioid dependent for rapid detoxification and maintenance therapy
•• Chlordiazepoxide and Diazepam •• Used in alcohol withdrawal
•• Methadone and clonidine •• Used in opium withdrawal
Sleep
REM sleep
NREM sleep
Neurotransmitters in Sleep
•• ↑Serotonin
•• ↑acetylcholine
•• ↑Dopamine
•• ↑RR
•• ↑PR
•• Absence of sk. Muscle movement
•• Alpha rhythm
Events in NREM
•• Night terrors
•• Nocturnal enuresis
•• Bruxism
•• Sleepwalking (somnambulism)
•• Sleep talking (somniloquy)
Narcolepsy
Narcolepsy is both a disorder of the ability to sustain wakefulness voluntarily and a disorder of REM sleep regulation
The classic ‘narcolepsy tetrad’ consists of excessive daytime somnolence plus three specific symptoms related to an intrusion of
REM sleep characteristics (e.g. muscle atonia, vivid dream imagery) into the transition between wakefulness and sleep:
•• Sudden weakness or loss of muscle tone without loss of consciousness, often elicited by emotion (cataplexy)
•• Hallucinations at sleep onset (hypnogogic hallucinations) or upon awakening (hypnopompic hallucinations) and
•• Muscular paralysis upon awakening (sleep paralysis).
Miscellaneous Parasomnias
Other clinical entities fulfill the definition of a parasomnia in that they occur selectively during sleep and are associated with some
degree of sleep disruption. Examples include:
•• Jactatio capitis nocturna (nocturnal headbanging)
•• Sleep talking
•• Nocturnal paroxysmal dystonia
•• Nocturnal leg cramps
Rapid Time Zone Change (JET LAG) Syndrome
•• People experience transmeridian air travel annually, which is often associated with excessive daytime sleepiness, sleep
onset insomnia, and frequent arousals from sleep, particularly in the latter half of the night
•• Gastrointestinal discomfort is common
•• The syndrome is transient, typically lasting 2 to 14 depending on the number of time zones crossed, the direction of travel and the
traveler’s age and phase-shifting capacity.
Shift Work Sleep Disorder
People work at night, either on a permanent or rotating schedule. Leading to both sleep loss and misalignment of their
circadian rhythms with respect to their sleep wake cycle.
•• Chronic shift workers have higher rates of cardiac, gastrointestinal and reproductive disorders
•• Studies of regular night-shift workers indicate that the circadian timing system usually fails to adapt successfully to such inverted
schedules. This leads to a misalignment between the desired work-rest schedule and the output of the pacemaker and in disturbed
daytime sleep
•• Consequent sleep deprivation, increased length of time awake prior to work, and misalignment of circadian phase produce
decreased alertness and performance, increased reaction time, and increased risk of performance lapses, thereby resulting in
greater safety hazards among night workers and other sleep-deprived individuals.
Dissociative Disorders
•• Dissociative amnesia
•• Dissociative fugue
•• Dissociative identity disorder
•• Depersonalization disorder
Dissociative Amnesia
A navy officer cannot recall an unfortunate event of fire on his ship in which 45 sailors lost life
•• MC type
•• Sudden inability to recall important personal information
•• Patchy impairment of personal memory
•• Patient is aware of amnesia.
Dissociative Fugue
A doctor in California for 5 years has no memory of coming and living in California
•• Feature is wandering away from home
•• Purposeful new identity with complete amnesia for earlier life
•• Not aware of amnesia
Dissociative Identity Disorder
A female is shown her previous photograph and she has no recollection of same as to who’s photograph it is
•• Person is dominated by two or more personalities
•• One personality is not aware of existence of other
Depersonalization
A female says that she is outside of herself watching her life in a movie. She knows that she is really living her life
•• An alteration in perception or experience of self to the extent that persons sense of his/her own reality is lost
•• It is detachment from self, social situation or environment. (Derealization)
358 USMLE Step 2 CK Platinum Notes
Ganser’s Syndrome
•• Hysterical Pseudodementia
•• Feature is ‘approximate answers’
•• Answer is wrong but person understands the nature of question asked
•• Hallucinations and apparent clouding of consciousness is a feature.
Newer Concepts
•• Holiday heart syndrome Atrial or ventricular arrhythmia after taking a binge of alcohol with no other evidence of heart
disease.
•• van Gogh syndrome Self mutilation in schizophrenia.
•• Othello syndrome A psychosis in which jealousy involving spouse is noticed.
•• Clerambault’s syndrome Erotomania in which usually a female has got a conviction that a person of higher status is in
love with her.
•• Capgras syndrome Patient sees a familiar person as a stranger. Delusion of doubles.
•• Cotard’s syndrome Delusion that one has lost everything (possessions, strength and even organ such as heart).
Thinks that his bowels are rotting and he will never pass tools again.
•• Gilles de la tourette syndrome Multiple motor ticks.
•• Kleine Levin syndrome Hypersomnia, Hyperphagia and Hyper sexuality
Amygdala involved
•• Pickwickian syndrome Sleep apnea seen in obese persons associated with Hypersomnia.
•• Gelineau’s syndrome Narcolepsy with Hypersomnia
•• Stockholm syndrome Feelings of captives towards capitor
Lithium
•• Lithium acts by interfering with cell membrane ion transport and excitability, adenylatecyclase activation, neurotransmitter
(norepinephrine) release and Na+, K+-ATPase activity
•• Lithium is not bound to plasma proteins
•• Therapeutic serum levels are 0.6 to 1.2 mmol/l
•• Liver is not affected in lithium toxicity.
Clinical Toxicity
Effects begin 1 to 4 hour after acute ingestion
Gastrointestinal effects include nausea, vomiting and diarrhea
Polyuria is the commonest symptom
Neuromuscular effects include weakness, confusion, ataxia, tremors, fasciculations, myoclonus, choreoathetosis, coma and seizures
Cardiovascular effects include arrhythmias and hypotension
Hyperthermia can occur
Leukocytosis, hyperglycemia, albuminuria, glycosuria, nephrogenic diabetes insipidus, and a falsely elevated serum chloride
level (due to interference by lithium with its assays) resulting in a low anion-gap may be present
ECG changes include sinus tachycardia or bradycardia, flattened or inverted T waves, AV block, and a prolonged QT interval
Hypothyroidism is seen
Prolonged or permanent encephalopathy and movement disorders can occur in patients with severe poisoning
Diagnosis: A serum level must be requested specifically
Contraindicated in pregnancy
Causes CVS defects
Psychiatry 359
List of TCA’S
•• Amitriptyline
•• Amoxapine
•• Cloimipramine
•• Desipramine
•• Doxepin
SSRI’S are
•• Fluoxetine
•• Fluvoxamine
•• Paroxetine
•• Sertaline
•• Citalopram
Azapirones
•• Buspirone
•• Gepirone
•• Ipsapirone
Buspirone: 5 HT1A receptor agonist
•• Nonsedating
•• No tolerance
•• No physical dependence
ECT
Used in:
•• Major severe depression
•• Delusional depression
•• Severe psychosis
•• Severe catatonia
•• Absolute contraindication is: Raised ICT
•• Mc side effect in Direct ECT is T4-T8 spine #
•• Mc side effect of Modified ECT is retrograde amnesia
•• Antegrade amnesia also occurs but with ↓ frequency
•• ECT ↓IOT (intraoccular pressure)
Disulfiram Inhibits
•• Aldehyde dehydrogenase
•• Alcohol dehydrogenase
•• Dopamine β hydroxylase
•• Cytochrome P450 enzymes
Antipsychotics
Antipsychotics act as dopamine D2 receptor antagonists, blocking dopaminergic transmission in the mesolimbic pathways.
Conventional antipsychotics are associated with problematic extrapyramidal side-effects which has led to the development of
atypical antipsychotics such as clozapine.
•• Extrapyramidal side-effects
•• Parkinsonism
•• Acute dystonia (e.g. torticollis, oculogyric crisis)
•• Akathisia (severe restlessness)
•• Tardive dyskinesia (late onset of choreoathetoid movements, abnormal, involuntary, may occur in 40% of patients, may be
irreversible, most common is chewing and pouting of jaw.
Other Side-effects
•• Antimuscarinic: dry mouth, blurred vision, urinary retention, constipation
•• Sedation, weight gain
•• Raised prolactin: galactorrhea
•• Neuroleptic malignant syndrome: pyrexia, muscle stiffness
•• Reduced seizure threshold (greater with atypicals)
•• Antipsychotics are not addictive
4. Methadone
5. Propranolol
Ans. 4. Methadone
Cataplexy
•• Cataplexy is ‘sudden and transient episode of loss of muscle tone, often triggered by emotions’
•• It is a rare disease, but frequently affects people who have narcolepsy, a disorder whose principal signs are EDS (Excessive Daytime
Sleepiness), sleep attacks, sleep paralysis, hypnagogic hallucinations and disturbed night-time sleep
•• The term cataplexy originates from the Greek ‘kata’, meaning down, and plexis, meaning a stroke or seizure
•• Cataplexy manifests itself as muscular weakness which may range from a barely perceptible slackening of the facial muscles to
the dropping of the jaw or head, weakness at the knees, or a total collapse
•• Usually the speech is slurred, vision is impaired (double vision, inability to focus), but hearing and awareness remain normal. These
attacks are triggered by strong emotions such as exhilaration, anger, fear, surprise, orgasm, awe, embarrassment and laughter.
Psychiatry 363
Cataplexy may be partial or complete, affecting a range of muscle groups, from those controlling facial features to (less commonly)
those controlling the entire body
Arm weakness
•• Sagging jaw
•• Drooping head
•• Slumping of the shoulders
•• Slurred speech
•• Generalized weakness
•• Knee buckling
Adjustment Disorder
•• It is the commonest group of psychiatric disorders as most patients do not fulfill the criterion for depression
•• Precipitants of Adjustment disorder are: Terminal illness, new marriage, separation, divorce, death of companion, natural disaster
•• Symptoms vary from anxiousness, depressive or mixed
•• It starts within 3 months. Depression is common in
–– AIDS
–– Postsurgery
–– Post MI
–– Postpartum
–– Myxedema
Cancer patients have highest fear for relapse of disease (Damocles syndrome)
‘Obsessive-Compulsive’ Disorder
It is characterized by obsessive thoughts and compulsive behaviors that impair everyday functioning. Serotonin is implicated
Obsessive-compulsive disorder describes patients with recurrent thoughts or impulses
(Obsessions) and/or recurrent behaviors or actions (compulsions) that cause marked dysfunction in their occupational and/
or interpersonal lives. It is not the same as obsessive-compulsive personality disorder. Look for washing rituals (washing of
hands repeatedly in a day) and/or checking rituals (checking to see if the window is locked repeatedly per day).
364 USMLE Step 2 CK Platinum Notes
Medication-Related
•• Thiazide diuretics and beta blockers
•• Calcium channel blockers and angiotensin-converting enzyme inhibitors are less frequently cited. Estrogens
•• GnRH agonists
•• H2 antagonists
•• Spironolactone
•• Antidepressant and antipsychotic agents particularly neuroleptics, tricyclics and SSRIs Digoxin
•• Sildenafil is the only approved and effective oral agent for the treatment of ED
•• Vacuum Constriction Devices are a reasonable treatment alternative for select patients who cannot take sildenafil or do not
desire other interventions
•• Intraurethral Alprostadil If a patient fails to respond to oral agents, a reasonable next choice is intraurethral or self-injection or
vasoactive substances. Intraurethral prostaglandin E1 (alprostadil)
•• Intracavernosal Self-Injection
•• Surgery A less frequently used form of therapy for ED involves the surgical implantation of a semi-rigid or inflatable penile
prosthesis
Important Points
Counsel patients about injury risk and prevention
•• All physicians, regardless of specialty, have the opportunity to intervene with the ‘host.’ It is standard practice during history taking
for physicians to inquire about tobacco and alcohol use, sexual practices, and exercise habits of adult patients, and to counsel
parents about specific risks at different stages of child development. There are known risks for injury and protective strategies that
patients can use to lower the risk of injury for themselves and their families
•• Identify and refer abused patients. Family violence is a serious and underdiagnosed problem in American society. Abused
patients include children, battered spouses and/or partners, elders, and those raped and sexually abused by those with whom
they have a personal or intimate relationship. Hospitals, clinics, and doctors’ offices may provide the first opportunity for abused
patients, particularly adult women, to acknowledge the abuse, receive support, find protection, and break the cycle of violence
•• Emphasize rehabilitation and community follow-up. Tertiary prevention involves minimizing functional disability, a
consequence of serious injury. Physicians can help their patients return to productive lives by ensuring that patients receive
appropriate physical and occupational therapy and that they have access to community services after discharge.
DERMATOLOGY
Dermatology 4
Primary Skin Lesions
•• Macule:
A flat, colored lesion, < 2 cm in diameter, not raised above the surface of the surrounding skin. A ‘freckle,’ or ephelid, is a prototype
pigmented macule.
•• Patch:
A large (> 2 cm), flat lesion with a color different from the surrounding skin. This differs from a macule only in size.
•• Papule:
A small, solid lesion, < 1 cm in diameter, raised above the surface of the surrounding skin and hence palpable (e.g. a closed comedone,
or whitehead, in acne).
•• Nodule:
A larger (1–5 cm), firm lesion raised above the surface of the surrounding skin.
•• Tumor: A solid, raised growth > 5 cm in diameter.
•• Plaque:
A large (> 1 cm), flat-topped, raised lesion; edges may either be distinct (e.g. in psoriasis) or gradually blend with surrounding skin (e.g.
in eczematous dermatitis)
•• Vesicle:
A small, fluid-filled lesion, <1 cm in diameter, raised above the plane of surrounding skin. Fluid is often visible, and the lesions are often
translucent.
•• Pustule:
A vesicle filled with leukocytes. Note: The presence of pustules does not necessarily signify the existence of an infection.
•• Bulla:
A fluid-filled, raised, often translucent lesion > 1 cm in diameter.
•• Cyst:
A soft, raised, encapsulated lesion filled with semisolid or liquid contents.
•• Wheal:
A raised, erythematous papule or plaque, usually representing short-lived dermal edema.
•• Telangiectasia:
Dilated, superficial blood vessels.
Basics of Dermatology
Important Questions asked as definition
•• Acanthosis: Increase in thickness of prickle cell layer.
•• Acantholysis: Loss of coherence between epidermal cells. Seen in Pemphigus
•• Corps Grains: Acantholytic, Dyskeratotic, Basophilic cells with rounded nuclei and perinuclear halo. Seen in Dariers disease
•• Dyskeratosis: Abnormal development of epidermal cells. (Feature of Premalignant lesions)
•• Hyperkeratosis: Increased thickness of Stratum Corneum
•• Para keratoses: Presence of immature nucleated cells in Stratum Corneum
372 USMLE Step 2 CK Platinum Notes
MILIA
•• These represent retention cysts of the pilosebaceous follicles
•• They occur in approximately 50% of neonates as firm pearly white 1 to 2 mm papules particularly on the face
•• They usually disappear by 4 weeks of age
•• Epstein’s pearls are epidermal cysts on the palate present in the majority of newborns
•• Persistent milia maybe a marker for certain syndromes including
–– Bazex’s syndrome
–– Orofaciodigital syndrome type I and
–– Marie–Unna hypotrichosis
Scabies
An infestation caused by the mite Sarcoptes scabiei, and the treatment of choice is permethrin cream application overnight, repeated
once in 7 days.
All family members and caretakers of the child should be treated. Bedlinens and clothes should be laundered in hot water, or dry cleaned
if laundering is not feasible.
Ivermectin in a single oral dose is used to treat scabies in immunocompromised individuals who have overwhelming infestation that
would not respond to standard topical therapy. It is not recommended for use in ordinary scabies.
Lindane cream should not be used in small infants because of its potential for neurotoxicity. It is relatively safe for use in adults, but
permethrin cream has an established safety record that makes it the treatment of choice today.
It is very important to treat all family members and caretakers simultaneously for the same reason.
Precipitated sulfur in petroleum was often used to treat scabies in infants and pregnant women but has become rather obsolete, as
permethrin cream has proven.
Pediculosis
•• Caused by lice
•• Lays down eggs called nits
•• Pediculosis capitis head
•• Pediculosis corporis body (Vagabonds disease)
•• Pediculosis pubis pubic region
•• Maculae cerulea are a feature
•• Permethrin is drug of choice.
Scarlet fever
•• Caused by Group A β Hemolytic streptococci
•• Common in children
•• Balanchable erythema
•• Circumoral pallor with malar flush
–– Sand paper skin
–– Pastias lines
–– Red strawberry tongue.
Psoriasis
Most important topic in Dermatology * (DETAILED)
•• ‘Chronic inflammatory skin disorder’ clinically characterized by erythematous, sharply demarcated papules and rounded
plaques, covered by ‘silvery micaceous scale’
•• The skin lesions of psoriasis are variably pruritic
•• Traumatized areas often develop lesions of psoriasis (Koebner or isomorphic phenomenon)
•• Additionally, factors that may exacerbate psoriasis including infections, stress, and medications (lithium, beta blockers, and
antimalarials).
374 USMLE Step 2 CK Platinum Notes
•• The most common variety of psoriasis is called ‘plaque type.’ Patients with plaque-type psoriasis will have stable, slowly
growing plaques, which remain basically unchanged for long periods of time
•• The most common areas for plaque psoriasis to occur are the ‘extensors’ elbows, knees, gluteal cleft, and the scalp. Involvement
tends to be symmetric
•• ‘Inverse psoriasis’ affects the intertriginous regions including the axilla, groin, submammary region, and navel, it also tends to
affect the scalp, palms, and soles
•• Eruptive psoriasis (Guttate psoriasis) is most common in children and young adults. It develops acutely in individuals without
psoriasis or in those with chronic plaque psoriasis. Patients present with many small erythematous, scaling papules, frequently
after upper respiratory tract infection with α-hemolytic streptococci
•• About half of all patients with psoriasis have fingernail involvement, appearing as punctuate pitting, nail thickening, or
subungual hyperkeratosis
•• About 5 to 10 % of patients with psoriasis have associated joint complaints and these are most often found in patients with
fingernail involvement
•• Although some have the coincident occurrence of classic rheumatoid arthritis many have joint disease that falls into one of three
types associated with psoriasis: (1) asymmetric inflammatory arthritis most commonly involving the distal and proximal
interphalangeal joints and less commonly the knees, hips, ankles and wrists; (2) a seronegative rheumatoid arthritis-like
disease; a significant portion of these patients go onto develop a severe destructive arthritis; or (3) disease limited to the
spine (psoriatic spondylitis)
•• Psoriasis has been linked to HLA-Cw6 and, to a lesser extent, to HLA-DR7.
Nail Changes
•• Thimble pitting
•• Nail plate thickening
•• Subungual hyperkeratosis
•• Oncholysis
•• Oil spots
•• Brownish discoloration
Histopathology
Hyperkeratoses
Parakeratosis
Orthokeratosis
Micro Munro abscesses in stratum Corneum
Suprapapillary thinning
Dermatology 375
Also Remember
•• Turnover time in Psoriasis is 4 days only instead of 28 days
•• Psoriatic involvement on flexor aspects is called Inverse Psoriasis
•• Burkleys Membrane is seen in Psoriasis
•• Arthritis mutilans is severe but rare form of Psoriatic arthritis
–– Treatment of choice for Psoriasis is PUVA therapy
–– Treatment of choice for Psoriatic arthopathy is Methotrexate
–– Retinoids are the drugs of choice for Psoriasis in AIDS, Pustular Psoriasis, Psoriatic Erythoderma
–– In pregnancy safest drug for pustular Psoriasis is Prednisolone
•• Lithium, NSAIDS, Beta blockers, Antimalarials cause deterioration of disease
•• Auspitz sign: Rubbing of skin leading to removal of scales leads to pinpoint bleeding is. It is seen in Psoriasis
•• Grattage sign is other sign in Psoriasis which is scratching of skin with glass slide accentuates scale formation. Seen in Psoriasis
•• On continuous scratching a glistening white membrane appears called Burkleys membrane. Seen in Psoriasis
•• von Zumbusch phenomenon: Combination of ‘Erythroderma plus sterile pustular psoriasis’.
Pemphigus
•• Uncommon autoimmune skin disorder characterized by blistering and erosions involving the mucous membranes and skin.
The autoimmune attack is on the junctions between epithelial cells in the epidermis. The blisters occur high in the epithelium and
can rupture easily, producing painful erosions. Nikolsky’s sign, in which rubbing of apparently unaffected skin causes a separation
of the outer layers, is a helpful diagnostic clue
•• Pemphigus tends to begin in the mouth, where rapid rupture of the blisters may lead to the impression that the initial lesion is an
ulcer rather than a blister
•• Biopsy with immunofluorescence studies can demonstrate blister formation high in the epithelium that is accompanied by IgG
deposition on epithelial cell surfaces.
There are three types of pemphigus which vary in severity: pemphigus vulgaris, pemphigus foliaceus, and paraneoplastic pemphigus.
•• It occurs when antibodies attack Desmoglein 3
•• Antibodies against intercellular substance
•• Autoimmune disease
•• Intradermal acantholytic bullae present
•• Intradermal acantholytic bullae present with oral mucosal lesions
•• Acantholysis is seen in epidermis
•• Mucus membrane lesions seen
•• Pemphigus Foliaceus is the least severe of the three varieties
•• Desmoglein 1, the protein that is destroyed by the autoantibody, is only found in the top dry layer of the skin
•• Pemphigus foliaceus (PF) is characterized by crusty sores that often begin on the scalp, and may move to the chest, back, and face
•• Mouth sores do not occur. It is not as painful as pemphigus vulgaris, and is often mis-diagnosed as dermatitis or eczema.
376 USMLE Step 2 CK Platinum Notes
•• The least common and most severe type of pemphigus is Paraneoplastic pemphigus (PNP)
•• This disorder is a complication of cancer, usually lymphoma and Castleman’s disease. It may precede the diagnosis of the tumor
•• Painful sores appear on the mouth, lips, and the esophagus
•• In this variety of pemphigus, the disease process often involves the lungs, causing Bronchiolitis obliterans (constrictive
bronchiolitis)
•• Complete removal and/or cure of the tumor may improve the skin disease, but lung damage is generally irreversible.
Hailey-Hailey Disease
•• Also called Familial Benign Pemphigus
•• It is an inherited (genetic) skin disease
•• Not an autoimmune disease
•• No subepidermal blistering
•• It is therefore not considered part of the Pemphigus group of diseases
Treatment
If not treated, pemphigus can be fatal due to overwhelming infection of the sores
The most common treatment is the administration of oral steroids, especially prednisone. The side effects of cortico-steroids may
require the use of so-called steroid-sparing or adjuvant drugs. The immuno-suppressant (Mycophenolic acid) is among those being
used:
Intravenous gamma globulin (IVIG) may be useful in severe cases, especially paraneoplastic pemphigus.
Mild cases sometimes respond to the application of topical steroids.
Recently, Rituximab, an anti-CD20 antibody, was found to improve otherwise untreatable severe cases of Pemphigus vulgaris.
If skin lesions do become infected, antibiotic may be prescribed.
Pemphigus Erythematosus
•• Also known as Senear-Usher syndrome
•• It is an overlap syndrome
•• It has erythematous, scaly plaques in a lupus-like butterfly distribution on the face as well as involving the scalp, back, chest, and
intertriginous areas
•• Sunlight may exacerbate the disease.
Pemphigus Pemphigoid
•• Common in age group 40–60 years •• Common greater 60 years
•• Nikolskys sign present •• Nikolskys sign absent
•• Intraepidermal bullae •• Subepidermal bullae
•• Flaccid bullae •• Tense bullae
•• Mucosal involvement common •• Mucosal involvement uncommon
•• Acantholysis present •• Acantholysis absent
Dermatology 377
A 46-year-old female presents to Dermatology clinic with painful oral ulcers. Physical examination demonstrates widespread
erosions of her mucous membranes. Dermatological examination reveals a friable mucosa. Biopsy of perilesional mucosa
demonstrates acantholysis; direct immunofluorescence demonstrates an intraepidermal band of IgG and C3. The most likely
diagnosis is: Pemphigus vulgaris.
An 88 year old elderly patient from Carolina complains to a dermatologist of sores in the oral cavity and on the skin. These
lesions have developed over about a month. Physical examination demonstrates three painful erosions on the oral mucosa
and tongue. Raw areas with crusting on the skin is seen on the face and trunk. The patient states that the skin lesions had
started as blisters that had quickly broken. While the involved areas were painful, no itchiness had been experienced. Careful
examination of the edge of the skin lesions demonstrates a few flaccid bullae. Rubbing of the skin near an affected area
easily detaches the superficial part of the epidermis from the underlying skin. No target-like lesions are seen. The most likely
diagnosis is:
PEMPHIGUS VULGARIS
USMLE Case Scenario
Bullous pemphigoid is an autoimmune disease in which the characteristic finding is linear deposition of IgG antibodies and
complement in the basement membrane zone. Reactivity also occurs in the basal cell-basement membrane attachment plaques
(hemidesmosomes), where most of the bullous pemphigoid antigen (BPAG) is located. Early lesions show a perivascular infiltrate
of lymphocytes and variable numbers of eosinophils, occasional neutrophils, superficial dermal edema, and associated basal cell
layer vacuolization. Bullous pemphigoid is characterized by a:
1. Epidermal, nonacantholytic blister
2. Epidermal, acantholytic blister
3. Subepidermal, acantholytic blister
4. Subepidermal, nonacantholytic blister
Ans. 4. Subepidermal, nonacantholytic blister
Lichen Planus
•• Flat toppled papules with white lacy markings on mucosa (Wickham’s striae)
•• Associated with Hepatitis C
•• ‘Civatte bodies’ seen
•• ‘Basal cell degeneration’ seen
•• ‘Max Josephs’ space seen
Multiple, erythematous, annular lesions with ‘collarette of scales’ predominantly on ‘trunks’ is diagnostic of Ptyriasis Rosacea.
•• This pathology is associated with Epstein-Barr virus (EBV) and occurs mostly in people with HIV, both immunocompromised
and immunocompetent, albeit it can affect patients who are HIV negative.
•• Unilateral or bilateral nonpainful white lesions can be seen on the margins, dorsal or ventral surfaces of the tongue, or on buccal
mucosa. The lesions may vary in appearance from smooth, flat, small lesions to irregular ‘hairy’ or ‘feathery’ lesions with prominent
folds or projections.
•• Lesions are adherent, and only the most superficial layers can be removed by scraping. There is no associated erythema or edema
of the surrounding tissue. Hairy leukoplakia may also involve dorsal and ventral tongue surfaces, the buccal mucosa, or the gingiva.
On the ventral tongue, buccal mucosa, or gingiva, the lesion may be flat and smooth, lacking the characteristic ‘hairy’ appearance.
TINEA
•• A dermatophyte is a parasitic fungus (mycosis) that infects the skin
•• The term embraces the imperfect fungi of the genera Epidermophyton, Microsporum and Trichophyton
•• Dermatophytes (name based on the Greek for ‘skin plants’) are a common label for a group of three types of Fungus that commonly
causes skin disease in animals and humans
•• They are anamorphic (asexual or imperfect)
•• Dermatophytes cause infections of the skin, hair and nails due to their ability to obtain nutrients from Keratinized material.
(Superficial)
•• The organisms colonize the keratin tissues and inflammation is caused by host response to metabolic by-products
•• They are usually restricted to the nonliving cornified layer of the epidermis because of their inability to Penetrate viable tissue
of an immunocompetent host. Invasion does elicit a host response ranging from mild to severe
•• Acid proteinases, elastase, keratinases, and other proteinases reportedly act as virulence factors.
Treatment
•• Tinea corpora (body), tinea manus (hands), tinea cruris (groin), tinea pedis (foot) and tinea facie (face) can be treated
topically
•• Tinea unguium (nails) usually will require oral treatment with terbinafine, itraconizole, or griseofulvin
•• Griseofulvin is usually not as effective as terbinafine or itraconizole
•• Tinea capitis (scalp) must be treated orally, as the medication must be present deep in the hair follicles toeradicate the fungus.
Usually griseofulvin is given orally for 2 to 3 months
•• Clinically dosage up to twice the recommended dose might be used due to relative resistance of some strains of dermatophytes
•• Tinea pedis is usually treated with topical medicines, like ketoconazole or terbinafine, and pills, or with medicines that contains
miconazole, clotrimazole, or tolnaftate. Antibiotics may be necessary to treat secondary bacterial infections that occur in addition
to the fungus (for example, from scratching)
Dermatophytes are filamentous fungi that infect only superficial keratinized tissue skin, hair and nail.
Tinea Causes
•• Tinea capitis: Ringworm of scalp
•• Tinea circinata: Ringworm of glabrous skin
•• Tinea barbae: Ringworm of beard
•• Tinea pedis/Trichophyton rubrum: Athletes foot
•• Tinea cruris: Dhobi itch or Jock itch
•• Tinea unguium: Ringworm of nails
Remember: ‘Tinea In cognito’ is extensive ringworm infection with atypical appearance due to steroid misuse.
Malasezia Furfur
•• Causes Tinea Versicolor (Versatile colors)
•• Scaly, Hypopigmented macules appearing on trunk and shoulders
•• Head and lower limbs are spared usually
•• Short Hyphae and round spores (Sphagetti and meat balls) appearance on KOH is seen
•• Tinea versicolor is a common, benign, superficial cutaneous fungal infection usually characterized by Hypopigmented or
hyperpigmented macules and patches on the chest and the back. In patients with a predisposition, the condition may chronically
recur
•• The fungal infection is localized to the stratum corneum
•• Ketocanozole and itracanazole as systemic therapy are useful
•• Griseofulvin is ineffective
•• Tinea versicolor is caused by the dimorphic, lipophilic organism, Malassezia furfur, M furfur is now the accepted name for the
organism. ‘Pityrosporon orbiculare, Pityrosporon ovale, and Malassezia ovalis’ are synonyms for M furfur.
•• M furfur is a member of normal human cutaneous flora.
•• The involved skin regions are usually the trunk, the back, the abdomen, and the proximal extremities. The face, the scalp, and the
genitalia are less commonly involved.
380 USMLE Step 2 CK Platinum Notes
•• The color of each lesion varies from almost white to reddish brown or fawn colored.
•• Patients often complain that the involved skin lesions fail to tan in the summer.
•• Occasionally, a patient also complains of mild pruritus.
•• Greater than 20% of patients report a positive family history of the condition. This subset of patients records a higher rate of
recurrence and longer duration of disease.
Acne Vulgaris
•• Acne vulgaris affects the majority of adolescents and hormonal influences, abnormal keratinization of pilosebaceous units and
colonization by bacteria (Propionibacterium acnes) are important pathogenetic elements
•• Acne vulgaris is a self-limited disorder primarily of teenagers and young adults, Involves pilosebacious glands
•• The permissive factor for the expression of the disease in adolescence is the increase in sebum release by sebaceous glands after
puberty
•• Small cysts, called comedones, form in hair follicles due to blockage of the follicular orifice by retention of sebum and keratinous
material
•• The activity of lipophilic yeast (Pityrosporum orbiculare) and bacteria (Proprionobacterium acnes) within the comedones releases
free fatty acids from sebum, causes inflammation within the cyst, and results in rupture of the cyst wall. The clinical hallmark of
acne vulgaris is the comedone, which may be closed (whitehead) or open (blackhead).Comedones are usually accompanied by
inflammatory lesions: papules, pustules, or nodules
•• The most common location for acne is the face, but involvement of the chest and back is not uncommon
•• Systemic medications such as lithium, isoniazid, halogens, phenytoin, and phenobarbital may produce acneiform eruptions
Treatment
•• Minimal to moderate, pauci-inflammatory disease may respond adequately to local therapy alone
•• Topical agents such as retinoic acid, benzoyl peroxide, or salicylic acid may alter the pattern of epidermal desquamation,
preventing the formation of comedones and aiding in the resolution of preexisting cysts. Topical antibacterial agents such as
benzoyl peroxide, azelaic acid, topical erythromycin (with or without zinc), clindamycin, or tetracycline are also useful
adjuncts to therapy
•• Patients with moderate to severe acne with a prominent inflammatory component will benefit from the addition of systemic
therapy. Oral tetracyclines or erythromycin in doses of 250 to 1000 mg/d will decrease follicular colonization with some of the
lipophilic organisms
•• Severe nodulocystic acne not responsive to oral antibiotics, hormonal therapy, or topical therapy may be treated with the
synthetic retinoid isotretinoin.
Acne vulgaris affects the majority of adolescents. Hormonal influences, abnormal keratinization of pilosebaceous units and
colonization by bacteria (Propionibacterium acnes) are important pathogenetic elements
Treatment of acne depends on the severity of the condition
Topical application of comedolytic agents such as retinoids (tretinoin, adapalene, and the new yeast-derived agent azaleic acid)
is effective for mild to moderate forms of noninflammatory acne, characterized by open comedones. Daily application of these
compounds will result in improvement within several weeks after starting treatment. Mild skin irritation and scaling may be minimized
by starting with low-concentration creams and then progressively increasing the concentration. Another side effect of retinoids is
increased photosensitivity. The patient must be instructed to avoid prolonged exposure to the sun and to use a sunscreen.
Acne Rosacea
•• Acne rosacea is an inflammatory disorder predominantly affecting the central face
•• It is seen almost exclusively in adults,
•• Rosacea is seen more often in women, but those most severely affected are men
•• It is characterized by the presence of
–– Erythema,
–– Telangiectases, and
–– Superficial pustules
Dermatology 381
TREATMENT
Acne rosacea can generally be treated effectively with oral tetracycline in doses ranging from 250 to 1000 mg/d. Topical
metronidazole or sodium sulfacetamide has also been shown to be effective. In addition, the use of low-potency, nonfluorinated
topical glucocorticoids, particularly after cool soaks, is helpful in alleviating facial erythema.
Erythrasma
•• Erythrasma is a chronic superficial infection of intertriginous areas
•• Organism causing erythrasma is Corynebacterium Minutissimum
•• Erythrasma produces coral red fluorescence under wood light secondary to production of porphyrins
•• Erythrasma is more common in blacks
•• Typical lesion is well demarcated, reddish brown macule with wrinkled appearance of skin
•• Inner thighs, crural region, scrotum and toe webs are commonly involved.
Corynebacterium Minutissimum
It is a part of normal skin flora
•• It is lipophilic
•• Gram-positive
•• Aerobic
382 USMLE Step 2 CK Platinum Notes
–– Catalase positive
–– Nonspore forming diphtheroid
It ferments
•• Glucose
•• Dextrose
•• Sucrose
•• Maltose and
•• Mannitol
Atopic Dermatitis
•• Atopic dermatitis (AD) is the cutaneous expression of the atopic state, characterized by a family history of asthma, hay fever,
or dermatitis
•• Also termed as ITCH DISEASE
•• The infantile pattern is characterized by weeping inflammatory patches and crusted plaques that occur on the face (MC), neck,
extensor surfaces, and groin
•• The childhood and adolescent pattern is marked by dermatitis of flexural skin, particularly in the antecubital and popliteal
fossae
•• Pruritus is a prominent characteristic of AD and many of the cutaneous findings in affected patients are secondary to rubbing and
scratching
•• Other cutaneous stigmata of AD are perioral pallor, an extra fold of skin beneath the lower eyelid (Dennie’s line), increased palmar
markings, and increased incidence of cutaneous infections, particularly with Staphylococcus aureus
•• Atopic individuals often have dry itchy skin, abnormalities in cutaneous vascular responses, and, in some instances, elevations
in serum IgE
•• Biopsy is the best test
•• ‘Berloque dermatitis’ is due to contact with Cosmetics
•• Excoriation, Lichenification, Hyperlinear palms are seen.
Contact Dermatitis
•• Contact dermatitis is an inflammatory process in skin caused by an exogenous agent or agents that directly or indirectly injure the skin
•• This injury may be caused by an inherent characteristic of a compound irritant contact dermatitis (ICD). An example of ICD would
be dermatitis induced by a concentrated acid or base. Agents that cause allergic contact dermatitis (ACD) induce an antigen-
specific immune response. The clinical lesions of contact dermatitis may be acute (wet and edematous) or chronic (dry, thickened
and scaly), depending on the persistence of the insult
•• The most common presentation of contact dermatitis is hand eczema and it is frequently related to occupational exposures
ICD
•• It is generally strictly demarcated and often localized to areas of thin skin (eyelids, intertriginous areas) or to areas where the
irritant was occluded
•• Lesions may range from minimal skin erythema to areas of marked edema, vesicles, and ulcers
•• Chronic low-grade irritant dermatitis is the most common type of ICD and
•• The most common area of involvement is the hands
•• The most common irritants encountered are chronic wet work, soaps, and detergents. Treatment should be directed to
avoidance of irritants and use of protective gloves or clothing
ACD
•• It is a manifestation of delayed type hypersensitivity mediated by memory T lymphocytes in the skin
•• The most common cause of ACD is exposure to plants, specifically to members of the family Anacardiaceae; including the
genera Toxicodendron, Anacardium, Gluta, Mangifera, and Semecarpus
•• Poison ivy, poison oak and poison sumac are members of the genus Toxicodendron and cause an allergic reaction marked by
erythema, vesiculation and severe pruritus
•• The eruption is often linear, corresponding to areas where plants have touched the skin.
Dermatology 383
Patch Tests
Patch testing is used to validate a diagnosis of allergic contact sensitization and to identify the causative allergen. Because the
entire skin of sensitized humans is allergic, the test reproduces the dermatitis in one small area where the allergen is applied,
usually on the back. The suspected allergen is applied to the skin, occluded, and left in place 48 hours. A positive test reproduces an
eczematous response at the test site from 48 hours up to a week later. The latter is a delayed hypersensitivity reaction. Considerable
experience is required to accurately perform and interpret patch tests. Photopatch testing is performed to detect photocontact
allergy. Suspected photoallergens are placed on the skin in two sets. One set of allergens is irradiated with appropriate wavelengths
of light after the patches are in place on the skin 24 hours; the second set of the same photoallergens is kept covered to serve as
controls. Photoallergens cause an erythematous reaction that is evident 24 hours after exposure to light.
Nummular Eczema
•• Nummular eczema is characterized by circular or oval ‘coinlike’ lesions
•• Initially, this eruption consists of small edematous papules that become crusted and scaly. The most common locations are on the
trunk or the extensor surfaces of the extremities, particularly on the pretibial areas or dorsum of the hands
•• It occurs more frequently in men and is most commonly seen in middle age.
Hand Eczema
Hand eczema is a very common, chronic skin disorder
It represents a large proportion of occupation-associated skin disease
It may be associated with other cutaneous disorders such as atopic dermatitis or may occur by itself
It may present with dryness and cracking of the skin of the hands as well as with variable amounts of erythema and edema. Often,
the dermatitis will begin under rings where water and irritants are trapped
A variant of hand dermatitis, dyshidrotic eczema, presents with multiple, intensely pruritic, small papules and vesicles occurring on
the thenar and hypothenar eminences and the sides of the fingers.
Asteatotic Eczema
•• Asteatotic eczema, also known as xerotic eczema or ‘winter itch,’ is a mildly inflammatory variant of dermatitis that develops
most commonly on the lower legs of elderly individuals during dry times of year
•• Fine cracks, with or without erythema, characteristically develop on the anterior surface of the lower extremities. Pruritus is variable
•• Asteatotic eczema responds well to avoidance of irritants, rehydration of the skin, and application of topical emollients.
•• Chronic stasis dermatitis is often associated with dermal fibrosis that is recognized clinically as brawny edema of the skin
•• Stasis dermatitis is often complicated by secondary infection and contact dermatitis
•• Severe stasis dermatitis may precede the development of stasis ulcers.
Squamous Cell Carcinoma: Primary cutaneous SCC is a malignant neoplasm of keratinizing epidermal cells.
Unlike BCC, which has a very low metastatic potential, SCC can metastasize and grow rapidly. The clinical features of SCC vary widely.
Predisposition to SCC
–– Actinic keratosis
–– Bowens disease
–– Lichen planus
–– DLE
Commonly, SCC appears as an ulcerated nodule or superficial erosion on the skin or lower lip, but it may present as a verrucous
papule or plaque.
Unlike BCC, overlying telangiectasias are uncommon.
The margins of this tumor may be ill-defined, and fixation to underlying structures may occur. Cutaneous SCC may develop anywhere
on the body, but it usually arises on sun-damaged skin.
Keratoacanthoma
•• Typically appears as a dome-shaped papule with a central keratotic crater, Expands rapidly, and commonly regresses without
therapy
•• This lesion can be difficult to differentiate from SCC.
Dermatology 385
Melanomas
Originate from melanocytes, pigment cells normally present in the epidermis and sometimes in the dermis.
The individuals most susceptible to development of melanoma are:
•• Those with fair complexions, red or blonde hair, blue eyes and freckles and who tan poorly and sunburn easily.
•• Family history of melanoma
•• The presence of a clinically atypical mole (dysplastic nevus)
•• A giant congenital melanocytic nevus, or a small to medium-sized congenital melanocytic nevus the presence of a higher than
average number of ordinary melanocytic nevi; and immunosuppression. Melanoma is relatively rare in heavily pigmented people
Melanomas often express cell-surface antigens that may be recognized by host immune cells, e.g. Melanoma antigens (MAGEs)
and Tyrosinase
The presence of ‘microscopic satellites’ is also predictive of microscopic metastases to the regional lymph nodes.
An alternative prognostic scheme for clinical stages I and II melanoma, proposed by Clark, is based on the anatomic level of invasion in
the skin. (Clarks Level)
•• Level I is intraepidermal (in situ)
•• Level II penetrates the papillary dermis
•• Level III spans the papillary dermis
•• Level IV penetrates the reticular dermis
•• Level V penetrates into the subcutaneous fat
•• No induration
•• No role of radiotherapy
•• Not known usually before puberty
Adnexal Tumors
Adnexal Or Appendageal tumors arise from cutaneous appendages
They may serve as markers for internal malignancies
They are mostly in the form of solitary or multiple nodules/papules
•• Eccrine porcoma is seen on p alms and soles
•• Cylindroma or turban tumor is seen on forehead and scalp
•• Syringomas are seen in vicinity of lower eyelids
•• Trichoepitheliomas and trichilemmoma arise from hair follicles and are seen on face, scalp, neck and upper trunk
•• Sebaceous adenoma arises from sebaceous glands
Pyogenic granuloma
•• Total misnomer
•• Neither’ infectious’ nor ‘Granulomatous.’
•• Also called as lobular capillary hemangioma
•• Usually occurs in children, young adults
•• Called as Pregnancy (pregnancy tumor) or Granuloma Gravidarum
•• This lesion most commonly develops at a site of previous injury, and is best considered a variation of granulation tissue formation.
The name is a misnomer, since the lesion is neither related to bacterial infection nor a true granuloma. Microscopically, proliferating
blood vessels in an immature fibrous stroma are seen. The lesions can occasionally be confused with dysplastic nevi or malignant
melanoma, but are completely benign. Treatment is with surgical excision, curettage, or electrodesiccation.
Actinic Keratosis
•• Actinic keratosis (AK)
•• It is a UV light–induced lesion of the skin that may progress to invasive squamous cell carcinoma
•• Most common lesion with malignant potential to arise on the skin
•• Actinic keratoses arise on fair-skinned people in areas of long-term sun exposure, such as the face, ears, bald scalp, forearms,
and backs of the hands
•• They may occur on any area that is repeatedly exposed to the sun, such as the back, the chest, and the legs
•• Long-term UV light exposure is implicated as the cause from both epidemiologic observations and molecular analysis of tumor
cells
•• Clinically, actinic keratoses range from barely perceptible rough spots of skin to elevated, hyperkeratotic plaques several
centimeters in diameter.
Dermatology 387
Medical Eponyms
•• Turban tumor: •• Cylindroma
•• Marjolins ulcer: •• Squamous cell cancer
•• Molluscum sebaceum: •• Keratocanthoma
•• Cocks Peculiar tumor: •• Infected sebaceous cyst
•• Brooks tumor: •• Tumor of hair follicles
Dermatitis Herpetiformis
•• (DH) is an
–– Intensely pruritic
–– Papulovesicular skin disease characterized by lesions symmetrically distributed
–– Over extensor surfaces (i.e. elbows, knees, buttocks, back, scalp, and posterior neck)
–– Pruritus is prominent, patients may present with excoriations and crusted papules
•• Patients have an associated, usually subclinical, gluten-sensitive enteropathy
•• More than 90% express the HLA-B8/DRw3 and HLA-DQw2 haplotypes
•• Biopsy of early lesional skin reveals neutrophil-rich infiltrates within dermal papillae. Neutrophils, fibrin, edema, and
microvesicle formation at these sites are characteristic of early disease. Older lesions may demonstrate nonspecific features of a
subepidermal bulla or an excoriated papule
•• Direct immunofluorescence microscopy of normal-appearing perilesional skin demonstrate granular deposits of IgA (with or
without complement components) in the papillary dermis and along the epidermal basement membrane zone
•• Drug of choice Dapsone (D for D).
Acanthosis Nigricans
1. Acanthosis nigricans is a skin condition characterized by ‘dark, thick, velvety skin in body folds and creases’
2. Most often, acanthosis nigricans affects ‘armpits, groin and neck.’ Sometimes the lips, palms orsoles of the feet are affected as
well
3. ‘Slow progression’. The skin changes appear slowly, sometimes over months or years
4. Possible itching. ‘Rarely,’ the affected areas itch
5. Acanthosis nigricans is often associated with conditions that increase insulin level, such as type2 diabetes or being overweight.
If insulin level is too high, the extra insulin may trigger activity in skin cells. This may cause the characteristic skin changes
6. In some cases, acanthosis nigricans is inherited. Certain medications —such as ‘human growth hormone, oral contraceptives
and large doses of niacin’ — can contribute to the condition
Other hormone problems, endocrine disorders or tumors may play a role as well.
•• The most common cause of Acanthosis nigricans is Type 2 diabetes mellitus followed by obesity or endocrinopathies, such as
hypothyroidism or hyperthyroidism, acromegaly, polycystic ovary disease, insulin-resistant diabetes, or Cushing’s disease
•• Other causes of Acanthosis nigricans are familial, drug-induced, and idiopathic as mentioned.
In the context of a malignant disease, Acanthosis nigricans is a paraneoplastic syndrome and is then commonly referred to as
‘Acanthosis nigricans maligna’
Involvement of mucous membranes is rare and suggests a coexisting malignant condition
When seen in individuals older than age 40, this disorder is commonly associated with an internal malignancy, usually:
•• Adenocarcinoma of lungs
•• Cancers of GI tract
•• Uterine cancers
•• Cancers of prostate
•• Cancers of breast
•• Cancers of ovary
•• The skin of patients with HIV infection is often a ‘Target Organ’ for drug reactions
•• Although most skin reactions are mild and not necessarily an indication to discontinue therapy, patients may have particularly
severe cutaneous reactions, including Erythroderma and Stevens-Johnson syndrome, as a reaction to drugs, particularly sulfa
drugs, the nonnucleoside reverse transcriptase inhibitors, abacavir, and amprenavir
•• Similarly, patients with HIV infection are often quite ‘photosensitive’ and ‘burn easily’ following exposure to sunlight or as a side
effect of radiation therapy.
Lepromatous Leprosy
•• Lepromatous Leprosy at the more severe end of the leprosy spectrum is lepromatous disease, which encompasses the LL and BL
forms
•• The initial skin lesions of lepromatous leprosy are skin-colored or slightly erythematous papules or nodules. In time, individual
lesions grow in diameter up to 2 cm; new papules and nodules then appear and may coalesce
•• Patients later present with symmetrically distributed skin nodules, raised plaques, or diffuse dermal infiltration which, when on
the face, results in leonine facies
•• Late manifestations include loss of eyebrows eyelashes, pendulous earlobes, and dry scaling skin, particularly on the feet
•• Gynacomastia, madriosis, collapse of nasal bridge
•• Dermatopathology in lepromatous leprosy is confined to the dermis and particularly affects the dermal appendages
•• Histologically, the dermis characteristically contains highly vacuolated cells (foam cells) and fewer or absent noncaseating
granuloma
•• The dermis in lepromatous leprosy contains few lymphocytes and giant cells, and granulomas are absent
•• In LL leprosy, bacilli are numerous in the skin (as many as 109/g), where they are often found in large clumps (globi), and in
peripheral nerves, where they initially invade Schwann cells, resulting in foamy degenerative myelination and axonal degeneration
and later in Wallerian degeneration.
Lepra reactions
Type 1 reaction is also called as ‘Downgrading’ or ‘reversal’ reaction
•• It occurs in Borderline Leprosy
•• It is type ‘IV delayed Hypersensitivity reaction’
•• Signs of inflammation in previous lesions, Neuritis and fever are characteristic. (Ulnar nerve is most commonly affected)
Treatment:
•• Continue antileprotics
•• Use Analgesics
•• Corticosteroids are the ‘drug of choice’
•• Unresponsive to thalidomide
Type II reaction is also called as ‘Erythema Nodosum Leprosum‘
•• It occurs in Lepromatous Leprosy
•• It is type ‘III Hypersensitivity reaction’
Treatment:
•• Corticosteroids are the initially used drugs
•• Thalidomide is effective
•• ‘Follicular plugging ‘and ‘Telengiactasis’ are characteristic of DLE. (Discoid lupus erythematosus (DLE) is characterized by discrete
lesions, most often on the face, scalp, or external ears. The lesions are erythematous papules or plaques with a thick, adherent
scale that occludes hair follicles (follicular plugging). When the scale is removed, its underside will show small excrescences that
correlate with the openings of hair follicles and is termed a ‘carpet tack’ appearance)
•• Lesions occur in face, scalp and sun exposed areas of the body
•• Only 5% patients with DLE develop SLE
•• When SLE lesions are in the form of urticaria, bullae, erythema multiformae, Lichen planus, the condition called ‘Lupus Profondus.’
Erythroderma
Erythroderma is the term used when the majority of the skin surface is erythematous (red in color)
There may be associated scale, erosions, or pustules as well as shedding of the hair and nails. Potential systemic manifestations include
fever, chills, hypothermia, reactive lymphadenopathy, peripheral edema, hypoalbuminemia and high-output cardiac failure. The major
etiologies of erythroderma are:
1. Cutaneous diseases such as psoriasis and dermatitis
2. Drugs
3. Systemic diseases and
4. Idiopathic.
Alopecia
•• The two major forms of alopecia are scarring and nonscarring
•• In scarring alopecia there is associated fibrosis, inflammation, and loss of hair follicles
•• In nonscarring alopecia the hair shafts are gone, but the hair follicles are preserved, explaining the reversible nature of nonscarring
alopecia
•• The most common causes of nonscarring alopecia include:
–– Telogen effluvium,
–– Androgenetic alopecia,
–– Alopecia areata,
Dermatology 393
Pemphigoid Gestationis
•• Pemphigoid gestationis (PG), also known as herpes gestationis, is a rare, nonviral, subepidermal blistering disease of pregnancy
and the puerperium
•• PG may begin during any trimester of pregnancy or present shortly after delivery
•• Lesions are usually distributed over the abdomen, trunk, and extremities; mucous membrane lesions are rare
•• Skin lesions in these patients may be quite polymorphic and consist of erythematous urticarial papules and plaques, vesiculopapules,
and/or frank bullae
•• Lesions are almost always very pruritic. Severe exacerbations of PG frequently occur after delivery, typically within 24 to 48 h
•• PG tends to recur in subsequent pregnancies, often beginning earlier during such gestations. Occasionally, infants of affected
mothers demonstrate transient skin lesions.
Pyoderma Gangrenosum
The border of the ulcers has a characteristic appearance of an undermined necrotic bluish edge and a peripheral erythematous halo
Most commonly found on the lower extremities, they can arise anywhere on the surface of the body, including sites of trauma
(pathergy)
An estimated 30 to 50% of cases are idiopathic, and the most common associated disorders are:
•• Ulcerative colitis and Crohn’s disease
•• Chronic active hepatitis
•• Seropositive rheumatoid arthritis
•• Acute and chronic granulocytic leukemia
•• Polycythemia vera, and
•• Myeloma
Note
•• MYCOSIS CELL: •• T lymphocyte
•• LE CELL: •• Neutrophil
•• LEPRA CELL: •• Plasma cell
•• TZANCK CELL: •• Modified keratinocyte
•• VIRCHOW’S CELL: •• Leprosy
•• MAJOCCHI’S GRANULOMA: •• Tinea rubrum
•• PALISADING GRANULOMA: •• Granuloma annulare
•• GRANULOMA INGUINALE: •• Calymmatobacterium granulomatosis
Dermatology 395
Impetigo Contagiosa
A staphylococcal, streptococcal, or combined infection characterized by discrete, thin-walled vesicles that rapidly become pustular
and then rupture. Impetigo occurs more frequently on the exposed parts of the body-the face, hands, neck, and extremities-although
it may appear at sites of friction as well. Over 50% of cases are due to Staphylococcus aureus, with the remainder being due to
Streptococcus pyogenes or acombination of the two bacteria. Group B streptococci are associated with newborn impetigo. Impetigo
is most commonly seen in early childhood and during hot, humid summers in temperate climates.
Erysipelas
A superficial cellulitis of the skin with marked lymphatic involvement, caused by beta-hemolytic group A streptococci. Erysipelas
most commonly occurs in infants, very young children, and the elderly. In most cases, the organism gains access by direct inoculation
through a break in the skin, but infrequently, hematogenous infection may occur. It begins with an area of redness and enlarges to
form the typical tense, painful, bright red, shiny, brawny infiltrated plaque with a well demarcated, distinct, and slightly raised border
(as opposed to deepercellulitis, which has no distinct border and is flush with surrounding skin).
396 USMLE Step 2 CK Platinum Notes
Bartonella Henselae
It is the pathogen of cat-scratch disease, which is a very common cause of chronic lymphadenitis in children. It typically presents as
an enlarging nontender lymph node, often located in the cervical, axillary, or inguinal regions. After being scratched by a kitten, a papule
develops at the scratch site. Affected lymph nodes draining the involved area become enlarged in 2 weeks. Other symptoms include
low-grade fever, malaise, fatigue and nonspecific body aches. The diagnosis of cat-scratch disease is best made by a serological test, such
as an indirect fluorescence antibody test for antibodies to B henselae. Usually, treatment is necessary only in severe systemic infection.
Collodion baby: Babies are borne with shiny, transparent membrane which peels off after a week or so, the peel looking like collodion
Harlequin fetus: Child is borne encased in a thick, abnormal fissured hyperkeratotic skin, abnormality of keratin synthesis
Café au lait spots:
•• Smooth (Coast of California): Neourofibromatosis
•• Irregular (Coast of Maine): Mc Cune Albright Syndrome
Serum Sickness
It is a condition commonly caused by hypersensitivity to drugs. It is suggested that the drug acts as a hapten, which binds to plasma
proteins. This drug-protein complex is recognized as being foreign to the body and induces the serum sickness. Common signs and
symptoms of serum sickness include fever, cutaneous eruptions (morbilliform and/or urticarial), lymphadenopathy, and arthralgias.
Erythema multiforme may also appear in severe cases. With respect to cefaclor, the incidence of serum sickness is much higher infants
and children than in adults.
Dermatology 397
Erythema Multiforme
It is a reaction pattern that clinically manifests with ‘target’ lesions on the palms and soles in milder cases, whereas more severe forms
have mucosal blisters and a widespread vesicular/bullous skin eruption. ‘Target’ lesions are characteristic for erythema multiforme
and consist of a dusky red center with or without a central vesicle surrounded by an erythematous halo. Erythema multiforme is most
commonly seen after herpes simplex infection, streptococcal pharyngitis, or Mycoplasma pneumonia.
Tuberous sclerosis:
•• Ash leaf spot
•• Adenoma sebaceum
•• Shagreen patches
•• Infantile spasms
•• Delayed milestones
Nummular Dermatitis
It is a chronic inflammation of the skin, the etiology of which is still unknown. ‘coin-shaped’ or ‘discoid’ are seen. Microscopically, the
dominant feature is a localized spongiosis (corresponding to edema) of the epidermis, which may also contain minute fluid-filled
holes that correspond to the tiny vesicles seen clinically in early lesions. Treatment of these patients is problematic, and numerous
regimens involving corticosteroids or antibiotics have been recommended, each of which appears to work with some but not all
patients.
Diabetic Ulcers
A 76-year-old man has had an indolent, nonhealing ulcer at the heel of the right foot for several weeks. The patient began wearing
a new pair of shoes shortly before the ulcer started and noticed a blister as the first anomaly at the site where the ulcer eventually
developed. The ulcer is painless, 3.5 cm in diameter, the ulcer base looks dirty, and no granulation tissue. The skin around the
ulcer looks normal. The patient has no sensation to pin prick anywhere in that foot. Peripheral pulses are weak but palpable. He
is obese and has varicose veins, high cholesterol, and poorly controlled type 2 diabetes mellitus. It is an example of diabetic ulcer.
Diabetic ulcers typically develop at pressure points, and the heel is a favorite location. The patient has evidence of neuropathy, and the
correlation with the trauma inflicted by the new shoes is classic.
Hypomelanosis
The differential diagnosis of localized hypomelanosis includes the following primary cutaneous disorders:
•• Idiopathic guttate hypomelanosis
•• Postinflammatory hypopigmentation
•• Tinea (pityriasis) versicolor
•• Vitiligo
•• Chemical leukoderma
•• Nevus depigmentosus
•• Piebaldism
Disorders of Hyperpigmentation
•• Seborrheic keratoses are common lesions, but in one clinical setting they are a sign of systemic disease, and that setting is the
sudden appearance of multiple lesions, often with an inflammatory base and in association with acrochordons (skin tags) and
acanthosis nigricans. This is termed the sign of Leser-Trelat and signifies an internal malignancy
•• Acanthosis nigricans can also be a reflection of an internal malignancy, most commonly of the gastrointestinal tract, and it
appears as velvety hyperpigmentation, primarily in flexural areas. In the majority of patients, acanthosis nigricans is associated
with obesity, but it may be a reflection of an endocrinopathy such as acromegaly, Cushing’s syndrome, the Stein-Leventhal
syndrome, or insulin-resistant diabetes mellitus (type A, type B, and lipoatrophic forms).
Dermatology 399
•• A proliferation of melanocytes results in the following pigmented lesions: Lentigo, melanocytic nevus, and melanoma
•• LEOPARD Syndrome: Lentigines; ECG abnormalities, ocular hypertelorism; pulmonary stenosis and subaortic valvular stenosis;
abnormal genitalia (cryptorchidism, hypospadias); retardation of growth; and deafness (sensorineural) syndromes
•• The lentigines in patients with Peutz-Jeghers syndrome are located primarily around the nose and mouth, on the hands and feet,
and within the oral cavity. While the pigmented macules on the face may fade with age, the oral lesions persist
•• Lentigines are also seen in association with cardiac myxomas and have been described in two syndromes whose findings overlap
•• LAMB (lentigines, atrial myxomas, mucocutaneous myxomas, and blue nevi) syndrome and NAME [nevus, atrial myxoma,
myxoid neurofibroma and ephelides (freckles)] syndrome
•• The third type of localized hyperpigmentation is due to a local increase in pigment production, and it includes ephelides and
cafe au lait macules (CALM). The latter are most commonly associated with two disorders, neurofibromatosis (NF) and McCune-
Albright syndrome.
•• Localized hyperpigmentation is seen as a side effect of several other systemic medications, including those that produce fixed
drug reactions [phenolphthalein, nonsteroidal anti-inflammatory drugs (NSAIDs), sulfonamides, and barbiturates] and
those that can complex with melanin (antimalarials). Chloroquine and hydroxychloroquine
•• Estrogen in oral contraceptives can induce melasma, symmetric brown patches on the face, especially the cheeks, upper lip, and
forehead
•• The endocrinopathies that frequently have associated hyperpigmentation include Addison’s disease, Nelson syndrome, and
ectopic ACTH syndrome
•• The metabolic causes of hyperpigmentation include porphyria cutanea tarda (PCT), hemochromatosis, vitamin B12 deficiency,
folic acid deficiency, pellagra, malabsorption, and Whipple’s disease
•• Of the autoimmune diseases associated with diffuse hyperpigmentation, biliary cirrhosis and scleroderma (Calcinosis cutis)
•• Actual deposits of a particular drug or metal in the skin are seen with silver (argyria), where the skin appears blue-gray in color;
gold (chrysiasis), where the skin has a brown to blue-gray color.
NEWER CONCEPTS
Subcorneal Pustular Dermatosis
•• Subcorneal pustular dermatosis is a rare, benign, chronic, relapsing sterile pustular eruption involving the flexural aspect of
trunk and proximal extremities
•• Palmar, plantar and facial forms of disease are rare
•• It commonly affects females
•• It is a disease of elderly
•• IgA has been found to be associated with this condition
•• It is associated with:
–– Paraproteinemias
–– Inflammatory bowel diseases
–– SLE
–– Rheumatoid arthritis
–– Lymphoproliferative disorders
•• ‘Half- Half blister’ is the classic lesion in which purulent fluid accumulates in the lower half of the blister
•• Dapsone is most effective in the treatment of this disorder and the ‘treatment of choice’.
Reiter’s Syndrome
Mucocutaneous lesions: Small, painless superficial ulcers are commonly seen on the oral mucosa, tongue, and glans penis (balanitis
circinata).
Patients may also develop hyperkeratotic skin lesions of the palms and soles and around the nails (keratoderma blennorrhagica)
Cardiovascular involvement with aortitis, aortic insufficiency, and conduction defects occurs rarely.
PUVA Therapy
Methoxsalen is a psoralen derivative with photosensitizing activity. Exact mechanism of erythemogenic, melanogenic, and
cytotoxic response in the epidermis is unknown, involves increased tyrosinase activity in melanin-producing cells, as well as
inhibition of DNA synthesis, cell division, and epidermal turnover. Successful pigmentation requires the presence of functioning
melanocytes. Used in treatment of:
•• Mycosis fungoides: Photopheresis, using methoxsalen with ultraviolet radiation of white blood cells, is indicated for use with
the UVAR System in the palliative treatment of the skin manifestations of mycosis fungoides (also known as cutaneous T-cell
lymphoma) in persons who have not been responsive to other forms of treatment
•• Psoriasis: Indicated in the treatment of severe, refractory, disabling psoriasis that has not responded to other therapy
•• Vitiligo is indicated for repigmentation in the treatment of vitiligo
•• Lichen Planus
Drugs Causing:
Toxic epidermal necrolysis Photodermatitis Hyperpigmentation Steven Johnson Syndrome
Barbiturates Sulfonamides Phenolphthalein, (NSAIDs), Barbiturates
Phenytoin OCP Sulfonamides and Barbiturates Phenytoin
Phenylbutazone Tetracyclines Chloroquine Hydroxychloroquine Phenylbutazone
Pencillins Phenothiazines Pencillins
Sulfonamides Thiazides Sulfonamides
NSAIDS Furosemide
Allopurinol Griseofulvin
Sulfonylureas
Dermatological Signs
Frequently asked Dermatological signs
•• Asboe Hansen Sign (Bulla Spread Sign): Pemphigus
•• Carpet Tack sign: DLE
Dermatology 401
Steroids
Low potency steroids •• Hydrocortisone
•• Betamethasone valerate
•• Clobetasol 17 butyrate
Medium potency •• Flucinonide
•• Triamcinolone acetonide 1
High potency •• Betamethasone dipropionate
•• Halcinonide
•• Triamiclinolone acetonide
Highest potency •• Clobetasol 17 propionate
402 USMLE Step 2 CK Platinum Notes
Eponyms
•• ‘Micro Munro’ abscesses: Degenerated polymorphonuclear leucocytes in horny layer (stratum corneum) seen in Psoriasis
•• Spongiform Pustules of Kagoj: Pustules of flattened keratinocytes. Seen in Psoriasis
•• Pautrier Microabscesses: Atypical mononuclear cell collection in epidermis in Mycosis Fungoides
•• Grenz Zone: Area of uninvolved dermis between epidermis and dermis in inflammatory neoplastic conditions
•• Max Joseph space is a feature of Lichen Planus
Nails in Dermatology
Important Points about nails
•• Half and half nails: White proximally, pink distally. Seen in CRF
•• Terrys nails: White nails with pink tip, seen in cirrhosis
•• Blue nails: Chloroquine, Wilsons disease
•• Mees lines: Arsenic poisoning
•• Onchyolysis: Psoriasis
•• Koilonychia: Iron deficiency anemia
•• Yellow nail syndrome: Seen with pleural effusions, edema
•• Nail Patella Syndrome: Hypoplastic nails and patella.
•• Muehrcke’s nails: Associated with hypoalbumenia
•• Koenens periungal fibroma: tuberous sclerosis
Terry Nails
Have a nail bed that is white or light pink with a distal band measuring 0.5 to 3.0 mm that is pink to brown in color
•• This finding is associated with advancing age and a variety of medical conditions, including type 2 diabetes mellitus,
congestive heart failure, chronic renal failure, and cirrhosis
•• In young patients, this finding should prompt one to think of serious underlying pathology
•• In elderly patients, the finding is usually incidental.
Dermatology 403
Diascopy
Diascopy is designed to assess whether a skin lesion will blanch with pressure as, for example, in determining whether a red lesion is
hemorrhagic or simply blood-filled. A hemangioma will blanch with pressure, whereas a purpuric lesion caused by necrotizing
vasculitis will not
Diascopy is performed by pressing a microscope slide or magnifying lens against a specified lesion and noting the amount of blanching
that occurs. Granulomas often have an ‘apple jelly’ appearance on diascopy.
Wood’s Light
A Wood’s lamp generates 360-nm ultraviolet (or ‘black’) light that can be used to aid the evaluation of certain skin disorders
Wood’s lamp filter is made of nickel and silica
Wood’s lamp will cause erythrasma (a superficial, intertriginous infection caused by Corynebacterium minutissimum) to show a
characteristic coral red color
Wounds colonized by Pseudomonas to appear pale blue
Tinea capitis caused by certain dermatophytes such as Microsporum canis or M audouinii exhibits a yellow fluorescence
Pigmented lesions of the epidermis such as freckles are accentuated, dermal pigment such as postinflammatory hyperpigmentation
fades under a Wood’s light
Vitiligo appears totally white under a Wood’s lamp, and previously unsuspected areas of involvement often become apparent
A Wood’s lamp may also aid in the demonstration of tinea versicolor and in recognition of ash leaf spots in patients with tuberous
sclerosis.
•• Candidal vulvovaginitis
•• Candidal intertrigo
•• Diaper candidiasis
•• Perianal candidiasis
•• Candidal paronychia
•• Erosio interdigitalis blastomycetica
•• Chronic mucocutaneous candidiasis
•• Systemic candidiasis
Tuberculosis of Skin
Lupus vulgaris:
•• It is skin TB with no underlying active focus
•• MC type of cutaneous TB
•• Form of cutaneous TB common in children and young adults
•• Feature is indurated plaque, annular in shape
•• Heals with scarring Blanching with glass slide (diascopy) reveals grey or green foci (Apple jelly nodules)
Scrofulodema:
It is skin TB secondary to involvement of underlying structure, e.g. joint, lymph node
Tuberculosis Verrucosa cutis
TB bacillus here is inoculated into skin and seen in TB patient, Pathologists, Veterinary surgeons
It is a form of post primary tuberculosis with good resistance
Tuberculosis cutis orifacialis
TB of orifices as oral cavity, anal canal, urogenital tract. Other lesions noted are:
–– Acne agminata
–– Erythema induratum
–– Erythema nodosum
–– Rosea like lesions
•• Tuberculids is hypersensitivity reaction to Mycobacterium. Tuberculosis where evidence of etiology is not definite but shows
tubercular granuloma in histology and positive response to antitubercular treatment. Seen in
•• Lichen scrofulosorum
•• Erythema nodosum
Dermatology 405
Repeated NAMES
•• Candida albicans — Yeast like fungi
•• Histoplasma capsulatum — Darling’s disease
•• Aspergillus — Fungus ball disease
•• Coccoides immitis — Desert rheumatism
3. Pemphigus vulgaris
4. Lichen planus
5. Eryhthema multiforme
Ans. 2. Urticaria pigmentosa
3. Mycosis fungoides
4. Pityriasis lichenoides chronica
Ans. 3. Mycosis fungoides
2. Reiter’s syndrome
3. Behçet’s syndrome
4. Sarcoidosis
Ans. 2. Reiter’s syndrome
Fluids
•• Water constitutes between 50% and 70% of total body weight
•• The water of the body is divided into three functional compartments
•• The intracellular water represents between 30% and 40% of body weight
•• The extracellular water represents approximately 20% of body weight and is divided between intravascular fluid, or plasma (5%
of body weight) and interstitial, or extravascular, extracellular fluid (15% of body weight)
•• Intracellular fluid. Measurement of intracellular fluid (ICF) is determined indirectly by subtraction of the measured extracellular
fluid (ECF) from the measured total body water. The intracellular water is between 30% and 40% of body weight, with the
largest proportion in the skeletal muscle mass. Because of the smaller muscle mass in the female, the percentage of intracellular
water is lower than in the male. The chemical composition of ICF with potassium and magnesium as the principal cations, and
phosphates and proteins as the principal anions
•• Extracellular fluid. The total ECF volume represents approximately 20% of body weight. The ECF compartment has two major
subdivisions. The plasma volume is approximately 5% of body weight in the normal adult. The interstitial, or extravascular,
ECF volume, obtained by subtracting the plasma volume from the measured total ECF volume, accounts for approximately 15%
of body weight
•• The interstitial fluid is further complicated by having a rapidly equilibrating or functional component as well as several more
slowly equilibrating, or relatively nonfunctioning, components. The nonfunctioning components include connective tissue water
as well as transcellular water, which includes cerebrospinal and joint fluids. This nonfunctional component normally represents
only 10% of interstitial fluid volume (1%–2% of body weight) and is not to be confused with the relatively nonfunctional ECF, often
called a third space, found in burns and soft tissue injuries
•• The normal constituents of ECF are with sodium as the principal cation and chloride and bicarbonate as the principal anions.
Stork mark
•• It is a superficial capillary hemangioma which may be seen on the forehead, bridge of the nose and upper eyelids. The lesion is
often V-shaped pointing down to the nose and there is a corresponding mark on the nape of the neck
•• Capillary hemangiomas are bright red to blue and vary from a few millimeters to several centimeters in diameter; hemangiomas
can be level with the surface of the skin or slightly elevated and have an intact overlying epithelium.
Port-wine stain (Nevus flammeus)
•• Unlike a strawberry nevus, this is present at birth and may be very disfiguring, as it becomes darker and increasingly nodular with
age
•• Persists (P for P)
•• Port-wine stains are vascular malformations on the skin, and most commonly occur on the face. Over time, port-wine stains darken
and pose psychological stress to the affected person because of cosmetic reasons. Port-wine stains rarely disappear on their own,
and the most effective therapy is pulsed dye laser. Pulsed dye laser reduces the size of most port-wine stains, and in some cases,
can eradicate the stain completely. Factors affecting the response to pulsed dye laser include location, timing, and size. Smaller
port-wine stains respond better than the bigger ones. The earlier the treatment takes place, the better the response.
Sturge-Weber syndrome
•• It is a severe form of port-wine stain on the scalp and face, in the distribution of one of the branches of the trigeminal nerve,
associated with an underlying vascular anomaly of the arachnoid covering the cerebral hemisphere
•• This leads to epilepsy, hemiplegia and mental retardation.
Cavernous hemangioma
These may occur alone or in association with a capillary lesion in the overlying skin. They increase in size after birth but usually in
proportion to the growth of the infant. Most resolve spontaneously but some persist requiring excision
Grossly, cavernous hemangiomas appear as red-blue, soft, spongy masses 1 to 2 cm in diameter; rare giant forms can affect large
subcutaneous areas of the face, extremities, or other body regions. Histologically, the mass is sharply defined but not encapsulated,
and it is composed of large, cavernous blood-filled vascular spaces separated by a mild-to-moderate amount of connective tissue
stroma. Intravascular thrombosis with associated dystrophic calcification is common.
Kasabach-Merritt syndrome
Hemangiomas associated with a generalized bleeding disorder caused by the trapping of platelets within them which produces a
profound thrombocytopenia.
Lymphangioma
•• Lymphagioma are similar to hemangiomas but involve lymphatics. They may also occur anywhere in the body but, in particular,
they may present as a cystic hygroma
•• Most commonly arising in the cervical region.
Klippel-Trenaunay Syndrome
•• Congenital AV fistulas
•• Hemangiomas
•• Varicose veins
•• Hypertrophy of limbs
Aneurysms
•• The most common type of true aneurysm is fusiform type
•• The most common site of arterial aneurysm is Infrarenal part of abdominal aorta
•• Popliteal aneurysms are the most common peripheral aneurysms
•• The most common site for dissecting aneurysms is ascending aorta
•• MC cause of AAA is A: Abdominal Aortic Aneurysm is atherosclerosis
•• ‘Cirsoid aneurysms’ are common in superficial temporal artery
Types of Aneurysms
•• Berry aneurysm occurs in circle of willis
•• Microaneurysms are seen in diabetes and hypertension
•• Mycotic aneurysms are seen in bacterial infections
•• Aortic dissecting aneurysms are due to degeneration of tunica media. Occur in Marfan’s syndrome and hypertension
•• Syphilitic aneurysms or luetic aneurysms involve ascending aorta
•• Pseudoaneurysm follows trauma usually.
ARTERIAL DISEASES
Buerger’s Disease (Thromboangiitis Obliterans)
Thromboangiitis obliterans (Buerger disease) is a distinctive disease that often leads to vascular insufficiency; it is characterized by
segmental, thrombosing acute and chronic inflammation of medium-sized and small arteries, principally the tibial and radial arteries,
with occasional secondary extension into extremity veins and nerves. Buerger disease is a condition that occurs almost exclusively in
heavy smokers of cigarettes, usually beginning before age 35.
•• Affects small and medium-sized arteries
•• Affects young
•• Affects smokers
•• Ankle pressure index is low here
–– Coldness and numbness of toes are the first sign
–– Xanthiol nicotinate is used for treatment
426 USMLE Step 2 CK Platinum Notes
Raynaud’s disease: Affects arterioles. Features are Blanching, Dusky cyanosis, Red engorgement
Raynaud’s phenomenon is vasospastic condition of diverse etiology
Leriche’s syndrome is due to aortoiliac occlusion
Butcher’s thigh is due to injury to femoral artery in femoral triangle.
Signs of DVT
•• Hoffmann’s sign •• Forced dorsiflexion of ankle causes pain in calf
•• Moses’ sign •• Pain in calf on squeezing calf muscles
•• Pratt’s sign •• Lateral squeezing of calf muscles causes pain
•• Phlegmasia alba dolens •• Swollen leg (whitish) with edema and blanching
•• Phlegmasia cerulea dolens •• Painful (blue) leg
Surgery 427
Varicose Veins
•• These are dilated, tortuous superficial veins that result from defective structure and function of the valves of the saphenous veins, from
intrinsic weakness of the vein wall, from high intraluminal pressure, or, rarely, from arteriovenous fistulas
•• Primary varicose veins originate in the superficial system and occur two to three times as frequently in women as in men.
Approximately half of patients have a family history of varicose veins
•• Secondary varicose veins result from deep venous insufficiency and incompetent perforating veins or from deep venous
occlusion causing enlargement of superficial veins that are serving as collaterals
•• Varicose dilation renders the venous valves incompetent and leads to stasis, congestion, edema, pain, and thrombosis.
The most disabling sequelae include persistent edema in the extremity and secondary ischemic skin changes including stasis
dermatitis and ulcerations; poor wound healing and superimposed infections can become chronic varicose ulcers. Notably,
embolism from these superficial veins is very rare
•• Duplex imaging is gold standard
•• Varicose veins can usually be treated with conservative measures
–– External compression stockings provide a counterbalance to the hydrostatic pressure in the veins
–– Small symptomatic varicose veins can be treated with sclerotherapy, in which a sclerosing solution is injected into the
involved varicose vein and a compression bandage is applied
–– Ethanolamine oleate is used as sclerosant
–– Surgical therapy usually involves extensive ligation and stripping of the greater and lesser saphenous veins
–– Brodie Trendelenburg test demonstrates saphenofemoral incompetence
–– Ecchymosis is the most common complication of stripping
–– Cockett and Dodd’s operation is subfascial ligation operation.
Lymphatic Diseases
•• Lymphedema congenita •• Present at birth
•• Lymphedemapraecox •• Starts at puberty
•• Lymphedema tarda •• Starts at adulthood
•• Milroy’s disease •• Congenital lymphedema of familial type
A scar is loosely defined as an abnormal, disorganized collection of collagen following wound repair. The collagens are a large group of
triple-helix structural matrix proteins
•• Type I collagen is the major structural component of bones, skin, and tendons
•• Type II is found predominantly in cartilage
•• Type III is found in association with Type I, although the ratio varies in different tissues
•• Type IV is found in basement membranes in association with mucopolysaccharides and laminin
•• Type V is found in the cornea in association with Type III and is important in maintaining transparency.
428 USMLE Step 2 CK Platinum Notes
Keloids
•• Keloids are made of dense connective tissue
•• Keloids are best treated by intrakeloidal injection of triamiclonolone
Wounds
Clean wound •• No violation of mucosa
•• No inflammation
•• No drains
•• No need of any prophylactic antibiotic, e.g. Varicose Vein Surgery
Elective Herniotomy
•• Violation of mucosa but no spillage
Clean contaminated wound •• Need of prophylactic antibiotic, e.g. Elective cholecystectomy
Surgery 429
Grafting-related Terms
•• Imbibition: refers to absorption of nutrients into the graft
•• Inosculation: donor and recipient capillaries become aligned
•• Revascularization is completed by connecting vessels into arterioles and venules.
Skin grafts
•• Skin grafts are harvested from a donor site and transferred to a recipient site on which they must survive, a process known as
take
•• All skin grafts initially adhere to the recipient bed by the formation of fibrin. Oxygen and nutrients diffuse by a process known as
plasmatic imbibition to keep the graft alive
•• New blood vessels then grow from the recipient site and link up with dermal capillaries to re-establish a blood supply, a process
known as inosculation
•• Thin skin grafts are more likely to survive by imbibition and will revascularize readily and are, therefore, more likely to take than
thicker grafts.
Partial-thickness skin grafts (Thiersch graft)
•• Consist of epidermis and a variable thickness of dermis
•• Partial-thickness grafts are used to resurface relatively large areas of skin defect and are particularly useful in burns
Full-thickness grafts (Wolfe’s graft)
•• Consist of epidermis and all of the dermis
•• The donor site will not epithelialize and must be closed, usually directly. Full-thickness grafts are most commonly used in repairing
defects on the face.
Composite grafts
•• Consist of skin and some underlying tissue such as fat and cartilage
•• Again, donor sites must be closed directly. Composite grafts carry the highest risk of failure
Biologic Dressings
Viable cutaneous allograft is the biologic dressing of choice, against which all other available materials must be evaluated. In
the operating room using sterile technique, allograft skin is harvested from cadavers free of jaundice, cutaneous malignancy or
infection, and viral disease. The harvested grafts are spread on fine-mesh gauze that is thinly impregnated with petrolatum, placed in
sterile containers, and then refrigerated for up to 2 weeks. Alternatively, the tissue can be frozen using cryoprotective techniques. If
refrigerated, such tissue performs better as a biologic dressing the sooner it is used after harvest.
Cutaneous Allograft
•• Prevents wound desiccation
•• Promotes maturation of granulation tissue
•• Limits bacterial proliferation in the burn wound
•• Prevents exudative protein and red blood cell loss
•• Decreases wound pain, thereby facilitating movement of involved joints
•• Diminishes evaporative water loss from the burn wound surface, thus decreasing heat loss; and serves to protect tendons, vessels,
and nerves.
430 USMLE Step 2 CK Platinum Notes
Cutaneous Xenografts
•• These are less effective as physiologic dressings and allow survival of greater numbers of subgraft bacteria, presumably because such
tissue is not vascularized by the host
•• Such tissues are not rejected in the true sense of the word but slough following necrosis.
Amnion
•• It is a physiologic dressing that is readily available and inexpensive
•• Since the amniotic tissue will desiccate and spontaneously separate from the wound bed if left exposed, it must be covered with
occlusive dressings that preclude continuous observation of the dressing and underlying wound bed
•• Amnion, like cutaneous xenografts, is not vascularized by the host, and biologic union occurs by ingrowth of granulation tissue.
Swellings of Skin
•• Lipoma •• Universal tumor
•• Adiposis dolorosa (Dercum’s disease) •• Multiple lipomata
•• Cavernous hemangioma •• Arises from veins
•• Plexiform hemangioma •• Arises from arteries
•• Warts •• Human papilloma virus
•• Rhinophyma (Potato nose) •• Acne rosacea
•• Cavernous lymphangioma •• Cystic hygroma
•• Sebaceous cyst •• Wen
•• Buschke-Lownstein tumor •• Massive veneral wart
•• Marjolin’s growth •• Squamous cell carcinoma
•• Bedsore •• Trophic ulcer
•• Pilomatrixoma •• Calcifying epithelioma
•• Pott’s puffy tumor •• Skull osteomyelitis
•• Raspberry tumor •• Umbilical adenoma
Sebaceous Cyst
•• It is also called Wen
•• It usually lies in dermis
•• It is a retention cyst
•• Its contents are sebum and keratin
•• Common sites are face and scalp
•• They don’t occur on palm and scalp
•• Cock’s peculiar tumor is a complication
Surgery 431
Lipoma
•• Most common site: Trunk
•• ‘Slipping margin’ sign positive
•• Pedunculated lipoma is called: lipoma arborscens
•• Mc type: Subcutaneous
•• Painful: Neurofibroma
•• Vascular: Naevo lipoma
Dercum’s disease
–– Adiposis dolorosa
–– Fatty deposits on limbs/trunk with facial sparing
–– Rare disease
–– Multiple painful lipomas in adult life
Ulcers of Skin
Ulcers of skin
•• Rodent ulcer: Basal cell carcinoma
•• Martorell’s ulcer: Hypertensive/Atherosclerotic ulcer
•• Bazin’s ulcer: Erythrocyanoid ulcer
•• Snail-track ulcer: Syphilitic ulcer
•• The squamous dyplasia is broad and occupies the full thickness of the epithelium
•• Invasive squamous cell carcinomas exhibit variable differentiation, ranging from tumors formed by atypical squamous
cells arranged in orderly lobules showing large zones of keratinization to neoplasms formed by highly anaplastic, rounded
cells with foci of necrosis and only abortive, single-cell keratinization (dyskeratosis)
•• While morphologic variation is wide, all squamous cell carcinomas share the feature of keratinization
Unlike BCC, which has a very low metastatic potential, SCC can metastasize and grow rapidly. The clinical features of SCC vary widely
–– Actinic keratosis predisposes to SCC
–– Bowen’s disease
–– Lichen planus
–– DLE
Marjolin’s Ulcer
•• Squamous cell carcinoma in scar/burn/long-standing ulcer
•• Slow-growing
•• Less malignant
•• Painless without lymphatic spread
•• Wide excision is the treatment of choice.
Keratoacanthoma
Keratoacanthoma typically appears as a dome-shaped papule with a central keratotic crater, expands rapidly, and commonly
regresses without therapy
This lesion can be difficult to differentiate from SCC
•• Benign
•• Self-limiting course
•• Affects elderly men usually
Melanomas
Melanomas originate from melanocytes, pigment cells normally present in the epidermis and sometimes in the dermis.
An alternative prognostic scheme for clinical stages I and II melanoma, proposed by Clark, is based on the anatomic level of invasion in
the skin (Clark’s level).
•• Melanomas may spread by the lymphatic channels or the bloodstream
•• The earliest metastases are often to regional lymph nodes
•• Most common site for malignant melanoma is skin (90%)
•• Cutaneous melanoma arises from epidermal melanocytes
•• Junctional melanoma predisposes to MM
•• It may be familial.
Melanomas can develop either de novo or in an existing mole. Sunlight exposure is a significant risk factor and fair-skinned persons are
at an increased risk of developing melanoma. The most significant factor for long-term prognosis is the depth of the lesion, since the
superficial dermis lies about 1 mm under the skin surface, and penetration to this depth is associated with a much higher incidence of
metastasis than is seen with a more superficial location.
Early detection of melanoma may be facilitated by applying the ‘ABCD rules’:
•• A–asymmetry, benign lesions are usually symmetric
•• B–border irregularity, most nevi have clear-cut borders
•• C–color variegation, benign lesions usually have uniform light or dark pigment
•• D–diameter > 6 mm (the size of a pencil eraser).
Surgery 433
•• ‘Clarks level’ is the classification based on the level of ‘invasion into skin’
•• ‘Breslow’s staging’ is based on thickness of invasion
Nos of MM (Malignant Melanoma)
•• No incisional biopsy
•• No induration
•• No role of radiotherapy
•• Not known usually before puberty
SALIVARY GLANDS
Pleomorphic Adenoma
•• Mc benign salivary gland tumor
•• Most common site is superficial lobe
•• Malignant transformation is uncommon
•• Treatment is by superficial parotidectomy (Patey’s operation)
434 USMLE Step 2 CK Platinum Notes
Warthin’s Tumor
•• Second most common benign tumor of parotid gland
•• Consists of epithelial+lymphoid elements (adenolymphoma)
•• Common in males in 5th-7th decade
•• Bilateral in about 10% cases
•• Encapsulated
•• Shows Hot Spot in Tc-Pertechnetate scan
•• Superficial parotidectomy is the treatment.
Mucoepidermoid Cancer
•• Mc malignant salivary gland tumor
•• Arises from mucinsecreting cells and epidermal cells
•• Mc malignant tumor of parotid
•• Mc radiation-induced salivary tumor
Remember
•• Most tumors of major salivary glands are benign
•• Pleomorphic adenoma is the most common benign salivary gland tumor
•• Most of the minor salivary gland tumors are malignant
•• Most common site of minor salivary gland tumor is oral cavity
•• Most of salivary gland tumors are radioresistant
•• Godwin’s tumor is benign lymphoepithelial tumor of parotid
•• Calculi are most common in submandibular duct
•• Most common organism as a cause of acute parotitis is staph aureus
•• Most common cause of acute parotitis is mumps
•• Most of submandibular calculi are radiopaque
•• Sialolithiasis refers to stones in salivary ducts
•• Sialadenosis is noninflammatory swelling affecting salivary glands
•• Frey’s syndrome (gustatory sweating) occurs after parotid surgery due to involvement of auriculotemporal nerve
•• Sialography is not done in acute infection of salivary glands.
Breast
•• Modified sweat gland
•• ‘Axillary tail of Spence’ is prolongation of breast tissue into axilla
•• ‘Foramen of LANGER’ is the space through which axillary tail passes
•• ‘Ligaments of cooper attach breast to superficial fascia
•• ‘Mammary ridge’ is the embryonic precursor
•• ‘Subareolar plexus of Sappy’ is a lymphatic plexus
•• Peau d’orange is lymphatic permeation
•• Blood supply: Internal mammary artery, lateral thoracic, superior thoracic and acromiothoracic arteries
Surgery 435
Clinical Scesnarios
•• A 16-year-old female has firm, rubbery mass in right breast moving with palpation Fibroadenoma
•• A 16-year-old female has firm, rubbery mass in right breast is 8 cms in diameter Giant fibroadenoma
•• A 30-year-old female has history of bilateral breast tenderness related to menstrual cycle with lumps Fibrocystic disease
coming and going
•• A 30-year-old female has bloody discharge from nipple. No other palpable masses are seen Intraductal papilloma
•• A 30-year-old lactating mother has red, hot, tender mass with fever and leukocytosis Breast abscess
•• A 55-year-old woman has 3.5 cms hard mass in her left breast with ill-defined borders and not mobile. Breast ca
The skin overlying has orange peel appearance
•• A 60-year-old has headaches not responding to medications. She had undergone modified radical Brain metastasis from
mastectomy 1 year back breast ca
Galactocele
•• It is a milk-filled cyst
•• Round, well-circumscribed, and easily movable within the breast
•• Usually occurs after the cessation of lactation or when feeding frequency has been curtailed significantly
•• Inspissated milk within a large lactiferous duct is responsible
•• The tumor is usually located in the central portion of the breast or under the nipple.
436 USMLE Step 2 CK Platinum Notes
•• Needle aspiration produces thick, creamy material that may be tinged dark-green or brown. Although it appears purulent, the
fluid is sterile
•• The treatment is needle aspiration. Withdrawal of thick milky secretion confirms the diagnosis
•• Operation is reserved for those cysts that cannot be aspirated or that become superinfected.
Phyllodes Tumor
•• Serocystic disease of Brodie or Cystosarcoma Phyllodes
•• Name is misnomer as it is rarely cystic or rarely develops into sarcoma
•• Phyllodes: Leaf. It is a benign breast tumor. It is a stromal breast tumor
•• They present as a large, massive tumor with uneven bosselated surface
•• They are however mobile and resemble fibroadenoma
•• They account for less than 1% of all breast neoplasms
•• This is predominantly a tumor of adult women, with very few examples reported in adolescents
•• Patients typically present with a firm, palpable mass
•• These tumors are very fast growing, and can increase in size in just a few weeks
•• Occurrence is most common between the ages of 40 and 50, prior to menopause
•• The common treatment for phyllodes is wide local excision. Other than surgery, there is no cure for phyllodes, as chemotherapy
and radiation therapy are not effective.
Risk Factors
•• Conditions with hyperestrogenic and hypoandrogenic states
•• Cirrhosis of liver
•• Undescended testis
•• Mumps orchitis
•• Klinefelter’s syndrome
•• May accompany gynecomastia
•• Most common variety is infiltrating ductal cancer
•• Presents with unilateral lump with majority being Estrogen receptor positive.
Aortic Injury
•• Widening of superior mediastinum > 8 cms is considered a reliable sign of Aortic injury
•• ‘PA view’ is better than AP view in this type of injury
•• X-ray is used as a ‘screening tool’ here
•• ‘Acceleration or deceleration’ injuries are associated with aortic rupture.
Other Signs
•• Obscured aortic knob
•• Widened mediastinum
•• Deviation of left main stem bronchus
•• Obliteration of aorticopulmonary window
•• Deviation of nasogastric tube
•• Left apical cap
•• Opacification of aortopulmonary window
•• Left pulmonary Hilar hematoma
438 USMLE Step 2 CK Platinum Notes
Blast Injury
•• Rupture of tympanic membrane, dislocation of ear ossicles
•• Injury to lungs
•• Perforation of stomach, gut
•• Conjuctival hemorrage in eyes
NECK
Dermoid Cysts
•• These usually occur at sites of embryological fusion
•• These may be in the midline
•• A dermoid cyst in the neck may be mistaken for a thyroglossal cyst, although it will not move on swallowing or protrusion of
the tongue
•• A common site is the external angular dermoid cyst in the eyebrow area at the outer angle of the eye. Occasionally, there may
be a dumbbell extension intracranially
•• They occur if ectodermal cells become buried beneath the skin surface during development
•• An inclusion dermoid cyst may similarly arise secondary to trauma.
Cystic Hygroma
•• Commonly arising in the neck, these fluid-filled lesions of lymphatic origin, lymphangioma
•• MC site: Posterior triangle of neck
•• Seen in Turner’s syndrome, trisomies, fetal alcohol syndrome, multiple pterygium syndrome
•• Brilliantly translucent, positive cough impulse
•• They are either present at birth, sometimes being diagnosed on antenatal ultrasonography, or may appear within the first 2 years
or sometimes later
•• Infection leads to difficulty with subsequent surgery, which is thus best performed soon after diagnosis.
Surgery 439
Sternomastoid Tumor
•• It is due to infarction of sternomastoid muscle
•• Usually during delivery
•• Infarcted muscle replaced by fibrosis
•• Present immediately after birth as a ‘swelling’ with the newborn keeping his/her neck to one side.
Branchial Cyst
•• Remanant of second branchial cleft
•• Mc site is upper part of neck at junction of upper anterior border of sternomastoid
•• Wall is made of lymphoid tissue
•• ‘NON’-translucent
•• Complete excision is treatment of choice.
Chemodectoma
•• ‘Potato tumor’ or carotid body tumor
•• ‘Benign tumor’ but rarely metastasizes (unusual combination)
•• Arises from ‘chemoreceptor zone’
•• Nonchromaffin paraganglioma
•• Origin from Schwann cells: Association with high altitude. Family history and pheochromocytoma
•• Biopsy is contraindicated
•• Usually unilateral
•• Firm, rubbery, mobile from side to side, slow-growing
•• Bruit may be present
•• Angiogram is diagnostic
•• Good prognosis, rarely metastasizes
•• Surgical excision is ‘treatment of choice’
•• Branchial Cyst
–– Arises from 2nd branchial cleft
–– Located usually at anterior border of upper 1/3 of sternomastoid
–– Contains cholesterol crystals
•• Branchial Fistula
–– Represents persisting in 2nd branchial cleft
–– Located usually at anterior border of lower 1/3 of sternomastoid
440 USMLE Step 2 CK Platinum Notes
•• Cystic Hygroma
–– Represents lymphatic venous anastomotic failure
–– Located usually in the neck lower in posterior triangle
–– Brilliantly translucent
•• Cervical Rib
–– Mostly unilateral
–– Mostly on right side
Thyroglossal Cyst
•• This may be present in any part of the thyroglossal tract
•• MC site is beneath the hyoid, in the region of the thyroid cartilage, and above the hyoid bone
•• Occupies the midline, except in the region of the thyroid cartilage, where the thyroglossal tract is pushed to one side, usually to
the left.
Surgery 441
•• The swelling moves upwards on protrusion of the tongue as well as on swallowing because of the attachment of the tract to
the foramen cecum
Never to be forgotten:
–– Subhyoid
–– Painless
–– Midline
–– Nontransluminant but fluctuant swelling
A thyroglossal cyst should be excised (Sistrunk’s operation) because infection is inevitable, owing to the fact that the wall contains
nodules of lymphatic tissue which communicate by lymphatics with the lymph nodes of the neck
Thyroglossal Fistula
•• Thyroglossal fistula is never congenital
•• It follows infection or inadequate removal of a thyroglossal cyst
•• The cutaneous opening of such a fistula is drawn upwards on protrusion of the tongue
•• A thyroglossal fistula is lined by columnar epithelium, discharges mucus, and is the seat of recurrent attacks of inflammation.
(Sistrunk’s Operation)
Because the thyroglossal tract is so closely related to the body of the hyoid bone, this central part must be excised, together with the
cyst or fistula, or recurrence is certain. When the thyroglossal tract can be traced upwards towards the foramen cecum, it must be
excised with the central section of the body of the hyoid bone, and a central core of lingual muscle
Riedel’s Thyroiditis
•• Thyroid tissue is replaced by cellular fibrous tissue which infiltrates through the capsule into adjacent muscles, paratracheal
connective tissue and the carotid sheaths
•• It may occur in association with retroperitoneal and mediastinal fibrosis
•• The goiter may be unilateral or bilateral and is very hard and fixed
•• The differential diagnosis from anaplastic carcinoma can only be made with certainty by biopsy, when a wedge of the isthmus
should also be removed to free the trachea
•• If unilateral, the other lobe is usually involved later and subsequent hypothyroidism is common.
442 USMLE Step 2 CK Platinum Notes
MEN Syndromes
The multiple endocrine neoplasia (MEN) syndromes are distinct genetic entities that are expressed in several specific patterns of
involvement.
MEN 1 is characterized by the concurrence of:
•• Parathyroid hyperplasia
•• Pancreatic islet cell neoplasms
•• Adenomas of the anterior pituitary gland
MEN 2A is characterized by the concurrence of:
•• Medullary thyroid carcinoma (MTC)
•• Pheochromocytomas, and
•• Parathyroid hyperplasia
MEN 2B consists of:
•• MTC
•• Pheochromocytoma
•• Mucosal neuromas, and distinctive marfanoid habitus
These syndromes may occur de novo (especially MEN 2B) and are transmitted as ‘autosomal dominant’ traits.
•• The most important product of MTC cells is calcitonin, which is a sensitive tumor marker for the presence of MTC whether in
preoperative screening or postoperative evaluation
•• Patients with clinically evident MTC most commonly present with a palpable thyroid nodule or multinodular thyroid gland.
Enlarged, firm cervical lymph nodes suggest metastatic disease. Patients with locally advanced disease may present with
hoarseness, dysphagia, respiratory difficulty, or signs of distant metastases (e.g. lung, liver and bone).
Papillary Carcinoma
•• Mc thyroid cancer
•• Encapsulated tumor
•• Psammoma bodies seen
•• It can be diagnosed by FNAC (contrast follicular type)
•• Usually multifocal.
Thyroid Lymphoma
•• Seen in Hashimoto’s disease
•• Diffuse large type is a variant
•• Radiosensitive
•• Rapidly growing
•• Radiotherapy + chemotherapy is used for treatment
Complications of Thyroidectomy
1. Hemorrhage
2. Recurrent laryngeal nerve palsy
3. Superior laryngeal nerve palsy
4. Hypoparathyroidism
5. Hypothyroidism
6. Tracheal collapse
7. Thyroid crisis
•• Hypocalcemia does not occur in hemithyroidectomy
•• Neck swelling immediately after thyroid operation is usually tension hematoma
•• In such cases: open immediately.
444 USMLE Step 2 CK Platinum Notes
Masses in Mediastinum
Anterior Mediastinal Masses Middle Mediastinal Masses Posterior Mediastinal Masses
•• Thymoma •• Bronchogenic cysts •• Neurogenic tumors
•• Lymphoma •• Pleura pericardial cysts •• Lymphoma
•• Teratoma •• Aneurysms •• Metastatic germ cell tumor
•• Thyroid/parathyroid masses •• Myxoma
•• Lipoma •• Esophageal lesions
•• Hiatal hernia •• Lipoma
•• Retrosternal goiter
Mediastinal Emphysema
•• Air may enter the mediastinum from the esophagus, trachea, bronchi, lung, neck, or abdomen, producing mediastinal
emphysema or pneumomediastinum
•• Can occur from blunt or penetrating trauma, intraluminal injury, such as during endoscopy, as well as barotrauma. Positive-
pressure ventilation
•• Blunt trauma due to compressive forces on the thorax, especially when the glottis is closed and ventilation is being achieved
with high pressures (usually in the setting of decreased lung compliance), may cause sufficient pressures at the intra-alveolar
region to rupture alveoli
•• Dissection of air along vascular structures into the hilum and mediastinum creates a pneumomediastinum
•• Mediastinal emphysema may also be caused by intra-abdominal air dissecting through the diaphragmatic hiatus
•• Spontaneous pneumomediastinum is usually seen in patients with exacerbation of bronchospastic disease
•• The clinical manifestations of this include substernal chest pain, which may radiate into the back and crepitation in the region of
the suprasternal notch, chest wall and neck
•• With increasing pressure, the air can dissect into the neck, face, chest, arms, abdomen, and retroperitoneum
•• Frequently, pneumomediastinum and pneumothorax ‘occur simultaneously’
•• Auscultation over the pericardium demonstrates a characteristic crunching sound that is accentuated during systole and is termed as
‘Hamman’s sign.’
Mediastinitis
•• Infection of the mediastinal space
•• It is a serious and potentially fatal process
Surgery 445
•• These factors responsible include perforation of the esophagus due to instrumentation, foreign bodies, penetrating or, more rarely,
blunt trauma, spontaneous esophageal disruption (Boerhaave’s syndrome), leakage from an esophageal anastomosis, tracheobronchial
perforation, and mediastinal extension from an infectious process originating in the pulmonary parenchyma, pleura, chest wall,
vertebrae, great vessels, or neck
•• Mediastinitis is manifested clinically by fever, tachycardia, leukocytosis, and pain that may be localized to the chest, back, or neck,
although, in some patients, the clinical course remains indolent for long periods.
Mediastinal Hemorrhage
Mediastinal hemorrhage is most frequently caused by blunt or penetrating trauma, thoracic aortic dissection, rupture of aortic
aneurysm, or surgical procedures within the thorax
Spontaneous mediastinal hemorrhage is a recognized entity with predisposing factors related to the following:
(1) complication of a mediastinal mass, of which thymoma, malignant germ cell tumor, parathyroid adenoma, retrosternal
thyroid, and teratoma are the most common;
(2) sudden sustained hypertension;
(3) altered hemostasis due to anticoagulant therapy, thrombolytic therapy, uremia, hepatic insufficiency, or hemophilia; and
(4) transient, sharp increases in intrathoracic pressure, which occur during coughing or vomiting, an entity initially described by
Retrosternal pain radiating to the back or neck is common
‘Compression of mediastinal structures’ (primarily the great veins) develop, including dyspnea, venous distention, cyanosis, and
cervical ecchymosis due to blood dissecting into soft tissue planes
Sufficient accumulation of blood causes mediastinal tamponade manifested by tachycardia, hypotension, reduced urinary output,
equalization of right- and left-sided cardiac filling pressures, and diastolic collapse of the right ventricle
The development of mediastinal tamponade is more insidious than pericardial tamponade because of the larger volume of the mediastinum.
Cardiac Tumors
Primary cardiac tumors are rare and usually require an intensive work-up to pinpoint the diagnosis. Seventy-five percent of primary
cardiac tumors are benign and among these, myxoma is the most common. The tumors are usually single; the most common location is
the left atrium. They may cause syncopal episodes or even shock and death due to obstruction by a ‘ball valve’ mechanism.
Esophagus
•• 25 cms long
•• Upper 1/3: Striated, middle 1/3 mixed. Lower 1/3: smooth muscle
•• Constrictions: 15 cms, 25 cms, 40 cms from incisor teeth
•• Most common site of impaction is below cricopharyngeus
•• No submucosal plexus, only Auerbach’s plexus present
•• No serosa present
•• Three parts: Cervical, thoracic, abdominal
•• Arterial supply: Inferior thyroid artery, branches of aorta and left gastric artery respectively
Esophagitis
Esophagitis
Most common causes are:
•• Reflux esophagitis (mc)
•• Infectious esophagitis
•• Eosinophilic esophagitis
•• Drug-induced esophagitis
•• Radiation esophagitis
•• Corrosive esophagitis
Monilial Esophagitis
•• Esophagitis due to Candida albicans is relatively common in patients taking steroids, including steroid inhalers for asthma
•• It may present with dysphagia or odynophagia
•• There may be visible thrush in the throat
•• Endoscopy shows numerous white plaques that cannot be moved, unlike food residues
•• Biopsies are diagnostic. In severe cases, a barium swallow may show dramatic mucosal ulceration and irregularity that is
surprisingly similar to the appearance of esophageal varices
Achalasia Cardia
•• Esophegal motility disorder
•• Increased difficulty in swallowing for both liquids/solids
•• Failure of relaxation of lower esophageal, increased pressure
•• Dysphagia is the presenting symptom
•• Loss of ganglion cells in Auerbach’s plexus
•• Bird-beak appearance
•• Cucumber esophagus
•• Heller’s myotomy is treatment of choice
•• Pseudoachalasia: Caused by adenocarcinoma of cardia, bronchogenic ca, pancreatic ca.
Dysphagia Lusoria
It is dysphagia secondary to vascular anomalies:
•• Double aortic arch
•• Right aortic arch
•• Aberrant right subclavian artery
•• Abnormal innominate artery
•• So diagnosis is by arteriography
Zenker’s Diverticulum
•• Mc esophegeal diverticulum
•• In elderly age group usually
•• Weakness in the posterior hypopharyngeal wall (Killian’s triangle)
Surgery 449
•• Pulsion diverticulum
•• Intermittent dysphagia, regurgitation, gurgling sound in the neck
•• Causes halitosis, regurgitation of saliva and food, dysphagia or complete obstruction. Symptomatic Zenker’s diverticula are treated
by cricopharyngeal myotomy with or without diverticulectomy
•• Very large symptomatic esophageal diverticula are removed surgically by:
–– Excision of pouch or cricopharyngeal myotomy
–– Dohlman’s procedure
Esophageal Hernias
Type I
•• ‘Sliding’ hernia
•• It is dislocation of cardia of stomach into posterior mediastinum and presents with GERD
•• MC type
•• Lesser complications occur
•• Treatment is generally medical for uncomplicated cases
•• Treatment is surgical for all complicated cases
Type II
•• ‘Paraesophegal’ hernia
•• Cardia is normal but gastric fundus, colon or small gut is dislocated
•• Cardiorespiratory arrest, dysphagia are seen
•• Complications are more common and severe
•• Treatment is surgical even for uncomplicated cases
Barrett’s Esophagus
•• Metaplasia seen
•• Metaplasia of ‘squamous epithelium’ to columnar in esophagus at lower end
•• Because of prolonged GERD
•• Intestinal type metaplasia is the commonest
•• Progression to adenocarcinoma occurs
Complications:
–– Barrett’s ulcer
–– Stricture
–– Perforation
–– Progression to adenocarcinoma occurs
–– Diagnosis suggested by endoscopy
–– Diagnosis confirmed by biopsy
Barrett’s esophagus may occur in a small number of patients who have gastroesophageal reflux disease (GERD). This condition
is a metaplasia of the normal squamous mucosa of the esophagus to a columnar (glandular) type of epithelium, and is usually
seen as a response to repeated acid exposure to the distal esophagus. Tobacco and alcohol use are also thought to contribute
to the process. The significance of Barrett’s esophagus is that it may lead to the development of low-grade dysplasia, high-
grade dysplasia, or esophageal adenocarcinoma.
Esophageal Diverticula
These are epithelial-lined mucosal pouches that protrude from the esophageal lumen. Almost all of them are acquired and
occur predominantly in adults.
450 USMLE Step 2 CK Platinum Notes
•• A true diverticulum: Contains all layers of the normal esophageal wall, including mucosa, submucosa, and muscle, whereas
a false diverticulum consists primarily of only mucosa and submucosa
•• Pulsion diverticula arise because elevated intraluminal pressure forces the mucosa and submucosa to herniate through
the esophageal musculature; therefore, they are false diverticula.
Traction diverticula are the result of external inflammatory reaction in adjacent mediastinal lymph nodes that adhere to the
esophagus and pull the entire wall toward them as they heal and contract; they are true diverticula.
Pharyngoesophageal Diverticula
•• The pharyngoesophageal diverticulum is the most common esophageal diverticulum, generally occurring in patients
between 30 and 50 years of age and, therefore, believed to be acquired
•• The diverticulum characteristically arises within the inferior pharyngeal constrictor, between the oblique fibers of the
thyropharyngeus muscle and the more horizontal fibers of the cricopharyngeus muscle, the upper esophageal sphincter
(UES).
•• The transition in the direction of these muscle fibers (Killian’s triangle) represents a point of potential weakness in the
posterior pharynx and is the site of formation of the pharyngoesophageal diverticulum
•• Pharyngoesophageal diverticula are usually associated with complaints of cervical dysphagia, effortless regurgitation of
undigested particles of food or pills sometimes consumed hours earlier, a gurgling sensation in the neck on swallowing,
choking, and recurrent aspiration
•• A barium esophagogram establishes the diagnosis
•• Surgical therapy in symptomatic patients is indicated in most cases, regardless of the size of the pouch and, it is hoped,
before complications occur.
Esophageal Varices
•• Important site for communication between the intra-abdominal splanchnic circulation and the systemic venous circulation is
through the esophagus
•• The increased pressure in the esophageal plexus produces dilated tortuous vessels called varices. Persons with cirrhosis develop
varices so that varices are present in approximately two-thirds of all cirrhotic patients
•• Varices appear as tortuous dilated veins lying primarily within the submucosa of the distal esophagus and proximal stomach.
Intramural Hematoma
•• Emetogenic injury, particularly in patients with bleeding abnormalities, can cause bleeding between the mucosal and muscle
layers of the esophagus
•• The patients develop sudden dysphagia
•• The diagnosis is made by barium swallow and computed tomography
•• Resolution is usually spontaneous.
Foreign Bodies
•• Foreign bodies may lodge in the cervical esophagus
•• Impaction of a bolus of food, particularly a piece of meat or bread, may occur when the esophageal lumen is narrowed due to
stricture, carcinoma, or a lower esophageal ring (nonprogressive dysphagia)
•• Acute impaction causes a complete inability to swallow and severe chest pain
•• Both foreign bodies and food boluses may be removed endoscopically.
Esophageal Cancer
Predisposing factors for esophageal cancer
•• Excess alcohol
Surgery 451
•• Cigarette smoking
•• Ingested (nitrates, fungal toxins in pickles)
•• Smoked opiates
•• Hot tea
•• Corrosives
•• Radiation strictures
•• Chronic achlasia
•• Caustic ingestion
•• External beam irradiation
•• Diverticulae
•• Plummer-Vinson syndrome
•• Tylosis
•• Molybdenum, Zinc and Vitamin A deficiency
•• Celiac sprue
•• Barrett’s esophagus
Features: Progressive dysphagia (mc) weight loss, weakness, anemia, hoarseness
Esophagoscopy is the diagnosis of choice.
•• Mc type of esophageal cancer: Squamous cell ca
•• Mc site of esophageal cancer: Lower end
•• Mc site of adenocarcinoma: Lower end
•• Mc site of squamous cell ca: Middle 1
•• Chemotherapy has limited role and cisplatin, bleomycin and 5 fluorouracil are used.
Esophagectomies
•• Cervical esophagus: Transhiatal esophagectomy
•• Upper thoraxic: Transincisional/Transhiatal esophagectomy
•• Mid thoraxic: Transincisional/Ivor Lewis surgery
•• Lower thoraxic: Ivor-lewis surgery
Abdomen
The abdominal wall is composed of nine layers.
From outward in, they are:
(1) Skin
(2) Subcutaneous tissue
(3) Superficial fascia (Scarpa’s fascia)
(4) External abdominal oblique muscle
(5) Internal abdominal oblique muscle
(6) Transversus abdominis muscle
(7) Transversalis fascia
(8) Extraperitoneal adipose and areolar tissue
(9) Peritoneum
Omphalitis
•• This is infection of the umbilicus
•• Results from poor hygiene and is treated with appropriate cleansing and local care to the umbilicus
•• However, in the neonatal period, omphalitis may result from bacterial infection and may potentially be associated with serious
sequelae, such as portal vein thrombosis.
452 USMLE Step 2 CK Platinum Notes
Desmoid Tumors
•• These are benign fibrous tumors that arise from the musculoaponeurotic abdominal wall.
•• These tumors are histologically benign but frequently are locally invasive and are prone to recurrence after local excision.
•• They present as firm, subcutaneous masses that grow slowly. They should be widely excised to prevent local recurrence. They do not
have a propensity toward metastasis.
Celiac Trunk
•• LEFT GASTRIC ARTERY→ esophageal branch, gastric branch
•• COMMON HEPATIC ARTERY→ right hepatic, left hepatic, gastroduodenal artery → supraduodenal, right gastroepiploic, superior
pancreaticoduodenal artery
•• SPLENIC ARTERY→ short gastric, left gastroepiploic, pancreatic branches
Chronic Gastritis
It is defined as the presence of chronic inflammatory changes in the mucosa leading eventually to mucosal atrophy and epithelial
metaplasia.
Surgery 453
Classification of Gastritis
•• Type A: Autoimmune gastritis—Associated with pernicious anemia. Body of the stomach is usually affected
•• Type B: Environmental gastritis—Results from dietary or intraluminal factors, specially Helicobacter pylori. Usually localized in the
antrum. In severe cases, the inflammation spreads to the proximal part of the stomach
•• Type C: Chemical gastritis—Associated with ingestion of nonsteroidal anti-inflammatory drugs or bile influx.
Features
•• Earliest symptom is vague postprandial heaviness
•• Features: weight loss, (MC), ascites, jaundice
•• Enlarged It. Supraclavicular node (Virchow’s node) ‘Troisier’s’ sign
•• Sister Mary Joseph’s nodule around the umbilicus
•• Secondaries in ovary (Krukenberg’s tumor)
•• Peritoneal implants in pelvis (Blumer’s shelf)
•• ‘Trousseau’s’ sign and DVT (both Troiser’s and Trousseau’s sign)
•• Best diagnosis is by: ‘ENDOSCOPY WITH BIOPSY’
•• Dumping syndromes
•• Osteomalacia/osteoporosis
•• Anemia
•• Intestinal obstruction
•• Carcinoma
Gastric Lymphoma
•• Most common in the sixth decade
•• The common symptoms being pain, weight loss and bleeding
•• Acute presentations of gastric lymphoma such as hematemesis, perforation or obstruction are not common
•• Primary gastric lymphomas are B-cell-derived, the tumor arising from the mucosa-associated lymphoid tissue (MALT)
•• Primary gastric lymphoma remains in the stomach for a prolonged period before involving the lymph nodes
•• Diagnosis is by endoscopic biopsy
•• Infection with H. pylori, increases the risk for gastric lymphoma in general and MALT lymphomas in particular
•• Microscopically, the vast majority of gastric lymphoid tumors are non-Hodgkin’s lymphomas of B-cell origin; Hodgkin’s disease
involving the stomach is extremely uncommon
•• Radiotherapy plus chemotherapy is used for treatment
•• Bleeding, perforation and residual disease warrant surgery
•• Rotation can occur in the horizontal (organoaxial) ‘most common’or vertical (mesenterioaxial) direction
•• This condition is usually associated with paraesophageal herniation
•• The transverse colon moves upwards to lie under the left diaphragm, thus taking the stomach with it, and the stomach and
colon may both enter the chest through the eventration of the diaphragm
•• The condition is commonly chronic, the patient presenting with difficulty in eating. An acute presentation with ischemia may
occur.
Ulcers
These are defined as a break in the mucosal surface >5 mm in size, with depth to the submucosa.
Duodenal ulcer Gastric ulcer
•• More common •• Less common
•• More associated with H. pylori •• Associated with H. pylori
•• Early age of onset •• Later age of onset
•• Occurs most often in the first portion of duodenum, with •• Occurs most often in the lesser curvature
~90% located within 3 cm of the pylorus •• More chances of malignancy
•• Less chances of malignancy •• Loss of weight
•• No loss of weight •• Tenderness usually midline
•• Tenderness usually in R. Hypochondrium •• Night pain uncommon
•• Night pain common
Dumping Syndrome
Dumping syndrome consists of a series of vasomotor and gastrointestinal signs and symptoms and occurs in patients who
have undergone vagotomy and drainage (especially Billroth procedures)
Early Dumping
•• Takes place 15 to 30 min after meals
•• Consists of crampy abdominal discomfort, nausea, diarrhea, belching, tachycardia, palpitations, diaphoresis, light-headedness and,
rarely, syncope
•• Cause: Rapid emptying of hyperosmolar gastric contents into the small intestine
•• Fluid shift into the gut lumen with plasma volume contraction and acute intestinal distention
•• Release of vasoactive gastrointestinal hormones (vasoactive intestinal polypeptide, neurotensin, motilin)
Late Dumping
•• Typically occurs 90 min to 3 h after meals
•• Vasomotor symptoms (light-headedness, diaphoresis, palpitations, tachycardia, and syncope) predominate during this phase
•• This component of dumping is thought to be secondary to hypoglycemia from excessive insulin release
•• Dumping syndrome is most noticeable after meals rich in simple carbohydrates (especially sucrose) and high osmolarity
•• Ingestion of large amounts of fluids may also contribute
•• Up to 50% of postvagotomy and drainage patients will experience dumping syndrome to some degree
•• Signs and symptoms often improve with time
•• Dietary modification is the cornerstone of therapy for patients with dumping syndrome
•• Small, multiple (six) meals devoid of simple carbohydrates coupled with elimination of liquids during meals is important
•• Antidiarrheals and anticholinergic agents are complimentary to diet
The somatostatin analogue octreotide has been successful in diet-refractory cases.
‘Pseudomembranous’ Enterocolitis
A 14-year-old girl presents to the GI Clinic with the complain of watery diarrhea tinged with blood. Her medications are topical
benzoyl peroxide and oral clindamycin for acne vulgaris. Physical examination reveals a slightly distended abdomen that is diffusely
tender.
458 USMLE Step 2 CK Platinum Notes
Surgery in UC
There are several well-identified complications that require urgent operation for survival. These include:
•• Massive, unrelenting hemorrhage
•• Toxic megacolon with impending or frank perforation
•• Fulminating acute ulcerative colitis that is unresponsive to steroid therapy
•• Obstruction from stricture
•• Suspicion or demonstration of colonic cancer
The largest number of colectomies for ulcerative colitis are performed for less dramatic indications, as the disease enters an intractable
chronic phase and becomes both a physical and a social burden to the patient
For Hemorrage/Bleeding: Prompt surgical intervention is indicated after hemodynamic stabilization. More than 50% of patients with
acute colonic bleeding have toxic megacolon. So one should be suspicious of the coexistence of the two complications. Uncontrollable
hemorrhage from the entire colorectal mucosa may be the one clear indication for emergency proctocolectomy. If possible, the rectum
should be spared for later mucosal proctectomy with ileoanal anastomosis.
Ulcers in GIT
•• Transverse ulcers: TB
•• Longitudinal ulcer: Crohn’s disease
•• ‘Flask’-shaped ulcer: Amebic ulcer
MALIGNANT:
•• Adenocarcinoma
•• Carcinoid
•• Lymphoma
•• Sarcoma
Small Intestine
•• Blood Supply: The small intestine receives its blood supply from the superior mesenteric artery, the second largest branch of
the abdominal aorta
•• Peyer’s patches are lymph nodules aggregated in the submucosa of the small intestine. These lymphatic nodules are most
abundant in the ileum, but the jejunum also contains them
•• Mucosa: The mucosal surface of the small intestine contains numerous circular mucosal folds called the plicae circulares
(valvulae conniventes, or valves of Kerckring). These folds are 3 to 10 mm in height; they are taller and more numerous in the
distal duodenum and proximal jejunum, becoming shorter and fewer distally
•• Intestinal villi barely visible to the naked eye resemble tiny finger-like processes projecting into the intestinal lumen.
•• Significant weight loss due to decreased food intake (food fear) may be present
•• Arteriographic studies should be performed to confirm the diagnosis
•• The only definitive treatment is vascular surgery or balloon angioplasty to remove the thrombus or the construction of bypass
arterial grafts to the ischemic bowel
Appendix
•• MC position is retrocecal
•• Appendicular artery is an end artery
•• McBurney’s point is in relation to appendicitis
Acute Appendicitis
•• The peak incidence of acute appendicitis is in the second and third decades of life
•• It is relatively rare at the extremes of age
•• The most commonly accepted theory of the pathogenesis of appendicitis is that it results from obstruction followed by infection.
The lumen of the appendix becomes obstructed by hyperplasia of submucosal lymphoid follicles, a fecalith, tumor, or other
pathologic condition. Once the lumen of the appendix is obstructed, the sequence of events leading to acute appendicitis
•• Infection with Yersinia organisms cause the disease (PSEUDOAPPENDICITIS)
•• Left sided appendicitis is diverticulosis
•• Most commonly caused by a fecalith, which results from accumulation and inspissation of fecal matter around vegetable fibers
•• Enlarged lymphoid follicles associated with viral infections (e.g. measles), inspissated barium, worms (e.g. Oxy uris)
•• (Pinworms, Ascaris, and Taenia), and tumors (e.g. carcinoid or carcinoma) may also obstruct the lumen
•• Tenderness at Mcburney’s point
•• Retrocecal appendicitis: Rigidity absent
•• Pelvic and postileal appendicitis: Diarrhea (MISSED FREQUENTLY)
•• Gangrene and perforation occur. If the process evolves slowly, adjacent organs such as the terminal ileum, cecum, and omentum
may wall off the appendiceal area so that a localized abscess will develop, whereas rapid progression of vascular impairment may
cause perforation with free access to the peritoneal cavity
•• Signs:
–– Hamburger sign (significant anorexia)
–– Pointing sign
–– Rovsing’s sign
–– Psoas sign
–– Obturator test
•• Acute appendicitis may be the first manifestation of Crohn’s disease
•• Fever greater than 102 °F and leucocyte count greater than 18,000/mm3 suggests ruptured appendix
•• The history and sequence of symptoms are important diagnostic features of appendicitis
•• The initial symptom is almost invariably abdominal pain usually poorly localized in the periumbilical or epigastric region with an
accompanying urge to defecate or pass flatus, neither of which relieves the distress
•• Appendicitis is the most common extrauterine condition requiring abdominal operation
•• USG is helpful in diagnosis
•• USG and CT scan are confirmatory
•• The treatment is early operation and appendectomy as soon as the patient can be prepared
•• The only circumstance in which operation is not indicated is the presence of a palpable mass 3 to 5 days after the onset of symptoms.
Mucocele of Appendix:
•• It is a benign tumor/retention cyst
•• It can progress to malignancy.
Surgery 461
Appendicular Carcinoid:
•• Mc malignant tumor of appendix
•• Adenocarcinoma of appendix is rare
•• Treated by right hemicolectomy.
Peritoneal cavity
•• It is the largest cavity in the body
•• Mc causative organism of acute bacterial peritonitis is E. coli followed by Klebsiella
•• Peritoneal mesotheliomas are associated with asbestos exposure
•• Lymphoma and retroperitoneal sarcoma are the most common malignant lesions of retroperitoneum
•• ‘Ormond’s disease’ is idiopathic retroperitoneal fibrosis
•• ‘Peritoneal mice’ is Appendicis epiploicae
•• Commonest site of intraperitoneal abscess is ‘pelvic’.
Pneumococcal Peritonitis
There are two forms of this disease: (1) primary, and (2) secondary to pneumonia
•• Primary pneumococcal peritonitis is more common
•• The patient is often an undernourished girl between 3 and 6 years of age, and it is probable that the infection sometimes occurs
via the vagina and Fallopian tubes, for pneumococci have been cultured from patients’ vaginas
•• In males, the infection is blood-borne from the upper respiratory tract or the middle ear. After the age of 10, pneumococcal
peritonitis is most unusual
•• Associated with nephritic syndrome
•• Children with nephritis are more liable to this condition than others.
Familial Mediterranean fever (periodic peritonitis)
•• Characterized by abdominal pain and tenderness, mild pyrexia, polymorphonuclear leucocytosis and occasionally pain in
the thorax and joints
•• The duration of an attack is 24–72 hours, when it is followed by complete remission but exacerbations recur at regular intervals
•• Most of the patients have undergone appendicectomy in childhood
•• This disease, often familial, is limited principally to Arabs, Armenians and Jews; other races are occasionally affected
•• The peritoneum—particularly in the vicinity of the spleen and the Gallbladder is inflamed
•• There is no evidence that the interior of these organs is abnormal. Colchicine may prevent recurrent attacks.
Peritoneal bands and adhesions
•• Congenital bands and membranes. Intestinal obstruction is rarely seen except by an obliterated vitellointestinal duct
•• Peritoneal adhesions
•• Peritoneal adhesions are abnormal deposits of fibrous tissue that form after peritoneal injury
•• They follow operation or peritonitis and are the commonest cause of small bowel obstruction and secondary female infertility in
developed countries.
Talc granuloma
•• Talc (silicate of magnesium) should never be used as a lubricant for rubber gloves for it is a cause of peritoneal adhesions and
granulomas in the Fallopian tubes.
Starch peritonitis
•• Like talc, starch powder has found disfavor as a surgical glove lubricant
•• In a few starch-sensitive patients, it causes a painful ascites, fortunately of limited duration.
462 USMLE Step 2 CK Platinum Notes
Frequently asked
Subphrenic abscess
Anatomy
The complicated arrangement of the peritoneum results in the formation of four peritoneal and three extraperitoneal spaces in which
pus may collect. Three of these spaces are on either side of the body, and one is approximately in the midline
Left superior (anterior) intraperitoneal (left subphrenic)
•• It is bounded above by the diaphragm, and behind by the left triangular ligament and the left lobe of the liver, the gastrohepatic
omentum and anterior surface of the stomach. To the right is the falciform ligament and to the left the spleen, gastrosplenic
omentum and diaphragm
•• Patient is toxic
•• The common cause of an abscess here is an operation on the stomach, the tail of the pancreas, the spleen or the splenic
flexure of the colon.
Left inferior (posterior) intraperitoneal (left subhepatic)
•• It is another name for the ‘lesser’ sac
•• The commonest cause of infection here is complicated acute pancreatitis. In practice, a perforated gastric ulcer rarely causes a
collection here because the potential space is obliterated by adhesions.
Right superior (anterior) intraperitoneal (right subphrenic)
•• Lies between the right lobe of the liver and the diaphragm. It is limited posteriorly by the anterior layer of the coronary and the
right triangular ligaments, and to the left by the falciform ligament
•• Common causes here are perforating cholecystitis, a perforated duodenal ulcer, a duodenal cap ‘blow-out’ following gastrectomy
and appendicitis.
Right inferior (posterior) intraperitoneal (right subhepatic)
•• It lies transversely beneath the right lobe of the liver in Rutherford Morison’s pouch
•• It is bounded on the right by the right lobe of the liver and the diaphragm. To the left is situated the foramen of Winslow and below
this lies the duodenum. In front are the liver and the gallbladder, and behind, the upper part of the right kidney and diaphragm
•• The space is bounded above by the liver and below by the transverse colon and hepatic flexure. It is the deepest space of the four
and the commonest site of a subphrenic abscess which usually arises from appendicitis, cholecystitis, a perforated duodenal ulcer
or following upper abdominal surgery.
Subphrenic spaces
•• Gardner’s Syndrome. This syndrome is generally characterized by rectal and colonic polyposis, but generalized polyposis has
been recorded. These polyps are involved in the development of adenocarcinoma. The syndrome also includes cysts of the skin,
osteomas, fibrous and fatty tumors of the skin and mesentery, follicular odontomas, and dentigerous cysts and changes in
the bony structures of the jaws. This syndrome is familial and is transmitted as an autosomal dominant trait
•• Gordon’s Disease. This is a protein-losing gastroenteropathy, usually manifested as Ménétrier’s disease, which involves
mucosal hypertrophy, hyperplasia of the superficial epithelium, degeneration in the glandular layer, and hypoproteinemia due
to leakage of proteins through the mucous membranes. A diffuse gastrointestinal polyposis associated with protein loss has also
been reported
•• Juvenile Polyposis. Juvenile polyposis is most commonly found in the colon and rectum, but isolated examples of generalized
gastrointestinal polyposis have been reported with and without family history or other congenital abnormalities.
•• Muir-Torre Syndrome. This syndrome was described to include sebaceous adenomas, epidermoid cysts, fibromas, desmoids,
lipomas, fibrosarcomas and leiomyomas with visceral cancers
•• Peutz-Jeghers Syndrome. This syndrome is characterized by hamartomatous polyps of the gastrointestinal tract (stomach,
small bowel, colon) that are associated with mucocutaneous pigmentation (lips, oral mucosa, fingers, forearm, toes, umbilical
area). The skin pigmentation may fade after puberty, but that of the mucous membrane is retained
•• Pseudoxanthoma elasticum. Benign vascular lesions of the intestinal tract have been reported in association with this disease
•• Rendu-Osler-Weber Disease. This disease is described as telangiectasia of the nasopharynx or gastrointestinal tract
•• Turcot’s Syndrome. Malignant brain tumors are associated with inherited intestinal adenomatous polyposis
•• von Recklinghausen’s Disease. Generalized neurofibromatosis with café au lait skin pigmentation may also include
neurofibromas of the gastrointestinal tract.
Non-neoplastic Polyps
•• Hyperplastic polyps
•• Hamartomatous polyp
•• Inflammatory polyp
•• Almost all other polyps are malignant
•• Juvenile polyp is most common in children
•• Villous adenoma has the highest malignant potential than other histological types.
Malignant polyps
Meckel’s Diverticulum
•• A true congenital diverticulum
•• It is a vestigial remnant of the omphalomesenteric duct (also called the vitelline duct), and is the most frequent malformation
of the gastrointestinal tract
•• Meckel’s diverticulum is located in the distal ileum, usually within about 60-100 cm of the ileocecal valve
•• It is typically 3–5 cm long, runs antimesenterically and has its own blood supply
•• It is a remnant of the connection from the umbilical cord to the small intestine present during embryonic development.
464 USMLE Step 2 CK Platinum Notes
•• An umbilical polyp is a small excrescence of omphalomesenteric duct mucosa that is retained in the umbilicus. Such polyps
resemble umbilical granulomas except that they do not disappear after silver nitrate cauterization.
•• Umbilical sinuses result from the continued presence of the umbilical end of the omphalomesenteric duct. These resemble
umbilical polyps, but close inspection reveals the presence of a sinus tract deep to the umbilicus.
•• Persistence of the entire omphalomesenteric duct is heralded by the passage of enteric contents from the umbilicus.
•• Cystic remnants of the omphalomesenteric duct may persist and be asymptomatic for long periods of time. The cysts may be
connected to the ileum with a fibrous band that is a remnant of the obliterated omphalomesenteric duct. Patients may present
with acute volvulus and intestinal obstruction or with acute abdomen because of cyst infection.
•• Meckel’s diverticulum results when the intestinal end of the omphalomesenteric duct persists. This is a true diverticulum of
the intestine with all layers of the intestinal wall represented.
•• Hereditary Non-Polyposis Coli •• This consists of three family members having at leat two generations with colonic
Syndrome (Lynch Syndrome) cancer
•• There is a high risk of associated ovarian and endometrial cancer in this syndrome.
•• Cowden’s Syndrome •• Hamartomas with increased risk of bleeding especially in a child presenting with
rectal bleeding.
Colonic Diverticula
•• Herniations or sac-like protrusions of the mucosa through the muscularis, at the point where a nutrient artery penetrates the
muscularis
•• Most commonly in the sigmoid colon and decrease in frequency in the proximal colon
•• They increase with age
•• Related to an increase in intraluminal pressure
•• Diet, deficient in dietary fiber or roughage. Results in decreased fecal bulk, narrowing of the colon, and an increase in intraluminal
pressure in order to move the smaller fecal mass
•• Colonic diverticula are usually asymptomatic and are an incidental finding on barium enema or colonoscopy.
Diverticulitis
•• Inflammation can occur in diverticular sac
•• The cause is mechanical, related to retention in the diverticula of undigested food residue and bacteria, which may form a hard
mass called a fecalith
•• More often in the left as in the right colon
•• Acute colonic diverticulitis is a disease of variable severity characterized by fever, left lower quadrant abdominal pain, and
signs of peritoneal irritation—muscle spasm, guarding, rebound tenderness
•• Rectal bleeding, usually microscopic, is noted in 25% of cases; it is rarely massive. Polymorphonuclear leukocytosis is common
•• Massive hemorrhage from colonic diverticula is one of the most common causes of hematochezia in patients over age 60
•• Complications include free perforation, which results in acute peritonitis, sepsis, and shock, particularly in the elderly
•• Abscess formation or fistulas then occur as the inflammatory mass burrows into other organs. Severe pericolitis may cause a
fibrous stricture around the bowel, which can be associated with colonic obstruction and may mimic a neoplasm
•• Diagnosis: During the acute phase of diverticulitis, barium enema and sigmoidoscopy may be hazardous, since contrast material
or air under pressure may lead to rupture of an inflamed diverticulum and convert a walled-off inflammatory lesion to a free
perforation
•• Colonoscopy or surgical excision may be required for accurate diagnosis. Abdominal computed tomography scan may demonstrate
the presence of a pericolic abscess.
Complications of diverticulitis
466 USMLE Step 2 CK Platinum Notes
Acquired Megacolon
In Central and South America,
Infection with Trypanosoma cruzi (Chagas disease)
Result in destruction of the ganglion cells of the colon
Onset is in adult life
•• Schizophrenia
•• Depression
•• Cerebral atrophy
Spinal cord injury and
•• Parkinsonism also may cause megacolon
•• Myxedema and primary systemic sclerosis also can reduce colonic motility and produce marked colonic distention.
examination, and there is no evidence of occult blood. X-ray films show a few distended loops of small bowel and a much
distended colon. The cecum measures 9.5 cm in diameter. The gas pattern of distention extends throughout the entire large
bowel, including the sigmoid and rectum. No stool is seen in the films. Vital signs are normal for his age.
Most likely diagnosis is: Ogilvie Syndrome
Intussusception
•• Outer tube: Intussuscipiens
•• Inner Tube: Intussusceptum
•• Stool Red currant jelly
•• Lump Sausage-shaped
•• Emptiness in Iliac Fossa Sign de dance
•• Barium enema Claw sign
Ischemic Colitis
•• Most often affects the elderly
•• Ischemic colitis is almost always nonocclusive
•• In acute fulminant ischemic colitis, the major manifestations are severe lower abdominal pain, rectal bleeding, and
hypotension
•• Dilation of the colon and physical signs of peritonitis are seen in severe cases
•• Abdominal films may reveal thumb printing from submucosal hemorrhage and edema
•• Barium enema is hazardous in the acute situation because of the risk of perforation. Sigmoidoscopy or colonoscopy may detect
ulcerations, friability, and bulging folds from submucosal hemorrhage
•• Surgical resection may be required in some patients with fulminant ischemic colitis to remove gangrenous bowel; others with
lesser degrees of ischemia may respond to conservative medical management
•• Subacute ischemic colitis is the most common clinical variant of ischemic colonic disease. It produces lesser degrees of pain and
bleeding, often occurring over several days or weeks.
Colorectal Cancer
•• Most colorectal cancers, regardless of etiology, arise from adenomatous polyps
•• Probability of an adenomatous polyp becoming a cancer depends on the gross appearance of the lesion, its histologic features,
and its size
•• Adenomatous polyps may be pedunculated (stalked) or sessile (flat-based). Cancers develop more frequently in sessile polyps
•• Villous adenomas, most of which are sessile, become malignant more than three times as often as tubular adenomas
•• The likelihood that any polypoid lesion in the large bowel contains invasive cancer is related to the size of the polyp, being negligible
(< 2%) in lesions < 1.5 cm, intermediate (2 to 10%) in lesions 1.5 to 2.5 cm in size and substantial (10%) in lesions > 2.5 cm
–– Mc site: Rectum
–– Mc site in colon: Sigmoid colon
–– Mc type: Adenocarcinoma
–– Right-sided common in females
–– Right-sided presents as anemia
–– Left-sided presents as obstruction/pain.
Protective Factors
•• Aspirin and other nonsteroidal anti-inflammatory drugs, which are thought to suppress cell proliferation by inhibiting
prostaglandin synthesis
•• Oral folic acid supplements and oral calcium supplements have been found to reduce the risk of adenomatous polyps and
colorectal cancers in case-control studies
•• Ascorbic acid, tocopherols, and b-carotene
•• Estrogen replacement therapy
Clinical Features
•• Lesions of the right colon commonly ulcerate, leading to chronic, insidious blood loss without a change in the appearance of the
stool. Patients with tumors of the ascending colon often present with symptoms, such as fatigue, palpitations, and even angina
pectoris and are found to have a hypochromic, microcytic anemia indicative of iron deficiency
•• Radiographs of the abdomen often reveal characteristic annular, constricting lesions (‘apple-core’ or ‘napkin-ring’)
Surgery 469
•• Cancers arising in the rectosigmoid are often associated with hematochezia, tenesmus, and narrowing of the caliber of
stool; anemia is an infrequent finding
•• Liver is the most common site of metastasis. 1/3rd patients show hepatic metastasis
•• Surgery is treatment of choice
–– Anterior resection: for tumors of upper 2/3rd of rectum
–– Combined abdominoperineal resection: for extensive tumors of lower 1/3rd of rectum
–– Immunotherapy: for disseminated carcinoma colon
Anal Canal Above Dentate Line Anal Canal Below Dentate Line
•• Endodermal •• Ectodermal
•• Cuboidal epithelium •• Stratified squamous
•• Superior rectal artery •• Inferior rectal artery
•• Superior rectal vein •• Inferior rectal vein
•• Internal iliac group of lymph nodes •• Superficial inguinal group of lymph nodes
•• Pain-insensitive •• Pain-sensitive
The Anal Valves of Ball
•• These are a series of transversely placed semilunar folds linking the columns of Morgagni. They lie along and actually constitute
the waviness of the dentate line.
The Crypts of Morgagni (syn. anal crypts)
•• These are small pockets between the inferior extremities of the columns of Morgagni. Into several of these crypts, mostly those
situated posteriorly, open one anal gland by a narrow duct
•• Infection of an anal gland can give rise to an abscess, and infection of an anal gland is the most common cause of anorectal
abscesses and fistulae.
The Anorectal Ring
•• Marks the junction between the rectum and the anal canal
•• It is formed by the joining of the puborectalis muscle, the deep external sphincter, conjoined longitudinal muscle and the
highest part of the internal sphincter
•• The anorectal ring can be clearly felt digitally, especially on its posterior and lateral aspects
•• Division of the anorectal ring results in permanent incontinence of feces
•• The position and length of the anal canal, as well as the angle of the anorectal junction, depend to a major extent on the integrity
and strength of the puborectalis muscle sling.
Fistula-in-Ano
•• A fistula-in-ano is a track, lined by granulation tissue, which connects deeply in the anal canal or rectum and superficially on the
skin around the anus
•• It usually results from an anorectal abscess which burst spontaneously or was opened inadequately
•• The fistula continues to discharge and, because of constant reinfection from the anal canal or rectum, seldom,
if ever, closes permanently without surgical aid
•• An anorectal abscess may produce a track, the orifice of which has the appearance of a fistula, but it does not communicate with
the anal canal or the rectum. By definition, this is not a fistula, but a sinus
Types of anal fistulae
•• Low-level fistulae open into the anal canal below the anorectal ring
•• High-level fistulae open into the anal canal at or above the anorectal ring.
•• Mucoepidermal carcinoma
•• Basal cell carcinoma
•• Malignant melanoma
•• Anal intraepithelial neoplasia (AIN)
Obturator’s sign Internal rotation of flexed right hip causing abdominal pain •• Appendicitis
Grey-Turner’s sign Discoloration of the flank •• Retroperitoneal hemorrhage (hemorrhagic
pancreatitis, abdominal aortic aneurysm
rupture)
Chandelier’s sign Manipulation of cervix causes patient to lift buttocks off •• Pelvic inflammatory disease
table
Rovsing’s sign Right lower quadrant pain with palpation of the left lower •• Appendicitis
quadrant
Surgery 471
Important Appearances
•• Corkscrew esophagus Diffuse esophageal spasm
•• Rosary esophagus Diffuse esophageal spasm
•• ‘Double Bubble sign in X-ray abdomen’ Annular pancreas, Duodenal atresia
•• ‘Scalloping of sigmoid colon’ Ulcerative colitis
•• ‘Microcolon on barium enema.’ Ileal atresia
•• ‘Bird of prey sign’ Sigmoid volvolus
•• ‘String sign’ Crohn’s disease
•• ‘Pipestem colon’ Ulcerative colitis
•• Saw tooth appearance of colon Diverticular disease
•• Apple core lesion/Napkin ring appearance Lt Colonic Ca
•• Coffee bean sign Volvulus
•• Claw sign/Signe de dance Intussusception
•• Thumb printing sign Ischemic colitis
•• Composition of a hernia
As a rule, a hernia consists of three parts — the sac, the coverings of the sac and the contents of the sac.
472 USMLE Step 2 CK Platinum Notes
•• The sac
The sac is a diverticulum of peritoneum consisting of mouth, neck, body and fundus. The neck is usually well-defined, but in some direct
inguinal hernias and in many incisional hernias, there is no actual neck. The diameter of the neck is important because strangulation of
bowel is a likely complication where the neck is narrow, as in femoral and paraumbilical hernias.
•• The body of the sac
The body of the sac varies greatly in size and is not necessarily occupied. In cases occurring in infancy and childhood, the sac is
gossamer-thin. In long-standing cases, the wall of the sac may be comparatively thick.
•• The covering
Coverings are derived from the layers of the abdominal wall through which the sac passes. In long-standing cases they become
atrophied from stretching and so amalgamated that they are indistinguishable from each other.
•• Contents
These can be:
–– Omentum = omentocele (syn. epiplocele)
–– Intestine = enterocele. More commonly small bowel, but may be large intestine or appendix
–– A portion of the circumference of the intestine Richter’s hernia
–– A portion of the bladder (or a diverticulum)
–– Ovary with or without the corresponding Fallopian tube
–– A Meckel’s diverticulum = a Littre’s hernia
–– Fluid — as part of ascites or as a residuum thereof.
Hernia-en-Glissade
•• Hernia-en-Glissade is also called sliding hernia
•• The contents of hernia on left side are sigmoid colon and mesentery
•• Most common content is sigmoid colon
•• The contents of hernia on right side are the cecum
•• Sliding hernia occurs mostly in males
•• Five out of every six cases occur on left side
•• The incidence of hernia increases with increasing age
Surgery 473
Spigelian Hernia
•• Spigelian hernia is an interparietal hernia
•• It lies at the level of arcuate line mostly
•• The arcuate line lies below umbilicus. Hence, it is a subumbilical hernia
•• Lies lateral to rectus abdominis
•• The spigelian fascia is present only below the umbilicus and most of the spigelian hernias are present subumbilically
•• In addition to this, the fibrous bands of spigelian fascia run transversely with small defects in between which are the sites of
spigelian hernia
•• The spigelian hernia may contain loops of bowel, colon, omentum or a part of circumference of bowel only.
Other Hernias
•• Beclard’s hernia: Hernia through opening for saphenous vein
•• Treitz’s hernia: Duodenojujenal hernia
•• Rokintansky hernia: Mucous membrane herniation through muscular layer of bowel
•• Mc type of hernia in men: •• Indirect inguinal
•• Mc type of hernia in women: •• Indirect inguinal
•• Femoral herna is more common in: •• Women
•• Femoral hernia is common on: •• Right side
•• Inguinal hernia is common on: •• Right side
•• Femoral hernia strangulates: •• Commonly
Pancreas
•• The pancreas occupies a retroperitoneal position in the abdomen, lying posterior to the stomach and lesser omentum
•• It extends obliquely from the duodenal C loop to a more cephalad position in the hilum of the spleen
•• The gland is divided into four portions: the head (which includes the uncinate process), the neck, the body, and the tail
•• It includes the posteroinferior extension arising from the ventral primordium, designated the uncinate process
•• Pancreatic agenesis
Very rarely, the pancreas may be totally absent, a condition usually (but not invariably) associated with additional severe malformations
that are incompatible with life. IPF1 gene mutations on chromosome 13q12.1 have been associated with pancreatic agenesis
•• Pancreas divisum
It is the most common clinically significant congenital pancreatic anomaly, with an incidence of 3% to 10%. It occurs when the fetal
duct systems of the pancreatic primordia fail to fuse. As a result, the main pancreatic duct (Wirsung) is very short and drains only a
small portion of the head of the gland, while the bulk of the pancreas (from the dorsal pancreatic primordium) drains through the
minor sphincter. The relative stenosis caused by the bulk of the pancreatic secretions passing through the minor sphincter predisposes
such individuals to chronic pancreatitis
•• Annular pancreas
A ring of pancreatic tissue that completely encircles the duodenum. It can present with signs and symptoms of duodenal obstruction
such as gastric distention and vomiting
•• Ectopic pancreas
Aberrantly situated, or ectopic, pancreatic tissue occurs in about 2% of the population; favored sites are the stomach and duodenum,
followed by the jejunum, Meckel’s diverticulum, and ileum.
474 USMLE Step 2 CK Platinum Notes
Heterotopic Pancreas
•• The development of pancreatic tissue outside the confines of the main gland is a congenital abnormality referred to as
heterotopic pancreas
•• Most commonly, heterotopic pancreatic tissue is found in the stomach (MC)
•• Duodenum, small bowel, or Meckel’s diverticulum
•• In most locations, heterotopic pancreatic tissue resides in a submucosal location, presenting as firm, yellow, irregular nodules
that vary in size from millimeters to several centimeters
•• The clinical significance of heterotopic pancreas is dependent on resultant complications. Intestinal obstruction may ensue,
rarely as a result of the size of the mass, and more commonly following intussusception, with the ectopic pancreatic tissue serving
as the intussusceptum. Other complications of heterotopic pancreas include ulceration and hemorrhage.
Pancreas Divisum
•• Follows failure of fusion of the two primordial ductal systems
•• The major portion of the pancreas is drained via the duct of Santorini via the minor duodenal papilla
•• The major duodenal papilla usually communicates with a small duct of Wirsung, which drains the ventral pancreas,
consisting of the inferior head and uncinate process
•• The significance of pancreas divisum remains controversial. It is unknown whether the ductal anomaly has any causal relationship
to the pancreatitis
•• Normal pancreatic tissue completely or partially encircles the second portion of the duodenum
•• Annular pancreas is thought to arise from failure of normal clockwise rotation of the ventral pancreatic bud
•• Varying degrees of duodenal obstructive symptoms may be observed in this condition
•• There is a common association with other serious congenital anomalies, such as intracardiac defects, Down’s syndrome, and
intestinal malrotation
•• In adults, symptoms may appear to be those of upper gastrointestinal obstruction, chronic pancreatitis, or peptic ulcer
•• Obstructive symptoms are an indication for operation
•• Retrocolic duodenojejunostomy is treatment of choice
Acute pancreatitis: Causes: The vast majority of cases in the UK are caused by gallstones and alcohol. Popular mnemonic is GET
SMASHED
–– Gallstones (Mc cause)
–– Ethanol
–– Trauma
–– Steroids
–– Mumps (other viruses include Coxsackie B)
–– Autoimmune (e.g. polyarteritis nodosa), Ascaris infection
–– Scorpion venom
–– Hypertriglyceridemia, Hyperchylomicronemia, Hypercalcemia, Hypothermia
–– ERCP
–– Drugs (azathioprine, sulfasalazine, didanosine, bendroflumethiazide, frusemide, pentamidine, steroids, sodium valproate)
A faint blue discoloration around the umbilicus (Cullen’s sign) may occur as the result of hemoperitoneum, and a blue-red-purple or
green-brown discoloration of the flanks (Turner’s sign) reflects tissue catabolism of hemoglobin
The diagnosis of acute pancreatitis
•• ↑level of serum amylase. Values threefold or more above normal virtually clinch the diagnosis if overt salivary gland disease and
gut perforation or infarction are excluded
•• There is no definite correlation between the severity of pancreatitis and the degree of serum amylase elevation. After 48 to 72 h,
even with continuing evidence of pancreatitis, total serum amylase values tend to return to normal
•• However, pancreatic isoamylase and lipase levels may remain elevated for 7 to 14 days
•• Fetal fat estimation diagnoses insufficiency
•• Gallstone pancreatitis has best prognosis.
Plain abdominal X-ray shows:
–– Sentinel loop
–– Colon cut-off sign
–– Renal halo sign
Purtscher’s Retinopathy, a relatively unusual complication, is manifested by a sudden and severe loss of vision in a patient with acute
pancreatitis. It is characterized by a peculiar funduscopic appearance with cotton-wool spots and hemorrhages confined to an area
limited by the optic disk and macula; it is believed to be due to occlusion of the posterior retinal artery with aggregated granulocytes.
Pancreatitis in patients with AIDS The incidence of acute pancreatitis is increased in patients with AIDS for two reasons: (1) the high
incidence of infections involving the pancreas, such as infections with cytomegalovirus, cryptosporidium, and the mycobacterium
avium complex; and (2) the frequent use by patients with AIDS of medications, such as didanosine, pentamidine, and trimethoprim-
sulfamethoxazole
Chronic Pancreatitis
•• The classic triad of ‘Pancreatic calcification, Steatorrhea, and Diabetes mellitus’ usually establishes the diagnosis of chronic
pancreatitis and exocrine pancreatic insufficiency but is found in less than one-third of chronic pancreatitis patients
•• The secretin stimulation test, which usually gives abnormal results when 60% or more of pancreatic exocrine function has been
lost. The radiographic hallmark of chronic pancreatitis is the presence of scattered calcification throughout the pancreas
Hereditary Pancreatitis
Hereditary pancreatitis is a rare disease that is similar to chronic pancreatitis except for an early age of onset and evidence of
hereditary factors (involving an autosomal dominant gene with incomplete penetrance). Genetic linkage analysis identified the
hereditary pancreatitis gene on chromosome 7.
Macroamylasemia
•• In macroamylasemia, amylase circulates in the blood in a polymer form too large to be easily excreted by the kidney
•• Patients with this condition demonstrate an elevated serum amylase value, a low urinary amylase value, and a Cam/Ccr ratio of
less than 1%
•• Usually macroamylasemia is an incidental finding and is not related to disease of the pancreas or other organs
•• Macrolipasemia has been documented in a few patients with cirrhosis or non-Hodgkin’s lymphoma.
Pancreatic Pseudocyst
•• Pancreatic pseudocysts are localized collections of pancreatic secretions in a cystic structure that lack an epithelial lining
–– These are usually solitary
–– They are commonly attached to the surface of the gland and involve peripancreatic tissues such as the lesser omental sac or
the retroperitoneum between the stomach and transverse colon or liver
–– They can range from 2 to 30 cm in diameter
–– Since pseudocysts form by walling off areas of hemorrhagic fat necrosis, they are typically composed of necrotic debris
encased by fibrous walls of granulation tissue lacking an epithelial lining
•• Mc site body or tail
•• Mc cause: pancreatitis (adults)
•• Mc cause: trauma (children)
•• Pseudocysts contain high concentrations of pancreatic enzymes, including amylase, lipase, and trypsin
•• Pancreatic pseudocysts develop in up to patients after an attack of acute alcoholic pancreatitis
•• Pseudocysts are also associated with acute pancreatitis with other causes, as well as with chronic pancreatitis, pancreatic
trauma, and pancreatic neoplasm
•• Most often present with upper abdominal pain
•• Physical examination reveals abdominal tenderness in the majority of patients
•• A CT scan of the abdomen is the favored study in an initial assessment for determining the presence of a pancreatic pseudocyst
Pancreatic pseudocyst
Complications:
•• Infection (MC)
•• Rupture
•• Hemorrhage
•• < 5 cm managed conservatively
•• > 5 cm, cyst > 6 weeks, infection, complication: Surgery
•• Treatment of choice is internal drainage (Cystojejunostomy).
Surgery 477
Pancreatic Carcinoma
•• More common in males than in females and in blacks than in whites
•• Cigarette smoking is the most consistent risk factor
•• Chronic pancreatitis
•• Diabetes mellitus
•• Alcohol abuse or cholelithiasis are not risk factors for pancreatic cancer
•• Hereditary pancreatitis, Peutz-Jeghers syndrome, VHL, Ataxia telangiectasia, Gardner’s syndrome and Lynch syndrome II
•• Mutations in K-ras genes
Clinical Features
•• Most common is: Ductal adenocarcinomas
•• Most common in: Pancreatic head (70% of cases)
•• Most common symptom is: Pain and weight loss
•• Most common physical sign is: Jaundice
–– Although the gallbladder is usually enlarged in patients with carcinoma of the head of the pancreas, it is palpable in 50%
(Courvoisier’s sign)
–– Other initial manifestations include venous thrombosis and migratory thrombophlebitis (Trousseau’s sign),
gastrointestinal hemorrhage from varices due to compression of the portal venous system by tumor, and splenomegaly
caused by cancerous encasement of the splenic vein
–– CECT is the most effective technique for diagnosis
–– Whipple’s operation is indicated for periampullary cancer or cancer head of pancreas
–– ‘Gemcitabine’, a deoxycytidine analogue, produces improvement in the quality of life for patients with advanced pancreatic
cancer.
Zollinger-Ellison Syndrome/Gastrinoma
•• Mc site is duodenum followed by pancreas
•• Mc malignant pancreatic endocrine tumor
•• More common in males
•• Nonbeta cell tumor
•• Tumor of delta cells
Gastrinoma Triangle
90% of extrapancreatic gastrinomas lie here
Formed by:
•• 3rd part of duodenum
478 USMLE Step 2 CK Platinum Notes
•• Cystic duct
•• Pancreatic neck
Ulcers in ZES
•• Unusual location
•• Ulcers in absence of predisposing factors
•• Ulcers in presence of hypercalcemia or family history of MEN 1
•• Multiple ulcers
•• Ulcers refractory to treatment
•• Recurrence of ulcers
•• Giant ulcers
•• Ulcers with complications
Insulinomas
•• Insulinomas are MC endocrine tumors of the pancreas thought to be derived from beta cells that autonomously secrete insulin, which
results in hypoglycemia
•• The most common clinical symptoms are due to the effect of the hypoglycemia on the central nervous system (neuroglycemic symptoms)
and include confusion, headache, disorientation, visual difficulties, irrational behavior, or even coma
•• Also, most patients have symptoms due to excess catecholamine release secondary to the hypoglycemia, including sweating,
tremor, and palpitations. Characteristically these attacks are associated with fasting
•• Usually single
•• Whipple’s triad is a feature
•• Weight gain occurs
•• Insulinomas are generally, usually solitary (90%), and only 5 to 15% are malignant
•• They almost invariably occur only in the pancreas, distributed equally in the pancreatic head, body and tail
•• Insulinomas should be suspected in all patients with hypoglycemia, especially with a history suggesting attacks provoked by
fasting or with a family history of MEN-1
•• In insulinomas, in addition to elevated plasma insulin levels, elevated plasma proinsulin levels are found and C-peptide
levels can be elevated.
Glucagonomas
•• Glucagonomas are endocrine tumors of the pancreas that secrete excessive amounts of glucagon that causes a distinct
syndrome characterized by dermatitis, glucose intolerance or diabetes, and weight loss
•• Glucagonomas mainly occur in persons between 45 and 70 years old. They are heralded clinically by a characteristic dermatitis
(migratory necrolytic erythema; accompanied by glucose intolerance weight loss, anemia, diarrhea and thromboembolism
•• The characteristic rash usually starts as an annular erythema at intertriginous and periorificial sites, especially in the groin or
buttock. It subsequently becomes raised and bullae form; when the bullae rupture, eroded areas form. The lesions can wax and
wane
•• A characteristic laboratory finding is hypoaminoacidemia
•• Glucagonomas are generally large tumors at diagnosis, with an average size of 5 to 10 cm. Between 50 and 80% occur in the
pancreatic tail and 50 to 82% have evidence of metastatic spread at presentation, usually to the liver. Glucagonomas are rarely
extrapancreatic and usually occur singly.
Surgery 479
Somatostatinoma Syndrome
Somatostatinomas are endocrine tumors that secrete excessive amounts of somatostatin, which causes a syndrome characterized
by:
•• Diabetes mellitus
•• Gallbladder disease
•• Diarrhea
•• Steatorrhea
Somatostatinomas occur primarily in the pancreas and small intestine, and the frequency of the symptoms differs in each.
Somatostatinomas occur in the pancreas in 56 to 74% of the cases, with the primary location being in the pancreatic head.
VIPomas
Are endocrine tumors that secrete excessive amounts of VIP, which causes a distinct syndrome characterized by large-volume
diarrhea, hypokalemia, and dehydration. This syndrome is also called Verner-Morrison syndrome, pancreatic cholera, or WDHA
syndrome
•• Watery diarrhea
•• Hypokalemia
•• Achlorhydria/hypochloridia
•• Hypercalcemia
Spleen
•• The gastrosplenic ligament contains the short gastric arteries and veins
•• The splenorenal ligament contains the splenic artery and vein, lymphatic structures, and often the tail of the pancreas
•• The arterial supply to the spleen is derived from the celiac artery from both the splenic artery and the short gastric arteries,
which usually arise as branches of the gastroepiploic or the splenic arteries
•• The splenic vein is formed by a coalescence of polar veins in the splenic hilum and courses with the splenic artery along the dorsal
surface of the pancreas to enter the portal system
•• The normal adult spleen is a slightly concave, solid, dark red organ that measures approximately 3 × 8 × 14 cm
•• MC cyst of spleen is pseudocyst
•• Aneurysmal rupture may occur and the rupture initially may be contained within the lesser sac
•• Initial aneurysmal rupture into the peritoneal cavity or delayed rupture from the lesser sac are associated with findings of
hemoperitoneum and exsanguinating hemorrhage. Rarely, a splenic artery aneurysm ruptures into the gastrointestinal tract,
pancreatic duct, or splenic vein.
Splenic Abscess
Usually results from
•• Bacteremia associated with a primary septic focus, such as bacterial endocarditis or lung abscess
•• Secondary infection in an area of the spleen damaged by infarction (sickle cell anemia or leukemia), trauma, or parasitic infestation
•• Clinical features of splenic abscess are those of left subphrenic suppuration and include fever, chills, left upper quadrant
tenderness, and often splenomegaly
•• Ultrasonography and radionuclide and CT scans are useful
•• CT is the most direct way of evaluating the spleen.
Splenunculi
These are single or multiple accessory spleens which are found
•• Near the hilum of the spleen
•• Splenic vessels
•• Behind the tail of the pancreas in 30 percent
•• In the splenic ligaments (gastrocolic, greater omentum, splenocolic)
•• Mesocolon in the remainder
•• Up to 20 percent of people have such splenunculi and most are no larger than 2 cm in diameter
Their importance lies in the fact that if not removed at the time of splenectomy, they will undergo hyperplasia and may well be the
site of persistent disease.
Splenic Trauma
•• Kehr’s sign is occurrence of pain in the tip of shoulder due to presence of blood/irritant in peritoneal cavity
•• Kehr’s sign in left shoulder is considered a characteristic sign of splenic rupture
•• Kehr’s sign is a classic example of referred pain (phrenic nerve)
Other important conditions causing Kehr’s sign are:
–– Diaphragmatic lesions
–– Ectopic pregnancy
–– Renal calculi
–– Hemoperitonium
Surgery 481
The signs and symptoms of splenic trauma are those of hemoperitoneum. Generalized and nonspecific abdominal pain in the left
upper quadrant occurs in approximately one-third of patients with splenic injury. Pain referred to the tip of the left shoulder (Kehr’s
sign) is inconstant, varying in incidence from 15 to 75%, and is unreliable for excluding splenic injury but is useful for enhancing the
diagnostic probability if present. Kehr’s sign is elicited by bimanual compression of the left upper quadrant after the patient has
been in Trendelenburg’s position for several minutes preceding the maneuver. On rare occasions, patients with splenic injury have a
palpable tender mass in the left upper quadrant (Ballance’s sign), caused by an extracapsular or subcapsular hematoma with omentum
adherent to the injured spleen.
Spleen
•• Felty’s Syndrome: Rheumatoid arthritis with Hypersplenism
•• Banti’s Syndrome: Congestive splenomegaly with Hypersplenism
•• Egyptian Splenomegaly: Schistosomiasis
•• Angiosarcoma: MC malignant tumor of spleen
•• Splenosis: Rupture of spleen with dissemination into peritonium
•• Spontaneous rupture is seen in Infectious mononucleosis typhoid, leukemia
Splenectomy
•• Mc indication for elective splenectomy: ITP
•• HS
•• G6PD
•• Portal hypertension
•• Hypersplenism
•• Autosplenectomy is seen in sickle-cell anemia
•• Pneumococcal vaccine and N. meningitidis vaccine given
•• Splenorrhaphy is done in stable patient with lacerated spleen.
Complications
•• Hemmorage
•• Hematemesis
•• Left basal atelectasis
•• Pancreatitis, pancreatic abscess
•• Risk of infections
Liver
•• 80% blood supply is from portal vein. 20% is from hepatic artery
•• Liver is divided into surgical right and left lobes by a line between Gallbladder and middle hepatic vein. Cantlie line
•• Portal venous system lacks valves
Portal vein
484 USMLE Step 2 CK Platinum Notes
Portal Hypertension
•• (> 10 mm Hg) most commonly results from increased resistance to portal blood flow
•• Increased resistance can occur at three levels relative to the hepatic sinusoids:
–– Presinusoidal
–– Sinusoidal
–– Postsinusoidal
•• ‘Cirrhosis’ is the most common cause of portal hypertension in the United States
•• Portal venous pressure may be measured directly by percutaneous transhepatic ‘skinny needle’ catheterization or indirectly
through transjugular cannulation of the hepatic veins
Sites of portosystemic anastomosis involved in portal hypertension:
–– Umbilicus: Caput medusae
–– Lower end of esophagus: Esophageal varices
–– Rectum and anal canal: Hemorrhoids
–– Bare area of liver
–– Posterior abdominal wall
•• Bleeding varices: Propranolol is the drug of choice for prophylaxis of bleeding
•• Octreotide is the drug of choice for bleeding varices
•• Barium swallow shows string of beads in esophagus.
Hydatid Disease
•• Echinococcosis is also known as hydatid disease
•• Hydatid cyst is a parasitic infection of humans by the tapeworm of genus echinococcus
•• It is a zoonosis
–– Echinococcus granulosus causes Cystic Echinococcosis
–– Echinococcus multilocularis causing Alveolar Echinococcosis
–– Echinococcus vogeli causes Polycystic disease
Echinococcus granulosus:
The liver is the most common organ affected followed by lungs, muscles, bones and kidneys
Passage of hydatid membrane in emesis is called hydatid emesia
Passage of hydatid membrane in stools is called hydatid enterica
Flushing and urticaria occur in rupture of hydatid cyst
1. Abdominal tenderness is the most common sign
2. Tender hepatomegaly signifies secondary infection of the cyst
3. Ascites is rare
4. Splenomegaly can be the result of portal hypertension or splenic echinococcosis
•• Brain involvement depends on site of brain involved and may present as coma or herniation
•• Bone and muscle involvement presents as visible masses
•• Echinococcosis is caused by larval stages of parasite
•• Man is an accidental intermediate host. Other intermediate hosts are sheep and cattle
•• The dog is the definitive host
•• Serological assay (Weinberg reaction) is a specific example of complement fixation test used in detection
•• ELISA is also sensitive, blood culture not useful
•• Eosinophilia is seen only in case of rupture of cysts
•• Detachment of cysts and their collapse lead to characteristic water Lily sign or floating membrane sign or camellotte sign
•• Other X-ray signs of echinococcus are:
–– Serpent sign/Rising sun sign
–– Crumbled egg appearance
–– Anaphylaxis, intrapleural rupture, intraperitoneal rupture are secondary complications
•• PAIR Technique (Percutaneous aspiration, injection of scolicidal agents and reaspiration of cyst contents) has also been used for
treatment
•• Albendazole is the most effective drug most commonly used.
486 USMLE Step 2 CK Platinum Notes
Hepatic Adenoma
•• Benign neoplasm
•• Derived from hepatocytes
•• Seen in women taking OCPS
•• Usually asymptomatic but may bleed intraabdominally
•• Seen as cold defects in hepatic scintigraphy
•• Core biopsy contraindication
•• Aspiration biopsy can be done
•• > 6 cm tumor should be excised
HEPATOCELLULAR CARCINOMA
Predisposing Factors
•• Hepatitis B
•• Hepatitis C
•• Alcoholic liver disease
•• Alpha 1 antitrypsin deficiency
•• Hemachromatosis
•• Tyrosinemia
•• Aflatoxin B1
•• Vinyl chloride, Thorium and Androgenic hormones are also implicated
Hepatocellular Carcinoma
Hepatocellular carcinoma: The principal reason for the high incidence of hepatocellular carcinoma in parts of Asia and Africa is the
frequency of chronic infection with hepatitis B virus (HBV) and hepatitis C virus (HCV).
‘Chronic liver disease’ of any type is a risk factor and predisposes to the development of liver cell carcinoma. These conditions include
alcoholic liver disease, a1-antitrypsin deficiency, hemochromatosis and tyrosinemia.
A small percentage of patients with hepatocellular carcinoma have a paraneoplastic syndrome
Erythrocytosis may result from erythropoietin-like activity produced by the tumor.
Hypercalcemia may result from secretion of a parathyroid-like hormone. Other manifestations may include hypercholesterolemia,
hypoglycemia, acquired porphyria, dysfibrinogenemia and cryofibrinogenemia.
Imaging procedures to detect liver tumors include ultrasound, CT, MRI, hepatic artery angiography and technetium scans.
Ultrasound is frequently used to screen high-risk populations and should be the first test if hepatocellular carcinoma is suspected; it is
less costly than scans, is relatively sensitive, and can detect most tumors > 3 cm.
Helical CT and MRI scans are being used with increasing frequency and have higher sensitivities.
AFP levels > 500 ug/L are found in about 70 to 80% of patients with hepatocellular carcinoma. Lower levels may be found in patients
with large metastases from gastric or colonic tumors and in some patients with acute or chronic hepatitis.
High levels of serum AFP (> 500 to 1000 ug/L) in an adult with liver disease and without an obvious gastrointestinal tumor strongly
suggest hepatocellular carcinoma.
Percutaneous liver biopsy can be diagnostic if the sample is taken from an area localized by ultrasound or CT. Because these tumors
tend to be vascular, percutaneous biopsies should be done with caution.
Staging of hepatocellular carcinoma is based on tumor size (< or > 50% of the liver), ascites (absent or present), bilirubin (< or
> 3), and albumin (< or >3) to establish Okuda stages I, II, and III. The natural history of each stage without treatment is: stage I, 8
months; stage II, 2 months; stage III, less than 1 month.
Fibrolamellar Carcinoma
Fibrolamellar carcinoma differs from the typical hepatocellular carcinoma in that it tends to occur in young adults without
underlying cirrhosis
NOT associated with cirrhosis
This tumor is nonencapsulated but well circumscribed and contains fibrous lamellae
It grows slowly and is associated with a longer survival if treated
Surgical resection has resulted in 5-year survivals >50%; if the lesion is nonresectable, liver transplantation is an option, and the
outcome far exceeds that observed in the nonfibrolamellar variety of liver cancer.
Hepatoblastoma is a tumor of infancy that typically is associated with very high-serum AFP levels
The lesions are usually solitary, may be resectable, and have a better 5-year survival than that of hepatocellular carcinoma.
Angiosarcoma consists of vascular spaces lined by malignant endothelial cells. Etiologic factors include prior exposure to thorium
dioxide (Thorotrast), polyvinyl chloride, arsenic, and androgenic anabolic steroids.
This tumor occurs in early adulthood, presents with right upper quadrant pain, is heterogenous on sonography, hypodense on CT, and
without neovascularity on angiography
Immunohistochemical staining reveals expression of factor VIII antigen. In the absence of extrahepatic metastases, these lesions can
be treated by surgical resection or liver transplantation.
MELD
•• The ‘Model for End-Stage Liver Disease’ (MELD) system was implemented on February 27, 2002 to prioritize patients waiting
for a liver transplant
•• MELD is a numerical scale used for adult liver transplant candidates.
•• The range is from 6 (less ill) to 40 (gravely ill). The individual score determines how urgently a patient needs a liver transplant
within the next three months. The number is calculated using the most recent laboratory tests
•• Within the MELD continuous disease severity scale, there are four levels. As the MELD score increases, and the patient moves up to
a new level, a new waiting time clock starts
•• Waiting time is carried backwards but not forward. If a patient moves to a lower MELD score, the waiting time accumulated at the
higher score remains. When a patient moves to a higher MELD score, the waiting time at the lower level is not carried to the new
level.
Surgery 489
‘Carcinoid Syndrome’
•• Occurs in less than 10% of patients with carcinoid tumors
•• The carcinoid syndrome is encountered when venous drainage from the tumor gains access to the systemic circulation so
that vasoactive secretory substances escape hepatic degradation
•• This situation obtains in three circumstances:
–– When hepatic metastases are present,
–– When venous blood from extensive retroperitoneal metastases drains into paravertebral veins, and
–– When the primary carcinoid tumor is outside the gastrointestinal tract, e.g. a bronchial, ovarian, or testicular tumor.
•• The principal features of carcinoid syndrome include flushing, sweating, wheezing, diarrhea, abdominal pain, cardiac valvular
fibrosis, and pellagra dermatosis
•• Diarrhea is found in 83% of patients, flushing in 49%, dyspnea in 20%, and bronchospasm in 6%
•• Many patients develop right-sided cardiac valvular disease with congestive heart failure. Serotonin and possibly other
neurohumors produced by the tumor cause fibrosis, as well as eventual incompetence of the tricuspid and pulmonic valves
•• The lungs metabolize serotonin and the other mediators and protect the left heart from fibrosis. If one can establish that the tumor
is slow-growing, patients with carcinoid-induced cardiac lesions are candidates for valve replacement
•• Biochemical mediators
•• The specific etiologic agents for each of the protean manifestations of carcinoid tumors are not known. Serotonin, prostaglandins,
5-hydroxytryptophan, substance P, kallikrein, histamine, dopamine, and neuropeptide K are thought to be involved in the
clinical manifestations of carcinoid tumors
•• Serotonin is thought to be largely responsible for both the diarrhea and the fibrosis. The cardiac lesions, and tricuspid and
pulmonic insufficiency are components of this fibrosing phenomenon
•• Other substances, such as histamine, VIP, and prostaglandins, may also contribute to the systemic manifestations in the
carcinoid syndrome
Diagnosis
•• Urinary 5-HIAA or whole blood and platelet-poor plasma 5-HT is the most reliable test to confirm the diagnosis of carcinoid
syndrome
•• Occasionally, measurement of plasma levels of substance P and neurotensin by radioimmunoassay may also be helpful
•• Measurements of neuron-specific enolase and chromogranins, when available, provide nonspecific evidence of the presence
of a neuroendocrine tumor
•• A useful diagnostic aid is the pentagastrin provocative test, which induces facial flushing, gastrointestinal symptoms, elevation
in circulating 5-HT
Appendicular Carcinoids
–– Not the commonest site
–– Mc site: tip
–– Presents as recurrent appendicitis
–– It has good prognosis
•• Carcinoid tumors occur in ileum and appendix
•• Rectum is rarely involved
•• 5-year survival is more than 60%
•• Females are affected less
Floating gallbladder
The organ may hang on a mesentery which makes it liable to undergo torsion.
Double gallbladder
Rarely, the gallbladder is twinned. One of the twins may be intrahepatic.
Absence of the cystic duct
This is usually pathological, as opposed to an anatomical anomaly and indicates the recent passage of a stone or the presence of a
stone at the lower end of the cystic duct which is ulcerating into the common bile duct. The main danger at surgery is damage to the
bile duct, and particular care to identify the correct anatomy is essential before division of any duct.
Low insertion of the cystic duct
The cystic duct opens into the common bile duct near the ampulla. All variations of this anomaly can occur. At operation, they are
not important. Dissection of a cystic duct which is inserted low in the bile duct should be avoided as removal will damage the blood
supply to the common bile duct and can lead to stricture formation.
An accessory cholecystohepatic duct
Ducts passing directly into the gallbladder from the liver do occur and are probably not uncommon. Nevertheless, larger ducts should
be closed but before doing so, the precise anatomy should be carefully ascertained.
Extrahepatic biliary atresia
Etiology and pathology
Atresia is present in one per 14,000 live births, and affects males and females equally. The extrahepatic bile ducts are progressively
destroyed by an inflammatory process which starts around the time of birth. Intrahepatic changes also occur and eventually result in
biliary cirrhosis and portal hypertension. The untreated child dies before the age of 3 years of liver failure or hemorrhage.
The inflammatory destruction of the bile ducts has been classified into three main types:
–– Type I — atresia restricted to the common bile duct
–– Type II — atresia of the common hepatic duct
–– Type III — atresia of the right and left hepatic ducts
PREDISPOSING FACTORS
Cholelithiasis
Cholesterol and Mixed Stones
1. Familial disposition
2. Hereditary aspects
3. Obesity
4. Weight loss
5. Female sex hormones
6. Ileal disease or resection
7. Increasing age
8. Gallbladder hypomotility leading to stasis and formation of sludge:
–– Prolonged parenteral nutrition
–– Fasting
–– Pregnancy
–– Drugs such as octreotide
9. Clofibrate therapy
10. Decreased bile acid secretion
–– Primary biliary cirrhosis
–– Chronic intrahepatic cholestasis
–– Remember fat, flatulent, female, fertile, forty/fifty
Pigment Stones
1. Demographic/genetic factors: Asia, rural setting
2. Chronic hemolysis
3. Alcoholic cirrhosis
4. Chronic biliary tract infection, parasite infestation
5. Increasing age
CBD Stone
•• Seen in 15% patients with gallstones
•• Develop in gallbladder
•• Obstructive pattern seen
•• Mc cause of obstructive jaundice
•• It can present with cholangitis
•• Bile duct diameter > 6 mm on USG is suggestive
•• ERCP, Sphincterotomy, with ballon clearance is standard treatment used
•• In cholangiography appears as meniscus sign.
Indications of cholecystectomy
Gallstone Ileus
•• Gallstone ileus refers to mechanical intestinal obstruction resulting from passage of a large gallstone into bowel lumen
•• This particular complication follows internal fistulas in which a gallstone or common duct stone gains entrance into the
intestinal tract through an internal biliary fistula
•• The usual potential complication is ascending cholangitis
•• A typical presentation of gallstone ileus is a history of frequent previous episodes of partial bowel obstruction that exacerbate
and abate as the stone negotiates its way into a narrower region of the intestinal tract. This phenomenon is called a tumbling
obstruction
•• Three-fourths of the spontaneous fistulas underlying gallstone ileus occur between the gallbladder and the duodenum
•• Most gallstones that enter the gastrointestinal tract are either passed or vomited, but 10% to 15% may lead to gallstone ileus
•• The common site of obstruction, found in two-thirds of patients, is the ileum
•• The diagnosis is easily made if a large mass lesion is found at the site of bowel obstruction; this mass is readily identified if the
gallstone is opaque. Sometimes, even if it is nonopaque, it can be observed because of the surrounding intestinal air
•• In addition, the finding of air in the biliary tree makes the diagnosis almost certain
•• The proper treatment of gallstone ileus is relief of the intestinal obstruction, usually by the performance of an enterotomy and
removal of the stone
•• Concomitant definitive correction of the internal fistula is advocated if the patient is in good condition and has sustained no
prolonged desease.
•• The triad of sudden onset of RUQ tenderness, fever, and leukocytosis is highly suggestive. Typically, leukocytosis in the range
of 10,000 to 15,000 cells per microliter with a left shift on differential count is found
•• The radionuclide (e.g. HIDA) biliary scan is IOC
•• Ultrasound will demonstrate calculi in 90 to 95% of cases.
Acalculous Cholecystitis
Especially associated with:
•• Serious trauma or
•• Burns
•• Diabetes
•• TPN
•• With the postpartum period following prolonged labor, and orthopedic and other nonbiliary major surgical operations in
the postoperative period vasculitis, obstructing adenocarcinoma of the gallbladder, diabetes mellitus, torsion of the gallbladder,
‘unusual’ bacterial infections of the gallbladder (e.g. Leptospira, Streptococcus, Salmonella, or Vibrio cholerae), and parasitic
infestation of the gallbladder
•• Acalculous cholecystitis may also be seen with a variety of other systemic disease processes (sarcoidosis, cardiovascular disease,
tuberculosis, syphilis, actinomycosis, etc.) and may possibly complicate periods of prolonged parenteral hyperalimentation
•• USG is IOC.
Enzyme elevation
Emphysematous Cholecystitis
•• Begin with acute cholecystitis (calculous or acalculous) followed by ischemia or gangrene of the gallbladder wall and infection by
gas-producing organisms
•• Bacteria most frequently cultured in this setting include anaerobes, such as C. welchii or C. perfringens, and aerobes, such as
E. coli
•• This condition occurs most frequently in elderly men and in patients with diabetes mellitus
•• The diagnosis is usually made on plain abdominal film by the finding of gas within the gallbladder lumen, dissecting within the
gallbladder wall to form a gaseous ring, or in the pericholecystic tissues. The morbidity and mortality rates with emphysematous
cholecystitis are considerable. Prompt surgical intervention coupled with appropriate antibiotics is mandatory.
Chronic Cholecystitis
Chronic inflammation of the gallbladder wall is almost always associated with the presence of gallstones and is thought to result from
repeated bouts of subacute or acute cholecystitis or from persistent mechanical irritation of the gallbladder wall. The presence of bacteria
in the bile occurs in more than one-quarter of patients with chronic cholecystitis.
Cholangiocarcinoma
Adenocarcinoma of the extrahepatic ducts is more common. There is a slight male preponderance (60%), and the incidence peaks in
the fifth to seventh decades.
494 USMLE Step 2 CK Platinum Notes
Klatskin Tumor
Tumor of CBD between cystic duct and common hepatic duct
•• ↓metastasis
•• Slow growth
•• Shows sclerosing features
•• There is believed to be a 15% incidence of carcinoma of the gallbladder in patients who have or have had a cholecystoenteric
fistula and the tumor may develop as much as 16 years later.
•• There is believed to be a 15% incidence of carcinoma of the gallbladder in patients who have or have had a cholecystoenteric
fistula and the tumor may develop as much as 16 years later.
•• The incidence of carcinoma in a calcified or porcelain gallbladder is reported to range from 12.5 to 61%.
•• It is now generally accepted that adenoma of the gallbladder is a precancerous lesion. Adenomas present as polypoid lesions
which are best detected by ultrasonography.
•• It has been suggested that xanthogranulomatous cholecystitis, a rare form of chronic cholecystitis that may grossly mimic
cancer of the gallbladder, is also associated with a higher than expected incidence of carcinoma.
•• Cancer of the gallbladder is more frequent in the presence of congenital biliary dilatation in which case there appears to be a
lower incidence of associated gallstones.
•• Ulcerative colitis has a well-known association with biliary tract malignancy.
•• Although the majority of such malignancies involve the bile ducts, as many as 13% originate in the gallbladder.
Acute Cholangitis
•• MC organisms: E. coli, Klebsiella, Pseudomonas, Enterococcus
•• Obstruction of CBD leading to bacterial stasis, bacterial overgrowth, suppuration, biliary sepsis
•• Charcots triad: Fever + RUQ pain + Jaundice
•• Reynolds pentad: Fever + RUQ pain + Jaundice + Shock + Confusion
•• Suppurative cholangitis has a high mortality rate
Sclerosing Cholangitis
•• Progressive, inflammatory, sclerosing, and obliterative process affecting the ‘extrahepatic and/or the intrahepatic bile
ducts’
•• The disorder occurs in about 70% in association with inflammatory bowel disease, especially ulcerative colitis
•• Also associated (albeit rarely) with multifocal Fibrosclerosis Syndromes such as:
–– Retroperitoneal, mediastinal, and/or periureteral fibrosis
–– Riedel’s struma
–– Pseudotumor of the orbit
Cholangiopancreatography may demonstrate a broad range of biliary tract changes as well as pancreatic duct obstruction and
occasionally pancreatitis
•• Often present with signs and symptoms of chronic or intermittent biliary obstruction: jaundice, pruritus, RUQ abdominal pain,
or acute cholangitis
•• Late in the course, complete biliary obstruction, secondary biliary cirrhosis, hepatic failure, or portal hypertension with bleeding
varices may occur
•• The diagnosis is established beaded appearance on cholangiography
496 USMLE Step 2 CK Platinum Notes
Complications
–– Adenocarcinoma colon
–– Adenocarcinoma bile ducts
–– Cirrhosis
Hemobilia
•• Triad of:
–– Abdominal pain
–– Obstructive jaundice
–– Melana
•• Occurs as a result of bleeding into biliary tract
•• Causes:
–– Trauma
–– Gallstones
–– Cholangiohepatitis
–– Neoplasms of hepatobiliary system
–– Hepatic artery trauma
Kidney
Renal tuberculosis
•• Tuberculosis of the urinary tract arises from hematogenous infection from a distant focus which is often impossible to identify.
The lesions are usually confined to one kidney
•• A group of tuberculous granulomas in a renal pyramid coalesces and forms an ulcer. Mycobacteria and pus cells are discharged
into the urine
•• Sterile pyuria is a consistent feature
•• Untreated, the lesions enlarge and a tuberculous abscess may form in the parenchyma
•• The necks of the calyces and the renal pelvis stenosed by fibrosis confine the infection so that there is tuberculous pyonephrosis,
which is sometimes localized to one pole of the kidney
•• Extension of pyonephrosis or tuberculous renal abscess leads to perinephric abscess and the kidney is progressively replaced by
caseous material (putty kidney) which may be calcified (cement kidney)
•• At any stage, the plain radiograph may show areas of calcification (pseudocalculi)
•• Less commonly the kidneys may be bilaterally affected as part of the generalized process of miliary tuberculosis
•• Renal tuberculosis is often associated with tuberculosis of the bladder and typical tuberculous granulomas may be visible in the
bladder wall. In the male, tuberculous epididymo-orchitis may occur without apparent infection of the bladder
•• IVU is the most sensitive technique for detecting early renal TB.
Renal carbuncle
•• An abscess in the renal parenchyma as the result of blood-borne spread of organisms, especially coliforms or Staphylococcus
aureus, from a focus elsewhere in the body
•• Occasionally the condition results from infection of a hematoma following a blow to the kidney
•• Renal carbuncle is most commonly seen in diabetic patients, intravenous drug abusers, those debilitated by chronic disease
and patients with acquired immunodeficiency.
Pathology: the renal parenchyma contains an encapsulated necrotic mass
Clinical features: There is an ill-defined tender swelling in the loin, persistent pyrexia and leucocytosis, signs that closely simulate
those of perinephric abscess. In early cases, there is no pus or bacteria in the urine but they appear after a day or so. Urography shows
a space-occupying lesion in the kidney which may be confused with a renal adenocarcinoma on ultrasonography and CT
Treatment: Resolution by antibiotic treatment alone is unusual. Formal open incision of the abscess may be necessary if the pus is too
thick to be drained by percutaneous aspiration.
Surgery 497
‘Renal Calculi’
•• Calcium stones: Calcium oxalate and calcium phosphate stones make up 75 to 85% of the total renal calculi
•• Most common
•• Calcium stones are more common in men; the average age of onset is the third decade. Approximately, 60% of people who form
a single calcium stone eventually form another within the next 10 years
•• In the urine, calcium oxalate monohydrate crystals (whewellite) usually grow as biconcave ovals that resemble red blood cells in
shape and size but may occur in a larger, ‘dumbbell’ form.
•• Uric acid stones are radiolucent and are also more common in men
•• Half of patients with uric acid stones have gout; uric acid lithiasis is usually familial whether or not gout is present. In urine, uric
acid crystals are red-orange in color because they absorb the pigment uricine
•• Uric acid gravel appears like red dust, and the stones are also orange or red on some occasions.
Stones (Calculi)
•• Whewellite: (Calcium oxalate monohydrate crystals)
•• Weddilite: (Calcium oxalate dihydrate crystals)
•• Staghorn/Coffin lid: Struvite
•• Radiolucent: Uric acid
•• Radio-opaque: Calcium, Struvite, Cysteine
Acute Pyelonephritis
It is an infectious disease involving the kidney parenchyma and the renal pelvis. Gram-negative bacteria, such as Escherichia coli,
Proteus, Klebsiella, and Enterobacter, are the most common causative organisms in acute pyelonephritis. Laboratory evaluation will
often reveal leukocytosis with a left shift, and urinalysis typically shows pyuria, varying degrees of hematuria, and white cell casts.
Since bacteremia is present, the patient should be hospitalized and empirically started on IV ampicillin and gentamicin.
•• The presenting signs and symptoms include hematuria, abdominal pain, and a flank or abdominal mass. This ‘classic triad’
occurs in 10 to 20% of patients. Other symptoms are fever, weight loss, anemia, and a varicocele
•• Causes expansile osteolytic lesions
•• A spectrum of paraneoplastic syndromes has been associated with these malignancies, including:
–– Erythrocytosis
–– Hypercalcemia
–– Nonmetastatic hepatic dysfunction (Stauffers’ syndrome)
–– Acquired dysfibrinogenemia
Erythrocytosis is present at presentation in only about 3% of patients
–– More frequently anemia, a sign of advanced disease
–– Cushing’s syndrome
–– Galactorrhea
–– Amyloidosis
–– Hypertension
–– PUO
Pheochromocytoma
•• Tumor of chromaffin cells
•• (Headache) Hypertension is a common symptom but not a reliable symptom of pheochromocytoma
•• Pheochromocytoma is a cause of ‘Episodic/Paroxysmal’ Hypertension and not persistent hypertension
•• Pheochromocytoma is also a cause of ‘Orthostatic Hypotension.’
•• This rare tumor is most often found in the adrenal medulla, although it can also be found in other tissues derived from neural
crest cells. The tumor cells secrete catecholamine hormones or their precursors, which can cause either paroxysmal or persistent
hypertension. Urinary metabolites of epinephrine and norepinephrine are:
–– Vanillylmandelic acid (VMA) and homovanillic acid
–– So screening 24-hour urine collections for these substances can be helpful in establishing or excluding these diagnoses even
in cases in which a physician does not observe one of the paroxysms and thus blood cannot be drawn for serum catecholamine
levels at that time.
500 USMLE Step 2 CK Platinum Notes
•• NA production is ↑↑
–– Bilateral in 10% cases
–– Extra-adrenal in 10% cases
–– Familial in 10% cases
–– Malignant in 10% cases
–– Multiple in 10% cases
•• ↑urinary VMA levels seen
•• ↑urinary metanephrine
•• MIBG scan is most specific and sensitive for diagnosis
Associated with:
•• MEN 2 syndromes
•• von-Hippel-Lindau syndrome
•• Neurofibromatosis
•• Sturge-Weber syndrome
Nutcracker phenomenon
•• It is seen in varicocele
•• It refers to nut-inside-a-cracker-like image
•• The left renal vein is distended in varicocele on the left side
•• As a result, ‘Left renal vein gets compressed’ in between superior mesenteric artery and the aorta leading to renal vein
hypertension
•• The remedy is surgical transposition of left renal vein
The two diseases most commonly leading to renal failure and treatable by kidney transplantation are glomerulonephritis and
insulin-dependent diabetes mellitus
Other important causes include:
•• Polycystic kidney disease
•• Hypertensive nephrosclerosis
Surgery 501
•• Alport’s disease
•• IgA nephropathy
•• Systemic lupus erythematosus
•• Nephrosclerosis
•• Interstitial nephritis
•• Pyelonephritis
•• Obstructive uropathy
•• Hypertensive nephrosclerosis
The left kidney is chosen, if possible, since its longer renal vein facilitates the recipient operation
However, if the arteriogram shows multiple renal arteries on one side, the kidney with a single artery is usually selected to facilitate
the anastomosis
A flank incision is used. After Gerota’s fascia is incised, the greater curvature of the kidney and the upper pole are mobilized, and the
hilar structures are exposed
On the left side, the adrenal and gonadal veins are ligated so that the full length of the renal vein can be utilized
Traction on the renal artery should be avoided, since it causes spasm and decreased kidney perfusion, possibly compromising early
function
The ureter should be mobilized, along with its blood supply and a generous amount of periureteric tissue. It is divided close to the
bladder after ligating the distal end.
Schistosomiasis
•• S. Haematobonium is causative agent
•• Resides in vesical venous plexus
•• Urticaria at the site of penetration occurs (Swimmers’ itch)
•• Calcifications with small contracted bladder.
Intermittent Painless Terminal Hematuria
•• Praziquantel is the DOC
Surgery is done in case of:
•• Secondary bacterial cystitis
•• Urinary calculi
•• Ureteric stricture
•• Seminal vesiculitis
•• Fibrosis of bladder and bladder neck
•• Cancer
Cystoscopic Features
•• ‘Pseudotubercles’ on cystoscopy are the earliest specific features. The pseudotubercles are larger, more prominent, more
numerous, more yellow and more distinctly grouped.
•• ‘Nodules’ are due to fusion of tubercles.
•• ‘Sandy patches’ are a result of calcified dead ova.
•• ‘Ulceration’ is the result of sloughing of mucous membrane containing dead ova. Ulcers are shallow, bleed readily.
•• ‘Carcinoma’ is the end result of neglected bilharziasis.
•• Carcinoma commences in ulcer and not papilloma.
•• It is usually well advanced and requires radical cystectomy.
•• It is sometimes associated with abacterial urethritis and is commonly sexually acquired. Tuberculous infection and carcinoma in
situ must be ruled out
•• The underlying causative organism may be mycoplasma or herpes.
Frequency dysuria syndrome (urethral syndrome)
•• This is common in women
•• It consists of symptoms suggestive of urinary infection, but with negative urine cultures and absent pus cells
•• Carcinoma in situ, tuberculosis and interstitial cystitis should be excluded
•• Adopt general measures such as wearing cotton underwear, using simple soaps, general perineal hygiene and voiding after
intercourse
•• Other treatments include cystoscopy and urethral dilatation, although the benefits remain doubtful.
Tuberculous urinary infection
•• Tuberculous urinary infection is secondary to renal tuberculosis
•• Cystoscopy shows that early tuberculosis of the bladder commences around the ureteric orifice or trigone, the earliest evidence
being pallor of the mucosa due to submucous edema
•• Subsequently tubercles may be seen cobble-stone appearance
•• Long-standing cases: There is much fibrosis and the capacity of the bladder is greatly reduced, scarred, fibrosed, small capacity
(thimble bladder)
•• Rigid wide-mouthed ureter (golf hole ureter)
Reiter’s Syndrome
Patients typically present with the acute onset of arthritis (usually asymmetric and additive), with involvement of new joints occurring
over a period of a few days to 2 weeks. Joints of the lower extremities are the most commonly involved, but wrists and fingers can
also be affected. Dactylitis (sausage digit), a diffuse swelling of a solitary finger or toe, is a distinctive feature of Reiter’s arthritis and
psoriatic arthritis. Tendonitis and fasciitis are common. Spinal pain and low back pain are common. Conjunctivitis, urethritis, diarrhea,
and skin lesions are also associated with Reiter’s syndrome. Up to 75% of patients are HLA-B27 positive. Micro-organisms which can
trigger Reiter’s syndrome include Shigella spp., Salmonella spp., Yersinia spp., Campylobacter jejuni, and Chlamydia trachomatis.
Most patients are younger males.
Incontinence
Genuine stress incontinence is defined as urinary leakage occurring during increased bladder pressure when this is solely due to
increased abdominal pressure and not due to increased true detrusor pressure. It is caused by sphincter weakness. The commonest
cause of leakage of urine in women is genuine stress incontinence (GSI), although in some parts of the world, vesicourethral fistulae
owing to neglected labor are very common. GSI occurs secondary to weakness of the distal sphincter mechanism associated with
laxity of the pelvic floor. It is usually found in multiparous women with a history of difficult labor often accompanied by the use of
forceps.
Chronic urinary retention with overflow incontinence. This is recognized by a large residual volume of urine and is usually associated
with high pressures during bladder filling.
Neurogenic incontinence. The common causes include:
•• Myelodysplasia
•• Multiple sclerosis
•• Spinal cord injuries
•• Cerebral dysfunction [cerebrovascular accident (CVA), dementia]
•• Parkinson’s disease (paralysis agitans)
These conditions lead to a combination of neurogenic vesical dysfunction often associated with loss of mobility. Careful investigation
of the whole urinary tract is always required, and the treatment needs to strike a fine balance between preventing hydronephrosis
from abnormally high bladder pressures, yet at the same time maintaining continence.
The mainstay of management is accurate urodynamic assessment to assess bladder emptying, incontinence and the risks to the upper
tract. The upper tracts should be assessed with regular ultrasound scanning, and assessment of the patient’s mobility, intelligence and
motivation is vital. The important factors to assess urodynamically are:
504 USMLE Step 2 CK Platinum Notes
•• Bladder emptying
•• Bladder capacity and bladder pressure during filling
•• Continence
Small bladder capacity. The capacity of the bladder may be considerably diminished in several conditions. This can cause crippling
urinary frequency and incontinence. It may follow tuberculosis, radiotherapy or interstitial cystitis. Radiotherapy for pelvic cancer can
also cause this problem.
Drug-induced incontinence. The detrusor muscle is basically under postganglionic parasympathetic control and the main
neurotransmitter system is cholinergic. A number of drugs can induce urinary retention (anticholinergic agents, tricyclic
antidepressants, lithium and some antihypertensives).
Constant dribbling of urine coupled with normal micturition. This occurs when there is a ureteric fistula or an ectopic ureter
associated with a duplex system opening into the urethra beyond the urethral sphincter in females, or into the vagina. The history
is diagnostic, and intravenous pyelography or ultrasound scanning may reveal the upper pole segment which is often poorly
functioning. These segments are very liable to infection. Treatment is by excision of the aberrant ureter and portion of kidney which
needs it. A ureteric fistula can be difficult to diagnose and may require retrograde ureterography and a high degree of suspicion to
demonstrate.
Urethral Stricture
The causes of urethral stricture are:
•• Congenital
•• Traumatic; MC Cause
•• Inflammatory:
–– Postgonorrheal
–– Posturethral chancre
–– Tuberculous
•• Instrumental:
–– Indwelling catheter
–– Urethral endoscopy
•• Postoperative:
–– Open prostatectomy
–– Amputation of penis
Extravasation of Urine
Superficial extravasation is likely with complete rupture of the bulbar urethra and in ruptured urethral abscess.
•• The extravasated urine is confined in front of the midperineal point by the attachment of Colles fascia to the triangular ligament,
and by the attachment of Scarpa’s fascia just below the inguinal ligament. The external spermatic fascia stops it getting into the
inguinal canals
•• Extravasated urine collects in the scrotum and penis and beneath the deep layer of superficial fascia in the abdominal wall.
Treatment is by urgent operation to drain the bladder by suprapubic cystostomy. This prevents further extravasation.
Deep extravasation occurs with extraperitoneal rupture of the bladder or intrapelvic rupture of the urethra
•• It can also occur if the ureter is damaged or if there is perforation of the prostatic capsule or bladder during transurethral resection
•• Urine extravasates in the layers of the pelvic fascia and the retroperitoneal tissues.
Treatment is by suprapubic cytostomy and drainage of the retropubic space.
Fracture of the pubic and ischial rami is most likely to result when sudden force is applied to one lower limb in a car accident or in
landing on one leg after falling from a height. There is an associated disruption of the sacroiliac joint so that one half of the pelvis and
ischiopubic ramus is pushed up above the other. This applies a traction force on the prostate which is firmly bound by ligaments to the
back of the symphysis pubis. The torn ends of the urethra may be widely displaced by this type of injury
Catheterization is best avoided.
Hypospadias
•• Hypospadias occurs in one in 350 male births
•• It is the most common congenital malformation of the urethra
•• The external meatus opens on the underside of the penis or the perineum and the inferior aspect of the prepuce is poorly developed
(‘hooded prepuce’)
•• Meatal stenosis also occurs
•• Bifid scrotum.
Hypospadias is classified according to position of the meatus.
Glandular hypospadias — this is the most common type and does not usually require treatment. The normal site of the external
meatus is marked by a blind pit, although it occasionally connects by a channel to the ectopic opening on the underside of the glans.
Coronal hypospadias — the meatus is placed at the junction of the underside of the glans and the body of the penis.
Penile and penoscrotal hypospadias — the opening is on the underside of the penile shaft.
Perineal hypospadias — this is the most severe abnormality. The scrotum is split and the urethra opens between its two halves.
There may be testicular maldescent which may make it difficult to determine the sex of the child.
The more severe varieties of hypospadias represent an absence of the urethra and corpus spongiosum distal to the ectopic opening.
The absent structures are represented by a fibrous cord which deforms the penis in a downward direction (chordee). The more distant
the opening from its normal position, the more pronounced the bowing
6-10 months of age is the best time for surgery.
Cryptorchidism
The term ‘cryptorchidism’ (Greek cryptos = hidden, orchis = testis) should be reserved for impalpable, usually abdominal,
testes
•• There is a higher incidence of undescended testes in premature than in full-term babies.
Surgery 507
•• Two-thirds of undescended testes in newborn infants will descend, usually by 6 weeks in term and 3 months in preterm babies
•• There is an increased incidence of cryptorchidism in anencephalics and other cerebral anomalies
Ectopic testes
These have descended as far as the external inguinal ring and then become deviated into the:
•• Superficial inguinal pouch
•• Perineal
•• Suprapubic or
•• Femoral ectopic sites
•• The commonest by far is the superficial inguinal pouch, above and lateral to the external inguinal ring.
Retractile testes
The cremasteric reflex in young children will draw the testes into the region of the superficial inguinal pouch very readily but they can
be manipulated back down to the bottom of the scrotum. The testis would normally reside in the scrotum if such a child is in a warm
bath or is relaxed in bed.
Ascending testis
Some boys with recorded testicular descent at routine clinic checks in infancy may be found later at preschool or school medicals to
have an undescended testis. This phenomenon of the ‘ascending testis’ was noted first by Atwell. It has been suggested that this is
caused by failure of elongation of the spermatic cord during differential body growth, so that the testis is drawn up by absorption of
the processus vaginalis.
Anorchia
Anorchia may be on one or both sides. If on one side alone, there may be ipsilateral renal agenesis. If the baby is fully masculinized but
both testes are absent, it must be assumed that they have atrophied subsequent to torsion or infarction during development. Absence
of testicular tissue and, therefore, lack of Müllerian inhibitory hormone during early gestation can lead to Müllerian development
along the female lines. The lack of androgenic stimulation (testosterone) from the testes leads to failure of Wolffian duct development.
•• Torsion of the testicle should be corrected as soon as possible after the diagnosis is entertained
•• Incomplete torsion can cause partial strangulation, the effects of which may be overcome if surgical intervention is accomplished
within 12 hours, whereas severe torsion with complete compromise of the blood supply results in loss of the testis unless surgical
intervention occurs within approximately 4 hours
•• The contralateral scrotum should also be explored at the time of the operation, since the primary anatomic defect—insufficient
attachment of the testicle to the scrotal sidewall—most often is a bilateral phenomenon
•• If the contralateral scrotum is not explored, the patient runs a very high risk of undergoing torsion on the other side and the
possible complication of loss of both testes.
508 USMLE Step 2 CK Platinum Notes
Remember
•• Aspermia: No ejaculate
•• Oligospermia: Sperm count less than 20 million/ml
•• Polyzoospermia: Count >350 million/ml
•• Azoospermia: No spermatozoa in semen
•• Asthenozoospermia: Reduced sperm motility
•• Necrozoospermia: Spermatozoa are dead
•• Teratozoospermia: >70% Spermatozoa with abnormal morphology
Oligospermia, by definition, indicates a sperm count of less than 20 million per ml. and under such circumstances, fertility is difficult
The principal causes of defective spermatogenesis include:
•• Congenital inadequacy of the seminiferous tubules;
•• Testicular damage as a consequence of infection, trauma, or infarction;
•• Klinefelter’s syndrome;
•• Hypopituitarism;
•• Varicocele; and
•• Cryptorchidism
•• Other causes of oligospermia may relate to the transport of spermatozoa. Chronic prostatitis and seminal vesiculitis may result in
fibrosis and impede transport and delivery of sperm. Infection spreading into the vas deferens may induce fibrosis and stricture
even to the point of total occlusion.
Azoospermia, complete absence of spermatozoa in the ejaculate, may be caused by:
•• Total occlusion of the sperm transport system, vasa, seminal vesicles, or ejaculatory ducts. Congenital absence of the vasa and
seminal vesicles may occur as an isolated anatomic defect, and congenital absence of the vasa is the rule in males with cystic
fibrosis
•• Gonococcal epididymitis and vasitis may cause complete stenosis and azoospermia
•• Complete nonresponsiveness of the germinal epithelium as in primary gonadal failure may also produce a picture of azoospermia
despite elevated follicle-stimulating hormone levels
•• Trauma to the vasa in the course of an inguinal hernia operation or orchidopexy
Surgery 509
Testicular Cancer
•• Primary germ cell tumors (GCTs) of the testis constitute 95% of all testicular neoplasms. Infrequently, GCTs arise from an
extragonadal site, including the mediastinum, retroperitoneum and, very rarely, the pineal gland
•• Bilateral in 10% cases
•• Cryptorchidism is associated with a several fold higher risk of GCT
•• Abdominal cryptorchid testes are at a higher risk than inguinal cryptorchid testes
•• Orchiopexy should be performed before puberty, if possible
•• Testicular feminization syndromes increase the risk of testicular GCT
•• Klinefelter’s syndrome is associated with mediastinal GCT
•• An isochromosome of the short arm of chromosome 12 is pathognomonic for GCT
•• A painless testicular mass is pathognomonic for a testicular malignancy
Mc tumor of testis: Seminoma
Mc histological subtype: Mixed
Mc tumor in infants: Yolk sac tumor
Mc tumor in aged elderly: Lymphoma
Seminoma
•• It has a median age in the fourth decade
•• Generally follows a more indolent clinical course
•• Seminomas are radiosensitive
•• But surgery is the TOC
•• Seminomas metastasize by lymphatics
•• Seminomas correspond to dysgerminomas of ovary.
Spermatocele
•• Spermatocele is a unilocular retention cyst
•• It almost always lies in relation to the head of the epididymis
•• The fluid contains spermatozoa and resembles ‘barley fluid’
•• Treatment for small spermatoceles is conservative follow-up while larger ones should be aspirated or excised
•• A spermatocele is a diverticulum of the epididymis that contains cloudy fluid with spermatozoa. It is unilocular or multilocular
and often confused with hydrocele because both a spermatocele and a hydrocele can be transilluminated. Differential diagnosis
of spermatocele and hydrocele is aided by the localization of the mass: hydrocele generally surrounds the testis, while the
spermatocele is more eccentric, can often be palpated in direct conjunction with the epididymis, and is often tender.
Spermatocele
•• This is a unilocular retention cyst derived from some portion of the sperm-conducting mechanism of the epididymis. A spermatocele
nearly always lies in the head of the epididymis above and behind the upper pole of the testis
•• It is usually softer and laxer than other cystic lesions in the scrotum but like them it transilluminates
•• The fluid contains spermatozoa and resembles barley water in appearance
•• Spermatoceles are usually small and unobtrusive. Less frequently they are large enough to make the patient think that he has
three testicles.
Infantile hydrocele
Infantile hydrocele does not necessarily appear in infants. The tunica and processus vaginalis are distended to the inguinal ring but
there is no connection with the peritoneal cavity.
Congenital hydrocele
The processus vaginalis is patent and connects with the general peritoneal cavity. The communication is usually too small to allow
herniation of intra-abdominal contents. Digital pressure on the hydrocele does not usually empty it but the hydrocele fluid may drain
into the peritoneal cavity when the child is lying down. Ascites or even ascitic tuberculous peritonitis should be considered if the
swellings are bilateral.
Encysted hydrocele of the cord
There is a smooth oval swelling near the spermatic cord which is liable to be mistaken for an inguinal hernia. The swelling moves
downwards and becomes less mobile if the testis is pulled gently downwards
Hydrocele of the canal of Nuck is a similar condition. It occurs in females and the cyst lies in relation to the round ligament. Unlike a
hydrocele of the cord, a hydrocele of the canal of Nuck is always at least partially within the inguinal canal.
Postherniorrhaphy hydrocele
Postherniorrhaphy hydrocele is a relatively rare complication of inguinal hernia repair. It is possibly due to interruption to the
lymphatics draining the scrotal contents.
Hydrocele of a hernial sac
Hydrocele of a hernial sac occurs when the neck is plugged with omentum or occluded by adhesions.
Filarial hydroceles and chyloceles
Filarial hydroceles and chyloceles account for up to 80 percent of hydroceles in some tropical countries where the parasite is endemic.
Filarial hydroceles follow repeated attacks of filarial epididymo-orchitis. They vary in size and may develop slowly or very rapidly.
Occasionally the fluid contains liquid fat which is rich in cholesterol. This is due to rupture of a lymphatic varix with discharge of chyle
into the hydrocele. Adult worms of the Wuchereria bancrofti have been found in the epididymis removed at operation or at necropsy.
In long-standing chyloceles, there are dense adhesions between the scrotum and its contents. Filarial elephantiasis supervenes in a
small number of cases.
Acute tuberculous epididymitis should come to mind when the vas is thickened and there is little response to the usual antibiotics
Acute epididymo-orchitis of mumps
•• Develops in about 18 percent of males suffering from mumps, usually as the parotid swelling is waning
•• The main complication is testicular atrophy which may cause infertility if the condition is bilateral (which is not usual). Partial
atrophy is associated with persistent testicular pain. The epididymitis of mumps sometimes occurs in the absence of parotitis,
especially in infants. The epididymis and testis may be involved by infection with other enteroviruses and in brucellosis and
lymphogranuloma venereum.
Varicocele
•• Dilatation and tortuosity of the veins of the pampiniform plexus
•• Most commonly observed on the left
•• Development of a varicocele may be an indicator of left renal tumor because the left spermatic vein system drains into the renal
vein and obstruction at that point could produce dilatation of the veins of the left cord
•• There may be a heavy, dragging, aching sensation in the scrotal compartment
•• Bag of Worms
•• May be a feature of renal cancer
•• Negative transillumination test
•• Reducible
•• Cough impulse +
•• MC cause of surgically treated male infertility.
Hydrocele
The tunica vaginalis, derived from the peritoneum as the processus vaginalis at the time of testicular descent, is a secretory membrane.
Fluid is generated by the serous surface of the tunica vaginalis
•• Congenital hydrocele may follow failure of obliteration of the processus vaginalis, and fluid formed within the peritoneal cavity
may gravitate into the tunica vaginalis
•• There may sometimes be an associated palpable inguinal hernia
•• In older persons, hydrocele is frequently the result of epididymo-orchitis or trauma
•• Lord’s plication is used for hydrocele.
Prostate Gland
•• Shape: Chest-nut-shaped
•• Type: Fibromusculoglangular organ
•• Corresponding female organ: Paraurethral glands of Skene
•• Volume: 8-12 ml
•• Carcinoma arises from: Peripheral zone
•• BHP arises from: Periurethral zone
•• Fracture of the bony pelvis may often result in laceration and transection of the membranous urethra just distal to the prostate and
urinary extravasation as well as bleeding may displace the prostate and bladder superiorly
•• MC site of urethral cancer is also membranous urethra.
512 USMLE Step 2 CK Platinum Notes
Complications of Prostatectomy
•• Hemorrhage
•• Bladder perforation
•• Retrograde ejaculation/Impotence
•• Incontinence
•• Urethral stricture
•• Hyponatremia
•• Sepsis
•• Osteitis pubis
Phimosis
Adhesions between the foreskin and the glans penis may persist until the boy is 6 years of age or more, giving the false impression that
the prepuce will not retract. Rolling back the prepuce causes its inner lining to pout and the meatus comes into view. This condition
should not be confused with true phimosis in small boys where there is scarring of the prepuce which will not retract without fissuring.
In these cases, the aperture in the prepuce may be so tight as to cause urinary obstruction.
Paraphimosis
•• When the tight foreskin is retracted, it may sometimes be difficult to return and a paraphimosis results
•• In this condition, the venous and lymphatic return from the glans and distal foreskin is obstructed and these structures swell
alarmingly causing even more pressure within the obstructing ring of prepuce. Gangrene may occur
•• Ice bags, gentle manual compression and injection of a solution of hyaluronidase in normal saline may help to reduce the swelling
•• Such patients can be treated by circumcision if careful manipulation fails
•• A dorsal slit of the prepuce under local anesthetic may be enough in an emergency.
Balanoposthitis: Inflammation of the prepuce is known as posthitis; inflammation of the glans is balanitis. The opposing surfaces of
the two structures are often involved — hence the term balanoposthitis.
Chordee. Chordee (French = corded) is a fixed bowing of the penis due to hypospadias or, more rarely, chronic urethritis. Erection is
deformed and sexual intercourse may be impossible. Treatment is usually surgical.
514 USMLE Step 2 CK Platinum Notes
Peyronie’s Disease
•• Peyronie’s disease is a relatively common cause of deformity of the erect penis
•• On examination, hard plaques of fibrosis can be palpated in the tunica of one or both corpora cavernosa. The plaques may be
calcified. The presence of the unyielding plaque tissue within the normally elastic wall of the corpus cavernosum causes the erect
penis to bend, often dramatically, towards the side of the plaque
•• The etiology is uncertain, but it may be a result of past trauma — there is an association with Dupuytren’s contracture
•• Usually self-limiting.
Paget’s disease of the penis (syn. erythroplasia of Quérat) is ‘a persistent rawness of the glans like a long-standing balanitis
followed by cancer of the substance of the penis’ (Sir James Pager). Treatment is by circumcision, observation and excision if the lesion
does not resolve.
Buschke–Lowenstein tumor is uncommon. It has the histological pattern of a verrucous carcinoma. It is locally destructive and
invasive, but appears not to spread to lymph nodes or to metastasize. Treatment is by surgical excision.
Peyronie’s Disease
•• A localized induration of the fibrous investments of the penile shaft, first described by the French surgeon Peyronie
•• After age of 40 years usually
•• Penile fibromatosis
•• A firm fibrotic thickening of the ‘Fascia of the Corpora Cavernosa’ is observed, usually involving the dorsolateral aspects of the
penile shaft or the intracavernous septum between the corpora cavernosa
•• Histologically similar to Keloid or Dupuytren’s contracture
•• The fibrous plaques themselves may be painless
•• Often compromise of erectile capacity of the penis with ‘deviation of the penis on erection’ and pain as a consequence of
this derangement
•• Deviation of the penis that interferes with intromission and coitus
•• Progression is slow, and spontaneous remissions are observed
•• Nesbitt’s operation is done
Surgery 515
Priapism
•• Prolonged
•• Pathologic
•• Painful erection of the penis
•• In recognition of the Greek god of sexual excess, Priapus
•• Pelvic venous thrombosis predisposes to priapism, metastatic malignant diseases, leukemia, pelvic trauma, sickle-cell disease
or trait, trauma to the corpora, or spinal cord injury
•• Prompt recognition and therapy are essential because prolonged unrelieved priapism almost inevitably leads to subsequent
Permanent impotence from fibrosis of the corpora cavernosa.
Neurosurgery
Glioblastoma
•• Most common primary bone tumor
•• Prognosis : grave
•• Pseudopalisading pattern of tumor cells
Meningioma
•• Psammoma bodies
•• Arises from arachnoid cells
•• Second most common brain tumor in adults
Oligodendroglioma
•• Most common in frontal lobes
•• Fried egg appearance of tumor cells
Schwannomas
•• Origin from Schwann cells
•• Bilateral schwannomas found in NF2
•• Two patterns: Antoni A and Antoni B
Pituitary tumors
•• Secrete prolactin
•• Derived from Rathke’s Pouch
516 USMLE Step 2 CK Platinum Notes
CNS Tumors
•• MC site of CNS Lipoma Corpus Callosum
•• MC site of Germinoma Pineal Gland
•• MC site of Chordoma Clivus
•• MC source of Metastasis to Brain (females) Breast
•• MC source of Metastasis to Brain (males) Lung
•• Imaging of choice is: Contrast-enhanced MRI
•• Most specific test: Stereotactic needle biopsy
•• Bleeding brain tumors: Glioblastoma multiforme
Waterhouse-Friderichsen Syndrome
•• Massive bilateral adrenal cortical hemorrhage occurs in cases of fulminating meningococcal septicemia and in some cases of
streptococcal, staphylococcal or pneumococcal septicemia
•• Most cases occur in infants and young children, but it can happen in adults with severe hemorrhage or burns. The onset is
catastrophic, with rigors, hyperpyrexia, cyanosis and vomiting
•• Petechial hemorrhages into the skin which coalesce rapidly into purpuric blotches are a constant feature. Profound shock follows,
and before long the patient passes into coma
•• The condition is one of overwhelming sepsis that pursues a galloping course, death occurring in most cases within 48 hours of the
onset of symptoms unless correct treatment is given without delay.
These tumors usually affect children and produce symptoms of increased intracranial pressure (headache, vomiting, somnolence)
as well as visual disturbances
Calcification within the tumor is readily seen on CT
Transfrontal resection is the treatment of choice for craniopharyngioma
Craniopharyngiomas are generally resistant to radiation therapy.
Pituitary Apoplexy
Pituitary apoplexy follows sudden hemorrhage into or infarction of a pituitary tumor. Symptoms occur suddenly due to expansion of
blood within the sella and include severe headache, stiff neck, loss of vision, and extraocular nerve palsies
Secondary adrenal insufficiency may lead to hypotension and shock
Pituitary apoplexy most often occurs in an undiagnosed pituitary tumor but can appear during radiation therapy for pituitary tumors,
during anticoagulation, or after closed-head trauma
Acute pituitary apoplexy is a neurosurgical emergency that requires acute transsphenoidal decompression of the sella.
Sheehan’s Syndrome
Pituitary necrosis may occur rarely after postpartum hemorrhage and hypovolemia. The degree of subsequent hypopituitarism
reflects the extent of pituitary necrosis and may include adrenal insufficiency, hypothyroidism and amenorrhea. An inability to
breastfeed postpartum due to destruction of oxytocin containing neurons of the posterior pituitary is an early clue to this diagnosis.
Empty Sella Syndrome
Results from arachnoid herniation through an incomplete diaphragma sellae. This syndrome may occur in the absence of a
recognized pituitary tumor and is either primary due to a congenital diaphragmatic defect or secondary due to an injury to the
diaphragm by pituitary surgery, radiation, or infarction.
Primary empty sella syndrome occurs in obese, multiparous, hypertensive women who experience headaches but have no
underlying neurologic disorders. Pituitary function is usually normal, but occasionally PRL is increased and GH reserve is reduced.
Secondary empty sella syndrome is observed in patients with otherwise benign CSF hypertension and in patients with a loss of
pituitary function due to apoplexy or surgical therapy.
ICU Management: A to J
A : Asepsis/Airway
B : Bed sore/encourage Breathing/Blood pressure
C : Circulation/encourage Coughing/Consciousness
D : Drains
E : ECG
F : Fluid status
G : GI losses/Gag reflex
H : Head positioning/Height
I : Insensible losses
J : Jugular venous pulse
Pulmonary Aspiration
Aspiration of material from the alimentary tract into the airway can lead to two distinct complications: aspiration pneumonitis, which
is due to the chemical composition of the aspirate; and aspiration pneumonia, which is due to its bacterial content. The two can
coexist, with aspiration pneumonia as a complication of aspiration pneumonitis.
Aspiration Pneumonitis
Aspiration pneumonitis is caused by the aspiration of gastric contents, which are normally sterile. The consequences depend on the
nature of the aspirated material. If the pH of the aspirate is less than 2.5, it can cause a chemical burn to the airways. The syndrome was
first described by Mendelson.
The clinical features of dyspnea, cyanosis, and tachycardia usually appear within half-an-hour of aspiration, with rales and expiratory
wheezes on auscultation. Chest films reveal interstitial pulmonary edema. Arterial blood gases demonstrate hypoxia and hypercapnia.
The condition may rapidly deteriorate into respiratory failure and is associated with high mortality.
Aspiration Pneumonia
Aspiration pneumonia is usually due to inhalation of contents from the oropharynx that are normally at physiologic pH but contain
bacteria, particularly anaerobes. It is a common cause of postoperative pneumonia and is particularly associated with poor oral
hygiene and the prolonged use of nasogastric and endotracheal tubes.
Pulmonary Edema
Pulmonary edema in a postoperative patient may be due to left ventricular failure following cardiac complications or to injury to the
alveolar membrane as a result of sepsis, oxygen toxicity, and so forth, but it is generally the result of circulatory overload following
excessive administration of intravenous fluids. Circulatory overload may also follow absorption of fluids used for irrigation of the
peritoneum or hollow viscera, such as the bladder, or it may occur during mechanical bowel preparation, especially in those with a
history of cardiac disease.
Gas Gangrene
Results from the following clostridial species:
•• Clostridium welchii
•• Clostridium oedematiens
•• Clostridium septicum
•• Microscopy of wound exudate shows gram-positive bacilli
•• Anaerobic culture on blood agar shows hemolytic colonies (Clostridium welchii)
•• ‘Stormy’ clot reaction with litmus milk
•• Clostridium welchii also shows positive Nagler’s reaction, due to lecithinase reaction of alpha exotoxin
Clinical features
•• Patients are generally toxic and unwell
•• Often have features of shock, jaundice, haemolysis or acute renal failure
Local signs of gas gangrene include:
•• Myositis or myonecrosis
•• Gas formation with palpable crepitus
•• Mottled discoloration of the overlying skin
•• Plain X-ray often shows gas in the subcutaneous tissue and fascial plains
SURGICAL ONCOLOGY
Latest Tumor Markers
•• Alphafetoprotein •• Hepatocellular carcinoma
•• CEA •• Adenocarcinoma colon, pancreas, lung, ovary
•• PSA •• Prostate cancer
•• Neuron-specific enolase •• Small cell lung cancer, Neuroblastoma
•• LDH •• Lymphoma
•• Catoecolamines •• Pheochromocytoma
•• Beta 2 microglobulin •• Multiple myeloma
•• Lymphoma
•• Bladder tumor antigen •• Bladder tumor, UTI, Renal calculi
•• CA 27.29 •• Breast cancer
•• CA 72.4 •• Ovarian and pancreatic cancer
•• LASA–P (Lipid Associated Sialic Acid) •• Ovarian cancer
•• NMP 2 •• Bladder cancer
•• HCG •• Gestational trophoblastic disorders
•• CA 125 •• Ovarian cancer
•• Placental Alkaline Phosphatase •• Seminoma
•• S100 •• Melanoma, neural tumors
Also Remember
•• Intermediate filament •• Tumor
•• Keratin •• Carcinoma
•• Vimentin •• Sarcoma
•• Desmin •• Muscle
•• Neurofilaments •• Phechromocytoma, Neuroblastoma
•• Glial Fibrillary Acidic Protein (GFAP) •• Astrocytomas, Ependymomas
Types of Secondaries
Osteolytic Secondaries Osteoblastic Secondaries
•• Kidney •• Prostate, Seminoma
•• Lung •• Breast, Uterus, Ovary
•• Thyroid •• Carcinoid
•• GIT •• Osteosarcoma
Surgery 521
Pulsating Tumors
•• RCC secondaries
•• Osteosarcoma
•• Osteoclastoma
•• Sec from follicular Ca thyroid
Hirschsprung’s Disease
•• Modified Duhamel
•• Swenson’s
•• Soave’s
Subperichondrial Hematoma
Blunt trauma to the pinna causes a subperichondrial hematoma
When bleeding occurs between the cartilage and the perichondrium, the pinna becomes a reddish purple shapeless mass
Because the perichondrium carries the blood supply to the cartilage, the cartilage undergoes avascular necrosis if the hematoma is
present on both sides of the cartilage, and with time, the pinna becomes shriveled.
Hematoma may become organized and calcify, which produces the cauliflower ear characteristic of wrestlers and boxers
Cauliflower ear is Perichondritis in wrestlers.
INFECTIOUS DISEASES
External Otitis
External Otitis. Infection of the ear canal occurs in a:
•• Diffuse form involving the entire canal, termed otitis externa diffusa
•• A localized form due to furunculosis, termed otitis externa circumscripta.
Malignant External Otitis
•• An unusually virulent form of external otitis due to infection with P. aeruginosa occurs in immunosuppressed diabetics,
particularly elderly diabetics with poor metabolic control
•• It produces pain, purulent otorrhea, and hearing loss
•• Facial nerve is most common cranial nerve involved
•• Granulation tissue found
•• Petrositis
•• Labyrinthitis
•• Facial paralysis
•• Conductive and sensorineural hearing loss
•• Epidural abscess
•• Meningitis
•• Brain abscess
•• Lateral sinus thrombosis
•• Subdural empyema, and
•• Otitic hydrocephalus
•• Mastoid reservoir phenomenon seen
Cholesteatoma/Epidermosis/Keratoma
•• A cholesteatoma occurs when the middle ear is lined with stratified squamous epithelium (white, amorphous debris in the
middle ear)
•• The stratified squamous epithelium desquamates in this closed space
•• Basically a bone erosion.
•• The desquamated epithelial debris cannot be cleared and accumulates in ever-enlarging concentric layers. This debris serves as a
culture medium for microorganisms.
•• Cholesteatomas have the ability to destroy bone, including the tympanic ossicles, probably because of the elaboration of
collagenase
•• Usually found in apex of petrous temporal bone
•• Attic/posterior superior marginal region is usually involved.
•• Pars flaccida and marginal perforations are frequently associated with cholesteatomas
•• Lateral semicircular canal is the commonest to perforate.
•• The presence of a cholesteatoma greatly increases the probability of the development of a serious complication, such as
purulent labyrinthitis, facial paralysis, or intracranial suppurations. Intracranial infections include meningitis, brain abscess, lateral
sinus thrombosis, subdural empyema, and epidural abscess
•• Modified radical mastoidectomy is used for treatment
Complications of CSOM
•• Commonest complication is Mastoiditis
•• Commonest cause of brain abscess is CSOM
–– Luc’s abscess (Root of Zygomatic Process)
524 USMLE Step 2 CK Platinum Notes
•• Radical mastoidectomy: Here the middle ear, including the attic and the antrum, and the mastoid air cell area are converted
into one cavity that communicates with the exterior through the ear canal
•• Modified radical mastoidectomy: If the cholesteatoma lies superficial to the remnants of the tympanic membrane and ossicles,
a modified radical mastoidectomy can be performed. The modified radical mastoidectomy spares the tympanic membrane
remnants and ossicles and preserves the remaining hearing
•• It is the commonest operation done for CSOM.
•• H. influenzae used to be the most common cause of bacterial meningitis under 5 years
•• Since the introduction of vaccine, its occurrence has greatly decreased
•• Risk factors for H. influenzae include Otitis media, Sinusitis, Pharyngitis and other URTI
•• As such, its occurrence also is associated with significant sensorineural hearing loss in up to 20 percent of cases
•• It is recommended to give steroids immediately along with antibacterials to decrease hearing loss.
Otosclerosis
•• Autosomal dominant
•• Begins in ‘fissula ante fenestram.’
•• Common in females
•• Family history positive
–– The most common cause of a progressive conductive hearing loss usually bilateral in an adult with a normal eardrum
–– Reversible conductive deafness seen
–– Otosclerosis is a disease of the bone of the otic capsule, with a predilection for the anterior part of the oval window
–– Flamingo pink tympanic membrane seen
–– But in majority, TM is normal (kindly remember)
–– Paracusis willisii (ability to hear better in noisy environment)
–– Positive Schwartz test
–– Carhart’s notch at 2000 Hz present
–– Gelle’s test is done for otosclerosis
–– Stapedectomy is surgery of choice
–– Sodium fluoride is of benefit
–– Used in pinkish tympanic membrane
–– Other operations done are: Fenestration and stapedectomy.
•• The patient experiences vertigo when lying on or rolling over onto the affected ear or when tilting the head back to look up.
There is a latency of a few seconds after assuming the provocative position before the vertigo and nystagmus begin
•• The vertigo is characterized by an intense sensation of spinning, and the nystagmus is rotary and counterclockwise when the
affected right ear is placed under and clockwise when the affected left ear is placed under
•• The quick component of the nystagmus is always toward the affected ear
•• The finding of basophilic calcium-containing concretions in the ampulla of the posterior semicircular canal has led some to refer
to this condition as cupulolithiasis.
Bell’s Palsy
Bell’s palsy is a unilateral facial paralysis that develops suddenly and is accompanied by pain in the postauricular area (LMNL).
It is thought to be of viral etiology.
All divisions of the nerve are paralyzed; this distinguishes the disease from a supranuclear lesion. The lesion is in the internal auditory meatus
or the intratemporal course of the nerve. The initial pathologic changes are hyperemia and edema
•• Acute onset
•• Spontaneous remission
•• Increased predisposition in diabetes mellitus
The edema compresses the blood supply to the nerve because of the bony confines of the fallopian canal. A conduction block
develops without death or degeneration of the axons. Release of the pressure on the nerve produces rapid recovery of function. This
type of paralysis is termed neurapraxia.
Corticosteroid therapy is initiated as soon as possible after the onset of the paralysis and is continued for 10 days to minimize the
inflammatory reaction.
It is an indication for decompression of the facial nerve by removing the bone of the fallopian canal. Approximately 85% of all
patients with idiopathic facial nerve paralysis recover spontaneously.
Chemodectomas
These nonchromaffin paragangliomas are termed glomus jugulare or glomus tympanicus tumors, depending on their site of origin.
The glomus tympanicus tumor arises from the area of Jacobson’s nerve in the tympanic plexus on the promontory of the
middle ear.
The glomus jugulare tumor arises from the glomus jugulare body in the jugular bulb. Both tumors consist of rich networks of vascular
spaces surrounded by epithelioid cells
Usually the neoplasms grow slowly, and symptoms may not be evident until the neoplasm is quite large
•• Pulsatile tinnitus
•• Facial nerve paralysis
•• Otorrhea
•• Hemorrhage
•• Vertigo
•• Paralysis of cranial nerves IX, X, XI, and XII are often the presenting symptoms and signs.
Characteristically, a ‘red mass’ that pulsates and blanches with compression with a pneumatic otoscope can be seen in the ear
canal or middle ear (Brown sign).
The pulsation can also be demonstrated with tympanometry. There may be evidence of bone erosion in the mastoid process, middle
ear, or petrous pyramid on CT. The extent of the lesion is best demonstrated with MRI with enhancement with gadolinium. Angiography
is a necessary part of the preoperative evaluation.
•• Arises from glomus bodies in the jugular bulb of internal jugular vein
–– Red reflex
–– Rising sun appearance
–– Bluish reflex through tympanic membrane is a feature
•• Earliest symptom is deafness and tinnitus
•• Deafness is of conductive type
•• Vertigo and cranial nerves IX and XII may get involved
•• Multicentric with lymphatic metastasis
•• Modified radical mastoidectomy and excision of petrous temporal bone are standard procedures.
Memorize
•• AC > BC Normal
•• AC < BC (Negative Rinne) Conductive deafness
•• Sound lateralized to deaf ear (Weber)
•• Absolute bone conduction (ABC) normal
•• AC > BC Sensorineural deafness
•• Sound lateralized to better ear
•• Absolute bone conduction
•• (ABC) Shortened
528 USMLE Step 2 CK Platinum Notes
Other Syndromes
•• Crouzon
•• Apert
•• Klippel-Feil
•• Down
•• Goldenhar
•• Pierre Robinson
•• van der Hoeve
•• Stickler
•• Wilder Ranch Syndrome
Presbycusis
•• Presbycusis (age-associated hearing loss) is the most common cause of sensorineural hearing loss in adults
•• In the early stages, it is characterized by symmetric, gentle to sharply sloping high-frequency hearing loss
•• With progression, the hearing loss involves all frequencies.
Surgery 529
•• More importantly, the hearing impairment is associated with significant loss in clarity
•• There is a loss of discrimination for phonemes, recruitment (abnormal growth of loudness), and particular difficulty in
understanding speech in noisy environments
•• Hearing aids may provide limited rehabilitation once the word recognition score deteriorates below 50
•• Significant advancements and improvements in cochlear implants have made them the treatment of choice.
Ear Signs
•• Brown Sign—blanching of redness on increasing pressure more than systemic pressure seen in Glomus jugulare
•• Hitselberger’s sign: In Acoustic neuroma—loss of sensation in the ear canal supplied by Arnold’s nerve (branch of vagus nerve
to ear)
•• Light house Sign—seeping out of secretions in acute otitis media
•• Lyre’s Sign—splaying of carotid vessels in carotid body tumor
•• Milian’s Ear Sign—Erysipelas can spread to pinna (cuticular affection), whereas cellulitis cannot
•• Tragus Sign—External otitis, Pain on pressing tragus
•• Waquino’s Sign is the blanching of the tympanic mass with gentle pressure on the carotid artery. Seen in Glomus tumors.
•• Bezold’s Sign: Inflammatory edema at the tip of the mastoid process in Mastoiditis.
Mucormycosis
•• Rhizopus and Rhizomucor species are ubiquitous, appearing on decaying vegetation, dung, and foods of high sugar content
•• Mucormycosis is uncommon and is largely confined to patients with serious pre-existing diseases
•• Mucormycosis originating in the paranasal sinuses and nose predominantly affects patients with poorly controlled diabetes
mellitus, organ transplantation, who have a hematologic malignancy, or who are receiving long-term deferoxamine
therapy
•• Vascular invasion by hyphae is a prominent feature
•• Ischemic or hemorrhagic necrosis is the foremost histologic finding.
Surgery 531
•• Mucormycosis originating in the nose and paranasal sinuses produces a characteristic clinical picture. Low-grade fever, dull
sinus pain, and sometimes nasal congestion or a thin, bloody nasal discharge are followed in a few days by double vision,
increasing fever and obtundation
•• Examination reveals a unilateral generalized reduction of ocular motion, chemosis, and proptosis. The nasal turbinates on the
involved side may be dusky red or necrotic
•• A sharply delineated area of necrosis, strictly respecting the midline, may appear in the hard palate. The skin of the cheek may
become inflamed. Fungal invasion of the globe or ophthalmic artery leads to blindness
•• Opacification of one or more sinuses is detected by computed tomography (CT) or by magnetic resonance imaging (MRI)
•• Carotid arteriography may show invasion or obstruction of the carotid siphon. Coma is due to direct invasion of the frontal lobe.
Septal hematomas lie between the quadrangular cartilage and the perichondrium. When the perichondrium has been elevated from
both sides of the septal cartilage, the cartilage undergoes avascular necrosis. Septal hematomas frequently become infected and abscess
formation produces avascular and septic necrosis of the septal cartilage, which causes a saddle deformity of the nose. Septal
hematomas are incised and drained as soon as the diagnosis is made
Septal abscesses are located between the cartilage and the perichondrium. They may involve both sides of the cartilage. Septal
abscesses are incised and drained under general anesthesia as soon as the diagnosis is established. Incisions are made bilaterally if
there is pus on both sides of the septum.
Epistaxis
Ninety percent of the time, epistaxis occurs from a plexus of vessels (Kiesselbach’s plexus) in the anteroinferior part of the septum
(Little’s area)
Which is an area of anastomosis between ICA and ECA
Arteries contributing to Little’s area:
•• Sphenopalatine
•• Anterior ethmoidal
•• Superior labial
•• Greater palatine artery
Silver nitrate is preferred as the cauterizing agent, since it produces satisfactory intravascular coagulation without a severe burn of
the mucous membrane.
Nose pricking is the commonest cause of epistaxis in children
Hypertension is the commonest cause of epistaxis in adults
Mc cause of epistaxis in pubescent male: Angiofibroma
Mc cause of epistaxis in pubescent female: Bleeding disorder
Mc cause of epistaxis in children: Trauma
A particularly debilitating form of epistaxis occurs in hereditary hemorrhagic telangiectasia (Rendu-Osler-Weber disease).
Patients with this disease have frequent bleeding from the nose and gastrointestinal tract.
Congenital Malformations
Choanal Atresia. Choanal atresia is a malformation in which the opening of the nasal cavity into the nasopharynx is obstructed
by a partition of mucous membrane and bone
The malformation may occur unilaterally or bilaterally
If it occurs bilaterally, it produces respiratory distress in the neonate
Newborn infants are obligatory nasal breathers. If there is obstruction to the nasal airway, asphyxia occurs
The newborn presses his tongue against the roof of his mouth during the inspiratory effort. Fortunately, crying, with its attendant
mouth breathing, often allows some ventilatory exchange
This diagnosis should be made in the delivery room
Choanal atresia should be considered in an infant who makes respiratory effort but fails to accomplish ventilatory exchange
The immediate solution to the problem is the insertion of an oral airway
ORAL CAVITY
Ludwig’s Angina
•• Edema of floor of mouth
•• Involves submandibular, sublingual spaces
•• Bilateral usually is a rapidly spreading, life-threatening cellulitis of the sublingual and submandibular spaces that usually starts in an
infected lower molar. Patients are febrile and may drool the secretions they cannot swallow
•• A brawny, board-like edema in the sublingual area pushes the tongue up and back
•• Airway obstruction may result as the infection spreads to the supraglottic tissues
Vincent’s Angina
•• Vincent’s angina, also called acute necrotizing ulcerative gingivitis or trench mouth
•• These have halitosis and ulcerations of the interdental papillae
•• Oral anaerobes are the cause
•• Therapy with oral penicillin plus metronidazole or with clindamycin alone is effective in both this condition and gingivitis.
Ranula
•• Sublingual
•• Thin-walled, retention cyst
•• Due to obstruction of mucous glands
534 USMLE Step 2 CK Platinum Notes
Cancrum oris
•• Rapidly developing gangrene in oral cavity
•• Postmeasles usually
•• Does not involve jaw
•• Noma, or cancrum oris, is a fulminant gangrenous infection of the oral and facial tissues that occurs in severely malnourished and
debilitated patients and is especially common among children. Beginning as a necrotic ulcer in the gingiva of the mandible,
noma is caused by oral anaerobes, especially fusospirochetal organisms (e.g. Fusobacterium nucleatum)
•• It is treated with high-dose penicillin, debridement, and correction of the underlying malnutrition.
Premalignant Conditions
•• Leukoplakia
•• Erythroplakia
•• Chronic candidiasis
•• Papillomas tongue/cheek
•• OSMF (oral submucous fibrosis)
•• Surgery treatment of choice
•• Radiosensitive.
However, from a functional point of view, the pharynx remains united by the constrictors of the pharynx. They have a common
insertion in the median pharyngeal raphe and form a musculomembranous tubular passage from the base of the skull to the opening
of the esophagus.
Waldeyer’s ring: The lymphoid structures of the pharynx include the pharyngeal tonsil or adenoid the palatine tonsils, the Tubal
bands and the lingual tonsils.
Surgery 535
Foreign Bodies
Foreign bodies of the pharynx are likely to be found in four locations: the palatine tonsils, the lingual tonsils, the valleculae, and the
pyriform sinuses
•• Foreign bodies in the palatine tonsil are removed by grasping the foreign body with a hemostat
•• Foreign bodies in the nasopharynx require general anesthesia for their removal
•• Foreign bodies of the hypopharynx are removed during direct laryngoscopy under local anesthesia.
Nasopharynx
Adenoids
•• Nasopharyngeal tonsil
•• No crypts and no capsule
•• Adenoid hypertrophy in childhood often leads to obstruction of the eustachian tubes and the choanae. Nasal obstruction, nasal
discharge, sinusitis, epistaxis and voice change
•• Adenoid facies:
–– Elongated face with dull expression
–– Open mouth
–– Prominent crowded upper teeth
–– Hitched up upper lip
•• This lymphoid hyperplasia may be physiologic or secondary to infectious and allergic manifestations
•• Obstruction of the eustachian tubes leads to serous or secretory otitis media, recurrent acute otitis media, and exacerbations of
chronic otitis media
Tornwaldt’s Cyst
•• Cysts occasionally form in the region of the medial recess of the nasopharynx. These cysts become symptomatic when they
become infected
•• There may be persistent purulent drainage that has a foul taste and odor
•• Symptoms of eustachian tube obstruction and sore throat may be prominent
•• Excision or marsupialization of the cyst with an adenotome is the treatment of choice.
536 USMLE Step 2 CK Platinum Notes
Nasopharyngeal Carcinoma
Lymphoepithelioma or squamous cell carcinoma is the most common type.
Carcinoma of the nasopharynx occurs at relatively young ages, and there is an unusually high incidence among the Chinese
Bimodal age distribution seen
Mc site is lateral wall of nasopharynx (Fossa of Rosenmuller)
Mc cranial nerve involved is VII CN.
Epstein-Barr virus is implicated.
Cervical lympadenopathy is the commonest presentation
Others present with nasal or eustachian tube obstruction
Obstruction of the eustachian tube may cause a middle-ear effusion
Serous otitis media seen.
The nasal obstruction may be associated with purulent, bloody rhinorrhea and frank epistaxis. The more dramatic symptoms
caused by cranial nerve paralysis and cervical lymph node metastasis are, unfortunately, common presenting complaints.
The diagnosis is made by biopsy of the primary tumor
Trotter’s Triad
•• Conductive deafness
•• Palatal paralysis
•• Temporoparietal neuralgia is seen.
The treatment of choice for carcinoma of the nasopharynx is irradiation with a supervoltage source. The radiation should be
delivered to the primary tumor-bearing area of the nasopharynx and to both sides of the neck, whether or not there is clinically
demonstrated metastasis. Operations have no role in the initial therapy of carcinoma of the nasopharynx. Those cervical metastases
that remain clinically palpable following radiation therapy or that subsequently become apparent should be eradicated by radical
neck dissection.
Peritonsillar Abscess/Quinsy
Peritonsillar abscess called Quinsy
MC organism: Streptococcus.
Surgery 537
Peritonsillar cellulitis and abscess are complications of acute tonsillitis in which the infection has spread deep to the tonsillar capsule
Pus forms between the tonsillar capsule and the superior constrictor of the pharynx
The tonsil is displaced medially
The uvula becomes tremendously edematous and is displaced to the opposite side
The soft palate is very red and displaced forward
•• There is marked trismus due to irritation of the pterygoid muscles, and the head is held tilted toward the side of the abscess. It is
painful for the patient to talk and to swallow
•• Odynophagia
•• Hot potato voice
•• Referred otalgia via IX cranial nerve are a feature
Swallowing is so painful that the patient drools. The breath is foul-smelling
Peritonsillar cellulitis or abscess is rare in children under the age of 10 to 12 years and is usually caused by a group A beta-hemolytic
streptococcus or anaerobe
Tonsillectomy done usually 6 weeks after appearance.
Parapharyngeal Abscess
Parapharyngeal abscess may occur in infants and young children as well as in adults
•• The abscess is usually secondary to streptococcal pharyngitis or tonsillitis
•• Pus forms in the parapharyngeal space secondarily from the breakdown of lymphadenitis
•• The pus is located lateral to the superior constrictor of the pharynx and adjacent to the carotid sheath
•• The tonsil and soft palate may be displaced medially but there may be no inflammatory reaction in the pharynx
•• There is marked swelling in the anterior cervical triangle.
Retropharyngeal Abscess
Retropharyngeal abscess occurs in infants and young children and is rare after the age of 10 years
These infections are located between the constrictors of the pharynx and the prevertebral fascia
Dysphagia fever torticollis bulge in posterior pharyngeal wall are a feature.
Laryngomalacia
•• Mc cause of stridor in a newborn
•• Inspiratory stridor seen
•• Omega-shaped epiglottis
•• Reassurance is the treatment modality
Vocal Nodules. Vocal nodules are caused by using a fundamental frequency that is unnaturally low and using the voice too
loudly and too long
Vocal nodules are condensations of hyaline connective tissue in the lamina propria at the junction of the anterior one-third and the
posterior two-thirds of the true vocal cord. These nodules produce hoarseness and give the voice a breathy quality. In adults, these le-
sions are removed during direct laryngoscopy to restore the voice. However, it is necessary to begin voice therapy prior to surgical therapy,
because if the underlying misuse of the voice is not corrected, the nodules recur. In children, surgical removal is not usually necessary,
because the vocal nodules regress with voice therapy, which consists of voice rest, reduction in intensity and duration of voice production,
and elevation of the pitch.
Heimlich maneuver (abdominal thrust). In this maneuver, the operator places his arms around the choking individual from behind,
grasps the fist of one hand in the other hand, and brings both hands up in the subxiphoid area briskly to apply pressure to the
diaphragm. The pressure increases the intrathoracic pressure and may expel the foreign body. This should maneuver fail, an alternative
airway must be established by the prompt performance of a tracheostomy.
Laryngotracheobronchitis
Laryngotracheobronchitis is Croup
MC causative agent: Parainfluenza
Haemophilus influenzae is the most frequently isolated agent in bacterial croup, but Staphylococcus and Streptococcus may also cause
croup.
Painful croupy cough, hoarseness and stridor.
Narrowing of subglottic region on X-ray (Steeple sign)
Epiglottitis
H. influenzae type b is the predominant microorganism in epiglottitis
Epiglottitis or supraglottic laryngitis is more likely to cause abrupt and complete airway obstruction
MC cause of death is also complete airway obstruction
•• Child prefers sitting position (Tripod sign)
•• Lateral X-ray shows swollen epiglottis (Thumb sign)
•• Hospitalization with IV antibiotics, steroids, humidification and intubation are a sequence.
Surgery 539
Laryngoceles
Laryngoceles are epithelium-lined diverticula of the laryngeal ventricle and may be located internal or external to the
laryngeal skeleton
An internal laryngocele may displace and enlarge the false vocal cord and may cause hoarseness and airway obstruction
External laryngoceles pass through the thyrohyoid membrane and present as a mass in the neck over the thyrohyoid membrane
The mass rises with the larynx on swallowing. Internal and external laryngoceles may co-exist
Laryngoceles are more common in glassblowers, wind instrument musicians, and others who develop high intraluminal pressures.
Initially, laryngoceles are filled with air and expand and collapse with changes in the intraluminal pressure
They are expanded during the Valsalva maneuver. They appear as smooth, ovoid, air-filled masses on CT scans of the neck.
Papilomas of Larynx
•• HPV-implicated
•• Common in children and infants
•• Multiple
•• Recurrent
•• Treated by removal/Surgical excision
Premalignant Lesions
•• Leukoplakia
•• Keratosis
•• Smoking
•• Papilloma
•• Chronic laryngitis
Supraglottic carcinomas
Smoking is a common factor
Pain is the mc manifestation.
540 USMLE Step 2 CK Platinum Notes
Glottic cancers
Mc cancers
Hoarseness is the mc manifestation
Hoarseness is the earliest manifestation
Best prognosis
No lymphadenopathy usually
Subglottic lesions
•• Represent more advanced glottic carcinomas in which the neoplasm has secondarily invaded the subglottic area as well as the
supraglottic area
•• Metastasis to the same side is present in 50% of patients
•• Subglottic extension of the carcinoma requires a total laryngectomy and radical neck dissection
•• With thyroid lobectomy on the same side.
Fractures in Children
The periosteum is extremely strong in children.
Children’s bones are much more resilient and less brittle than those of adults.
Bending moments applied to the bone of a child may cause a Greenstick fracture, in which there is distraction of the cortex on the
convex side and compression of bone on the concave side. (Cortex intact)
542 USMLE Step 2 CK Platinum Notes
Implicated in:
•• Ape thumb deformity
•• Carpal tunnel syndrome
•• Pointing index
•• Loss of opposition and abduction of thumb
•• Pen test is positive in median nerve injury
Radial Nerve
•• Supplies Extensor Compartment of Arm and Forearm
•• Commonly involved in ‘Spiral groove/Radial groove’
•• Accompanied by Profunda Brachii vessels
•• Supplies:
–– BEAST Muscles (Brachioradialis, Extensors of wrist and fingers, Anconeus, Supinator, Triceps)
•• The lesions appear to be degenerative and the gelatinous material found in the center has a high hyaluronic acid content
•• Many ganglion cysts regres spontaneously or after needle aspiration of the contents. Recurrent ganglia or ganglia that are
cosmetically unacceptable to the patient can be surgically excised, but may recur after excision.
Dupuytren’s Contracture
•• Progressive
•• Painless
•• Puckering of skin of Palmar Fascia
•• Flexion of MCP joints of ring and little fingers
Colles’ Fracture
•• Fall on outstretched hands
•• Common in elderly women
•• Distal fragment displaced dorsally, angulated dorsally, supinated
•• ‘Dinner Fork’ Deformity
•• Commonest complication is ‘Stiffness of fingers’
Scaphoid Fracture
•• Scaphoid is one of the carpal bones which undergo fracture commonly as well as avascular necrosis
•• Avascular necrosis of ‘proximal’ fragment is seen
•• Injury occurs by fall on outstretched hands
•• MC site of injury is ‘waist’
•• Tenderness in ‘Anatomical snuff box’ may be seen
•• Best radiological view is Oblique view
•• In absence of radiological findings, suspect scaphoid fracture
•• Most common site is between proximal 1/3 and distal 2/3
546 USMLE Step 2 CK Platinum Notes
Nerve Injuries
•• Neurapraxia: Complete recovery possible
•• Axonotmesis: Rupture of nerve fiber with intact sheath
•• Neurotmesis: Rupture of nerve fiber and sheath
–– Best recovery is seen in pure motor nerve (radial nerve)
–– In mixed nerve, recovery is poor because sensory and motor fibers may unite with each other.
Remember
•• Commonest type of hip dislocation: posterior
•• Flexion, adduction and internal rotation are a feature
•• Posterior dislocation can be seen on per rectal examination
Important Terms
•• Tennis Elbow is Medial Epicondylitis (Affects Common Flexor Origin)
•• Golfer’s Elbow is Lateral Epincondylitis (Affects Common Extensor Origin)
•• Student’s elbow is Olecranon bursitis
•• Painful arc syndrome is Supraspinatus Tendinitis in which patient can’t abduct his arm beyond 30° without pain
•• Frozen shoulder: Active movement at shoulder is not possible at all.
Spondylosis
•• It is degeneration of Intervertebral disk
•• Spondylolysis is a bony defect in the pars interarticularis (a segment near the junction of the pedicle with the lamina) of the
vertebra
Spondylolisthesis
•• It is the anterior slippage of the vertebral body, pedicles, and superior articular facets, leaving the posterior elements behind
•• Spondylolisthesis is associated with spondylolysis and degenerative spine disease and occurs more frequently in women
•• The slippage may be asymptomatic but may also cause low back pain, nerve root injury (the L5 root most frequently), or
symptomatic spinal stenosis
•• A ‘step’ may be present on deep palpation of the posterior elements of the segment above the spondylolisthetic joint
•• The trunk may be shortened and the abdomen protuberant as a result of extreme forward displacement of L4 on L5 in severe
degrees of spondylolisthesis
•• Spondylolisthesis is slipping of one vertebrae over other
•• Beheaded Scottish Terrier sign is seen
•• Beheaded Scottish Terrier sign is seen in oblique view.
Remember Eponyms
•• Marble bone disease •• Osteopetrosis
•• Spotted bone disease •• Osteopoikilosis
•• Stripped bone disease •• Osteopathia striata
•• Candle bone disease •• Melorheostosis
Perthes Disease
•• Is chronic disease with ischemia of upper end of femur causing ‘avascular necrosis’
•• Osteochondritis of femoral head
•• Boys are affected more
•• Adduction of limb is unaffected
•• Not obese
•• Pain is the initial and most common complaint
•• Limping is the associated feature
•• MRI is the investigation of choice
Coxa vara
Term used to describe a hip in which angle between neck and shaft of femur is less than 125°
Osteochondritis
1. Perthes disease •• Osteochondritis of Femoral head
2. Panner’s disease •• Osteochondritis of Capitulum
3. Kienbock’s disease •• Osteochondritis of Lunate
4. Osgood-Schlatter disease •• Osteochondritis of Tibial tubercle
5. Sever’s disease •• Osteochondritis of Calcaneum
6. Kohler’s disease •• Osteochondritis of Navicular
7. Frieberg’s disease •• Osteochondritis of Metatarsal head
8. Scheuermann’s disease •• Osteochondritis of vertebrae
Diaphyseal Metaphyseal
•• Fibrous dysplasia and other fibrous tumors of Bone
•• Fibrosarcoma
•• Fibroxanthoma
•• Chondromyxoid fibroma
•• Fibrous cortical defect
•• Nonossifying fibroma
Ewing’s Sarcoma
•• Affects children < 15 years usually
•• Diaphysial Tumor
•• Common in second decade
•• Onion skin lesions on X-ray
•• Translocation (t; 11:22)
•• Mimics Osteomyelitis
•• PAS Positive material (Glycogen) found
•• Most radiosensitive bone tumor
Osteosarcoma
•• Second most common primary malignant bone marrow
•• Metaphyseal tumor
•• Affects knee joint
•• MC site lower end of femur
•• Sunburst appearance
•• Codman’s triangle
•• Presents with secondaries in lungs with pneumothorax
•• Pulsating bone tumor
•• Chemotherapy + limb salvage surgery used for treatment
Osteoclastoma
•• Epiphyseal tumor
•• Soap bubble appearance
•• Breech of cortex
•• Lower end of Femur is most common site
Chondrosarcoma
•• Metaphyseal Tumor
•• Dense Punctate Calcifications
•• Associated with endosteal scalloping
Multiple Myeloma
•• This malignant neoplasm of marrow cell origin is the ‘most common primary malignant neoplasm’ of bone.
•• Myeloma is often encountered in the late decades of life and is seen more frequently in males than in females.
•• The symptoms vary from local pain and discomfort to systemic symptoms of
–– Anemia
–– Fever
–– Hypercalcemia
–– Renal failure related to extensive skeletal involvement
–– Abnormal production of immunoglobulins
•• Myeloma characteristically produces lytic destruction of bone with little if any reactive bone formation
•• These lesions are described as ‘geographic or punched out’ in appearance.
•• The absence of reactive bone formation is borne out by the observation that myeloma lesions are often silent on radionuclide
scanning
•• Pathologic fracture is often the presenting symptom.
•• The diagnosis of myeloma is made by marrow aspiration or the demonstration of abnormal plasma cells at the site of bony
destruction.
•• Fifty percent of patients with disseminated disease have Bence Jones protein in their urine
•• Serum electrophoresis demonstrates an abnormal amount of immunoglobulins.
•• Urine electrophoresis is also helpful when the diagnosis is in doubt and may demonstrate abnormal proteins in patients with a
normal serum electrophoretic pattern.
•• Microscopically, myeloma produces sheets of plasma cells. Usually these are well-differentiated cells in which the characteristic
arrangements of nuclear chromatin can be recognized with ‘Russel Bodies’.
Chondroblastoma
•• Chondroblastoma is a benign cartilaginous tumor
•• Chondroblastoma is epiphyseal tumor
Chondroblastoma is characterized by
•• Multi nucleate giant cells
•• PAS positive reaction
•• ‘Dense punctuate/stippled or mottled calcification with sclerotic rim’
Most common site is proximal humerus
Adamantinoma
•• Epithelial tumor from dental epithelium
•• Mc sites: Mandible, Tibia
•• Diaphyseal tumor
•• Slowly growing, locally invasive benign, cystic lesion
•• Occurs in late adolescence/middle age
•• ‘Bubble defect’ in anterior tibial cortex is a feature
Osteoid Osteoma
•• One of the commonest benign bone tumor
•• Severe Pain at night responding to NSAIDS
•• Mc site: Femur and tibia
Paget’s Disease
Paget’s disease is a focal disorder of bone remodeling that results in increased bone turnover and abnormal bone architecture.
Patients are often asymptomatic but may present with gross skeletal abnormalities (bowing, long-bone fractures, increased skull
circumference), deafness, nerve compression syndromes, or simply an abnormally elevated alkaline phosphatase. Serum calcium and
phosphorus levels are usually normal
•• Paget’s Disease is also called Osteitis Deformans.
Surgery 555
Differentiate
•• Paget’s disease of nipple: Which is associated with underlying ductal malignancy of breast
•• Paget’s disease of vulva: Which is not associated with underlying malignancy but can cause vulval cancer
•• Paget’s recurrent fibroid: A spindle cell sarcoma of low potential occurring in abdominal wall or thighs
•• Paget’s cells: Upward spread of breast cancer cells into epidermis which are mucin positive.
Fibrous Dysplasia
•• Fibrous dysplasia is a tumor like lesion
•• It is benign
•• It is developmental in origin and hormone dependent
•• Pathology is replacement of bone by fibrous tissue
•• X-ray shows ‘Ground glass’ appearance and ‘Chinese lettering of bone’
•• Rarely osteosarcoma can occur in fibrous dysplasia
•• Sexual precocity in girls with polyostotic fibrous dysplasia and cutaneous pigmentation constitutes ‘McCune Albright Syndrome’
•• Fibrous dysplasia and McCune-Albright syndrome represent a phenotypic spectrum of disorders caused by activating mutations
in the GNAS1 gene
Radiologic Changes
The roentgenographic appearance of the lesions is that of a radiolucent area with a well-delineated, smooth or scalloped border,
typically associated with focal thinning of the cortex of the bone
Fibrous dysplasia can cause bones to become larger than normal, a feature characteristic of Paget’s disease as well.
The ‘ground-glass’ appearance is due to the thin spicules of calcified woven bone.
556 USMLE Step 2 CK Platinum Notes
Septic Arthritis
It is microbial invasion of the synovial space. Pathogens enter the synovial space by either hematogenous spread, local spread from
contiguous infection, or a traumatic or surgical infection of the joint space. Accumulating fluid and pus rapidly raise the intra-articular
Pressure and permanently injure vessels and articular cartilage. More than 90% of cases of septic arthritis affect the joints of the lower
extremity, with the knee most commonly involved. Acute septic arthritis is bacterial, with Staphylococcus.
Surgery 557
Osteomyelitis
•• Osteomyelitis is the term used to denote infection of bone
•• Earliest site of bone involvement is in Metaphysis
•• The majority of bone infections are the result of Staphylococcus aureus
•• The isolated necrotic bone within the abscess cavity is called a sequestrum
•• New bone formation occurring about the periphery of the abscess represents the body’s attempt to wall off the infection. This new
bone is called involucrum
•• In some instances, a static abscess cavity remains without further enlargement. This equilibrium between host and organism is
known as a Brodie’s abscess in long bones
•• Young children may demonstrate the phenomenon known as pseudoparalysis. These children refuse to move the involved limb,
mimicking a neurologic deficit
•• Radiographs: It takes some time for osteomyelitic lesions to appear on radiographs
•• The lesion of osteomyelitis appears on radiograph in not less than two weeks
•• As a result alternative diagnostic modalities are used to pick-up osteomyelitis in the form of MRI and Bone scans
•• Most cases of acute osteomyelitis are due to Staph Aureus and MRSA strains are common nowadays
•• It is not advisible to wait for radiological changes to appear in case of acute osteomyelitis and then to start treatment.
Diagnosis
1. ESR Increased
2. CRP Increased
3. TLC Increased
4. MRI (90% sensitive 95% specific)
5. Bone scans 60% sensitive and 33% specific)
6. Blood cultures positive in 65% cases (Confirmatory)
•• Early diagnosis of acute osteomyelitis is critical because prompt antibiotic therapy may prevent the necrosis of bone
•• The evaluation usually begins with plain radiographs because of their ready availability, although they frequently show no
abnormalities during early infection
•• In 95% of cases, the technetium radionuclide scan using 99mTc diphosphonate is positive within 24 h of the onset of
symptoms. Falsely negative scans usually indicate obstruction of blood flow to the bone. Because the uptake of technetium
reflects osteoblastic activity and skeletal vascularity, the bone scan cannot differentiate osteomyelitis from fractures, tumors,
infarction, or neuropathic osteopathy
•• Ga citrate- and 111In-labeled leukocyte or immunoglobulin scans, which have greater specificity for inflammation, may help
distinguish infectious from noninfectious processes and indicate inflammatory changes within bones that for other reasons are
already abnormal on radiography and technetium scanning
•• Ultrasound can be used to diagnose osteomyelitis by the detection of subperiosteal fluid collections, soft tissue abscesses
adjacent to bone, and periosteal thickening and elevation
•• MRI is as sensitive as the bone scan for the diagnosis of acute osteomyelitis because it is able to detect changes in the water
content of marrow. MRI yields better anatomic resolution of epidural abscesses and other soft tissue processes than CT and is
currently the imaging technique of choice for vertebral osteomyelitis.
Types of Sequestrum
•• Ring Sequestrum •• Amputation stumps
•• Tubular Sequestrum •• Osteomyelitis (Hematogeneous)
•• Rice grain Sequestrum •• Tuberculosis
Skeletal Tuberculosis
•• In bone and joint disease, pathogenesis is related to reactivation of hematogenous foci or to spread from adjacent paravertebral
lymph nodes
•• Weight-bearing joints (spine, hips, and knees ¾ in that order) are affected most commonly
•• Spinal tuberculosis (Pott’s disease or tuberculous spondylitis) often involves two or more adjacent vertebral bodies
•• Upper thoracic spine is the most common site of spinal tuberculosis in children
•• The lower thoracic and upper lumbar vertebrae are usually affected in adults. From the anterior superior or inferior angle of the
vertebral body, the lesion reaches the adjacent body, also destroying the intervertebral disk
•• With advanced disease, collapse of vertebral bodies results in kyphosis (gibbus)
•• A paravertebral ‘cold’ abscess may also form
Surgery 559
•• In the upper spine, this abscess may track to the chest wall as a mass; in the lower spine, it may reach the inguinal ligaments or
present as a psoas abscess
•• Pathologic Considerations
•• The histologic appearance of the tuberculous lesion in bone resembles that observed in visceral tuberculosis. Histiocytes, Langhans
giant cells, and fibroblastic proliferation are all present
•• Caseous necrosis is less frequently seen in joint lesions than in pulmonary lesions
•• The destruction produced by granulomatous inflammation is characteristically slow
•• Within the joint, invasion of bone tends to occur at the margins where synovium is attached to bone, producing a characteristic
marginal defect
•• As destruction proceeds, the joint becomes filled with necrotic products and fragments of articular cartilage, material called rice
bodies because of its resemblance to grains of rice
•• Computed tomography (CT) or magnetic resonance imaging (MRI) reveals the characteristic lesion and suggests its etiology,
although the differential diagnosis includes other infections and tumors
•• Aspiration of the abscess or bone biopsy confirms the tuberculous etiology, as cultures are usually positive and histologic
findings highly typical
•• Skeletal tuberculosis responds to chemotherapy, but severe cases may require surgery
Skeletal Actinomycosis
•• Actinomyces Israelii is the MC organism
•• Orocervicofacial form is the MC type of skeletal actinomycosis
•• ‘Lower jaw’ (Mandible ) is the commonest site
•• Induration and Sinus Formation are a feature
•• Pencillin G is the DOC
Rickets
•• Rickets is characterized by defective mineralization of bones and cartilage
•• Osteomalacia is defective mineralization of bones
•• Rickets is seen before closure of growth plates
Causes
•• Nutritional rickets: vit D deficiency, Malabsorption
•• Accelerated loss of vit D: Phenytoin, Rifampicin, Barbiturates
•• Impaired hydroxylation in liver and Kidney
•• Liver disease, Hypoparathyroidism, Renal failure, Renal Tubular Acidosis
•• Vit D Resistant rickets, Fanconi’s syndrome, Wilson’s disease.
Clinical Features
Failure of calcification of cartilage and osteoid.
Epiphyseal enlargement occurs (thickening of knees, ankles, wrists)
•• Craniotabes
•• Genu valgum
•• Coxa vara
•• Short stature
•• Protrubent abdomen
560 USMLE Step 2 CK Platinum Notes
Radiographic Features
•• Thickening and widening of epiphysis
•• Cupping and fraying of metaphysis
•• Irregular metaphyseal margins
•• Flaring of anterior ends of ribs
•• Bowing of diaphysis
Biochemical
•• Serum calcium: normal or low
•• Serum phosphate: low
•• Alkaline phosphatase: high
•• PTH: high
Osteopetrosis
Osteopetrosis is also called:
•• Albers Schonberg disease or
•• Marble bone disease
•• It is characterized by
–– Anemia
–– Hepatoslpenomegaly
–– Infections
–– Pathological fractures
•• There is osteoclast dysfunction
•• Radiographically there is increased opacity of bones
•• Endobones (bone within a bone) is seen
Osteoporosis
Osteoporosis is defined as a reduction of bone mass (or density) or the presence of a fragility fracture. This reduction in bone tissue
is accompanied by deterioration in the architecture of the skeleton, leading to a markedly increased risk of fracture
Several noninvasive techniques are now available for estimating skeletal mass or density
These include:
•• Dual-energy X-ray absorptiometry (DXA)
•• Single-energy X-ray absorptiometry (SXA)
•• Quantitative computed tomography (CT)
•• Ultrasound
The diagnosis of Osteoporosis is made when a patient has characteristic osteoporotic fracture. In absence of such a fracture, evaluation
of osteoporosis is done by T score measurement based on bone mass.
Glucocorticoid-Induced Osteoporosis is by
•• Inhibition of osteoblast function and potential increase in osteoblast apoptosis, resulting in impaired synthesis of new bone
•• Stimulation of bone resorption, probably as a secondary effect
•• Impairment of the absorption of calcium across the intestine, probably by a vitamin D-independent effect
Surgery 563
•• Increase of urinary calcium loss and induction of some degree of secondary hyperparathyroidism
•• Reduction of adrenal androgens and suppression of ovarian and testicular secretion of estrogens and androgens
•• Potential induction of glucocorticoid myopathy, which may exacerbate effects on skeletal and calcium homeostasis, as well as
Increase the risk of falls.
•• Extra-articular Manifestations As a rule, these manifestations occur in individuals with high titers of autoantibodies to the Fc
component of immunoglobulin G (rheumatoid factors).
•• Rheumatoid nodules develop in 20 to 30% of persons with RA.
•• Clinical weakness and atrophy of skeletal muscle are common. Muscle atrophy may be evident within weeks of the onset of RA.
•• Rheumatoid vasculitis Neurovascular disease presenting either as a mild distal sensory neuropathy or as mononeuritis
multiplex may be the only sign of vasculitis. Cutaneous vasculitis usually presents as crops of small brown spots in the nail
beds, nail folds, and digital pulp. Larger ischemic ulcers, especially in the lower extremity, may also develop. Myocardial infarction
secondary to rheumatoid vasculitis has been reported, as has vasculitic involvement of lungs, bowel, liver, spleen, pancreas, lymph
nodes testes. Renal vasculitis is rare.
•• Pleuropulmonary manifestations, which are more commonly observed in men, include pleural disease, interstitial fibrosis,
pleuropulmonary nodules, pneumonitis, and arteritis
•• Clinically apparent heart disease attributed to the rheumatoid process is rare, but evidence of asymptomatic pericarditis is found
at autopsy in 50% of cases.
•• RA tends to spare the central nervous system directly, although vasculitis can cause peripheral neuropathy. Neurologic
manifestations may also result from atlantoaxial or midcervical spine subluxations. Nerve entrapment secondary to proliferative
synovitis or joint deformities may produce neuropathies of median, ulnar, radial (interosseous branch), or anterior tibial
nerves.
•• The rheumatoid process involves the eye in fewer than 1% of patients. Affected individuals usually have long-standing disease and
nodules. The two principal manifestations are episcleritis, which is usually mild and transient, and scleritis, which involves the
deeper layers of the eye and is a more serious inflammatory condition. Histologically, the lesion is similar to a rheumatoid nodule
and may result in thinning and perforation of the globe (scleromalacia perforans).
•• Felty’s syndrome consists of chronic RA, splenomegaly, neutropenia, and on occasion, anemia and thrombocytopenia.
•• Osteoporosis secondary to rheumatoid involvement is common and may be aggravated by glucocorticoid therapy.
•• RA in the Elderly Aggressive disease is largely restricted to those patients with high titers of rheumatoid factor. By contrast,
elderly patients who develop RA without elevated titers of rheumatoid factor (seronegative disease) generally have less severe,
often self-limited disease
•• The presence of rheumatoid factor does not establish the diagnosis of RA as the predictive value of the presence of rheumatoid
factor in determining a diagnosis of RA is poor. Thus fewer than one-third of unselected patients with a positive test for
rheumatoid factor will be found to have RA. Therefore, the rheumatoid factor test is not useful as a screening procedure
A number of additional autoantibodies may be found in patients with RA, including:
•• Antibodies to filaggrin
•• Antibodies to citrulline
•• Antibodies to calpastatin
•• Antibodies to components of the spliceosome (RA-33), and an unknown antigen, Sa.
Surgery 565
Goals of therapy of RA
•• Relief of pain
•• Reduction of inflammation
•• Protection of articular surfaces
•• Maintenance of function
•• Control of systemic involvement
Gout
•• Metatarsopharyngeal joints involved
•• Usually great Toe. (Podagra)
•• Assymetric joint involvement usually
•• Negatively birefringent crystals seen
•• NSAIDS, colchicines, Allopurinol used in treatment
Psoriatic Arthritis
•• Distal Interphalangeal Joints usually involved
•• Rheumatoid-like polyarthritis
•• Asymmetrical oligoarthritis: typically affects hands and feet
566 USMLE Step 2 CK Platinum Notes
•• Sacroilitis
•• DIP joint disease
•• Arthritis mutilans (severe deformity fingers/hand, ‘telescoping fingers’)
Joint Involvement
•• PIP joint involvement •• RA, OA, Psoriatic arthritis
•• PIP and DIP •• OA, Psoriatic arthritis
•• PIP and DIP and MCP and wrist •• Psoriatic arthritis
Ankylosing spondylitis
•• Typically a young man who presents with lower back pain and stiffness
•• HLA 27 association
–– Stiffness is usually worse in the morning and improves with activity
–– Enthesopathy is characteristic
–– Bamboo spine
–– Squaring of vertebrae
–– Pain improves with exercise
–– Peripheral arthritis (25%, more common if female)
Other features – the ‘A’s
•• Apical fibrosis
•• Anterior uveitis
•• Aortic regurgitation
•• Achilles tendonitis
•• AV node block
•• Amyloidosis and cauda equina syndrome
X-rays are often normal early in disease, later changes include:
•• Sacroilitis: Subchondral erosions, sclerosis
•• Squaring of lumbar vertebrae
•• ‘Bamboo spine’ (late and uncommon)
Association of HLA-B27
Osteoarthritis/OA
•• Involves hip, knee usually
•• Disease of elderly and obese usually
Surgery 567
Remember
•• Epiphyseal widening is a feature of Rickets
•• Epiphyseal dysgenesis is a feature of Hypothyroidism
•• Epiphyseal enlargement is a feature of Juvenile Rheumatoid Arthritis
Hemophilic Arthropathy
•• Hemophilia is a sex-linked recessive genetic disorder characterized by the absence or deficiency of factor VIII (hemophilia A,
or classic hemophilia) or factor IX (hemophilia B, or Christmas disease)
•• Hemophilia A is more common type
•• Spontaneous hemarthrosis is a common problem with both types of hemophilia and can lead to a chronic deforming arthritis
•• The frequency and severity of hemarthrosis are related to the degree of clotting factor deficiency
•• Hemarthrosis becomes evident after 1 year of age, when the child begins to walk and run
•• In order of frequency, the joints most commonly affected are the knees, ankles, elbows, shoulders, and hips. Small joints of the
hands and feet are occasionally involved.
•• Squaring of patella is a feature
•• In the initial stage of arthropathy, hemarthrosis produces a warm, tensely swollen, and painful joint. Widening of the femoral
intercondylar notch, enlargement of the proximal radius, and squaring of the distal end of the patella are seen
•• The patient holds the affected joint in flexion and guards against any movement
•• Blood in the joint remains liquid because of the absence of intrinsic clotting factors and the absence of tissue thromboplastin in
the synovium
•• The blood in the joint space is resorbed over a period of a week or longer, depending on the size of the hemarthrosis
•• Recurrent hemarthrosis leads to the development of a chronic arthritis
•• Bleeding into muscle and soft tissue also causes musculoskeletal disorders
•• When bleeding involves periosteum or bone, a pseudotumor forms. These occur distal to the elbows or knees in children and
improve with treatment of the hemophilia.
Remember
•• Iliotibial tract syndrome: Synovium deep to iliotibial tract is inflamed where it rubs on lateral femoral condyle. Common in
marathon runners
•• Medial shelf syndrome: Synovial fold above medial meniscus is inflamed
•• Fat pad syndrome: Tenderness deep to patellar tendon may be caused by fat caught in tibiofemoral joint.
Scoliosis
Associations
•• Hemivertebrae
•• Wedge vertebrae
•• Block vertebrae
Surgery 569
•• Unsegmented bar
–– Turnbuckle cast scoliosis
–– Risser’s cast scoliosis
–– Milwaukee’s brace scoliosis
–– Boston brace scoliosis
•• Cobb’s angle is measured in scoliosis
Important Diseases
•• Burton’s Disease Combination of scurvy and rickets
•• Engelmann Disease Progressive diaphyseal dysplasia
•• Trevor’s Disease Dysplasia epiphysis hemimelica
•• Caffey’s Disease Osteomyelitis of Jaw
•• De Quervain’s Disease Tenovaginitis of APL and EPB (tendon)
•• Blount’s Disease Tibia vara
572 USMLE Step 2 CK Platinum Notes
NEWER TOPICS
Pycnodysostosis
•• Pycnodysostosis is an autosomal recessive form of osteosclerosis that superficially resembles osteopetrosis
•• It is a form of short-limbed dwarfism associated with bone fragility and a tendency to fracture with minimal trauma
•• Life span is usually normal
•• In addition to a generalized increase in bone density, features include:
–– Short stature
–– Separated cranial sutures
–– Hypoplasia of the mandible
–– Kyphoscoliosis and deformities of the trunk
–– Persistence of deciduous teeth
–– Progressive acro-osteolysis of the terminal phalanges
–– High, arched palate
–– Proptosis
–– Blue sclerae
–– A pointed, beaked nose
•• Patients usually present because of frequent fractures
•• (Remember the clinical presentation)
Osteomyelosclerosis
•• In osteomyelosclerosis, the marrow cells are replaced by diffuse fibroplasia, occasionally accompanied by osseous metaplasia
and increased skeletal density on roentgenograms
Surgery 573
Melorheostosis
•• This rare, sporadic condition usually begins in childhood and is characterized by a slowly progressive linear hyperostosis in one
or more bones of one limb
•• Usually in a lower extremity
•• All segments of the bone may be involved, with sclerotic areas that have a ‘Candle Dripping Wax’ or ‘flowing’ distribution
•• The involved limb is often extremely painful.
Osteopoikilosis
Osteopoikilosis is characterized by dense spots of trabecular bone <1 cm in diameter, usually of uniform density, located in the
epiphyses and adjacent parts of the metaphyses
All bones may be involved except the skull, ribs, and vertebrae.
Stress Fractures
•• Stress fractures are undisplaced fatigue fractures, which develop as a result of repeated loading. The lower limbs are most
frequently affected
•• The metatarsals (march fractures) and the tibia being the most frequently involved bones
•• March fracture involves 2, 3 metatarsals most commonly
•• Younger children and even toddlers can present with stress fractures but they are more common in adolescents where the proximal
third of the tibia is the most frequent site
•• Symptoms of localized pain develop insidiously and are usually relieved by rest
•• Treatment involves protection from further trauma, which sometimes involves immobilization and always involves
abstinence from the causative stress, usually running. Once symptoms have resolved, a gradual return to activity can be begun.
Enthesitis-related Arthritis
Enthesitis is defined as inflammation of the insertion of tendons into bones. This new diagnostic group is intended to define those
with disease related to the HLA antigen B27, and avoids the term ‘juvenile spondyloarthropathy’; inaccurate because of the rarity of
spinal involvement in children
Enthesitis-related arthritis is defined as arthritis and enthesitis; or arthritis; or enthesitis with at least two of the following features:
1. Sacroiliac joint tenderness and/or inflammatory spinal pain
2. Presence of HLA B27
3. Family history in at least one first- or second-degree relative of medically confirmed HLA B27-associated disease
4. Anterior uveitis that is usually associated with pain, redness, or photophobia
5. Onset of arthritis in a boy after 8 years of age
Ehlers–Danlos syndrome
•• Ehlers-Danlos syndrome (EDS) consists of a group of disorders of connective tissue characterized by:
•• Joint hypermobility plus fragility and laxity of the skin
•• These conditions are characterized by abnormalities in collagen genes resulting in the production of abnormal collagen and
consequent tissue fragility
•• EDS has soft, hyperextensible skin, ‘cigarette paper’ scars, easy bruising and marked hypermobility. EDS type II is similar to
type I but less severe
•• Also seen is serious involvement with a high incidence of rupture of the arteries, the colon or the pregnant uterus. Management
of EDS consists of patient education and support, with genetic counseling for the more severe forms.
Synovial chondromatosis
•• It is a disorder characterized by multiple focal metaplastic growths of normal-appearing cartilage in the synovium or tendon
sheath. Segments of cartilage break loose and continue to grow as loose bodies
•• When calcification and ossification of loose bodies occur, the disorder is referred to as synovial osteochondromatosis
•• The disorder is usually monoarticular and affects young to middle-aged individuals. The knee is most often involved, followed
by hip, elbow and shoulder
•• Symptoms are pain, swelling, and decreased motion of the joint
•• Radiographs may show several rounded calcifications within the joint cavity.
•• Treatment is synovectomy; however, the tumor may recur.
•• An infant presents with failure to thrive, Pancytopenia. Bones are brittle and fracture •• Infantile osteopetrosis
easily. Defect is found in osteoclasts
•• A two-year-old presents with multiple fractures in different stages of healing brought by •• Battered baby syndrome
overconcerned parents
Surgery 575
•• A six-year-old with b/l symmetrical fractures and blue sclera. Bones are osteopenic and •• Osteogenesis imperfecta
brittle
•• An athelete after trauma and hematoma formation at elbowcomes to you with hard •• Myositis ossificans
calcific mass found in the muscle Belly
•• A patient with multiple injuries on day 2 develops Tachycardia, Tachypnea, ↓Po2 and •• Fat embolism
rash.
•• A patient after hip replacementdevelops chest pain and cvs collapse •• Pulmonary thromboembolism
Clinical Scenarios
•• An itching, squeezing pain sensation after amputation that an amputated limb is still •• Phantom limb pain
attached and functioning with intensly painful sensations
•• Complex and progressive disease with pain swelling and changes in skin without •• Reflex sympathetic dystrophy/
demonstrable nerve lesions after injury Sudeck’s atrophy
•• Complex and progressive disease with pain swelling and changes in skin with •• Causalgia
demonstrable nerve lesions after injury
•• A 13-year-old boy with a hot, tender swelling arising from diaphysis of bone with •• Ewing’s sarcoma
characteristic feature of onion skinning
•• An obese boy in 2nd decade presents with pain and with limited flexion, abduction and •• Slippped femoral epiphysis
femoral head seen downwards and posteriorly
•• A 10-year-old boy with pain and limp. X-ray shows distorted femoral neck and head •• Perthes’ disease
•• An elderly boy after trauma presents with expansile osteolytic and cystic bony lesion with •• Aneurysmal bone cyst
thin wall
•• An elderly man presents to orthopedic clinic with pain in bones. On X-ray, cortical •• Melorheostosis
widening of bones. ‘Molten wax’ or ‘Candle Dripping Wax’ appearance of bones is
noted
•• A male presents with triad of lytic bony lesions on skull, diabetes Insipidus and exomphalos •• Hand-schuller-christian disease
576 USMLE Step 2 CK Platinum Notes
2. Ortner’s syndrome
3. Ollier’s syndrome
4. Holt-Oram syndrome
5. Ogilvie’s syndrome
Ans. 5. Ogilvie’s syndrome
•• Omphalocele and gastroschisis result in evisceration of bowel and require emergency surgical treatment to effect immediate or
staged reduction and abdominal wall closure.
•• Patent urachal or omphalomesenteric ducts result from incomplete enclosure of embryonic connections from the bladder and
ileum, respectively, to the abdominal wall. They are appropriately treated by excision of the tracts and closure of the bladder or
ileum. In most children, umbilical hernias close spontaneously by the age of 4 and need not be repaired.
3. Lumbar stenosis
4. Myopathy
5. Normal pressure hydrocephalus (NPH)
Ans. 3. Lumbar stenosis
It is caused by degenerative changes in the lumbosacral spine. The history is that of vague low back pain with subtle physical
examination findings referable to impingement on motor and sensory roots.
3. GI malignancy
4. Gastritis
5. Hirschsprung’s disease
6. GERD
Ans. 6. GERD
3. Mallory-Weiss syndrome
4. GI malignancy
5. Gastritis
6. Hirschsprung’s disease
7. GERD
Ans. 4. GI malignancy
3. Ischiorectal abscess
4. Pelvic abscess
5. Ischemic colitis
6. Pseudomembranous colitis
Ans. 3. Ischiorectal abscess
3. Pseudomembranous colitis
4. Volvulus
5. Intussusception
6. Colonic atresia
7. Annular pancreas
Ans. 4. Volvulus
4. Diverticulitis
5. Hepatitis
6. Cholangitis
7. Pseudoobstruction
Ans. 1. Renal colic
3. Gastrinoma
4. Insulionma
5. Pheochromocytoma
Ans. 1. Glucagonoma
4. Carcinoid tumors arise from the ectoderm, are a type of apudoma and the most common site is the large bowel.
5. Carcinoid tumors arise from the neuroectoderm, are a type of apudoma and the most common site is the large bowel.
Ans. 2. Carcinoid tumors arise from the neuroectoderm, are a type of apudoma and the most common site is the small bowel.
3. Dercum’s disease
4. De Quervain’s disease
Ans. 3. Dercum’s disease
3. Zone III
4. Zone IV
Ans. 2. Zone II
Zone I is the base of the neck from 2 cm above the clavicles to the level of the clavicles.
Zone II is the midcervical region from 2 cm above the clavicle to the angle of the mandible.
Zone III is the top of the neck from the angle of the mandible to the base of the skull.
3. The gastrohepatic and hepatoduodenal ligaments, which consist of the anterior layer of lesser omentum and are
continuous with the left triangular ligament. The hepatoduodenal ligament contains the hepatic arteries, portal vein,
and extrahepatic bile ducts. It forms the anterior boundary of the epiploic foramen of Winslow and the communication
between the greater and lesser peritoneal cavities.
4. Cecal volvulus
5. Sigmoid cancer
6. Right colonic cancer
Ans. 4. Cecal volvulus
3. Urgent
4. Deferrable
Ans. 1. Exigent
1. Exigent—the most life-threatening conditions, requiring instantaneous intervention (e.g. laryngeal fracture with
complete upper airway obstruction and tension pneumothorax).
2. Emergency—those conditions requiring immediate intervention, certainly within the first hour (e.g. ongoing hemorrhage
and intracranial mass lesions).
3. Urgent—those conditions requiring intervention within the first few hours (e.g. open contaminated fractures, ischemic
extremity and hollow viscous injuries).
4. Deferrable—those conditions that may or may not be immediately apparent but will subsequently require treatment (e.g.
urethral disruption and facial fractures).
•• Postoperative and postinjury cases of necrotizing soft tissue infection are most often caused by mixed bacterial species, including
aerobic and anaerobic pathogens, both gram-positive and gram-negative, a very similar spectrum to that seen in intra-abdominal
infections.
Pagets disease
OPHTHALMOLOGY
Ophthalmology 7
Anatomy of Eye
Muscles of eye and their innervations
•• The ‘Extorter’ of Eye Ball is Inferior Oblique and Inferior Rectus
•• The ‘Intorter’ of Eye ball is Superior Oblique and Superior Rectus
•• Action of Superior oblique is Abduction, Intorsion and depression
•• Dilator Pupillae dilates pupil and is supplied by Sympathetics
•• Sphincter Pupillae constricts pupil and is supplied by Parasympathetics
LR6S04
•• Lateral Rectus is supplied by 6th Cranial Nerve (Abducent)
•• Superior Oblique is supplied by 4th Cranial Nerve (Trochlear)
•• Rest other ocular muscles are supplied by 3rd Cranial Nerve (Occulomotor)
•• Levator palpabrae superior is supplied by 3rd caranial nerve
•• Muscle attached to posterior tarsal margin: Mullers muscle
•• Third nerve gets affected in a variety of syndromes of Midbrain such as:
–– Benedict’s Syndrome
–– Claude’s Syndrome
–– Weber’s Syndrome
–– Nothnagel’s Syndrome
Trigeminal nerve (Fifth Cranial nerve) is involved in afferent pathway of corneal reflex
Cornea
•• The cornea is quite unlike most tissues in that it is perfectly transparent
•• The epithelium consists of a thin layer of nonkeratinized stratified squamous
628 USMLE Step 2 CK Platinum Notes
Layers of Retina
•• Retinal pigment epithelium (RPE)
•• Layer of rod and cone cells outer segments
•• Outer limiting membrane
•• Outer nuclear layer
•• Outer plexiform layer
•• Inner nuclear layer
•• Inner plexiform layer
•• Ganglion cell layer
•• Nerve fiber layer
•• Internal limiting membrane
‘The Annulus of Zinn’, also known as the ‘Annular tendon’ or ‘Common Tendinous Ring’
It is a ring of fibrous tissue surrounding the optic nerve at its entrance at the apex of the orbit? It is the origin for five of the six
extraocular muscles
Annulus of Zinn (Annulus communis tendinis) is seen to transmit: (NAO)
•• Occulomotor nerve
•• Nasociliary nerve
•• Abducent nerve
Associated with
•• Optic atrophy
•• Papilledema
•• Neurofibromatosis
Angles of Eye
•• Alpha angle: Between visual axis and optical axis
•• Kappa angle: Between puppilary axis and visual axis
•• Visual angle: Angle subtended by object at nodal point of lens
Ophthalmological Tests
Direct Ophthalmoscopy Indirect Ophthalmoscopy
Condensing lens not required Condensing lens required
Examination close to patient Examination at a distance
Image is virtual and erect Image is real and inverted
Magnification is 15 times Magnification is 5 times
Stereopsis absent Stereopsis present
Area of field in focus is 2 disk diopters Area of field in focus is 8 disk diopters
Examination through hazy media not possible Examination through hazy media possible
Illumination not so bright Illumination bright is done for examination of periphery of retina up to orra seratta.
Ophthalmology 631
•• Regurgitation of pus
•• Treated by: Massage over Lacrimal sac, Lacrimal syringing, Probing, Intubation with silicone tube, Dacrosystorhinostomy
•• Dacrocystectomy
•• Conjunctivocystorhinostomy
Field Defects
‘Visual Field Defects’
The main points for the exam are:
•• Left homonymous hemianopia means visual field defect to the left, i.e. lesion of right optic tract
•• Homonymous quadrantanopias: PITS (Parietal-Inferior, Temporal-Superior)
•• Incongruous defects = optic tract lesion; congruous defects= optic radiation lesion or occipital cortex
Homonymous Hemianopia
•• Incongruous defects: Lesion of optic tract
•• Congruous defects: Lesion of optic radiation or occipital cortex
•• Macula sparing: Lesion of occipital cortex
Homonymous Quadrantanopia
•• Superior: Lesion of temporal lobe
•• Inferior: Lesion of parietal lobe
•• Mnemonic = PITS (Parietal-Inferior, Temporal-Superior)
Bitemporal Hemianopia
•• Lesion of optic chiasm
•• Upper quadrant defect > lower quadrant defect = inferior chiasmal compression, commonly a pituitary tumor
•• Lower quadrant defect > upper quadrant defect = superior chiasmal compression, commonly a craniopharyngioma.
Functionally, the retina is composed of 3 elements, i.e. three neurones carry the impulses in the retina ‘Itself’, these are:
1. The receptor cells (The rods and cones)
2. The bipolar nerve cells
3. The ganglion cells
•• The rods and cones synapse with the bipolar cells (in the retina)
•• The bipolar cells synapse with the ganglion cells (in the retina also)
•• The axons of the ganglion cells from the optic nerve which contains nasal and temporal fibers.
Ophthalmology 633
•• The optic nerves pass to the optic chiasma (which lies immediately in front of the pituitary gland)
•• The nasal fibers of the optic nerve cross to the opposite side while the temporal fibers remain on the same side
•• The part of the visual pathway which passes backwards from the chiasma is called the optic tract
•• Each optic tract carries «temporal» fibers of its «own» side and «nasal» fibers from the «opposite» side
•• Most of the fibers of the optic tract end in the lateral geniculate body while some fibers leave the optic tract to end in the superior
colliculus of the midbrain and in the pretectal nucleus.
The fibers of the optic tract which reach the superior Colliculus and the pretectal nucleus are concerned with the light reflex (i.e.
narrowing of the pupil in response to excess light) while those fibers of the optic tract which reach the lateral geniculate body from
part of the visual pathway which will reach the visual cortex
The optic radiation (or the geniculo-calcarine tract) [The pathway from the lateral geniculate body to the visual area of the cortex]
The lateral geniculate body (LGB) is the thalamic center for vision
The axons of the cells of the LGB form the «optic radiation» which passes in the internal capsule to reach to the «striate area» or the
Area 17 or the Visual Area in the occipital lobe
The optic radiation has fibers running downwards and forwards
The visual area
The «Visual Area» surrounds the calcarine fissure on the medial surface of the occipital lobe. It is formed of the cuneus and the lingual
gyrus.
Pituitary tumor
VISION
Color Vision is tested by
•• Ishihara plates
•• Hardy rand Rittler plates
•• Edridge green lantern test
•• City university test
•• Farnsworth Munsell 199 hue test
•• Nagels anamoloscope
•• Holmgrens wool test
Amaurosis
This term refers to a ‘transient ischemic attack of the retina.’
•• Because neural tissue has a high rate of metabolism, interruption of blood flow to the retina for more than a few seconds results
in transient monocular blindness, a term used interchangeably with amaurosis fugax
•• Patients describe a rapid fading of vision like a curtain descending, sometimes affecting only a portion of the visual field
•• Amaurosis fugax usually occurs from an embolus that becomes stuck within a retinal arteriole
•• Ophthalmoscopy reveals zones of whitened, edematous retina following the distribution of branch retinal arterioles
•• Complete occlusion of the central retinal artery produces arrest of blood flow and a milky retina with a cherry-red fovea
•• Emboli are composed of either cholesterol (Hollenhorst plaque), calcium, or platelet-fibrin debris
•• The most common source is an atherosclerotic plaque in the carotid artery or aorta, although emboli can also arise from the
heart, especially in patients with diseased valves, atrial fibrillation, or wall motion abnormalities
•• Retinal arterial occlusion also occurs rarely in association with retinal migraine, lupus erythematosus, anticardiolipin
antibodies, anticoagulant deficiency states (protein S, protein C, and antithrombin III deficiency), pregnancy, intravenous
drug abuse, blood dyscrasias, dysproteinemias, and temporal arteritis
•• Amaurosis fugax warns of a patient at high-risk for stroke
•• Marked systemic hypertension causes sclerosis of retinal arterioles, splinter hemorrhages, focal infarcts of the nerve fiber layer
(cotton-wool spots) and leakage of lipid and fluid (hard exudate) into the macula. In hypertensive crisis, sudden visual loss can
result from vasospasm of retinal arterioles and consequent retinal ischemia. In addition, acute hypertension may produce visual
loss from ischemic swelling of the optic disk.
OPTIC REFLEXES
The Light Reflex
(Constriction of the pupil in response to excess light)
This is a ‘protective’ reflex: the pupil becomes narrow to prevent excess light from injuring the rods and cones in the retina
Types: There are 2 types of light reflex — Direct and Indirect
Direct Pupillary Reflex:
Light falls on the right eye → constriction of the right pupil
indirect Pupillary Reflex: (Consensual light reflex)
Light falls on the right eye → constriction of the left pupil.
•• Edinger-Westphal parts of the oculomotor [III] nucleus of the Rt and Lt sides → (preganglionic fibers) in the oculomotor [III] nerves
(of both sides) → the ciliary ganglia (on both sides)
•• Each ciliary ganglion → (postganglionic fibers) in the short ciliary nerves → sphincter pupillae muscle → constriction of the pupil.
Light reflex
Accommodation Reflex
•• An object placed 6 meters or more from the eye considered a ‘far’ object and light rays falling on the eye from it are parallel and are
brought to focus on the retina. This is vision ‘at rest’
•• A ‘near’ object is placed nearer than 6 meters from the eye (e.g. a book). Light rays falling on the eye from a near object are
divergent. In order to bring them to focus on the retina the LENS MUST INCREASE ITS THICKNESS to have a stronger refractive
power. This is called accommodation. In order for the lens to increase its thickness, the ciliary muscle must contract; contraction of
the ciliary muscle → relaxation of the suspensory ligament of the lens → thicker and more powerful lens.
Pathway of Accommodation
•• Light from a near object → RETINA → optic nerve → optic chiasma → optic tract → Lateral geniculate body
(LGB) → optic radiation → AREA 17 of the occipital cortex
•• The first picture which falls on the eye does not fall in focus on the retina so the first picture which reaches the visual cortex is
blurred (not seen clearly). Area 17 now sends signal to area 19
•• In order to make the picture fall on the retina area 19 sends signal to the superior colliculus (in the midbrain) (by way of the
occipitotectal tract). The superior colliculus then sends orders to Edinger-Westphal nucleus of the third nerve → contractions
of the ciliary muscle → relaxation of the suspensory ligament → thicker and more powerful lens → picture NOW falls in
focus on the retina can be seen clearly when it reaches the visual area.
Corneal Reflex
Light touching of the cornea or conjunctiva results in blinking of the eyelids
On touching the cornea afferent impulses from the cornea or conjunctiva travel through the ophthalmic division of the trigeminal
nerve to the sensory nucleus of the trigeminal nerve. Internuncial neurons connect with the motor nucleus of the facial nerve on both
sides through the medial longitudinal fasciculus. The facial nerve which carries the efferent impulses supplies the orbicularis oculi
muscle, which causes closure of the eyelids.
Ophthalmology 637
Horner’s Syndrome
This results from paralysis of the sympathetic fibers at any site along its long course in the head, in the neck or mediastium as a result,
e.g. of neoplasm or trauma. It consists of:
•• Ptosis, which is partial: The lid may be slightly raised voluntarily
•• Enophthalmos
•• Miosis: Fixed construction, i.e. does not dilate on shading the eye
•• Anhydrosis: Loss of sweating on ipsilateral half of head and face when the lesion is proximal to sympathetic fiber separation along
the external carotid
•• Loss of the ciliospinal reflex
Causes:
•• Brainstem (vascular-syringobulbia)
•• Cervical cord (glioma- syringobulbia)
•• Damage to C8, T1
•• Cervical sympathetic chain damage by: pancoast tumor of lung.
Oculomotor Nerve: The third cranial nerve innervates the medial, inferior, and superior recti; inferior oblique; levator palpebrae
superioris; and the iris sphincter
Total palsy of the oculomotor nerve causes ptosis, a dilated pupil, and leaves the eye ‘down and out’ because of the unopposed
action of the lateral rectus and superior oblique
Injury to structures surrounding fascicles of the oculomotor nerve descending through the midbrain has given rise to a number of
classic eponymic designations
•• Nothnagel’s syndrome, injury to the superior cerebellar peduncle causes ipsilateral oculomotor palsy and contralateral
cerebellar ataxia
•• Syndrome, injury to the red nucleus results in ipsilateral oculomotor palsy and contralateral tremor, chorea and athetosis
•• Claude’s syndrome incorporates features of both the aforementioned syndromes, by injury to both the red nucleus and the
superior cerebellar peduncle
•• Weber’s syndrome, injury to the cerebral peduncle causes ipsilateral oculomotor palsy with contralateral hemiparesis
Trochlear Nerve: The fourth cranial nerve originates in the midbrain, just caudal to the oculomotor nerve complex
•• Fibers exit the brainstem dorsally and cross to innervate the contralateral superior oblique
•• The principal actions of this muscle are to depress and to intort the globe
•• Supplies superior oblique muscle
•• A palsy therefore results in hypertropia and excyclotorsion
•• The ‘head tilt test’ is a cardinal diagnostic feature
Ophthalmology 639
•• Blepharitis: This refers to inflammation of the eyelids. The most common form occurs in association with acne rosacea or
seborrheic dermatitis.
•• A chalazion is a painless, ‘granulomatous inflammation of a meibomian gland that produces a pealike nodule within the
eyelid. It can be incised and drained, or injected with glucocorticoids
•• Recurrent chalazion might indicate malignant transformation. (sebaceous cell carcinoma)
•• Basal cell, squamous cell, or meibomian gland carcinoma should be suspected for any ‘nonhealing, ulcerative lesion’ of the
eyelids
•• Basal cell carcinoma:
–– It is the most common tumor of eyelid
–– Lower lid is the most common site
Found in association with
–– Xeroderma pigmentosa
–– Gorlin Goltz syndrome
Ptosis
‘Ptosis’: Drooping of upper eyelids
Tylosis: hypertrophy and drooping of eyelids
•• Blepharoptosis: This is an abnormal drooping of the eyelid. Unilateral or bilateral ptosis can be congenital, from dysgenesis of the
levator palpebrae superioris, or from abnormal insertion of its aponeurosis into the eyelid
•• Mechanical Ptosis: This occurs in many elderly patients from stretching and redundancy of eyelid skin and subcutaneous
fat (dermatochalasis). The extra weight of these sagging tissues causes the lid to droop. Enlargement or deformation of the eyelid
from infection, tumor, trauma or inflammation also results in ptosis on a purely mechanical basis
•• Aponeurotic Ptosis: This is an acquired dehiscence or stretching of the aponeurotic tendon, which connects the levator
muscle to the tarsal plate of the eyelid. It occurs commonly in older patients, presumably from loss of connective tissue
elasticity. Aponeurotic ptosis is also a frequent sequela of eyelid swelling from infection or blunt trauma to the orbit, cataract
surgery, or hard contact lens usage
•• Myogenic Ptosis: The causes of myogenic ptosis include myasthenia gravis and a number of rare myopathies that manifest with
ptosis
•• In the Kearns-Sayre variant, retinal pigmentary changes and abnormalities of cardiac conduction develop. Peripheral muscle
biopsy shows characteristic ‘ragged-red fibers.’
•• Oculopharyngeal dystrophy is a distinct autosomal dominant disease with onset in middle age, characterized by ptosis, limited
eye movements, and trouble swallowing
•• Myotonic dystrophy, another autosomal dominant disorder, causes ptosis, ophthalmoparesis, cataract, and pigmentary
retinopathy. Patients have muscle wasting, myotonia and frontal balding and cardiac abnormalities
•• Neurogenic Ptosis: This results from a lesion affecting the innervation to either of the two muscles that open the eyelid: Muller’s
muscle or the levator palpebrae superioris
•• In Horner’s syndrome, the eye with ptosis has a smaller pupil and the eye movements are full
•• Patient with nonretractable ptosis gets corrected due to action like mimicking, chewingin marcus gunn jaw winking
syndrome
•• In oculomotor nerve palsy, the eye with the ptosis has a larger, or a normal, pupil. If the pupil is normal but there is limitation of
adduction, elevation, and depression, a pupil-sparing oculomotor nerve palsy is likely
•• Corrected by Blaskowicks operation.
Horner’s Syndrome
•• Ptosis
•• Miosis
•• Anhydrosis
640 USMLE Step 2 CK Platinum Notes
REFRACTIVE ERRORS
Myopia
•• Myopia is short sightedness
•• Rays of light focused in front of retina
•• Big eyeball is seen
•• Corrected by concave lens
•• Deep anterior chamber
•• Patient squeezes eyes to see distant things clearly
•• Myopic crescents (on temporal side)
•• Foster Fuchs spots
•• Cystoid Degeneration
•• Lattice and Snail track degeneration
•• Peripheral retinal degeneration
•• Posterior staphyloma
Hypermetropia
•• It is long sightedness
•• Rays of light focused behind retina
•• Corrected by convex Lens
•• Shallow anterior chamber
•• Eyeball and cornea are small
Presbyopia
•• Not an error of refraction
•• Weakness of ciliary muscles, ligaments
•• Loss of elasticity of lens capsule
•• Condition of physiological insufficiency of accommodation
•• Loss of power of accommodation is seen
•• Physiological change appropriate for age
•• Treatment is convex lens
Astigmatism
•• It is spherical aberration
•• Due to irregular curvature of cornea
•• Corrected by convex lens
Aphakia
It is absence of crystalline lens
•• Eye becomes hypermetropic
•• Accommodation is lost
–– Deep anterior chamber
–– Jet black pupil
•• Iridodenesis
•• Purkinje image test show only two test images
•• Post chamber IOL is the treatment of choice
•• Spectacles are prescribed to patient of aphakia after 6 weeks of surgery
Important Facts
–– ↑ inter pupillary distance: Hypertelorism
–– ↓ inter pupillary distance: Hypotelorism
–– ↑ inter canthal distance: Telecanthus
•• Convergent squint (esotropia) is due to 6th nerve palsy
•• Divergent squint (exotropia) is due to 3rd nerve palsy
•• In paralytic squint primary deviation < secondary deviation
•• Inconcomitant squint primary deviation = secondary deviation
Nystagmus
•• Down beat Nystagmus is seen in cerebellar/brainstem lesions, multiple sclerosis, alcohol, anticonvulsants and lithium intoxication
•• Upbeat Nystagmus is seen in pontine lesions
•• Vestibular Nystagmus is seen in Menier’s disease, Vestibular lesions
•• Rotatory Nystagmus: Miners Nystagmus
642 USMLE Step 2 CK Platinum Notes
Conjunctiva
Conjunctivitis and most frequently asked questions
Phylenticular Conjunctivitis
•• Type IV hypersensitivity reaction
•• Due to staph aureus (MC) and tuberculous protein
•• Koeppes nodule
•• Busaca nodules are manifestations
•• MC allergic manifestation is Koppes nodule
•• Phylycten is hence due to endogeneous causes
•• Doc : Topical steroids
Spring Catarrh
•• Recurrent, bilateral conjunctivitis
•• Horner Trantas spots on bulbar conjunctiva
•• More common in summer not spring
•• Burning and itching are common
•• Cobble stone appearance of palpebral conjunctiva
•• Pseudogerentoxon with cuspid bow appearance
•• Papillary hypertrophy
•• Type 1 Hypersensitivity
•• Maxwell lyon sign (stringy, ropy discharge)
•• Shield ulcer
•• DOC: steroids
Trachoma
•• Trachoma is a chronic Keratoconjunctivitis affecting conjunctiva and cornea simultaneously
•• It is caused by Chlamydia Trachomatis11 Serotypes (A-K)
•• It is one of the leading causes of preventable blindness.
Ophthalmology 643
Pinguecula:
•• It is a small, raised conjunctival nodule at the temporal or nasal limbus. In adults such lesions are extremely common and have
little significance, unless they become inflamed (pingueculitis)
•• It is hyaline infiltration and elastotic degeneration
Pterygium:
•• Resembles a pinguecula but has crossed the limbus to encroach upon the corneal surface. Removal is justified when symptoms
of irritation or blurring develop, but recurrence is a common problem
•• It is elastotic degeneration with proliferation of vascularized granulation tissue
•• Stocker’s line is seen
•• Mitomycin, thiotepa and β irradiation are used for treatment.
Glaucoma
•• Aqueous humor is produced by ciliary processes. At the rate of 2 µl/min
•• Aqueous humor has less protein than plasma
•• It maintains IOP. (Intraocular pressure)
•• It provides substrates and removes metabolites from cornea
•• It flows from posterior chamber to anterior chamber
•• Carbonic anhydrase is important in aqueous humor production
•• Conventional outflow is main pathway and uveoscleral outflow is minor pathway
‘Congenital Glaucoma’
•• Boys effected more
•• Bilateral
644 USMLE Step 2 CK Platinum Notes
•• Blepharospasm
•• Big, (Buphthalmos) ‘ox eyes’, Large cornea
•• Blue eyes
•• Presents as photophobia
•• Hazy frosted glass cornea due to edema with Habbs striae. ‘Habbs striae are due to breaks in descemet’s membrane where
corneal opacities appear as lines with double contours’
•• Lens flattened anteroposteriorly
•• Deep anterior chamber.
•• Goniotomy or Trabeculotomy is used for treatment.
•• Trabeculectomy is considered in case of failure of above procedures
•• Trabeculotomy and trabeculectomy (Combined) is preferential method of choice
•• ‘Hereditary glaucoma’ is due to optineurin abnormality.
Megalocornea:
•• Defined as diameter of cornea > 11.7 mm at birth or > 13 mm at age 2 years
•• Cornea usually clear with normal thickness and vision
•• Usually bilateral and nonprogressive
•• Associated with Marfan’s syndrome
Neovascular Glaucoma
Usually associated with neovascularization of Iris (Rubeosis iridis). Seen in association with:
•• Diabetic retinopathy
•• Eales disease
•• CRVO
•• Sickle cell retinopathy
•• CRAO
•• Intraocular tumors
•• Intraocular inflammation
•• Treated by Pan Retinal Photocoagulation
646 USMLE Step 2 CK Platinum Notes
Pigmentary glaucoma
•• It is a form of open angle glaucoma with AD pattern of inheritance with pigment disruption of iris and deposition in anterior
chamber
•• Corneal pigmentation (Krukenbergs spindle) is a feature.
•• Medical therapy (Topical beta blockers are the drugs of choice.) Timolol, betaxolol, cartelol
•• Carbonic anhydrase inhibitors (acetazolomide, dorzolamide)
•• Acetazolomide is used orally not topically
•• Prostaglandin analogues (latanoprost)
•• Pilocarpine and adrenergic agents. (Epinephrine and Dipivefrine)
•• Atropine is contraindicated
•• Timolol is a beta blocker
•• Argon or diode laser trabeculoplasty and trabeculectomy are surgical options
•• Carbonic anhydrase (CA) is an enzyme found in many tissues of the body including the eye. It catalyzes the reversible reaction
involving the hydration of carbon dioxide and the dehydration of carbonic acid.
•• In humans, carbonic anhydrase exists as a number of isoenzymes, the most active being carbonic anhydrase II (CA-II), found
primarily in red blood cells (RBCs), but also in other tissues.
•• Inhibition of carbonic anhydrase in the ciliary processes of the eye decreases aqueous humor secretion, presumably by slowing
the formation of bicarbonate ions with subsequent reduction in sodium and fluid transport. The result is a reduction in intraocular
pressure (IOP).
•• Latanoprost is a prostaglandin F2 analog that is believed to reduce IOP by increasing the outflow of aqueous humor. It is
indicated for lowering
•• IOP in patients with open-angle glaucoma and ocular hypertension who are intolerant to other agents.
Eponym Glaucoma’s
•• Hundred day glaucoma: Neovascular glaucoma
•• Hypersecretory glaucoma: Epidemic Dropsy
•• Ghost cell glaucoma: Vitreous Hemorrhage
•• Angle recession glaucoma: Blunt trauma
•• Glaucoma Fleckens: After Acute congestive glaucoma
•• Malignant glaucoma: Ciliary block glaucoma
Cornea
•• Dendritic ulcers are pathogonomic of herpes simplex virus
•• Acanthameba causes pseudodendritic corneal ulcers
•• Decreased corneal sensation is a feature
Band shaped keratopathy in cornea is due to deposition of calcerous salts after hyaline infiltration
Seen in:
•• Chronic uveitis
•• Hyperparathyroidism
•• Vitamin D toxicity
•• Sarcoidosis
Keratoconus
Keratoconus is protrusion of central part of cornea with thinning
Corneal nerves visible
Seen in association with
•• Downs
648 USMLE Step 2 CK Platinum Notes
•• Turner’s
•• Ehler-Danlos
•• Marfan’s
•• Osteogenesis imperfecta
•• Mitral valve prolapsed
•• Features:
–– Munson’s sign
–– Fleischer ring
–– Vogt lines
–– Oil droplet reflex are seen
Epiretinal Membrane
•• This is a fibrocellular tissue that grows across the inner surface of the retina, causing metamorphopsia and reduced visual
acuity from distortion of the macula
•• With the ophthalmoscope one can see a crinkled, cellophane-like membrane on the retina. Epiretinal membrane is most
common in patients over 50 years of age and is usually unilateral
•• Most cases are idiopathic, but some occur as a result of hypertensive retinopathy, diabetes, retinal detachment, or trauma
•• When visual acuity is reduced to the level of about 6/24 (20/80), vitrectomy and surgical peeling of the membrane to relieve
macular puckering are recommended. Contraction of an epiretinal membrane sometimes gives rise to a macular hole.
•• Vitrectomy may improve visual acuity in some patients with macular hole. Fortunately, fewer than 10% of patients with a macular
hole develop a hole in their other eye.
Important Indices
•• Power of lens: +16 D
•• Power of cornea: + 44 D cornea is the most important refractive surface of eye
•• Power of eye: + 60 D
•• Refractive index of air: 1
•• Refractive index of aqueous humor: 1.33
•• Refractive index of vitreous humor: 1.33
Ophthalmology 649
Lens
•• Lens is avascular
•• Lens has no nerve supply
•• Lens derives nutrition from aqueous humor, perilimbal capillaries and oxygen in air
•• Lens thicker anteriorly than posteriorly
•• Oldest cells are present in center and youngest cells in periphery
•• Crystallins and Major intrinsic protein MIP 26 enhance lens transparency
•• Glutathione, peroxidase, catalase, ascorbic acid, α tocopherol and β carotene are protective against free radicals.
Cataracts
Majority are age related and due to UV light
Systemic causes:
•• DM, Hypoglycemia, Hypocalcemia, Galactosemia, Hypothyroidism, Hypoparathyroidism, Alport’s, Down’s Syndrome, Dystrophia
myotonica, Lowe’s syndrome, Wilson’s Disease
•• Steroids, Chloroquin, Amidarone, Busulfan, Chlorambucil, Copper, Gold, Iron
•• Infection (Congenital rubella, CMV, Toxoplasmosis)
•• Smoking, HTN, Diabetes mellitus
Ocular causes:
•• Trauma
•• Radiation
•• High myopia
•• Atopic dermatitis, Icthyosis, Rothmund’s Syndrome, Wener’s syndrome (Dermatological causes)
•• Cataract is the most common cause of blindness in India.
Complicated Cataract
•• Complicated Cataract: Refers to opacification of the lens secondary to other intraocular disease
•• The opacity in complicated cataract is irregular in outline and variable in density
•• A slit lamp examination shows ‘Bread Crumb appearance’
•• A characteristic sign is the appearance of iridescent colored particles so called ‘Polychromatic lusture’ of reds, greens and blues
•• Posterior subcapsular cataract is visually handicapping and presents as marked diminution of vision
•• Here patient sees better in dark.
Cataract Surgeries
•• ICCE: (Intracapsular cataract extraction): Whole lens removed with intact capsule
•• ECCE: (Extracapsular cataract extraction): Lens removed, capsule left behind
•• Phecoemulsification: Whole lens removed with intact capsule
•• Phecoemulsification: Emulsifies lens nucleus and cortex by ultrasonic vibration
Lens Dislocation:
•• Inferior: Homocystinuria
•• Superior: Marfans
•• (Superotemporal)
•• Forward: Marchesani Weil Syndrome, Trauma, Hypermature cataract, Ehler-Danlos syndrome also cause lens Dislocation
Contact Lens
•• MC infection is pseudomonas infection
•• Most common predisposing factor for acanthameba is use of contact lens
•• Other complications related to use of contact lens:
–– Over wear syndrome
–– Giant papillary conjunctivitis
–– Allergic conjunctivitis
Papilledema
Causes: Raised ICT:
•• ICSOLS (Intracranial space occupying lesions), Postcranial fossa tumors
•• Intracranial infections (Cavernous sinus Thrombosis, cerebral abscess)
•• Malignant Hypertension, Toxemia of pregnancy
•• Pseudotumor cerebri (Tetracycline, Nalidixic acid, OCP, Vitamin A.)
•• Foster Kennedy Syndrome (Optic atrophy on same side and papilledema on other side)
•• Clinical features:
•• Headache, nausea, Projectile vomiting
–– Painless, Progressive loss of vision
–– Amourosis fugax
–– Enlargement of blind spot
–– Progressive contraction of visual field
–– Visual acuity and puppilary reaction remain normal until optic atrophy occurs
–– Leads to postneuritic ophthalmopathy
•• Axonal swelling
•• Stasis of axoplasmic flow
•• Extracellular edema are a feature
Papillitis/Optic Neuritis
Causes: Idiopathic plus:
•• Multiple sclerosis, Devices disease, Leucodystrophies, Postviral infections, Vitamin B complex deficiency
•• Diabetes
•• Syphilis
•• Drugs: Quinine, Chloroquine, Ethambutol, digitalis, INH, NSAIDS, Tobbaco, alcohol, arsenic
•• Pseudopapillitis is seen in hypermetropia.
652 USMLE Step 2 CK Platinum Notes
Features:
•• (Monocular) unilateral decrease in visual acuity over hours or days
•• Poor discrimination of colors, ‘Red Desaturation’
•• Inflammation of optic nerve
•• Pain worse on eye movement
•• Relative afferent pupillary defect/Marcus Gun pupil (EARLY)
•• Full afferent puppilary defect is seen in optic nerve lesion.
Defects:
•• Defective contrast sensitivity
•• Decreased color vision
•• Defective depth perception of moving objects (Pulfrich phenomenon)
•• Worsening of symptoms with exercise (Uhthoff Phenomenon)
•• Visual evoked potentials show latency and delay in amplitude
Macular Degeneration
•• This is a major cause of gradual, painless, bilateral central visual loss in the elderly
•• It occurs in a nonexudative (dry) form and an exudative (wet) form
•• The nonexudative process begins with the accumulation of extracellular deposits, called drusen, underneath the retinal pigment
epithelium
•• Exudative macular degeneration, which develops in only a minority of patients, occurs when neovascular vessels from the
choroid grow through defects in Bruch’s membrane into the potential space beneath the retinal pigment epithelium
•• Leakage from these vessels produces elevation of the retina and pigment epithelium, with distortion (metamorphopsia)
and blurring of vision
•• Verteporfin
•• Verteporfin is indicated for treatment of age-related macular degeneration in patients with predominantly classic subfoveal
choroidal neovascularization
•• Verteporfin is a photosensitizing agent composed of two isomers
•• It accumulates preferentially in neovasculature, including choroidal neovasculature
•• Activation of verteporfin by nonthermal light (689 nanometers wavelength) in the presence of oxygen generates highly reactive,
short-lived singlet oxygen and reactive oxygen radicals
•• The activated particles cause local damage to neovascular endothelium and subsequent vessel occlusion
•• Damaged endothelium releases procoagulant and vasoactive factors causing platelet aggregation, fibrin clot formation, and
vasoconstriction
•• These factors contribute to the temporary occlusion of choroidal neovascularization
•• Distribution: Verteporfin is transported in the plasma primarily by lipoproteins.
Lish nodules
•• Are melanocytic hamartomas of Iris
•• They are not seen on naked eye examination
•• They are seen on slit lamp examination
•• They are seen in Neurofibromatosis (von Recklinghausen’s disease)
Fundoscopy:
•• Chloroquine toxicity: Bulls Eye Maculopathy
•• CMV Retinitis: Pizza fundus, Tomato Ketchup Retinopathy
•• CRAO: Cattle trucking appearance
654 USMLE Step 2 CK Platinum Notes
Uveal Tract
Episcleritis
•• It is an inflammation of the episclera, a thin layer of connective tissue between the conjunctiva and sclera
•• Episcleritis resembles conjunctivitis but is a more localized process and discharge is absent
•• Most cases of episcleritis are idiopathic, but some occur in the setting of an autoimmune disease
Scleritis
•• Refers to a deeper, more severe inflammatory process, frequently associated with a connective tissue disease such as
•• Rheumatoid arthritis, (MC)
•• Lupus erythematosus, polyarteritis nodosa, Wegener’s granulomatosis, or relapsing polychondritis
•• The inflammation and thickening of the sclera can be diffuse or nodular
•• In anterior forms of scleritis, the globe assumes a violet hue and the patient complains of severe ocular tenderness and pain
•• With posterior scleritis the pain and redness may be less marked, but there is often proptosis, choroidal effusion, reduced motility
and visual loss
•• Episcleritis and scleritis should be treated with NSAIDs
•• If these agents fail, topical or even systemic glucocorticoid therapy may be necessary, especially if an underlying autoimmune
process is active.
Uveitis
Involving the anterior structures of the eye, this is called iritis or iridocyclitis
The diagnosis requires slit-lamp examination to identify inflammatory cells floating in the aqueous humor or deposited upon the
corneal endothelium (keratic precipitates)
Circum ciliary congestion
Cells and flare in aqueos are a feature.
•• Reiter’s syndrome
•• Behcet’s disease
It is also associated with herpes infections, syphilis, Lyme disease, onchocerciasis, tuberculosis and leprosy.
•• Treatment is aimed at reducing inflammation and scarring by judicious use of topical glucocorticoids. Dilation of the pupil by
atropinereduces pain and prevents the formation of synechiae
•• ‘Secondary glaucoma’ is the most common complication of recurrent anterior uveitis
•• ‘Pilocarpine’ and other cholinergics are contraindicated
•• ‘Steroids’ are the drug of choice followed by mydriatics
Posterior Uveitis
This is diagnosed by observing inflammation of the vitreous, retina, or choroid on fundus examination. It is more likely than anterior
uveitis to be associated with an identifiable systemic disease
Posterior uveitis is a manifestation of autoimmune diseases such as:
•• Sarcoidosis
•• Behçet’s disease
•• Vogt-Koyanagi-Harada syndrome
•• Inflammatory bowel disease
It also accompanies diseases such as toxoplasmosis, onchocerciasis, cysticercosis, coccidioidomycosis, toxocariasis and
histoplasmosis; infections caused by organisms such as Candida, Pneumocystis carinii, Cryptococcus, Aspergillus, herpes, and
cytomegalovirus; and other diseases such as syphilis, Lyme disease, tuberculosis, cat-scratch disease, Whipple’s disease, and brucellosis
In multiple sclerosis, chronic inflammatory changes can develop in the extreme periphery of the retina (pars planitis or intermediate
uveitis).
Vogt-Koyangi-Harada Syndrome
A disease of melanocyte containing tissue. (Soon)
•• Skin Changes: Vitiligo, Alopecia, Poliosis
•• Occular Changes: Uveitis, Retinal detatchment, Depigmented fundus, Dalen fuchs nodules
•• Otological Changes: Deafness, Vertigo, Tinitus
•• Neurological Changes: Meningitis, Encephalitis
656 USMLE Step 2 CK Platinum Notes
Sympathetic Ophthalmitis
•• Bilateral granulomatous panuveitis
•• Ciliary body is injured
•• After penetrating trauma in ciliary region. (Dangerous zone)
•• Keratatic precipitates or retrolental flares are IST signs
•• Traumatized eye is ‘Exciting eye’
•• Fellow eye is ‘Sympathizing eye’
•• Keratitic precipitates are seen early
•• ‘Dalen Fuchs nodule’ are characteristic
•• Early excision of injured eye is the best prophylactic measure
•• Steroids (Systemic) followed by topical steroids and cycloplegics are used for treatment
•• Example: Difficulty in reading in one eye after sustaining an injury in other eye after 3 to 4 weeks.
EYE Tumors
•• MC intraocular malignancy in children: Retinoblastoma
•• MC tumor of orbit/periorbitin children: Capillary hemangioma
•• MC primary orbital malignancy in children Rhabdomyosarcoma
•• MC cause of orbital metastasis in children: Neuroblastoma
•• MC benign orbital tumor in adults: Cavernous hemangioma
•• MC eyelid tumor is: BCC
•• MC tumor of lacrimal glands: Benign mixed tumor
Retinoblastoma
•• Retinoblastoma develops from ‘Neuroectoderm’ from ‘Photoreceptor cells’ of Retina. (Primitive Neuroectodermal Tumor)
•• There is mutation of Rb gene which is a tumor suppressor gene
•• Retinoblastoma metastasizes to brain via optic nerve
•• Retinoblastoma is bilateral in 30–40 % of cases
•• Majority of cases of Retinoblastoma are sporadic (85–95%)
•• Only a minor are familial
•• Familial ones are usually bilateral
•• The gene for Retinoblastoma is located on chromosome 13
•• Knudson Hypothesis applies to Retinoblastoma
•• Pathological features of Retinoblastoma are:
–– Flexner Wintersteiner Rosettes
–– Homer Wright Rosettes
–– Pseudorosettes
–– Flurettes
•• The most common manifestation of Retinoblastoma is Amauratic cats eye reflex or ‘Leukocoria’ (Do not confuse with
Leukorrhea)
•• Metastasis in the form of Orbital invasion, Invasion of regional Lymph nodes, Invasion of Brain, Meninges, bone marrow, is common
•• Treatment of Retinoblastoma depends on size
•• Diffuse Retinoblastoma is treated with Ennucleation
•• Ennucleation Removal of eye ball with portion of Optic Nerve from Orbit
•• Small tumors are removed by Brachytherapy
•• Enlargement of Orbit and Orbital canal is a feature
•• Treatment of metastatic disease is chemotherapy
Ophthalmology 657
Malignant Melanoma
•• It is commonest intraocular primary malignant tumor in adults
•• It involves vortex veins.
Orbital Pseudotumor
•• This is an idiopathic, inflammatory orbital syndrome
•• Symptoms are pain, limited eye movements, proptosis and congestion
•• Imaging often shows swollen eye muscles (orbital myositis) with enlarged tendons
•• The Tolosa-Hunt syndrome may be regarded as an extension of orbital pseudotumor through the superior orbital fissure into the
cavernous sinus
•• Biopsy of the orbit frequently yields nonspecific evidence of fat infiltration by lymphocytes, plasma cells and eosinophils
•• A dramatic response to a therapeutic trial of systemic glucocorticoids indirectly provides the best confirmation of the diagnosis.
Retinal Detachment
•• It is separation of neurosensory retina from retinary pigment epithelium
•• RD is of three types:
a. Rhegmatogenous: Formation of hole in retina
b. Exudative: Retina pushed from bed by fluid accumulation
c. Tractional: Mechanical detachment
Causes:
–– Myopia
–– Trauma, cataract surgery
–– Hypertension
–– Malignant neoplasms of eye
–– Penetrating trauma
–– Diabetes
–– Eales disease
–– Toxemia of pregnancy predispose to RD
–– Photopsia, floaters (muscae volitantes) are present
–– Vitreous shows Tobacco dusting which is also called as ‘Shafer’s Sign’
–– Indirect ophthalmoscopy is the investigation of choice
–– Primary aim of RD surgery is closure of break.
Retinitis Pigmentosa
This is a general term for a disparate group of rod and cone dystrophies characterized by progressive night blindness (nyctalopia)
•• Visual field constriction with a ring scotoma
•• Loss of acuity
658 USMLE Step 2 CK Platinum Notes
Parinaud’s Syndrome
It is a distinct supranuclear vertical gaze disorder from damage to the posterior commissure
It is a classic sign of hydrocephalus from aqueductal stenosis
Pineal region tumors (germinoma, pineoblastoma), cysticercosis, and stroke also cause Parinaud’s syndrome
Features Include:
•• Loss of upgaze (and sometimes downgaze)
•• Convergence-retraction nystagmus on attempted upgaze
•• Downwards ocular deviation (‘setting sun’ sign)
•• Lid retraction (Collier’s sign)
•• Skew deviation
•• Pseudoabducens palsy
•• Light-near dissociation of the pupils
Disorders of vertical gaze, especially downwards saccades, are an early feature of progressive supranuclear palsy, Parkinson’s disease,
Huntington’s chorea, and olivopontocerebellar degeneration.
Herpes Zoster
•• Herpes zoster from reactivation of latent varicella (chickenpox) virus causes a dermatomal pattern of painful vesicular
dermatitis
•• Ocular symptoms can occur after zoster eruption in any branch of the trigeminal nerve but are particularly common when vesicles
form on the nose, reflecting nasociliary (V1) nerve involvement (Hutchinson’s sign).
Ophthalmology 659
•• Herpes zoster ophthalmicus produces corneal dendrites, which can be difficult to distinguish from those seen in herpes simplex
•• Stromal keratitis, anterior uveitis, raised intraocular pressure, ocular motor nerve palsies, acute retinal necrosis, and postherpetic
scarring and neuralgia are other common sequelae
•• Herpes zoster ophthalmicus is treated with antiviral agents and cycloplegics. In severe cases, glucocorticoids may be added to
prevent permanent visual loss from corneal scarring.
Tunnel Vision
Tunnel vision is the concentric diminution of the visual fields
Causes:
•• Papilledema
•• Glaucoma
•• Retinitis pigmentosa
•• Choroidoretinitis
•• Optic atrophy secondary to tabes dorsalis
•• Hysteria
Diabetic Retinopathy
•• Diabetic retinopathy is the most common cause of blindness in adults aged 35–65 years old. Hyperglycemia is thought to cause
increased retinal blood flow and abnormal metabolism in the retinal vessel walls. This precipitates damage to endothelial cells and
pericytes
•• Endothelial dysfunction leads to increased vascular permeability which causes the characteristic exudates seen on fundoscopy
•• Pericyte dysfunction predisposes to the formation of microaneurysms
•• Neovascularization is thought to be caused by the production of growth factors in response to retinal ischemia
•• Person with NIDDM should have ophthalmic examination as early as possible
•• Person with IDDM should have fundus examination five years after diagnosis definitely
•• INCIDENCE increases with disease duration.
Background retinopathy:
•• Microaneurysms (dots)
•• Blot hemorrhages (< = 3)
•• Hard exudates
Preproliferative retinopathy:
•• Cotton wool spots (soft exudates; ischemic nerve fibers)
•• Blot hemorrhages
•• Venous beading/looping
•• Deep/dark cluster hemorrhages
•• More common in Type I DM, treat with laser photocoagulation
Proliferative retinopathy:
•• Retinal neovascularization may lead to vitrous hemorrhage
•• Fibrous tissue forming anterior to retinal disk
•• More common in Type I DM, 50% blind in 5 years
Maculopathy:
•• Based on location rather than severity, anything is potentially serious
•• Hard exudates and other ‘background’ changes on macula
•• More common in Type II DM
Sjögren’s Syndrome
Dry eye or Keratitis Sicca or Aqueous deficiency dry eye occurs in ‘Sjögren’s syndrome’. Mucosal involvement is widespread and
clinical manifestations include
•• Dry mouth
•• Dry nose
•• Dry eyes
•• Dry skin
•• Dry throat
•• Dry sex (vaginal involvement)
•• Both dry eye and dry mouth is a feature of Sjögren’s syndrome
•• This occurs as a result of decreased function of lacrimal glands
•• Cornea will be thickened and there will decrease visual acuity
•• Most common cause includes Sjögren’s syndrome and sometimes Rheumatoid arthritis and SLE Produce similar conditions.
Hypertensive Retinopathy
Characteristic changes seen with hypertensive retinopathy include:
Important Signs
•• Gunn’s sign: Crossing of artery over vein with compression of underlying vein
•• Gvist’s sign: Increased tortuosity of small venules surrounding maculae
•• Siegrist sign: Hyperpigmented flecks and choridal vessels arranged in radial fashion in Hypertension and Temporal arteritis
•• Salus Sign: Venous deflection
•• Bonnet’s sign: It is right angled deflection of vein under artery
•• LHON typically presents with painless, subacute, bilateral visual loss with central scotomas and dyschromatopsia
•• Mitochondrial disease
•• Males are affected three to four times more commonly than females.
Evisceration
It is removal of contents of eyeball leaving behind the sclera
Indications are:
•• Panophthalmitis
•• ‘Frills excision’ is done for pan ophthalmitis
•• Bleeding anterior staphyloma
Expulsive Choroidal hemmorhage
Enucleation:
It is removal of eye ball with a part of optic nerve from the orbit
Indications are:
•• Retinoblastoma in children
•• Malignant melanoma in adults
•• Sympathetic ophthalmitis
•• Endophthalmitis (DOES NOT INVOLVE SCLERA)
•• Pthisis bulbi
•• Blind and disfigured eye
•• Blind eye
•• Contraindicated in panophthalmitis
Ophthalmology 663
Trauma to Orbit
•• Blowout #: isolated comminuted # of orbital floor and medial wall. ‘Hanging drop’ or ‘Tear drop’ sign on Waters view
•• Orbital hematoma, caroticocavernous fistula
•• Iridodialysis
•• Subluxation/dislocation of lens
•• Rosette cataract
•• Vossius ring (Lens concussion)
•• Macular edema/hole
•• Commotio retinae (Berlin’s edema) milky white cloudiness involving posterior pole with cherry red spot in foveal region.
(concussion injury)
•• Optic nerve evulsion
•• Globe rupture
•• Bilateral black eye (Panda eye/Raccoon eye)
•• Unilateral Black eye
•• Berlins edema
•• D shaped pupil is seen in IRIDODIALYSIS
Mydriatics
•• Tropicamide: Quickest and shortest
•• Phenylephrine: No cycloplegia
•• Atropine: Used as an ointment in children
•• Commonest complication of topical steroids: Glaucoma
•• Drug used for blepharospasm: Botulinium toxin
Associations Asked
•• Candle wax spots: Sarcoidosis
•• Fleischer ring: Keratoconus
•• Arlt’s line: Trachoma
664 USMLE Step 2 CK Platinum Notes
2. Latanoprost
3. Dipiverine
4. Pilocarpine
Ans. 1. Timolol
Hormones of Placenta
Protein Hormones:
•• Human Chorionic Gonadotrophin (HCG)
•• Human Placental Lactogen (HPL)
•• Human Chorionic Thyrotrophin (HCT)
•• Human Chorionic Corticotrophin (HCC)
•• Pregnancy specific Beta-1 Glycoprotein
Steroid hormones:
•• Estrogens: Estriol, Estradiol and Estrone
•• Progesterone
Estrogen
•• Produced by the Syncytiotrophoblast
•• Its synthesis depends much on the precursor derived mainly from the fetus
•• In late pregnancy, estriol is the most important estrogen
•• Estrogen increases the secretion and ciliary beating in fallopian tubes
•• Estrogen changes the cuboidal lining of vagina to stratified
•• Estrogen changes the breakdown of glycogen into lactate in vagina
•• Estrogen initiates breast development
•• Estrogen causes early epiphyseal closure
•• Estrogen causes water retention
Progesterone
•• Produced by Syncytiotrophoblast
•• Chief hormone of Corpus luteum
•• After delivery, its level decreases rapidly and is not detectable after 24 hours
Amniotic Fluid
•• Green colored •• Fetal distress
•• Golden colored •• Rh Incompatibility
•• Saffron colored •• Postmaturity
•• Dark brown •• IUD
680 USMLE Step 2 CK Platinum Notes
Amnoicentesis
•• Done in:
•• Pregnancy above the age of 35 years (Chance of Downs Syndrome)
•• A previous child with chromosomal abnormalities (e.g. Autosomal Trisomy)
•• X-linked genetic recessive disorders
•• To detect inborn errors of metabolism
Investigation to be done:
•• Culture of the desquamated fetal cells in the amniotic fluid
•• Chromosomal study of the desquamated fetal cells in the amniotic fluid
•• Fetal sex is determined by karyotyping of cultured amniotic cells
•• Other related points to be remembered
•• Fetal serum also contains AFP in a concentration 150 times that of maternal serum
•• Acetyl cholinesterase levels in amniotic fluid is more specific than AFP in predicting Neural tube defects
Therapeutic Indications
First Half of Pregnancy:
•• Induction of abortion (Instillation of chemicals, e.g. hypertonic saline, urea, prostaglandin)
•• Repeated decompression of the uterus in acute hydramnios
Second Half of Pregnancy:
•• Decompression of uterus in unresponsive cases of chronic hydramnios
•• To give intrauterine fetal transfusion in severe hemolysis following Rh-isoimmunization
Hazards of Amniocentesis
Maternal Complications
•• Infection
•• Hemorrhage (placental or uterine injury)
•• Premature rupture of the membranes
•• Premature labor
Obstetrics and Gynecology 681
Fetal Complications
•• Trauma
•• Fetomaternal hemorrhage
•• Triple test was devised to increase sensitivity for pick-up rates of down’s syndrome
Triple Test
•• It includes:
•• HCG
•• Estriol
•• Alpha fetoprotein (Maternal)
In Down’s Syndrome:
HCG↑ AFP↓ Estriol ↓
In Edward’s syndrome:
HCG↓ AFP↓ Estriol ↓
Doppler Flow
•• Peak velocity of blood flow occurs during systole
•• Small amount of blood flow occurs in diastole
•• S/D ratio assessments are used
Abnormal flows:
•• Absence of blood flow
Obstetrics and Gynecology 683
•• Reverse flow
•• Notching of wave form at end of systole
Abnormal flows cause:
•• IUGR
•• Fetal distress
Remember
•• Cardiac activity of fetus seen by USG at: 6 weeks
•• Anencephaly is the earliest fetal anomaly detected by USG
•• External genitalia earliest diagnosed by USG at: 10 weeks
•• Fetal respiratory movements occur at: 12 weeks
•• Fetal bradycardias less than 120 bpm for 15 minutes period of continuous monitoring
•• Fetal scalp pH <7.2 is abnormal.
NORMAL PREGNANCY
Total Duration: 280 days
•• Gravidity •• Total number of previous pregnancies (normal or abnormal)
•• Parity •• State of having given birth to an infant or infant weighing 500 gms or more dead or alive
•• Immature infant •• Weighing <1000 gms and completed < 28 weeks
•• Premature infant •• Weighing >1000 gms and completed > 28 weeks
•• Low birth weight •• Live born infant weighing 2500 gms or less
•• Mature infant •• Live born infant completed 38 weeks of gestation and weighing more than 2500 gms
•• Postmature infant •• Live born infant completed 42 weeks of gestation or more
684 USMLE Step 2 CK Platinum Notes
Respiratory System
•• ↓TLC, FRC, RV
•• ↑Tidal volume
•• ↑Minute ventilation
•• ↑O2 requirements
Genitourinary System
•• ↑Urinary frequency
•• ↑UTI, Pyelonephiritis
•• Glycosuria
Other Changes
•• ↑Incidence of Carpal Tunnel Syndrome
•• ↑Total Thyroxine and ↑Thyroxine binding globulin levels. Normal TSH
•• ↑Incidence of Gallstones, GERD (Reflux)
•• ↑Renal blood flow, ↑GFR
•• ↑Transferrin
•• ↑Leukotrienes
‘Signs’of Pregnancy
•• Chadwick’s sign: Congestion of pelvis causes bluish/purplish hue of vagina/cervix
•• Leukorrhea: Non fern pattern of cervical mucus
•• Goodells sign: Cyanosis and softening of cervix
•• Ladin’s sign: Softening of uterus in anterior midline
•• Hegar’s sign: Compressibility of isthmus on bimanual examination
•• Mc Donald’s sign: Uterus becomes flexible at uterocervical junction
•• von Frendwal’s sign: Irregular softening of fundus at site of implantation
•• Piskacek’s sign: Softening of cervix with lateral implantation
•• Osianderr’s sign: Pulsations in lateral vaginal fornix
•• Palmer’s sign: Rythmic contractions of uterus
•• Weinberg sign: Abdominal pregnancy
Pseudocyesis
•• Patient having an intense desire for pregnancy and develops symptoms of pregnancy
•• No FHS
•• No USG documentation of gestational sac
•• Refer to pshyciatrist
Pelvis
•• Normal female pelvis is Gynecoid pelvis
•• Most common type of pelvis is Gynecoid pelvis
•• Transverse diameter is more than AP Diameter in Gynecoid pelvis
•• Occipitofrontal •• Vertex
•• Mentovertical •• Brow
•• Submentovertical •• Face
•• Submentobregmatic •• Face
•• Inlet contraction •• If AP diameter is 10 cms or less
•• Mid pelvic contraction •• If inter ischial diameter is < 9.5 cms
•• Mid pelvic contraction •• Prominent ischial spines, converging pelvic walls
•• Outlet contraction •• Intertuberous diameter is 8 cms or less
Bishops Scoring
It is used for labor
Bishops scoring includes:
•• Dilatation of cervix
•• Effacement of cervix
•• Consistency of cervix
•• Position of cervix
•• Station of head
Bishops score > 6 indicates beginning of labor.
Labor
•• Graphical representation of stages of labor is partogram
•• Assessment of labor is best done by Partogram
•• Graph showing relationship between cervical dilatation and labor is cervicograph
•• Cervical ripening is mainly due to PGE2
•• Sensitivity of uterine musculature is ↑ by: estrogen
•• Cervical ripening is done by PGE2, oxytocin, misoprostol
Trial of Labor
Indications: Minor CPD
Contraindications:
•• Elderly primi
•• Major CPD (disproportion)
•• Severe PET
•• Previous cesarean
•• Malpresentation
•• Outlet contraction
Bandl’s Ring
•• Retraction ring
•• Seen in CPD, malpresentation
Obstetrics and Gynecology 689
•• Average pressure of uterine contractions during first stage of labor: 30 mm Hg immediately following delivery, height of uterus
corresponds to 20 weeks
•• Graph showing relationship between cervical dilatation and duration of labor is called cervicograph
•• Assessment of labor is best done by Partogram
•• Pressure inside uterus during second stage of labor is 100–120 mm Hg
•• Precipitate labor is labor occurring in less than 2 hours
‘Preterm Labor’
It is defined as labor occurring before 37 completed weeks of gestation
Differentiate from premature baby: < 38 weeks
Prediction of preterm labor:
•• Symptoms of preterm labor
•• Uterine contraction greater than 4 per hour
•• Cervical length less than 2.5 cm
•• U shaped cervix is indicative
•• Presence of fibronectin in vaginal discharge between 24 and 36 weeks
•• Bishops score greater than 4
•• Cervical dilatation greater than 2 cm and effacement 80%
•• Vaginal bleeding
•• Prior preterm
Causes of preterm labor are:
•• Chorioamnionitis
•• Bacterial vaginosis
•• Previous history
•• Recurrent UTI
•• Smoking
•• Low socioeconomic and nutritional status
•• Uterine anomalies
•• Medical and surgical illnesses
•• Cervical incompetence
•• Increased amniotic fluid IL-6 levels and Neoptrin are considered as markers of preterm labor
Tocolytics
These are drugs used to arrest premature labor
Important Tocolytics are:
•• Beta mimetics: Ritodrine, Salbutamol, Isoxsuprine
•• Oxytocin antagonists: Atosiban
•• Magnesium Sulfate
•• Nifedepine
•• Glyceryl Trinitrate
•• Progesterone
•• Diazoxide
•• Ethyl Alchohol
•• Atropine
•• PG inhibitors: Indomethacin, Sulindac
690 USMLE Step 2 CK Platinum Notes
Abortion
•• Threatened abortion Os closed
•• Inevitable abortion Os open
•• Incomplete abortion Patulous os
•• Complete abortion Os closed with expulsion of gestational sac
Septic Abortion
•• Any abortion with clinical evidence of infection of uterus or its contents
•• Rise of temp at least 100 °F for at least 24 hours
•• Presence of offensive/purulent vaginal discharge
•• Presence of evidence of pelvic infection
•• Most commonly follows illegal abortion
•• Infection is polymicrobial
Ectopic Pregnancy
•• A young girl with 8 weeks ammenorrhea comes in shock. Likely diagnosis is: ectopic
•• A young girl with 6 weeks ammenorrhea presents with mass abdomen, USG shows empty uterus. Likely diagnosis is:
ectopic
•• A young girl with 6 weeks ammenorrhea comes with pain abdomen USG shows fluid in pouch of douglas Aspiration yields
dark colored fluid failing to clot. Likely diagnosis is: ruptured ectopic
It is associated with:
•• Tubal diseases (commonest)
•• Endometriosis
•• PID
•• Tuberculosis
•• IUCD
•• Following Tubal procedures Congenital tubal anomalies
•• Progastasert
•• Most common feature of ectopic is abdominal pain
•• Most reliable indicator of ectopic gestation is no gestational sac in USG
•• Expelled products in ectopic originate from: decidua vera
•• Medical treatment of ectopic pregnancy is by: methotrexate, actinomycin-D, mifeprestone, prostaglandins
•• STUDOFORD CRITERION is for primary abdominal pregnancy
•• Laproscopy is the best investigation for diagnosis
•• Commonest type which ruptures is: Isthmic
•• Transvaginal USG and HCG levels are also used
Bagel sign: It is a sign of ectopic pregnancy in Ultrasonography
It is a hyperechoic ring around gestational sac in adenexal region
Blob sign is in homogenous mass adjacent to ovary or moving separate from ovary. It is a gestational sac without cardiac activity
SALPINGOSTOMY FOR unruptured tubal pregnancy (Tubes to be preserved)
SALPINGECTOMY FOR ruptured tubal pregnancy with no desire for future pregnancy
•• Occasional mitosis
•• It occurs in 10–15% cases of ectopic pregnancy
•• It is because of progesterone influence
Post-term Pregnancy
•• Post-term pregnancy is characterized by:
•• Wrinkled skin appearance
•• Overgrown nails
•• Absence of Vernix caseosa
•• ‘Scanty saffron colored meconium and subsequent oligohydramnios’
•• Placental calcification
•• Hypoxia and RDS
•• Polycythemia
•• Hypoglycemia
–– Golden colored meconium: Rh incompatibility
–– Saffron colored meconium: Postmaturity
–– Dark brown meconium: IUD
Hydatid Mole
•• Molar pregnancy is an abnormal form of pregnancy, characterized by the presence of a hydatidiform mole (or hydatid mole, mola
hytadidosa)
•• Molar pregnancy comprises two distinct entities, partial and complete moles
•• Complete moles have no identifiable embryonic or fetal tissues and arise when an empty egg with no nucleus is fertilized by a
normal sperm
•• Partial mole occurs when a normal egg is fertilized by two spermatozoa. Hydatidiform moles may develop into choriocarcinoma
•• Caused by triploidy
Causes persistent GTD
•• A hydatidiform mole is a pregnancy/conceptus in which the placenta contains grapelike vesicles that are usually visible with the
naked eye. The vesicles arise by distention of the chorionic villi by fluid. When inspected in the microscope, hyperplasia of the
trophoblastic tissue is noted. If left untreated, a hydatidiform mole will almost always end as a spontaneous abortion
•• H
ydatidiform moles are a common complication of pregnancy, occurring once in every 1000 pregnancies in the US, with much
higher rates in Asia
•• Blood tests will show very high levels of human chorionic gonadotropin (hCG)
•• T he diagnosis is strongly suggested by ultrasound (sonogram), but definitive diagnosis requires histopathological examination.
The mole grossly resembles a bunch of grapes (‘cluster of grapes’ or ‘honeycombed uterus’ or ‘snow-storm’)
•• There is increased trophoblast proliferation and enlarging of the chorionic villi. Angiogenesis in the trophoblasts is impared as
well
•• S
ometimes symptoms of hyperthyroidism are seen, due to the extremely high levels of hCG, which can mimic the normal
Thyroid-stimulating hormone (TSH)
ydatidiform moles should be treated by evacuating the uterus by uterine suction or by surgical curettage as soon as
•• H
possible after diagnosis, in order to avoid the risks of choriocarcinoma. Patients are followed up until their serum human chorionic
gonadotrophin (hCG) level has fallen to an undetectable level
•• Invasive or metastatic moles (cancer) may require chemotherapy and often respond well to methotrexate. The response to
treatment is nearly 100%.
Obstetrics and Gynecology 693
Complications of H. Mole
•• Hemorrhage and shock due to separation of vesicles from its attachment to deciduas
•• Massive intraperitoneal hemorrhage
•• Sepsis
•• Perforation of uterus
•• Preeclampsia with convulsions
•• Acute pulmonary insufficiency due to pulmonary embolism
•• Thyroid storm
•• Development of choriocarcinoma
Nut Shell
•• Large for date uterus
•• Chromosome configuration is: 46XX
•• Excessive nausea/vomiting
•• Early onset PET
•• Absent fetal heart
•• Grape like vesicles per vaginum
•• Bilateral theca leutin cysts associated
•• High HCG
•• Snow storm appearance on USG
•• Investigation of choice to diagnose H mole is USG
•• Confirmation of diagnosis is by vaginal examination
•• Evacuate by suction evacuation
•• Immediate complication of evacuation of H mole is bleeding
Associations:
•• PIH
•• Thyrotoxicosis
•• Hyperemesis
Sarcoma Botyrides
•• Sarcoma botyrides is anembryonal rhabdomyosarcoma
•• Mesenchymal in origin
•• Readily bleeds on touching
•• Tumor cells resemble Tennis racket
•• Presents as grape like mass buldging through vaginal orifice
•• Locally invasive
•• Chemoradiation is treatment of choice
694 USMLE Step 2 CK Platinum Notes
Twinning
•• Negroes have the highest rate of twinning
•• Hellin’s law indicates chances of twinning
•• Most common types of twins are both vertex
•• Superfecundation is fertilization of two ova released at same time by sperms released at two different occasions of sexual
intercourse
•• Siamese twins have highest mortality
•• Thoracophagus is the most common type of conjoined twins
•• A double headed monoster is called dicephalous
•• Twin peak sign is seen in diamniiotic dichorionic twins
•• In multiple fetal pregnancy fetal reduction is done by: Potassium chloride
•• Most common cause of perinatal mortality in twins is prematurity
•• Twin pick sign is seen in dichorionicity
Anencephaly
•• Associated with:
–– Face presentation
–– Hydramnios
–– Postmaturity
–– Prematurity
•• Best diagnosed by USG at 10 to 12 weeks
MALPRESENTATIONS
Breech
•• Most common cause of breech is prematurity
•• Most common type of breech is frank breech
•• Engagement occurs earliest in frank breech
•• Mortality in breech is mainly due to intracranial hemorrhage
•• Commonest position is left sacroanterior
•• Engaging diameter is bitrochanteric
•• Head is borne by flexion
Shoulder Dystocia
•• Impaction of anterior shoulder of fetus against symphysis pubis after fetal head has been delivered
•• Occurs when breadth of shoulders is greater than BPD
•• Risk factors are:
–– Diabetes
–– Obesity
–– Post-term baby
–– Excessive weight gain in pregnancy Approach is:
•• Apply suprapubic pressure
•• Legs in flexion (Mc Robert’s maneuver)
•• Woods procedure
•• Release posterior shoulder
•• Manual corckscrew
•• Episiotomy
•• Rollover
Turtle sign positive: Head delivers but retracts against symphisis pubis
Complications: Erbs palsy, klumpkes palsy#clavicle, humerus, spine, PPH (injury to perineum), Fetal hypoxia
Treatment
•• Cleidotomy
•• Zanavelli maneuver: Replacement of fetus to uterine cavity and cesarean section
•• Symphisiotomy
696 USMLE Step 2 CK Platinum Notes
Rh Incompatibility
•• Immune hydrops
•• Father Rh+, mother Rh-, fetus Rh+
•• Ist child escapes usually
Their fetuses are at risk of hypoglycemia and birth trauma (Brachial Plexus) injury
•• Maternal Complications •• Fetal Complications
•• Abortion •• Fetal macrosomia
•• Infections •• Neonatal malformations (CVS and Neural Tube defects)
•• Preeclampsia •• Renal Agenesis
•• Hydramnios •• Hypoglycemia
•• Hypomagnesemia
•• Hypocalcemia
•• Maternal Distress •• RDS
•• Increased complications of labor •• Hyperviscosity Syndrome
•• Shoulder Dystocia •• Hyperbilirubinemia
•• PPH •• Hypertrophic Cardiomyopathy
•• Perineal Injuries •• Polycythemia
•• Erbs, Klumpkes palsy
Mitral Regurgitation and Aortic Regurgitation. These are both generally well tolerated during pregnancy
•• The pregnancy-induced decrease in systemic vascular resistance reduces the risk of cardiac failure with these conditions
•• As a rule, mitral valve prolapse does not present problems for the pregnant patient and aortic stenosis, unless very severe, is also
well tolerated. In the most severe cases of aortic stenosis, limitation of activity or balloon valvuloplasty may be indicated
•• For women with artificial valves contemplating pregnancy, it is important that warfarin be stopped and heparin initiated prior
to conception
•• Warfarin therapy during the first trimester of pregnancy has been associated with fetal chondrodysplasia punctata. In the
second and third trimester of pregnancy, warfarin may cause fetal optic atrophy and mental retardation.
Hypertension in Pregnancy
Pregnancy induced HTN:
Predicted by:
•• Rolling over test done at 20 weeks
•• Serum uric acid ↑
•• Weight gain > 2 kg/month
•• Earliest sign is ↑ BP
•• Preeclampsia: Hypertension+proteinuria+edema
•• Hypertension with proteinuria in a previously normotensive and nonproteinuric patient.
•• Mild preeclampsia: Systolic BP > 140 or Diastolic BP > 90 mm Hg on two or more occasions and proteinuria > 300 mg/24
hours.
•• Severe preeclampsia: Systolic BP >160 or Diastolic BP >110 mm Hg on two or more occasions and proteinuria > 5 gms/24
hours
•• Eclampsia is preeclampsia+convulsions
•• Imminent eclampsis is heralded by:
–– Headache
–– Blurring of vision
–– Epigastric pain
–– Brisk deep tendon reflexes
–– Diminished urinary output
•• Delivery of fetus is the definitive treatment
HELLP Syndrome
•• The HELLP (hemolysis, elevated liver enzymes, low platelets) syndrome
•• Microangiopathic hemolysis occurs
•• AST↑
•• LDH↑
•• Acute fatty liver of pregnancy, cholestasis of pregnancy, eclampsia, and the HELLP syndrome (can be confused with viral hepatitis
during pregnancy)
ANTEPARTUM HEMORRHAGE
Placenta Previa
Abnormal implantation of placenta usually near cervical os
Risk factors:
•• Prior cesarean
•• Grand multipara
•• Multiple gestations
•• Prior placenta previa
•• Painless
•• No tenderness
•• Soft uterus
•• No uterine irritability
•• Malpresentation
•• FHR normal usually
•• Coagulopathy uncommon
700 USMLE Step 2 CK Platinum Notes
Abruptio Placenta
Premature separation of a normally placed placenta
Risk factors:
•• Hypertension
•• Trauma to abdomen
•• Smoking
•• Cocaine use
•• Painful
•• Tenderness
•• Increased uterine tone
•• Uterine irritability
•• Malpresentation none usually
•• FHR abnormal usually
•• Coagulopathy common
Vasa Previa
•• Fetal vessels crossing cervical os
•• Associated with velemantous cord insertion
•• Higher incidence in twins
•• Painless vaginal bleeding + fetal distress
•• APT test + look for nucleated cells in cord blood by wrights stain
•• Emergency cesarean is TOC
PPH
•• ‘Any amount of bleeding from or into the genital tract following birth of baby until the end of puerperium which adversely
affects the general condition of the patient evidenced by rise in pulse rate and falling BP is called PPH.’
•• Commonest cause of PPH is: Atonic uterus
•• Prolonged labor
•• Uterine malformation
•• Mismanaged third stage
•• Constriction ring
Due to Trauma:
•• Instrumental delivery, (cervical lacerations)
•• Face to pubis delivery
•• Precipitate labor
•• Macrosomia
Due to Blood coagulopathies:
•• Abruption
•• Sepsis
•• IUD
•• HELLP Syndrome
Uterine Inversion
•• Inside out turning of uterus
•• Results from mismanaged third stage of labor
•• Patient presents with intense shock
•• On examination of abdomen, fundus of uterus cant be felt
•• Hemorrhage (mc), neurogenic shock, pulmonary embolism, uterine sepsis and subinvolution are complications
702 USMLE Step 2 CK Platinum Notes
FHR
Normal Pattern
•• Base line heart rate: 120–160 bpm
•• Base line variability: 5–25 bpm
•• Two accelerations in 20 minutes
•• No deceleration
Obstetrics and Gynecology 703
Abnormal Pattern
•• Unexplained tachycardia
•• Unexplained bradycardia
•• Base line variability < 5 bpm
•• Repetitive early or variable decelerations
•• Repetitive late decelerations
•• Sinusoidal pattern
Deceleration
•• Early deceleration: Head compression
•• Late deceleration: Placental insufficiency
•• Variable deceleration: Cord compression
Treatment of deceleration
The biophysical profile (BPP) is a noninvasive test that predicts the presence or absence of fetal asphyxia and, ultimately, the
risk of fetal death in the antenatal period. When the BPP identifies a compromised fetus, measures can be taken to intervene before
progressive metabolic acidosis leads to fetal death.
The BPP combines data from 2 sources (i.e. ultrasound imaging and fetal heart rate [FHR] monitoring). Dynamic realtime B-mode
ultrasound is used to measure the amniotic fluid volume (AFV) and to observe several types of fetal movement. The FHR is obtained
using a pulsed Doppler transducer integrated with a high-speed microprocessor, which provides a continuously updated reading
Originally described by Manning and colleagues, the BPP has become a standard tool for providing antepartum fetal surveillance.
The BPP integrates 5 parameters to yield a biophysical profile score (BPS) and includes:
•• The nonstress test (NST)
•• Ultrasound measurement of the AFV, (Amniotic fluid volume)
•• Fetal breathing movements
•• Fetal body movements and
•• Tone
The BPP allows 2 points for each parameter that is present, yielding a maximum score of 10
•• Reassuring score is 8–10
•• < 6 is fetal distress.
Cardiotocography
•• Monitoring the baby’s heartbeat is one way of checking babies’ well-being in labor. By listening to, or recording the baby’s
heartbeat, it is hoped to identify babies who are hypoxic and who may benefit from cesarean section or instrumental vaginal birth
•• The heartbeat can also be checked continuously by using a CTG machine
•• This method is sometimes known as electronic fetal monitoring (EFM) and produces a paper recording of the baby’s heart rate
and their mother’s labor contractions.
Chorioamnionitis
•• Fever+maternal tachycardia
•• Uterine tenderness
•• Foul odor of amniotic fluid
•• Leukocytosis
Complications are: Premature rupture of membranes
•• Premature labor
•• Endometritis
•• Parametritis
•• Abruptio placenta
Vaginal delivery is the mainstay of treatment
Neonatal Sepsis
Sepsis is asystemic response to infection. In newborns it can be classified as early onset, occurring in the first week of life, and late
onset, occurring between 8 and 28 days. Risk factors for neonatal sepsis include:
•• Maternal infection during pregnancy (urinary tract infection, chorioamnionitis), prematurity and prolonged rupture of membranes
•• Common organisms causing sepsis in the newborn are:
–– Group B streptococci
–– Escherichia coli
–– Listeria
•• Viral causes can be herpes simplex virus and enteroviruses
•• Signs and symptoms of neonatal sepsis are nonspecific, such as grunting, tachypnea, cyanosis, poor feeding, irritability, apnea,
bradycardia, jitters, tremors, and seizures
•• Newborns do not always develop fever, and hypothermia may be a presenting sign
•• A bulging fontanelle may be palpated on physical exam, but there is rarely nuchal rigidity
•• Laboratory studies should include a complete blood count, lumbar puncture, and blood and urine cultures, as well as cultures of
any visible lesions or drainage.
Remember
•• ACE inhibitors are contraindicated in pregnancy
•• Yellow fever vaccine is contraindicated in pregnancy
Obstetrics and Gynecology 705
Fallopian Tube
•• Has 4 parts
•• Lined by ciliated columnar epithelium
•• Has peg cells
•• Measures 10 cms
Female Urethra
•• 4 cms in length
•• 6 mm in diameter
•• Posterior urethrovesical angle is 100°
•• Has transitional epithelium
The larche, pubarche and menarche is the order of sexual development in girls.
Estrogen
•• Produced by the Syncytiotrophoblast
•• Its synthesis depends much on the precursor derived mainly from the fetus
•• In late pregnancy, estriol is the most important estrogen
•• Estrogen increases the secretion and ciliary beating in fallopian tubes
•• Estrogen changes the cuboidal lining of vagina to stratified
•• Estrogen changes the breakdown of glycogen into lactate in vagina
•• Estrogen initiates breast development
•• Estrogen causes early epiphyseal closure
•• Estrogen causes water retention
Progesterone
•• Produced by Syncytiotrophoblast
•• Chief hormone of Corpus luteum
•• Precursors from fetal origin are not necessary
•• After delivery, its level decreases rapidly and is not detectable after 24 hours
Ovulation
•• Best predictor of ovulation is preovulatory rise in progesterone
•• Source of progesterone in normal menstrual cycle is: Corpus luteum
Obstetrics and Gynecology 707
Supports of Uterus
•• Perineal body
•• Pelvic diaphragm
•• Transcervical ligament
•• Pubocervical ligament
•• Uterosacral ligament
•• Uterine axis
•• Round ligament of uterus
•• Broad ligament is not a support of uterus
Anomalies of Uterus
•• Uterus bicornis bicollis: Two uterine cavities, double cervix
•• Uterus bicornis unicollis: Two uterine cavities, single cervix
•• Septate uterus: Septum between two fused mullerian ducts
•• Retroverted uterus: Long axis of body of uterus and cervix are in line instead of normal anteverted and anteflexed uterus
Supports of Vagina
•• Perineal body
•• Pelvic diaphragm
•• Levator ani muscle
Bartholin’s Gland
•• Correspondes to bulbourethral glands of male (B-B)
•• Situated in the superficial perineal pouch
•• Lies close to the posterior end of vestibular bulb
•• Are pea sized
•• Lie near the junction of the anterior 2/3 and posterior 1/3 of labium majus
•• It is a Compound Racemose Gland and its duct measures about 2 cm, lined with columnar Epithelium
•• Its duct passes forward and inward to open external to the hymen on inner side of Labium minus
•• Infection of these glands or their ducts results in Bartholinitis
Acute bartholinitis
•• Although Gonococcus is always in mind but more commonly E.coli, Staphylococcus
Streptococcus or Chlamydia trachomatis are involved
•• The end results of acute bartholinitis are:
–– Complete resolution
–– Recurrence
–– Abscess formation
–– Cyst formation
Recurrent Bartholinitis
•• Periodic painful attacks cause annoyance to the patient
•• Treatment: Active Phase: Hot compress, Analgesics and Antibiotics after proper culture
•• Quiescent Phase: Excision of the gland along with its duct.
Bartholins Abscess
•• It is the end result of acute bartholinitis
•• Duct is blocked by fibrosis, exudates pent up inside to produce abscess
•• Treatment: Same as for bartholinitis. Abscess should be drained at the earliest before. It bursts
•• Marsupialization not only helps in drainage of pus but prevents recurrence of abscess and future cyst formation
•• It should be done under General Anesthesia
•• Reccurrent Bartholin’s abscess Rx _____Excision in Quiescent Stage
Bartholin’s Cyst
•• It commonly involves the duct and not the gland
•• Cyst of the duct or the gland can be differentiated by the lining epithelium
Obstetrics and Gynecology 709
Gartners Duct
•• Remnant of wolfian duct
•• Runs below but parallel to the fallopian tube in the mesosalpinx
•• The tubules of the gartners duct may be cystic
•• The outer ones are Kobelts tubules
•• The Middle set, the Epoophoron
•• The proximal set, the Paroopohoron
•• Small cyst may arise from any of the tubules
•• A cystic swelling from the Gartners duct may appear in the anterolateral wall of the vagina confusing with cystocele. Cystic
swelling is present at the junction of lower 1/3 and upper 2/3 of vaginal wall
Kartagener’s Syndrome
Infertility is common, due to defective ciliary action in the fallopian tube in affected females or diminished sperm motility in
affected males
In Kartagener’s syndrome (KS), primary defects of the ciliary axoneme cause dyskinetic ciliary motion. Because ciliary motion is an
important factor in normal ovum transport, ciliary dyskinesia may cause infertility. In active regions, beat frequency ranged from
5 to 10 Hz, approximately 30% of normal
•• Electron microscopy shows morphological defects in tubal mucosa
•• The number of cilia per cell is ↓
•• The major ultrastructural abnormality was an absence of the central microtubules
•• Electron microscopy demonstrated that the majority of cilia lack dynein arms and radial spokes and that various defects of
microorgans existed in the sperm
PID
•• Gram-negative diplococcic in PMNs in urethral Exudate •• Nisseria gonorrhea
•• Culture negative specimen with inclusion bodies •• Chlamydiae trachomatis
•• Organisms without cell wall and urease positive •• Ureaplasma urealyticum
•• Flagellate protozoa with motility •• Trichomonas vaginalis
Chancroid
•• Hemophilus ducreyi
–– Painful ulcer, genital ulceration and inguinal adenitis
–– Painful lymphadenopathy seen
–– Associated with infection with HIV
•• School of fish appearance
•• H. ducreyi is a highly fastidious coccobacillary gram-negative bacterium whose growth requires X factor (hemin)
•• Chancroid can be treated effectively with several regimens, including (1) ceftriaxone, 250 mg intramuscularly as a single dose;
(2) azithromycin, 1 g orally as a single dose; (3) erythromycin, 500 mg orally four times daily for 7 days; and (4) ciprofloxacin, 500
mg orally twice daily for 3 days.
Candida
•• Caused by candida albicans
•• Recurrent vulvovaginalis: 4 or more episodes per year
•• It is a gram-positive fungus
•• Common in pregnancy, diabetes mellitus
•• Commonly seen in pregnancy, diabetes, antibiotics, OCP use, Corticosteroids
Obstetrics and Gynecology 711
Trichomoniasis
•• Most common vaginal infection
•• Caused by flagellate parasite ‘trichomonas vaginalis’
–– Profuse, thin, greeny discharge which is malodorous
–– Strawberry vagina/colpitis macularis appearance
•• Best test: Culture
•• Metronidazole is DOC
•• Treat sexual partener concurrently
Bacterial Vaginosis
Poly microbial caused by:
–– Gardenella vaginalis
–– Hemophilus vaginalis
–– Mobilincus
–– Ureaplasma urealyticum
–– Mycoplasma hominis
•• ↓Lactobacilli
•• ↓Leucocytes
•• ALKALINE pH
•• ‘Clue cells’ are epithelial cells with bacteria adhering to their surface and sometimes obscuring their borders. Clue cells indicate
bacterial vaginosis
•• Clue cells were first described by Gardner and Dukes in 1955 and were so named as these cells give an important ‘clue’ to the
diagnosis of bacterial vaginosis (BV)
•• Clue cells are vaginal squamous epithelial cells coated with anaerobic Gram-variable coccobacilli Gardnerella vaginalis.
Pathogenesis:
The detection of clue cells is the most useful single procedure for the diagnosis of BV. Presence of more than 20% clue cells in vaginal
discharge is included in Amsel’s criteria for the diagnosis of BV. Other criteria for the diagnosis of BV include:
Milky, homogeneous, adherent discharge
vaginal pH greater than 4.5
Positive Whiff test, i.e. typical fishy odor on addition of one or two drops of 10% KOH to vaginal discharge and
Few or no lactobacilli
Metronidazole (DOC) or clindamycin is used in treatment
Metronidazole is used in pregnancy.
LGV
•• Chlamydia trachomatis L type
–– Painless ulcer
–– Esthiomine seen in LGV
–– Groove sign. (Double genitocrural fold)
–– Genital elephantiasis is seen
712 USMLE Step 2 CK Platinum Notes
•• Incubation period = 3 days to 3 weeks. Painless, vesicle, often transient, followed by suppurative LAP
•• Sign of Groove + (LAP present both above and below inguinal ligament)
•• Elephantiasis of vulva + Vaginal and rectal strictures seen
•• Frei’s intadermal test +
•• Doxycycline is DOC
Herpes Genitalis
•• HSV II virus usual
•• Painful vesicular lesions
Condyloma Acuminata
•• Most common viral STD
•• Caused by HPV: 6, 16, 18
•• Immunosuppression, diabetes, pregnancy are predisposing factors
•• Cauliflower like masses, pedunculated
Treated by:
•• Podophyllin, podophox
•• Trichloroacetic acid
•• Imiquimoid
•• Cryo
•• LASER
•• Surgical excision
Condyloma lata are seen in secondary syphilis
Chlamydia
•• Purulent discharge, urethritis, arthritis, conjuctivitis
•• Infertility associated with fallopian-tube scarring. It appears that subclinical tubal infection (‘silent salpingitis’) may produce
scarring
•• Ectopic pregnancy
•• Perihepatitis, or the Fitz-Hugh-Curtis syndrome, this syndrome should be suspected whenever a young, sexually active woman
presents with an illness resembling cholecystitis (fever and right-upper-quadrant pain of subacute or acute onset). Symptoms and
signs of salpingitis may be minimal. High titers of antibodies to C. trachomatis are generally present
•• PCR is gold standard for diagnosis
•• Azithromycin is DOC for uncomplicated chlamydiae
•• Azithromycin is DOC for chlamydiae in pregnancy
•• Erythromycin is safe in pregnancy
•• Azithromycin and contact tracing is the most effective for chlamydiae infections
Obstetrics and Gynecology 713
Genital Tuberculosis
•• Genital TB is almost always secondary to focus elsewhere
•• Primary focus is in lungs (50%), lymph nodes (40%) cases
•• Hematogeneous spread is the commonest mode followed by direct spread and ascending route
•• Major forms:
–– Tuberculous endosalpingitis
–– Tuberculous exosalpingitis
–– Interstitial salpingitis
•• Only about 10% of genital TB cases have children
•• First site of affection are the fallopian tubes
•• Commonest site of affection are the fallopian tubes
•• ‘Tobacco pouch appearance’ of fallopian tubes is a feature
•• ‘Salpingitis isthmica nodosa’ is nodular thickening of tubes in genital TB
•• Endometrium of uterus is involved in 60% cases
•• Cervix is involved in 10–15% of cases
•• Infertility is due to blockage of fallopian tubes. In the form of TB endosalpingitis
•• The first line of treatment is ATT (Antitubercular therapy)
Endometriosis
•• Commonest site: ovary
•• Followed by: pouch of Douglas, uterosacral ligaments, rectovaginal septum
•• Peak age: 30–40 years
•• Common innullipara
•• Chocolate cysts
•• Tenderness and nodularity of uterosacral ligaments
•• Sampson’s theory of retrograde menstruation. (Most accepted)
•• Meyer and ivanoffs coelomic metaplasia theory
•• Direct implantation theory
•• Halbans lymphatic theory
•• Rare in negroes
•• Pelvic pain (commonest manifestation)
•• Painful periods Dysmenorrhea
•• Painful intercourse Dyspareunia
•• Painful bowel movements Dyschezia
714 USMLE Step 2 CK Platinum Notes
•• Menorrhagia
•• Polymenorrhea
•• Infertility
Treatment:
–– Progesterone
–– OCP
–– CLOMIPHENE (in infertile women)
–– Danazol
–– GnRH analogues (leuprolide)
–– Ovarian cystectomy/oopherectomy/Wedge resection
–– Paratubal lysis by laproscopy
–– Total hysterectomy: For diffuse endometriosis interna
Pseudoxanthoma cells: Adjacent to lining of endometriosis, presence of polyhedral, phagocytic, pigmented cells with hemosiderin
A female of 34 years complain of dysmenorrheal, dyspareunia and dyschezia with fixed retroverted uterus and nodularity of
uterosacral ligaments
Adenomyosis
•• Endometrial glands without Myometrium of uterine wall
•• Secondary dysmenorrhea and menorrhagia are common
•• Uterus is enlarged and tender
•• Treatment: Hysterectomy
A female 36-year-old with dysmenorrhea, menorrhagia, chronic pelvic pain and symmetrically, enlarged smooth uterus tender
to palpation
CONTRACEPTION
Gossypol
•• Male contraceptive
•• Direct suppression of semineferous tubules causing azoospermia
•• Suppresses LH
Obstetrics and Gynecology 715
IUCD
Act by ovulation inhibition, causing aseptic endometritis
Act by prevention of fertilization
Act by interfering with implantation
Contraindications:
•• PID
•• Diabetes mellitus
•• Congenital uterine anomalies
•• Heart disease
•• Pelvic TB
•• HIV Positives
•• Suspected pregnancy
•• Previous ectopic
•• Menorrhagia
Commonest side effect is bleeding
Nova T has a silver core but contains both copper and silver
•• In CuT200, 200 mean 200 mm2 of copper
•• CuT200 is inserted postnatally after 8 weeks
•• If CuT200 is implanted in Myometrium treatment is hysteroscopic removal
•• CuT200 should be replaced after every 10 years
Contraceptive TODAY
Contraceptive TODAY
•• Contains 9 NON OXYNOL
•• It is a barrier contraceptive
•• Effective for 24 hours after insertion
•• Spermicidal in nature
Uses of OCP
•• Contraception, emergency postcoital (prophylaxis) A combination of levonorgestrel 50 or norgestrel with ethinyl estradiolis
used as emergency contraception (also called intraception, morning-after treatment, or postcoital contraception) for postcoital
birth control, after pregnancy has been ruled out. The dosing method using high doses of estrogen-progestin hormones is
commonly called the Yuzpe method
•• Acne vulgaris
•• Amenorrhea
•• Dysfunctional uterine bleeding (DUB)
•• Dysmenorrhea
–– Hypermenorrhea
–– Endometriosis (Prophylaxis and Treatment)
716 USMLE Step 2 CK Platinum Notes
–– Hirsutism, female
–– Hyper and rogenism, ovarian
–– Polycystic ovary syndrome
Levonorgestrel
•• Levonorgestrel is a synthetic progestogen used as an active ingredient in hormonal contraception
•• Makes endometrium unreceptive
•• Makes cervical mucus thick
•• It is used effectively in emergency contraception both in combined form with estrogens as well as levonorgestrel only method
•• Levonorgestrel only method uses ‘1500 µgm single dose’ or ‘750 µgm’ doses twelve hours apart within 3 days of unprotected
sexual activity
Progestasert
•• Third generation IUCD containing progesterone
•• Decreases blood loss
•• Decreases dysmenorrhea
•• BUT ↑ risk of ectopics
Obstetrics and Gynecology 717
Infertility
•• Failure to conceive within one or more years of regular unprotected sex
•• Male responsible for 30% cases
•• Female responsible for 30% cases
INTRAUTERINE INSEMINATION
Indications
•• Hostile cervical mucus
•• Cervical stenosis
•• Oligospermia
•• Immune factors
•• Unexplained fertility
INTRACERVICAL INSEMINATION
Indications
•• Hypospadias
•• Retrograde ejaculation
•• Impotence
•• Third degree retroversion of uterus
IN VITRO FERTILIZATION
Indications
•• Tubal disease
•• Unexplained infertility
•• Cervical hostility
•• Failed ovulation induction
•• Endometriosis
•• Male factor infertility
Prolapse
•• Cystocele: Prolapse of upper 2/3 of anterior vaginal wall. Formed by base of bladder
•• Urethrocele: Prolapse of lower 1/3 of anterior vaginal wall
•• Enterocele: Prolapse of upper 1/3 of posterior vaginal wall
•• Rectocele: Prolapse of lower 2/3 of posterior vaginal wall
•• Uterine prolapse: Abnormal descent of uterus through vagina
•• Procedentia: Complete uterine prolapse outside vulva
Complications
•• Elongation of cervix
•• Cystocele
•• Decubitus ulcer
•• Decubitus ulcer is because of venous congestion
Fistula
•• MC urinary fistula: Vesicovaginal
•• MC cause of VVF in India: obstructed labor (DOC cystoscopy)
•• MC cause of Uretrovaginal fistula: injury to ureter in Hysterectomy
•• MC cause of vesicouterine fistula: cesarean section
•• MC cause of Rectovaginal fistula: complete perineal tear
Menstrual Disorders
•• ‘Menarche’ (First Menstruation) occurs between 11 and 15 years
•• Normal duration of menstrual period is 4–5 days
•• Peak Secretory activity is seen on Day 22 of menstrual cycle
•• Amount of blood loss is 20–80 ml
•• AVERAGE blood loss in normal periods is 50 ml
•• Precocious menstruation occurs in females aged less than 10 years
Obstetrics and Gynecology 719
•• Polymenorrhea is cyclic bleeding where cycle length is reduced to less than 21 days and remains constant at that frequency
•• Metorrhagia is irregular, acyclic bleeding
•• Menometorrhagia is irregular and excessive bleeding to the extent that menstrual period is not recognized at all
•• Hypomenorrhea is scant bleeding lasting for less than 2 days
Primary Amenorrhea
Failure of onset of menses in a girl by 16 years of age
Causes:
•• Aplasia/hypoplasia of uterus
•• Turner’s syndrome
•• Pseudohermaphroditism
•• Cretinism
Mittelschmerz
•• Lower abdominal pain cyclically 2 weeks before menstruation
•• In mid menstrual period
•• Nausea and constipation invariably absent
•• May be associated with mucoid discharge
Mullerian Agenesis
•• Mullerian agenesis (The Mayer-Rokitansky-Kuster-Hauser syndrome) second in frequency only to gonadal dysgenesis as a
cause of primary amenorrhea
•• Caused by mutations in the genes encoding anti-mullerian hormone (AMH) or its receptor (AMHR)
•• Women with this syndrome have a
–– 46, XX karyotype
–– Female secondary sex characteristics
–– Normal ovarian function, including cyclic ovulation
–– Absence or hypoplasia of the vagina
•• The uterus usually consists of only rudimentary bicornuate cords, but if the uterus contains endometrium, cyclic abdominal pain
and accumulation of blood may occur, as in other forms of outlet obstruction
•• One-third of women with this syndrome have abnormalities of the urogenital tract and one-tenth has skeletal anomalies,
usually involving the spine
•• Demonstration of a 46, XX karyotype the biphasic basal body temperature curve characteristic of ovulation and elevated
levels of progesterone during the luteal phase establish the diagnosis of mullerian agenesis
Cancer Cervix
Predisposing factors:
•• Early age of coitus
•• Multiple sex partners
•• Multiparity
•• Poor hygiene
•• Poor socioeconomic status
•• Smoking, alcohol, drug abuse
•• Associated STDS
•• Immunosupression
The staging of cervical carcinoma is clinical and generally completed with a pelvic examination under anesthesia with cystoscopy
and proctoscopy. Chest X-rays, intravenous pyelograms and computed tomography are generally required and magnetic
resonance imaging (MRI) may be used to assess extracervical extension
•• Stage 0 is carcinoma in situ
•• Stage I is disease confined to the cervix
•• Stage II disease invades beyond the cervix but not to the pelvic wall or lower third of the vagina
•• Stage III disease extends to the pelvic wall or lower third of the vagina or causes hydronephrosis
•• Stage IV is present when the tumor invades the mucosa of bladder or rectum or extends beyond the true pelvis
Cone Biopsy
Indications:
•• Cervical lesion cannot be visualized by coloposcope
•• Squamo columnar junction is not seen by coloposcope
•• Endocervical curettage demonstrates findings positive for CIN II,CIN III
•• Lack of correlation between biopsy and coloposcopy
•• Microinvasive carcinoma or adenocarcinoma in situ on coloposcopy
•• Therapeutic in case of CIN III (Best approach in elderly is hysterectomy)
•• Done under general anesthesia ideally
Complications:
•• Hemorrhage
•• Infection
•• Cervical stenosis
•• In case of visible mass: Punch biopsy
•• In case of no mass: Coloposcopic directed biopsy
Obstetrics and Gynecology 723
Etiology
•• Fibroids are monoclonal Tumors resulting from somatic mutation. They arise from
•• Single neoplastic smooth muscle cell
•• Abnormalities in chromosomes 6, 7, 12 and 14 have been identified
•• Disruption or Dysregulation of the high mobility group genes on chromosome 12 contribute to fibroid development
Role of Hormones:
•• Estrogen is a promoter of fibroid growth and not its causal factor
•• Fibroids rarely found before puberty
•• They stop growing after menopause
•• New fibroids rarely appear after menopause
•• They often grow rapidly during pregnancy and amongst Pill Users
•• GnRH agonists create a hypoestrogenic environment that results in a reduction of the size of fibroid
•• Some Peptide growth factors play a role in etiology
•• Epidermal Growth Factor (EGF) induces DNA synthesis in fibroids and myometrial cells
•• Estrogen exerts its effect through EGF
PIRFENIDONE, an antifibrotic agent suppresses fibroid growth as it inhibits fibrogenic cytokines including basic FGF, PDGF, TGF-beta
and EGF.
Submucous Fibroid
•• Located beneath the endometrium and can grow into the uterine cavity
•• Least common (about 5%) BUT maximum symptoms
Fate of Submucous fibroid
–– Surface necrosis (abnormal bleeding and anemia)
–– Polypoid change following pedicle formation
–– Infection
–– Degenerations including Sarcomatous change
Subserous Fibroid (Subperitoneal)
•• Located just beneath the serosal surface and grow out towards the peritoneal cavity, causing bulging of the peritoneal surface
of the uterus
•• They may develop a pedicle, become pedunculated and reach a large size within the peritoneal cavity without producing symptoms
724 USMLE Step 2 CK Platinum Notes
Pathological Changes
1. Fibroids are pseudoencapsulated solid Tumors
2. Well demarcated from surrounding myometrium
3. The false capsule has more parallel arrangement while the tumor has whorled appearance
4. The capsule is pinkish in color in contrast to whitish appearance of the tumor
5. The capsule is separated from the tumor by A THIN LOOSE AREOLAR TISSUE
6. The blood vessels run through this plane to supply the tumor
7. The tumor is shelled out during Myomectomy through this plane
8. The periphery of tumor is more vascular and has more growth potentiality
9. The center of the tumor is least vascular and likely to degenerate
CELLULAR LEIOMYOMAS: Are Tumors with mitotic counts of five to ten per 10 consecutive high power fields that lack cytological
atypia
LEIOMYOSARCOMAS: Previously diagnosed on the basis of mitotic count of 10 mitotic figures per 10 high-power fields. The new
factors recently recognized are CELLULAR ATYPIA and COAGULATIVE NECROSIS of tumor cells.
Myomectomy
•• Indications
–– Infertile Women in her reproductive period desirous of having a baby
–– Recurrent pregnancy wastage due to fibroid
•• Prerequisites Prior to Myomectomy
–– Examination of Husband (From fertility point of view)
–– Hysterosalpingography (to detect a fibroid encroaching the uterine cavity OR polyp OR tube)
–– Diagnostic D+C (in cases of irregular cycles to exclude endometrial carcinoma)
•• Facts to be in Mind Prior to Consideration of Myomectomy
•• It should be done mainly to preserve the reproductive function
•• The wish to preserve the menstrual function in parous women should be judiciously complied
•• More risky operation when the fibroid is too big and too many
•• Chance of recurrence (5–10%)
•• Chance of persistence of menorrhagia (1–5%)
•• Increased rate of relaparotomy to the extent of 20–25%
•• Pregnancy rate is about 40–50%
•• Pregnancy following myomectomy should have a mandatory hospital delivery, although the chance of scar rupture is rare.
726 USMLE Step 2 CK Platinum Notes
Contraindications
•• Husband proved infertile. In the face of advent of ART, counseling is imperative
•• Associated bilateral infective tubo-ovarian mass
•• Infected fibroid
Technically Difficult Cases with Poor Reproductive Outcome
•• Big broad ligament fibroid
•• Too many fibroids
Advantages of Hysterectomy Over Myomectomy
•• There is no chance of recurrence
•• Adnexal pathology and the unhealthy cervix can also be removed
Role of Vaginal Hysterectomy in Fibroid Surgery
•• Fibroids with size of 10-12 weeks of pregnancy and associated with uterine prolapse are better dealt by Vaginal Hysterectomy
with repair of pelvic floor
Endometrial Carcinoma
Occurs most often in the sixth and seventh decades of life
Symptoms often include
•• Abnormal vaginal discharge
•• Abnormal bleeding which is usually postmenopausal
•• Leukorrhea
Between 75 and 80% of all endometrial carcinomas are adenocarcinomas
Adenocarcinoma with squamous differentiation is seen in 10% of patients; the most differentiated form is known as adenoacanthoma
and the poorly differentiated form is called adenosquamous carcinoma
Most Malignant: Clear Cell carcinoma
Other less common pathologies include:
•• Carcinoma (5%) and papillary serous carcinoma
•• Secretory (2%)
•• Ciliated, clear cell
•• Undifferentiated carcinomas
Lymph nodes involved are:
•• Para-aortic
•• Presacral
•• Inguinal
Histological Grade
•• Myometrial invasion
•• Lymph node metastasis
•• Cervical extension (Cervical involvement is associated with increased risk of extrauterine disease and Lymph node metastasis)
•• Tumor size
•• Hormone receptor status (Receptor positive-better prognosis)
•• Ploidy status (Aneuploid tumors have better prognosis than diploid tumors)
•• Oncogene expression/mutation (HER 2/neu, k ras: poor prognosis)
728 USMLE Step 2 CK Platinum Notes
‘The Lynch syndrome’ occurs in families with an autosomal dominant mutation of mismatch repair genes MLH1, MSH2, MSH6, and
PMS2, which predispose to nonpolyposis colon cancer as well as endometrial and ovarian cancer
OVARIAN TUMORS
Mucinous Cystadenoma
•• Largest benign ovarian tumor
•• Most common ovarian tumor which can lead to Pseudomyxoma peritoni
•• Usually unilateral
•• Epithelium resembles Epithelium of Endocervix
•• Occasionally, associated with dermoid cysts or Brenner’s tumor
•• Torsion is most frequently seen with this tumor
Fibroma
•• Most common benign ovarian tumor of connective tissue origin
•• Most common ovarian tumor associated with Meigs’ Syndrome
•• They resemble histologically with Brenner Tumor
Meigs’ Syndrome
•• Ascites
•• Hydrothorax (Right Sided)
•• Benign tumor of ovary (FIBROMA)
Pseudo-Meigs Syndrome
•• Ascites
•• Hydrothorax (Right Sided)
•• Malignant ovarian tumor of ovary
Remember
•• Among Gynecological cancers it Ranks 3rd
•• Among all Gynecological cancers in India it accounts for 5%
•• Among all deaths due to Gynecologic cancers, most common cause is due to ovarian cancer
•• Most common Ovarian cancer is Papillary Serous Cystadenocarcinoma
•• Second most common ovarian cancer is Endometroid Carcinoma
•• Most common in women < 20 years of age is Dysgerminoma
•• Most common hormonally active ovarian tumor is Granulosa-Theca cell Tumors
Epithelial Cancers
Remember
•• Constitute about 80% of all Primary ovarian Carcinomas
•• Bilateral 50% cases
•• Cystic type is more common than Solid type
•• Most common histological type is Papillary Serous Cystadenocarcinoma
•• Median age 60 years
•• The single most important risk factor for Epithelial ovarian cancer is Age > 40 years
Obstetrics and Gynecology 731
Krukenbergs Tumor
•• Are Secondary tumor of the ovary
•• They are invariably bilateral solid Tumors
•• The primary of these Tumors are: Carcinoma of stomach (70%) (Most Common Site), Carcinoma of large bowel (15%), Carcinoma
breast (6%)
Gross Features
•• SURFACE: Smooth with no tendency to form adhesions; No infiltration of capsule
•• SHAPE: Retains the shape of normal ovary
•• CONSISTANCY: Solid waxy; Cystic spaces may be seen due to degeneration.
•• MOBILITY: Freely movable in the pelvis
Microscopy
•• Cellular or myxomatous stroma with Scattered SIGNET RING CELLS. (Mucin Secreting cells)
•• Involvement of Ovary is by Retrograde Lymphatics
•• The first halt of tumor cells between Stomach and Ovary is Superior Gastric Lymph Nodes
TORSION: Torsion of the Pedicle (Axial Rotation) is very common during pregnancy (12%) cases.
Treatment
•• Surgically Debulk all Stages of Tumor that involves:
Total Abdominal Hysterectomy (TAH) + BSO+ Omentectomy + any other tumor > 2 cm diameter size
•• In Stage III and Stage IV Debulking Surgery is done or at least biopsy of the tumor is obtained
ADJUVANT THERAPY
•• Radiotherapy _________STAGE Ib and Ic
•• Chemotherapy ________STAGE II, III, IV
Feminizing Tumors
•• Granulosa cell tumor
•• Theca cell tumor
•• Fibromas
During Pregnancy
Uncomplicated Complicated
Principle: To remove the tumor as soon as the diagnosis is made The tumor is removed irrespective of the period of Gestation
Best Time: For Elective Operation is 14-18th week
•• If the diagnosis is made before this time the patient should be
kept under observation
•• If the diagnosis is made in 3rd trimester
Immediate removal is done
•• If the diagnosis is made beyond 36 weeks
The operation is withheld till delivery and the tumor is removed
as early in puerperium as possible
During Labor
•• If the tumor is well above the presenting part •• If the tumor is impacted in the pelvis causing obstruction
A Watchful Expectancy hoping for Vaginal delivery Cesarean Section followed by removal of tumor in the same sitting
Obstetrics and Gynecology 735
During Puerperium
On Occasion, the diagnosis is made following delivery. The tumor should be removals early in puerperium as possible.
Pseudomyxoma Peritonei
This is a rare condition resulting from
•• Rupture of a mucocele of the appendix
•• A mucinous ovarian cyst or
•• Mucin-secreting intestinal or ovarian adenocarcinoma
–– The abdomen becomes filled with masses of jelly-like mucus
–– Colloid carcinoma arising from the stomach or colon with peritoneal implants may resemble pseudomyxoma at laparotomy
–– The course of this type of highly malignant tumor is one of rapid cachexia and early death
–– The diagnosis usually can be made by the appearance of many highly malignant cells in the peritoneal implants
736 USMLE Step 2 CK Platinum Notes
Differentiate
Vaginismus Dyspareunia Desire Disorder Arousal Disorder Anorgasmia
Painful reflex spasm of Pain related to sexual Apathy for and Failure of vaginal Women never has had
paravaginal thigh adductors intercourse lack of enjoyment lubrication and organism by any means
Diagnosed by physical of sex lack of pelvic
examination engorgement
Almost always psychogenic
Vaginal dilators useful as
therapy
Remember
•• Hirsutism: Male pattern hair growth in females
•• Virilization: Male pattern hair growth in females + Clitoromegaly/Baldness/Loss of female body contours/Lowering of
voice/↑Muscle mass
•• ↑DHEA: Adrenal tumor
•• ↑Testosterone: PCOD
•• ↑↑↑Testosterone: Ovarian Tumor (Sertoli leyding, Hilus cell tumor)
•• ↑Serum 17 OH Progesterone levels: Congenital Adrenal Hyperplasia
•• Maternal virilizing tumor of pregnancy is called luteoma of pregnancy can result in masculinization of female fetus
Obstetrics and Gynecology 737
Clomiphene
•• Antiestrogen with weak agonistic activity as well
•• Enclomiphene has antiestrogenic effect
•• It is indicated in the treatment of anovulation or oligo-ovulation in patients desiring pregnancy, whose sexual partners have
adequate sperm, and who have potentially functional hypothalamic-hypophyseal-ovarian systems and adequate endogenous
estrogen
•• Chances of pregnancy are 3 fold as compared to placebo
•• Risk of multiple pregnancy is 6-10 fold
•• It may be used to treat corpus luteum dysfunction
•• It is used to detect abnormalities of the hypothalamic-pituitary-gonadal axis in males
•• Used in Stein Levinthal syndrome
•• It is used to treat infertility in males with oligospermia
•• It is sometimes given as a test dose to aid in predicting whether an ovulatory response might occur
Tamoxifen
•• Nonsteroidal antiestrogenic
•• It is indicated for adjuvant treatment of axillary node-negative breast cancer in women following total mastectomy or segmental
mastectomy, axillary dissection, and breast irradiation
•• Indicated for adjuvant treatment of axillary node-positive breast cancer in postmenopausal women following total
mastectomy or segmental mastectomy, axillary dissection, and breast irradiation
•• That women whose tumors are estrogen receptor-positive are more likely to benefit from tamoxifen therapy
•• It is indicated to reduce the risk of developing breast cancer in women who have been determined to be at high-risk for
developing this cancer
Danazol
•• Androgen derivative
•• Gonadotropin inhibitor
Indications
•• Endometriosis treatment
•• Fibrocystic Breast disease
•• Cyclic mastalgia, noncyclic mastalgia
•• Angioedema, hereditary (prophylaxis)
•• Menorrhagia, primary
•• Gynecomastia
•• Puberty precocious
GnRH Analogues
•• Goserelin
•• Buserelin
•• Nafarelin
•• Histrelin
•• Triptorelin
738 USMLE Step 2 CK Platinum Notes
Used for
•• Precocious puberty
•• Infertility
•• Ca breast
•• DUB
•• Endometriosis
•• Fibromyoma uterus
•• Hirsutism
Important Syndromes
•• Sheehan’s syndrome •• Postpartum pituitary necrosis
•• Asherman’s syndrome •• Overzealous curettage resulting in endometrial synechia
•• Stein levinthal syndrome •• PCOD
•• Kallaman’s syndrome •• Primary amenorrhea, hyposmia, failure of secondary sexual features
Sheehan Syndrome
•• It is a condition affecting women who experience life-threatening blood loss during or after childbirth
In Sheehan’s syndrome, the damage occurs to the pituitary gland
•• The result is the permanent underproduction of essential pituitary hormones (hypopituitarism)
•• For some women, Sheehan’s syndrome seems to cause few, if any, symptoms. For others, Sheehan’s syndrome can lead to an adrenal
crisis—a life-threatening shortage of the hormone cortisol. Treatment of Sheehan’s syndrome involves hormone replacement
therapy.
Symptoms:
In most cases, the signs and symptoms of Sheehan’s syndrome appear slowly, after a period of months or even years
Signs and symptoms of Sheehan’s syndrome include:
•• Slowed mental function, weight gain and difficulty staying warm, as a result of an underactive thyroid (hypothyroidism)
•• Difficulty breastfeeding or an inability to breastfeed
•• No menstrual periods (amenorrhea) or infrequent menstruation (oligomenorrhea)
•• Loss of pubic or underarm hair
•• Low blood pressure
•• Fatigue
•• Weight loss
Remember
•• Donovaniosis •• Calymmatobacter granulomatosis
•• LGV •• Chlamydia trachomatis
•• Chancroid •• H. ducreyi, Herpes hominis
•• Condyloma acuminate •• HPV 6, 11, 16, 18
•• Yaws •• T. pertune
•• Pinta •• T. carateum
Obstetrics and Gynecology 739
1. Miconazole
2. Penicillin
3. Spectinomycin
4. Spiramycin
5. Tetracycline
6. Metronidazole
Ans. 6. Metronidazole
3. Not recommend to her the use of herbal remedies because such products are classified as dietary supplements and are not
FDA-regulated for purity, safety, and efficacy and there is significant data regarding then on teratogenic potential of herbal
medications in humans
4. Not recommend to her the use of herbal remedies because such products are classified as dietary supplements and are not
FDA-regulated for purity, safety, and efficacy and there is no significant data regarding the teratogenic potential of herbal
medications in humans
Ans. 4. Not recommend to her the use of herbal remedies because such products are classified as dietary supplements and are
not FDA-regulated for purity, safety and efficacy and there is no significant data regarding the teratogenic potential of herbal
medications in humans.
4. Placenta previa
5. Membranaceous placenta
6. Placenta accreta
Ans. 4. Placenta previa